Ilovepdf Merged

Download as pdf or txt
Download as pdf or txt
You are on page 1of 503

Назва наукового напрямку (модуля): Семестр: 6

2019 КРОК_1 англ


Опис:
КРОК_1 англ
Перелік питань:
1. A 12-year-old child developed nephritic syndrome (proteinuria, hematuria, cylindruria) 2 weeks after
a case of tonsillitis, which is a sign of affected glomerular basement membrane in the kidneys. What
mechanism is the most likely to cause the basement membrane damage?
A. Cytotoxic
B. Granulomatous
C. Antibody-mediated
D. Reaginic
E. * Immune complex
2. Several minutes after a dentist administered Novocain for local anesthesia of a patient’s tooth, the
following symptoms sharply developed in the patient: fatigue, skin itching. Objectively the following
can be observed: skin hyperemia, tachycardia, BP dropped down to 70/40 mm Hg. What kind of
allergic reaction is this pathology?
A. Immune complex
B. Cytotoxic
C. Stimulating
D. Cell-mediated immune reaction
E. * Anaphylactic
3. During blood transfusion a patient has developed intravascular erythrocyte hemolysis. What kind of
hypersensitivity does the patient have?
A. IV type (granulomatosis)
B. I type (anaphylactic)
C. III type (immune complex)
D. IV type (cellular cytotoxicity)
E. * II type (antibody-dependent)
4. During autopsy of a 9-month-old girl’s body, who died due to severe pneumonia complicated by
sepsis, there is a lack of a thymus. In the lymph nodes, lymphoid follicles and cortex are absent;
spleen follicles are reduced in size, without light zones and plasma cells. What is the reason for such
structural changes?
A. Thymus aplasia
B. Accidental involution of thymus
C. Thymus hypoplasia
D. Thymus atrophy
E. * Thymus agenesis
5. A 10-year-old child was found to have a congenital hypoplasia of the left kidney. Ultrasound
examination revealed that the right kidney was markedly enlarged and had regular shape. No
functional disorders were revealed. Specify the process that developed in the right kidney:
A. Metaplasia
B. Working hypertrophy
C. Hypertrophic growth
D. Pseudohypertrophy
E. * Vicarious hypertrophy
6. Histological examination of the biopsy material obtained from the lower third of the esophagus of a
57-year-old male with the symptoms of continuous reflux revealed the change of the stratified
squamous epithelium to the single layer columnar glandular epithelium with signs of mucus
production. Specify the pathological process in the mucous membrane:
A. Regeneration
B. Hyperplasia
C. Hypertrophy
D. Organization
E. * Metaplasia
7. Chronic inflammation and transformation of the one-layer ciliated epithelium into multiple-layers flat
epithelium was revealed in the thickened mucous membrane of the bronchus bioptate of the patient
with smoke abuse. Which of the processes is the most likely?
A. Epithelium hypertrophy
B. Hyperplasia of the epithelium
C. Squamous cancer
D. Leucoplacia
E. * Metaplasia
8. 10 years ago a patient underwent extraction of his right kidney on account of a tumour. After that the
volume of his left tumour grew by 50%. What process was developed in the kidney?
A. Hypertrophic enlargement
B. Neurohumoral hypertrophy
C. Pseudohypertrophy
D. Functional hypertrophy
E. * Vicarious hypertrophy
9. Examination of a 40-year-old man ill with stenosing (without metastases) esophageal carcinoma
revealed the folowing changes: atrophy of skeletal muscles and fatty tissue.His skin is sallow,
epidermis is attenuated, heart has grown smaller. Myocardium and liver are brown. What is the most
probable diagnosis?
A. Addison’s disease
B. Myasthenia
C. Cancerous cachexia
D. Brown atrophy
E. * Alimentary cachexia
10. In a male patient, a visual examination of the skin of his back revealed some spherical tumour, 2 cm
in diameter, which was thick in consistency and had clear borders with the surrounding tissues.
Microscopically, the tumour consisted of some chaotically interlaced bundles of collagenous fibres
and a small numbt: of connective tissue cells. Name the tumour.
A. Leiomyoma
B. Haemangioma
C. Melanoma
D. Lipoma
E. * Fibroma
11. A 65-year-old woman underwent removal of some tumour, 1.0 x 1.0 x 0.8 cm in size, localized under
the skin of her thigh. Macroscopically, the tumour had a connective-tissue capsule and was
represented on section with a yellowish lobate tissue. Microscopically, there were large cells, which
had the sudanophilic cytoplasm and formed lobules separated with connective-tissue layers. Name
this tumour.
A. Hibernoma
B. Liposarcoma
C. Fibroma
D. Desmoid
E. * Lipoma
12. A thick encapsulated node, 2.0 cm in diameter, was surgically removed from the mammary gland of a
female patient. On section, the tissue of the node was white-pink and fibrous. Microscopically, the
tumour consisted of glandular structures, which had no signs of cellular atypism and were
compressed with a connective tissue vegetating around. In the tumour, the stroma prevailed over the
glandular parenchyma. What is your diagnosis?
A. Adenoma
B. Nonproliferative mastopathy
C. Proliferative mastopathy
D. Adenocarcinoma
E. * Fibroadenoma
13. On examination of a 6-year-old child with a tumour on the femoral diaphysis, several metastatic foci
of another osseous localization were found. A histological examination of the primary tumour
revealed that it consisted of some round cells, which had scanty cytoplasm, were characterized by an
insignificant tendency to formation of pseudorosettes, and manifested themselves with solitary
mitoses. What is your diagnosis?
A. Plasmacytoma
B. Chondroma
C. Cancer
D. Fibrosarcoma
E. * Ewing's sarcoma
14. A 40-year-old male patient underwent removal of a tumour, 2 cm in diameter, which was localized in
the region of the cerebellopontine angle of the brain stem and tended to grow into the auditory
meatus. Histologically, the tumour consisted of spindle cells with rod-shaped nuclei; the tumour cells
and fibres formed rhythmic structures. Name the kind of the tumour.
A. Medulloblastoma
B. Meningioma
C. Oligodendroglioma
D. Astrocytoma
E. * Schwannoma
15. A 6-year-old boy underwent removal of a tumour localized along the median line of the cerebellum.
Histologically, the tumour consisted of the cells which had a poor crown of the cytoplasm, a
hyperchromatic nucleus, demonstrated a mitotic activity and tended to form "rosettes". What is your
diagnosis?
A. Astrocytoma
B. Oligodendroglioma
C. Multiform spongioblastoma
D. Bipolar spongioblastoma
E. * Medulloblastoma
16. A tumour removed from the white matter of the right hemisphere of the brain is some soft "motley"
node, 4 cm in diameter, without any clear borders with the substance of the brain. Microscopically,
the tumour consists of polymorphous cells with numerous pathological mitoses, and it also reveals
foci of necrosis and haemorrhages which occurred at different time. Name the tumor.
A. Oligodendroglioma
B. Oligodendroglioblastoma
C. Astrocytoma
D. Astroblastoma

E. * Glioblastoma
17. A histological express examination of a tumour node of a mammary gland revealed some
encapsulated formation with proliferation of alveoli and intralobular ducts; the interstitial connective
tissue grew either around or inside the ducts. Which of the tumours took place?
A. Foliaceous tumour
B. Noninfiltrating intralobular carcinoma
C. Infiltrating intralobular carcinoma
D. Paget's disease
E. * Fibroadenoma
18. During an operation on a woman, her cyst-like changed ovary was removed; it was a thin-walled
cavity filled with some yellowish transparent fluid and having a smooth inner surface. Histologically,
the cavity wall was lined with the cubical epithelium. Name the kind of the tumour.
A. Mucinous cystadenoma
B. Serous cystadenocarcinoma
C. Granulosa cell tumour
D. * Serous cystadenoma
E. Pseudomucinous cystocarcinoma
19. Autopsy of a man who died of intoxication revealed cachexia, muscular atrophy, wrinkled skin,
decreased mass of the inner organs, stenosing tumor of the stomach with metastases to the liver and
regional nodes. Which type of cachexia is most probable?
A. Alimentary
B. Hypophyseal
C. Cerebral
D. In chronic infectious disease
E. * Cancerous
20. In a 37-year-old female patient, an enlarged dense mammary gland was revealed, the nipple with the
areola of the mammary gland were oedematous, the skin had an appearance of an "intradermal bleb".
On microscopic examination, the gland tissues revealed layers of tumour cells with polymorphous
nuclei and a large number of pathological mitoses. The tumour stroma was poorly expressed. Make
the diagnosis.
A. Adenofibroma
B. Scirrhous carcinoma
C. Paget's disease
D. Adenoma
E. * Medullary carcinoma
21. On bronchoscopy, an exophytic tumour was found; it was localized in the bronchus and significantly
narrowed its lumen. Histologically, the tumour consisted of complexes of polymorphous epithelial
cells with hyperchromatic nuclei and pathological mitoses. Among the tumour cells there were
eosinophilic concentric structures. Make a diagnosis of the tumour.
A. Nonkeratinizing squamous cell carcinoma
B. Large-cell carcinoma
C. Small-cell carcinoma
D. Adenoacanthoma
E. * Keratinizing squamous cell carcinoma
22. For a histological examination, an eyeball was sent; some black tumour, 1 x 0.4 cm in size, was
revealed in its vascular membrane. Microscopically, the tumour consisted of large polymorphous
cells grouped in alveolar structures. The cytoplasm of the cells contained some brown pigment. What
is your diagnosis?
A. Neurilemmoma

B. Angiosarcoma
C. Neuroblastoma
D. Ganglioneuroblastoma
E. * Melanoma
23. A microscopic examination of a biopsy from a large intestine revealed some tumour made of the
columnar epithelium which formed atypical glandular structures of various shapes and size. The
epithelial cells were polymorphous and with hyperchromatic nuclei, there were pathological mitoses.
What is your diagnosis?
A. Basal cell carcinoma
B. Solid carcinoma
C. Mucinous carcinoma
D. Carcinoma simplex
E. * Adenocarcinoma
24. A male patient, who suffered from chronic bronchitis for a long period of time, revealed a pulmonary
tumour, which was closely connected with the bronchial wall and grew in the form of a polyp.
Microscopically, the tumour consisted of complexes of polymorphous epithelial cells with a large
number of mitoses. Among the tumour cells there were stratified concentric oxyphilic structures.
Name the histological type of the tumour.
A. Mucinous carcinoma
B. Nonkeratinizing squamous cell carcinoma
C. Adenocarcinoma
D. * Keratinizing squamous cell carcinoma
E. Solid carcinoma
25. A male underwent surgical removal of a black tumour, 2 cm in diamete, from the skin of his thigh.
Microscopically, the tumour consisted of polymorphous cells, the cytoplasm of most of them having
some brown pigment (with a positive reaction to DOPA). A large number of pathological mitoses
was registered. Which of the tumours listed below was the most probable?
A. Carcinoma
B. Sarcoma
C. Carcinosarcoma
D. Nevus
E. * Melanoma
26. On bronchoscopy in the initial part of the upper lobe bronchus of the right lung some polyp-like
formation, 1.0 cm in diameter, with a superficial ulcer was found. A histological examination
revealed a tumour consisting of lymphocyte-like cells with hyperchromatic nuclei; the cells grew in
layers and bands. Indicate the most probable tumour.
A. Undifferentiated large-cell carcinoma
B. Squamous cell carcinoma
C. Adenocarcinoma
D. Glandular squamous cell carcinoma
E. * Undifferentiated small-cell carcinoma
27. A histological examination of some spherical neoplasm located under the surface of the skin,
revealed papilliform vegetations of the epithelium with phenomena of acanthosis and
hyperkeratinization. The tumour stroma consisted of a large amount of the connective tissue and
vessels. What tumour took place?
A. Keratoacanthoma
B. Carcinoma in situ
C. Keratinizing squamous cell carcinoma
D. Nonkeratinizing squamous cell carcinoma

E. * Papilloma
28. A 26-year-old male patient underwent surgical removal of a tumour, 4 x 5 cm in size, which was
surrounded by a capsule and located in the white matter of his brain. Microscopically, the tumour
consisted of the stellate and glia cells having various size and located among the glial fibres. Name
the tumour.
A. Oligodendroglioma
B. Astroblastoma
C. Glioblastoma
D. Ependymoma
E. * Astrocytoma
29. On supersonic examination of a 48-year-old male patient, a hepatic neoplasm was diagnosed and a
puncture biopsy was made. Microscopically, the tumour consisted of atypical hepatocytes which
formed trabeculae, acini or tubules. The tumour stroma was poor and had thin-walled blood vessels.
Which of the kinds of tumours listed below was the most probable?
A. Hepatocellular adenoma
B. Metastasis of adenocarcinoma
C. Cholangiocellular carcinoma
D. Solid carcinoma
E. * Hepatocellular carcinoma
30. Autopsy of a woman with cerebral atherosclerosis revealed in the left cerebral hemisphere a certain
focus that is presented by flabby, greyish and yellowish tissue with indistinct edges. What
pathological process is the case?
A. Senile encephalopathy
B. Multifocal tumor growth with cystic degeneration
C. Multiple foci of fresh and old cerebral hemorrhage
D. Focal encephalitis
E. * Ischemic stroke
31. Autopsy of a 75-year-old man with a long history of atherosclerosis revealed a grey irregular-shaped
focus of loose consistency in the right parieto-temporal region of brain. What is the most likely cause
of this process?
A. Thrombosis of tomentum cerebri
B. Thrombosis of the right anterior cerebral artery
C. Thrombosis of the right posterior cerebral artery
D. Thrombosis of basilar artery
E. * Thrombosis of the right medial cerebral artery
32. Autopsy of a 75-year-old patient who had been suffering from disseminated atherosclerosis and died
under chronic cardiac failure revealed constriction and deformation of coronary arteries, tuberous
intima whose section appeared to be white and petrosal. Specify the stage of atherosclerosis
morphogenesis:
A. Atheromatosis
B. Lipoidosis
C. Liposclerosis
D. Bilipid
E. * Atherocalcinosis
33. Morphological examination of an amputated gangrenous extremity revealed that the lumen of
femoral artery was constricted due to stony, partly ulcerated plaques with obturating thrombi. What is
the most likely diagnosis?
A. Nodular periarthritis
B. Obliterating endartheriitis

C. Non-specific aortoartheriitis
D. Obliterating thromboangiitis
E. * Atherosclerosis
34. A male patient developed substernal pains at 8 a.m., and at 9 a.m. myocardial infarction was
diagnosed by ECG data at the admission department. Ten minutes later the patient died. What most
reliable sign of myocardial infarction will be found on histological examination?
A. Vacuole dystrophy of cardiomyocytes
B. Fat infiltration of cardiomyocytes
C. Reduced activity of dehydrogenases in fibroblasts
D. Relaxation of myofibrils in cardiomyocytes
E. * Disappearance of glycogen in cardiomyocytes
35. On autopsy of a 68-year-old male, who died from cardiac decompensation, the myocardium of the
anterior wall in left ventricle of his heart contained an irregular grey focus, 5 x 4 cm in size, with a
dense consistency, fibrous structure and clear borders. What pathological process in the myocardium
did the pathologist reveal?
A. Myocarditis
B. Microfocal cardiosclerosis
C. Infarction
D. Rheumatism
E. * Postinfarction cardiosclerosis
36. On autopsy of a 66-year-old male, who died from acute cardiac failure, an acute venous plethora of
the internal organs was found. The cardiac cavities were dilated; a myocardial section revealed some
dim yellowish focus, 3.5 x 4 cm in size, in the anterior wall of the left ventricle. The coronary arteries
had stenosing atheromatous plaques. Which of the diagnoses listed below was the most probable?

A. Microfocal cardiosclerosis
B. Macrofocal cardiosclerosis
C. Fatty degeneration of myocardium
D. Myocarditis
E. * Myocardial infarction
37. Three weeks following a myocardial infarction, a 54-year-old man presents with fever, productive
cough, and chest pain. The pain is worse with inspiration, better when he is sitting up, and not
relieved by nitroglycerin. Physical examination finds a friction rub along with increased jugular
venous pressure and pulsus paradoxus (excess blood pressure drop with inspiration). Which of the
following is the most likely explanation for these findings?
A. Caplan’s syndrome;
B. Dressler’s syndrome;
C. Ruptured papillary muscle;
D. Ventricular aneurysm.
E. * Ruptured ventricular wall;
38. In a patient with hypertension disease, in the context of a hypertensive crisis, acute renal failure, from
which he died, developed. What are the most likely morphological changes in kidney arterioles?

A. Constrictive atherosclerosis
B. Hyperelastosis
C. Sclerosis
D. Hyalinosis
E. * Fibrinoid necrosis

39. Autopsy has revealed shrunken kidneys weighing 50 mg, with finegrained surface and uniformly
thinned substance. Microscopic investigation has shown the thickening of arteriole walls due to
accumulation of homogeneous anhistic pink-colored masses in them. Glomerules were undersized,
sclerotic, with atrophied tubules. What disease are these changes characteristic of?
A. Membranous nephropathy
B. Pyelonephritis with kidney shrinkage
C. Renal amyloidosis
D. Acute glomerulonephritis
E. * Essential hypertension
40. A 60-year-old woman has reported incrising fatigue over the past year. An abdominal ultrasound scan
shows that her kidneys are symmetrically smaller than normal whith fine-grained surface. The
microscopic appearance of the kidneys showed sclerosis and glomeruli hyalinosis. These finding are
most likely to indicate which of the following underlying conditions?
A. Escherichia coli septicemia
B. Polyarteritis nodosa
C. Atherosclerosis
D. Takayasu arteritis
E. * Systemic hypertension
41. In man of 60 years old, who had hypertension for a long time and died of chronic renal failure at
autopsy was revealed: both kidneys are greatly reduced in size, the surface is fine-grained;
histologically - the majority of glomerulars is hialinised and sclerosed, some glomerulars are -
hyperplastic; areas of sclerosis are detected in stroma, arteriolosclerosis and atherosclerosis,
elastofibrosis of large branches of renal artery. Name identified changes of kidneys.
A. Arteriosclerotic kidney
B. Atherosclerotic nephrosclerosis
C. Chronic glomerulonephritis
D. Chronic pyelonephritis
E. * Arteriolosclerotic nephrosclerosis
42. An autopsy of a 9-year-old child, who suffered from rheumatism and died of heart failure, revealed
dilatation of cavities in the ventricles of his heart. Microscopically, the myocardial stroma was
characterized by a plethora, oedema, diffuse infiltrations of histiocytes, lymphocytes, neutrophils and
eosinophils. What diagnosis was the most probable one?
A. Focal interstitial exudative myocarditis
B. Granulomatous productive myocarditis
C. Interstitial productive myocarditis
D. Alterative myocarditis
E. * Diffuse interstitial exudative myocarditis
43. An autopsy of a female revealed morphological manifestations of chronic heart failure in the right
ventricle, stenosis of the left atrioventricular aperture, insufficiency of the mitral valve.
Histologically, a connective-tissue disorganization in the form of some mucoid and fibrinoid swelling
was found with presence of blooming Aschoff s bodies against a background of focal cardiosclerosis
in the myocardium. Which of the diagnoses listed below was the most probable?
A. Scleroderma
B. Dermatomyositis
C. Polyarteritis nodosa
D. Systemic lupus erythematosus
E. * Rheumatism

44. A histological examination of the cusps of the mitral valve in the heart of a female, who died from
cardiac decompensation, revealed a focal desquamation of endotheliocytes with superimposed
thrombotic masses in these areas. The connective tissue of the valve cusp had signs of
disorganization, areas of sclerosis and angiomatosis. Diagnose the kind of valvular endocarditis.
A. Diffuse
B. Acute vegetative
C. Loffler's
D. Polypous-ulcerous
E. * Recurrent vegetative
45. An autopsy of a 7-year-old child, who died from progressing heart failure, revealed 200 ml of some
semitransparent fluid in the pericardial cavity, the surface of the epicardium had greyish filamentous
superpositions. Name the kind of pericarditis.
A. Serous
B. Purulent
C. Fibroplastic
D. -
E. * Fibrinous
46. A 45-year-old female, who complained of progressing muscular weakness, underwent a biopsy of
soft tissues on her shin. A histological examination of the biopsy revealed some microfocal
petrification of the derma and skeletal muscles, a reduced amount of glycogen and transversal
striation in the muscular fibres, some fibres were necrotized, the stroma was infiltrated by
lymphocytes, macrophages and plasma cells. Make a diagnosis of the disease.
A. Systemic scleroderma
B. Systemic lupus erythematosus
C. Rheumatism
D. Polyarteritis nodosa
E. * Dermatomyositis
47. A 38-year-old female suffers from an expressed deformity of joints of her lingers und Iocs.
Histologically, the periarticular connective tissue reveals some mucoid swelling, foci of
fibrinoid necrosis, clusters of macrophages and areas of sclerosis, the synovial membrane
has an oedema of villi, as well as their mucoid and fibrinoid swelling, the synovial cavity
contains "rice bodies". Make a diagnosis of the disease.
A. Rheumatism
B. Bekhterev's disease
C. Infectious polyarthritis
D. Polyarteritis nodosa
E. * Rheumatoid arthritis
48. An examination of the female, who died from renal insufficiency, revealed that her skin in the
regions of the bridge of the nose and lateral surfaces of the face was brown-reddish and
desquamative. On autopsy, the heart was enlarged, the cusps of the aortic valve were thickened,
dense and had thrombotic superpositions. The aortic wall had elastolysis and small scars in the
middle coat. The kidneys were motley, enlarged, with foci of haemorrhages in their cortical layer.
Microscopically, there were haematoxylin bodies in the epithelial nuclei, the basal membranes of
capillaries of the glomeruli were thickened and gave an appearance of wire loops, somewhere the
capillaries contained hyaline thrombi and foci of fibrinoid necrosis. Make a diagnosis.
A. Rheumatism
B. Arteriolar nephrosclerosis
C. Nephropathy amyloidosis
D. Arterial nephrosclerosis
E. * Systemic lupus erythematosus

49. Microscopy of the kidneys from a man died of systemic lupus erythematosus revealed sclerosed
glomeruli, the lumens of the small arteries and arterioles are narrow, the median membrane is thin,
homogeneous, eosinophilic masses are present in the subendothelial space. Immunologically these
masses contain immune complexes and fibrin. Which substance is present in the subendothelial
space?
A. Fat-protein detritus
B. Simple hyaline
C. Lipohyalin
D. Amyloid
E. * Complex hyalin
50. On autopsy of a 16-year-old male juvenile with reduced nourishment, who was ill with primary
pulmonary tuberculosis and died from an accompanying pneumonia, it was found that the 2nd
segment of the right lung had a Ghon’s focus, the peribronchial and bifurcation lymph nodes were
enlarged, united in packs, grey-yellow, dense in consistency and crumbled. Microscopically, the 3rd
segment of the right lung contained a primary affect surrounded by fibrosed tuberculous granulomata
and a connective-tissue capsule; the lymph nodes were characterized by caseous necrosis. Which of
the forms of tuberculosis was the most probable?
A. Caseous pneumonia
B. Acute focal pulmonary tuberculosis
C. Chronic course of primary tuberculosis
D. Haematogenous acute miliary tuberculosis
E. * Lymphogenous progressing of tuberculosis
51. The 48-year-old patient, suffered from fibrous-cavernous tuberculosis, has complained of weakness,
reduction of daily secretion of urine, edema of the body and extremities and increasing of blood
pressure to 180/90. The increasing of protein, presence of hyaline and grain cylinders and
erythrocytes were found in urine. The patient has died in a month because of the insufficiency of
kidney. In autopsy the enlargement of the heart and "lardaceous" kidneys with weight more than 240
g were found. What is the complication of fibrous-cavernous tuberculosis?
A. Nephrotic syndrome
B. Glomerulonephritis
C. Pyelonephritis
D. Nephrosclerosis
E. * Amyloidosis
52. In autopsy of patient, which suffered by tuberculosis for a long time, the cavity by the sizes 3х2 cm
connected with a bronchus was found out in the upper lobe of right lung. The wall of the
cavity dense has three layers: internal - pyogenic, middle - layer of tubercular granulation
tissue, external – layer of connective tissue. What is the most probable diagnosis which this
A. formation
Tuberculoma. relative to?
B. Acute focal tuberculosis.
C. * Fibrous-cavernous tuberculosis.
D. Acute cavernous tuberculosis
E. Fibrous-focal tuberculosis.
53. During autopsy of the patient, which suffered by the progressing form of a secondary tuberculosis,
enlarged dense left lung was found out. The tissue of the lung had yellow color, on the pleura there
were fibrinous coverings. Microscopic examination showed the predominance of necrotic changes.
What from the offered diagnosis is most probable?
A. Fibrous-cavernous tuberculosis.
B. Infiltrative tuberculosis.
C. Acute focal tuberculosis.
D. Fibrous-focal tuberculosis.

E. * Caseous pneumonia.
54. A male patient complained of a fever, a severe headache, dyspnoea, and tachycardia. He died under
the phenomena of heart failure. On visual examination of the body, roseolae and petechiae on the
skin of the thorax, bedsores of the shins and a gangrene of the toes were found. A histological
examination of the myelencephalon tissue revealed a hyperaemia, stases, perivascular cuffs of plasma
cells and foci of a proliferation of the microglia (Popov’s granulomata). What is your diagnosis?

A. Meningococcal meningoencephalitis
B. Anthracic meningoencephalitis
C. Typhoid fever
D. Epidemic typhus
E. * Rubella
55. A 34-year-old male injection addict died from progressing respiratory insufficiency. A
pathoanatomical examination revealed the following changes: enlarged peribronchial, bifurcation,
paratracheal and paraaortic lymph nodes; the pulmonary tissue was air-free, the surface of its section
discharged a large amount of some mucous-foamy viscous fluid. Microscopically, the lumens of
alveoli contained foamy eosinophilic masses and clusters of macrophages, among which on silvering
some small round grey-black corpuscles with a light area in the centre were identified. Make a
diagnosis of the disease.
A. Streptococcal pneumonia
B. Lung candidiasis
C. Pneumococcal pneumonia
D. Cytomegalovirus pneumonia
E. * Pneumocystic pneumonia
56. A serviceman, who returned from Chechnya, was admitted to a hospital in a bad state with
disturbances of consciousness, swallowing and respiration. The skin of his extremities, trunk and the
conjunctiva of his eyes had petechial eruptions. The death was caused by arrest of respiration. A
pathoanatomical examination of the capillaries and arterioles of the grey matter, myelencephalon,
pons and subcortical ganglia revealed destructive-proliferative endothrombovasculitis with
perivascular nodules consisting of the glial cells, lymphocytes and histiocytes. The skin and kidneys
revealed vasculitis, the heart had focal isolated myocarditis. What disease should be suspected?
A. Brucellosis
B. Poliomyelitis
C. Polyarteritis nodosa
D. Rheumatism
E. * Epidemic typhus
57. A geologist, who several months before had been on an expedition in the Central Asia, had
paroxysms of a fever, an icteric colouring of the skin, hypochromic anaemia and loss of body mass
during his life-time. On autopsy, a sharp enlargement of the spleen and liver, as well as hyperplasia
of the marrow were found. Microscopically, there was haemomelanosis of the liver, spleen and
marrow. What disease are such changes typical for?
A. Relapsing fever
B. Chronic septicaemia
C. Epidemic typhus
D. Amoebiasis
E. * Malaria

58. A 15-year-old female patient died at an infectious hospital on the 3rd day of some disease from
intoxication. On autopsy, a punctate rash on the skin, excluding the region of the nasolabial triangle,
was found. The fauces, tonsils and tongue were brightly hyperaemic (the (glowing fauces(, a
(strawberry tongue(). Microscopically, the internal organs revealed prevalence of dystrophic changes
and a sharp disturbance of circulation. Which of the diagnoses was the most probable?
A. Measles
B. Septic form of scarlet fever
C. Allergic period of scarlet fever
D. Diphtheria of respiratory tract
E. * Toxic form of scarlet fever
59. A disease in an 8-year-old boy began acutely with a catarrh of the upper respiratory tract and
conjunctivitis, on the 3rd day of the disease he developed some macrofocal papular eruption behind
the ears, and later on the face, neck and trunk. The patient died on the 10th day under the phenomena
of severe respiratory insufficiency. An autopsy revealed lobular pneumonia with foci of suppuration.
Microscopically, he had purulonecrotic panbronchitis and bronchiolitis, peribronchial interstitial
pneumonia with a focal infiltration of the interstice by lymphoid, plasma cells, macrophages and
giant multinucleate cells. Which of the diagnoses listed below was the most probable?
A. Influenza
B. Respiratory-syncytial infection
C. Adenovirus infection
D. Scarlet fever
E. * Measles
60. A 24-year-old female, who 2 months before had had diphtheria and suffered from paralysis of the soft
palate, suddenly died from heart failure. On autopsy, an eccentric hypertrophy of the heart and a dim
flaccid myocardium were found. A microscopic examination of the glossopharyngeal, phrenic nerves
and vagus revealed a lumpy breakdown of the myelin and a plethora of the epineural vessels. The 3rd
cervical sympathetic ganglion and the nodose one of the vagus were characterized by an impaired
circulation and phenomena of cytolysis. Which of the diagnoses was the most probable?
A. Exudative myocarditis
B. Alterative myocarditis
C. Isolated myocarditis
D. Toxic myocarditis
E. * Late paralysis of heart
61. Against a background of a satisfactory general state and a subfebrile body temperature, a visual
examination of a 6-year-old child revealed eruptions on the face, head and trunk in the form of red
spots and vesicles having transparent contents, some vesicles were covered with crusts. From the side
of the fauces and upper respiratory tract, expressed catarrhal changes were determined.
Microscopically, the areas of the eruptions were characterized by a plethora of the vessels, a balloon
dystrophy and giant multinucleate cells in the spinous layer of the epidermis. The oral mucosa had
small erosions, plethoric vessels and perivascular lymphohistiocytic infiltrates. Which of the
diagnoses was the most probable?
A. Scarlet fever
B. Measles
C. Smallpox
D. Rubella
E. * Varicella
62. 14 days after quinsy a 15-year-old child presented with morning facial swelling, high blood pressure,
"meat slops" urine. Immunohistological study of a renal biopsy sample revealed deposition of
immune complexes on the basement membranes of the capillaries and in the glomerular mesangium.
What disease developed in the patient?

A. Necronephrosis
B. Acute interstitial nephritis
C. Lipoid nephrosis
D. Acute pyelonephritis
E. * Acute glomerulonephritis
63. Autopsy of a 52-year-old woman with a long history of chronic glomerulonephritis revealed
significantly reduced in size, dense kidneys with a surface of fine granularity; fibrinous inflammation
of serous and mucous membranes; dystrophic changes in parenchymatous organs; cerebral edema.
The described changes of serous membranes and internal organs are caused by the following
complication:
A. Thrombocytopenia
B. Anaemia
C. Sepsis
D. * Uraemia
E. DIC syndrome
64. A 45-year-old patient with an 8-year history of tuberculosis died in a hospital of chronic renal failure.
At autopsy, the kidneys were enlarged, the cross-section surface looked greasy, histological study
revealed profuse deposits of structureless homogeneous eosinophilic masses exhibiting marked
metachromasia when stained with Congo red. What pathological process developed in the kidneys?
A. Nephrosclerosis
B. Hematogenous renal tuberculosis
C. Acute glomerulonephritis
D. Toxic nephritis against the background of antibiotic therapy
E. * Secondary amyloidosis
65. The man of 23 years old fell ill after acute hypothermia. There was high blood pressure, hematuria,
swelling of the face. The treatment was ineffective. After 6 months he died of uremia. At the section
of kidneys they are increased, flabby, cortical layer of broad, swollen, yellow-gray, dull, with a red
specs, well bordereded from crimson brain substance. Microscopically detected proliferation of
capsule glomerular epithelium, podocytes and macrophages with formation of a half-moons. Choose
a diagnosis.
A. Acute glomerulonephritis
B. Chronic glomerulonephritis
C. Acute pyelonephritis
D. Renal amyloidosis
E. * Subacute glomerulonephritis
66. The patient died at signs of azotaemic uremia. At the autopsy: enlarged kidney; swollen, red-blooded,
easily removable capsule. Cavities of kidney pelvises and cups are enhanced, filled with cloudy urine
pus, their membranes are dull, with focuses hemorrhage. At section kidney tissue is mottled,
yellow-gray areas are surrounded by a zone of hyperemia and hemorrhage. What disease does
correspond to the macroscopic view of the kidneys?
A. * Acute pyelonephritis
B. Acute glomerulonephritis
C. Renal amyloidosis
D. Chronic pyelonephritis
E. Chronic glomerulonephritis

67. 42 year old man who suffered severe typhoid fever, ARF (acute renal failure) developed, from which
he died. At autopsy the kidneys are increased in size, swollen, fibrous capsule is removed easily, at
section - cortical substance is pale gray, dark red pyramids. At histological examination in the
majority of tubules the lumen is narrowed, epithelial cells increased in size without nuclei;
glomerulars are collapsed; in stroma - edema, small leukocyte infiltration, minor hemorrhage.
Determine the renal pathology that caused the death of the patient.
A. * Necronephrosis
B. Acute pyelonephritis
C. Acute glomerulonephritis
D. Pionephros
E. Nephrotic syndrome
68. Patient of 49 years hospitalised because pain in the waist. At ultrasound examination in dramatically
enhanced renal pelvis and cups of right kidney stones were found. Nephrectomy was performed. At
morphological study: kidney is sharply increased, atrophied parenchyma, cups and pelvises are
distended, filled with clear yellowish liquid. Microscopically revealed diffuse sclerosis, tubular and
glomerular atrophy saved tubules are distended with cysts. What complications of renal stone disease
emerged in the patient?
A. * Hydronephrosis
B. Pionefrosis
C. Pyelonephritis
D. Glomerulonephritis
E. Chronic paranephritis
69. During the autopsy woman died of uremia, kidneys of different sizes were found with
macrotubercular surface, between the renal capsule and renal surface there are dense adhesions. At
microscopic examination the following focal renal changes were identified: Multiple encapsulated
abscesses, expressed limfohistiocytic interstitial infiltration and widening of the connective tissue.
Mucous membrane of kidneys with focuses of metaplasia of transitional epithelium into multilayerd.
Tubules are partially atrophied, partially distended and filled with colloid-like content Which of the
following diagnoses is the most likely?
A. * Chronic pyelonephritis
B. Chronic glomerulonephritis
C. Acute pyelonephritis
D. Acute glomerulonephritis
E. Tubuloyinterstitial nephritis
70. A scrape from the uterine cavity revealed in the blood some particles of a neoplasm which had an
organoid structure and consisted of the columnar epithelial cells which formed glandular structures;
the stroma of the neoplasm was significantly developed, the area of its (pedicle( had glomi of
thick-walled vessels. Which of the tumours was the most probable?
A. Glandular hyperplasia of endometrium
B. Uterine fibroadenoma
C. Endometriosis
D. Uterine adenocarcinoma
E. * Endometrial polyp
71. On gynaecological examination of a 36-year-old female, a red focus, which was 0.8 x 0.5 cm in size
and had uneven contours, was found in the mucous coat of the vaginal portion of the uterine cervix
on its border with the mouth of the womb. A microscopical examination revealed substitution of the
stratified squamous epithelium for the columnar one, as well as accumulation and new formation of
glands from the cambial elements of the columnar epithelium in the cervical channel were observed
under the integmentary epithelium. Which of the diagnoses listed below was the most probable?
A. Adenomatosis of uterine cervix

B. Healing endocervicosis
C. Simple endocervicosis
D. Dysplasia of epithelium of uterine cervix
E. * Proliferative endocervicosis
72. In a biopsy of сervix of a 26-year-old woman the diagnosis following was established:
pseudo-erosion. What microscopical changes has the pathologist revealed?
A. Cell-atypia of an epithelium of an mucosal epithelium
B. Keratinization of an epithelium
C. “Carcinomatous pearls”
D. Local inflammation and necrosis in mucosa
E. * Local changes of a stratified squamous epithelium on single-layer prismatic one
73. Diagnostic scraping was performed to the woman with dysfunctional uterine bleeding. Multiple
convoluted glands, ganglially dilated cavities of some glands were revealed histologically in the
scrape. Name the type of general pathological process.
A. Hypertrophic excrescence
B. Metaplasia
C. Displasia
D. Atrophy
E. * Glandular- cystic hyperplasia
74. A patient with gastric juice hypersecretion has been recommended to exclude from the diet rich
broths and vegetable infused water. A doctor recommended it, because these food products stimulate
production of the following hormone:
A. * Gastrin
B. Secretin
C. Cholecystokinin
D. Somatostatin
E. Neurotensin
75. A 49-year-old patient was found to have a disproportionate enlargement of hands, feet, nose, ears,
superciliary arches and cheek bones. Blood test revealed hyperglycemia, impaired glucose tolerance.
What is the most likely cause of this pathology development?
A. * Hypersecretion of growth hormone
B. Posterior pituitary hormone hypersecretion
C. Insulin hyposecretion
D. Vasopressin hyposecretion
E. Glucocorticoid hypersecretion
76. An autopsy of the body of an elderly man, who was suffering from acute intestinal disorder during
his last 2 weeks of life, has revealed the following change in the rectum and sigmoid colon: brown
and green film covering the mucosa is detected. The itestinal wall is thickened; the cavi- ty sharply
narrows down. Microscopy reveals mucosa necrosis of varying depth, necrotic tissue is pierced
through with fibrin threads, leucocytic infiltrati- on is observed. What diagnosis is the most probable?

A. * Fibrinous colitis
B. Catharrhal colitis
C. Ulcerative colitis
D. Follicular colitis
E. -
77. A 43-year-old woman complains of weight loss, hyperhidrosis, low-grade fever, increased irritability.
She has been found to have hyperfunction of the sympathetic-adrenal system and basal metabolism.
These disorders can be caused by hypersecretion of the followi- ng hormone:

A. * Thyroxine
B. Somatotropin
C. Corticotropin
D. Insulin
E. Aldosterone
78. Specify the type of jaundice, during which there is no direct bilirubin in blood, and urine
urobilinogen level is high:
A. * Suprahepatic
B. Hepatic
C. Subhepatic
D. Mechanical
E. -
79. A 35-year-old woman complai- ns of swollen neck. Subtotal thyreoidectomy is performed. On
histological examination of the removed part of the thyroid gland the following was detected: atrophy
of parenchyma, moderate sclerosis development, duffuse infiltration by lymphocytes and plasma
cells leading to formation of lymphatic follicles. What pathology has developed in the thyroid gland?

A. * Hashimoto’s thyroiditis
B. Follicular adenoma
C. Riedel’s thyroiditis
D. Papillary carcinoma of the thyroid gland
E. Diffuse toxic goiter
80. A 55-year-old man had been suffering from chronic glomerulonephritis. He died from chronic renal
failure. Macroscopical examination revealed on the surface of epicardium and pericardium some
greyish-white villous depositions. After their removal dilated and plethoric vessels were uncovered.
What process took place in the pericardium?
A. Organization
B. Haemorrhagic inflammation
C. Proliferative inflammation
D. Arterial hyperemia
E. * Fibrinous inflammation
81. A 39-year-old man who had been operated for the stomach ulcer died 7 days after the surgery.
Autopsy revealed that peritoneal leaves were dull, plephoric, covered with massive yellow-greenish
films, the peritoneal cavity contained for about 300 ml of thick yellow-greenish liquid. What
pathologic process was revealed in the peritoneal cavity?
A. Peritoneal commissures
B. Fibrinous serous peritonitis
C. Fibrinous haemorrhagic peritonitis
D. Serous peritonitis
E. * Fibrinous suppurative peritonitis
82. Preventive examination of a 55-year-old patient revealed type II diabetes mellitus. An endocrinologist
revealed an increase in body weight and liver enlargement. The man is non-smoker and doesn't abuse
alcohol but likes to have a good meal. Histological examination by means of diagnostic liver
puncture revealed that the hepatocytes were enlarged mostly on the lobule periphery, their cytoplasm
had transparent vacuoles showing positive reaction with sudan III. What liver pathology was
revealed?
A. Acute viral hepatitis
B. Chronic viral hepatitis
C. Alcohol hepatitis

D. Portal liver cirrhosis


E. * Fatty hepatosis
83. A female patient with a tumour of pancreas has developed mechanic jaundice resulting from
compression of a bile-excreting duct. Which duct is compressed?
A. Ductus hepaticus dexter
B. * Ductus choledochus
C. Ductus hepaticus sinister
D. Ductus cysticus
E. Ductus hepaticus communis
84. Examination of a 32-year-old patient revealed disproportional skeleton size, enlargement of
superciliary arches, nose, lips, tongue, jaw bones, feet. What gland's function was disturbed?
A. Epiphysis
B. Pancreas
C. Suprarenal
D. Thyroid
E. * Hypophysis
85. Autopsy of a 67-year-old man who died after presenting with hypoglycemic coma revealed some
areas of connective tissue growth and necrosis foci, atrophy of Langerhans islets in pancreas. What
disease might have induced such changes in pancreas?
A. Acute pancreatitis
B. * Diabetes mellitus
C. Cancer of the head of pancreas
D. Pancreas hypoplasia
E. Mucoviscidosis
86. A 55 year old man had been suffering from chronic glomerulonephritis. He died from chronic renal
failure. Macroscopical examination revealed on the surface of epicardium and pericardium some
greyish-white villous depositions. After their removal dilated and plethoric vessels were uncovered.
What process took place in the pericardium?
A. * Fibrinous inflammation
B. Organization
C. Proliferative inflammation
D. Haemorrhagic inflammation
E. Arterial hyperemia
87. Microscopic examination of periodontium revealed plethoric vessels, edema of connective tissue
along with infiltration by single neutrophils. What type of exudative inflammation in the
periodontium is it?
A. * Serous
B. Purulent
C. Putrid
D. Fibrinous
E. Catarrhal
88. A 60 year old patient complains of tongue burning, excessive salivation and glossalgia effects that
came 5 days after he started using a metal dental bridge. Objectively: mucous membrane of oral
cavity is edematic and hyperemic. What form of stomatitis is it?
A. * Catarrhal
B. Purulent
C. Ulcerous
D. Gangrenous

E. Fibrinous
89. A 23 year old man got perforation of hard palate. There was also a solid welldefined formation.
Postoperative microscopical examination of this formation revealed a large focus of caseous necrosis
surrounded by granulation tissue with endovasculitis and cellular infiltrate consisting mainly of
plasmocytes but also of lymphocytes and epithelioid cells. What is the most probable diagnosis?
A. * Syphilis
B. Tuberculosis
C. Scleroma
D. Sarcoma
E. Lepra
90. A 40 year old male patient died from cerebral edema. In anamnesis the face carbuncle was registered.
Autopsy revealed hyperemia and edema of cerebral tissue. White matter of the left hemisphere had
two cavities 6х5,5 and 5х4,5 cm large filled with yellowish-green cream-like fluid. Walls of the
cavities were built up by nerve tissue with irregular rands. What complication of carbuncle
A. * was
Acuteit?abscesses
B. Chronic abscesses
C. Empyema
D. Colliquative necroses
E. Cysts
91. Examination of puncture biopsy material of liver revealed dystrophy of hepatocytes, their necrosis
and sclerosis along with disturbance of beam and lobulous structure and formation of pseudolobules
of regeneration nodes. What is the most probable diagnosis?
A. Chronic hepatosis
B. Chronic hepatitis
C. Progressing massive liver necrosis
D. Acute hepatitis
E. * Liver cirrhosis
92. Autopsy of a 34 y.o. man who di- ed from rheumatism revealed that epi- cardium surface was villous
and covered with grey films that can be easily removed. After their removal the surface is edematic
and plethoric. What is the most probable diagnosis?
A. * Fibrinous pericarditis
B. Purulent pericarditis
C. Hemorrhagic pericarditis
D. Proliferative pericarditis
E. Catarrhal pericarditis
93. Opening of a patient’s abdominal cavity revealed for about 2,0 L of purulent fluid. Peritoneum is dull,
greyish, serous tunic of intestines has grayish layers that can be easily removed. It is most likely to
be:
A. * Fibrinopurulent peritonitis
B. Hemorrhagic peritonitis
C. Serous peritonitis
D. Tuberculous peritonitis
E. –
94. In course of gastric endoscopy the biopsy material of mucous membrane was taken. Its histological
examination revealed the following: mucous membrane is intact, thickened, edematic, hyperemic,
with small dropli- ke hemorrhages, coated with thick mucus. Name the form of acute gastritis:
A. * Catarrhal
B. Erosive
C. Fibrinous

D. Purulent
E. Necrotic
95. During the histologic lung analysis of a man who died from cardiac insufficiency the inflammation
focuses were revealed. Alveoles were full of light-pink fluid, here and there with pinkish fibers that
formed a close-meshed reticulum with a small number of lymphocytes. What type of exudate is
present in lungs?
A. * Serofibrinous
B. Hemorrhagic
C. Serous
D. Purulent
E. Fibrinous
96. Histological examination of thyroid gland of a man who died from cardiac insufficiency
accompanied by hypothyroi- dism revealed diffuse infiltration of the gland by lymphocytes and
plasmocytes with formation of lymphoid follicles, as well as atrophy of parenchyma and growth of
connective tissue. What is the most probable diagnosis?
A. * Autoimmune Hashimoto’s thyroiditis
B. Adenoma of thyroid gland
C. Purulent thyroiditis
D. Thyrotoxic goiter
E. –
97. An autopsy of a 7-year-old child, who died from progressing heart failure, revealed 200 ml of some
semitransparent fluid in the pericardial cavity, the surface of the epicardium had greyish filamentous
superpositions. Name the kind of pericarditis.
A. Serous
B. Purulent
C. Fibroplastic
D. * Fibrinous
E. -
98. A 40-year-old female patient underwent an operation of thyroidectomy. A histological examination of
the thyroid tissue revealed that its follicles differed in size, contained some foamy colloid, the
follicular epithelium was high and in some places it formed papillae. The stroma of the gland had
clusters of lymphocytes which formed follicles with light centres. Make a diagnosis of the disease of
the thyroid gland.
A. Hashimoto's disease
B. Ligneous thyroiditis
C. Acute nonsuppurative thyroiditis
D. Nodular goiter
E. * Toxic goiter
99. A histological examination of the thyroid gland revealed a significant infiltration of its tissue by
lymphocytes, formation of lymphoid follicles, an atrophy of parenchymatous elements and a
significant vegetation of the connective tissue. What disease is characterized by this picture?
A. Colloid goiter
B. Endemic goiter
C. Diffuse toxic goiter
D. Parenchymatous goiter
E. * Hashimoto's disease

100. An autopsy of a young female, who died from adrenal insufficiency, revealed diffuse hypermelanosis
of the skin, hyperplasia of the cells in islets of Langerhans in the pancreas, the adrenal glands were
sharply reduced in size and their thinned cortical substance had foci of necrosis, haemorrhages and
sclerosis. What is your diagnosis?
A. Waterhouse-Friderichsen syndrome
B. Primary aldosteronism
C. Cushing's disease
D. * Addison's disease
E. Pheochromocytoma
101. In a young male, an abundant quantity of the somatotropic hormone and enlargement of the nose,
lips, ears, loWER jaw, hands and feet were revealed. What is your diagnosis?
A. Pituitary dwarfismB.
B. shing's disease
C. Addison's disease
D. Adiposogenital dystrophy
E. * Acromegaly
102. In a male patient with an increased level of the parathormone, a histological examination in the area
of a pathological fracture of his femur revealed foci of a lacunar resolution of the osteoid beams and
new formation of a fibrous tissue. What is your diagnosis?
A. Multiple myeloma
B. Osteoblastoclastoma
C. Paget's disease
D. Osteopetrosis
E. * Parathyroid osteodystrophy
103. A male patient with phenomena of hypothyroidism died from heart failure. On histological
examination, his thyroid gland revealed a diffuse infiltration of the gland by lymphocytes and
plasmacytes, an atrophy of the parenchyma and a vegetation of the connective tissue. What disease
was it?
A. Acute nonsuppurative thyroiditis
B. Ligneous thyroiditis
C. Nodular goiter
D. Toxic goiter
E. * Hashimoto's disease
104. An autopsy of a 45-year-old female, who was suffering from arterial hypertension, diabetes mellitus
and ovarian dysfunction during past 15 years, revealed obesity by the upper type, a pituitary basophil
adenoma in the anterior lobe of the hypophysis, hyperplasia of the adrenal cortex. Which of the
diagnoses listed below was the most probable?
A. Cushing's syndrome
B. Hypertensive disease
C. Addison's disease
D. Adiposogenital dystrophy
E. * Cushing's disease
105. A 50-year-old female took medical advice complaining of excretion of a large amount of urine and
excessive thirst. On examination, her nourishment was reduced, the skin was dry, density of the urine
ranged from 1001 to 1010, data of an ultrasound examination and computed tomography of the brain
revealed a tumour in the posterior lobe of the hypophysis. Indicate the most probable disease.
A. Acromegaly
B. Babinsky-Frelich disease
C. Simmonds disease

D. Cushing's disease
E. * Diabetes insipidus
106. A 46-year-old male patient, who suffered from bulimia, polydipsia, polyuria with glucosuria and
albuminuria, died of renal insufficiency. On autopsy, the kidneys were reduced in size, dense and had
a fine-grained surface. The pancreas was reduced and partially substituted for a fatty tissue.
Microscopically, islets of Langerhans were fine, in some places they were substituted for a
connective tissue, solitary ones were hypertrophic. The kidneys reveal intracapillary
glomerulosclerosis. Which of the diagnoses was the most probable?
A. Subacute glomerulonephritis
B. Chronic indurative pancreatitis
C. Diabetes insipidus
D. Chronic glomerulonephritis
E. * Diabetes mellitus
107. A 36-year-old female patient underwent resection of the both lobes of her thyroid gland; each of them
was 5 x 6 cm in size, pink-yellow, moderately dense and had a tuberous surface. A microscopic
examination revealed follicles of various size, some of them were dilated like cysts and filled with
some colloid; the follicular walls were lined with the smoothed cuboidal epithelium; the stroma of the
gland was redundantly developed owing to the connective tissue, there were foci of calcinosis. Which
of the diseases listed below corresponded most to the changes found?
A. Parenchymatous goiter
B. Toxic goiter
C. Hashimoto's disease
D. Ligneous thyroiditis
E. * Colloid goiter
108. An autopsy of a 24-year-old female (from her case history it is known that a year before the woman
had given birth to a child) revealed a sharp decrease of the body weight down to 38 kg, the skin was
dry and thin, the weight of the internal organs was loWER ed. Also, there was a sharp decrease in the
weight of the adenohypophysis, the latter had cicatrices; there were foci of dystrophy, necrobiosis
and hyalinosis in the diencephalon. The ovaries, thyroid and adrenal glands had phenomena of
hypotrophy, the mucous membrane of the intestines was atrophied. Which of the diagnoses was the
most probable?
A. Nutritional dystrophy
B. Suprarenal cachexia
C. Cachexia associated with chronic amoebiasis
D. Cachexia associated with pellagra
E. * Cerebrohypophysial cachexia
109. An autopsy of a 45-year-old female patient, who suffered from obesity by the upper type, steroid
diabetes mellitus, arterial hypertension and secondary ovarian dysfunction, revealed hypertrichosis,
hirsutism, striae on the skin of the thighs and abdomen. The anterior lobe of the hypophysis
contained a white-pink encapsulated tumour, 2.5 cm in diameter (microscopically, it was a pituitary
basophil adenoma); the adrenal glands were characterized by bilateral hyperplasia of the fascicular
layer. Which of the diagnoses was the most probable?
A. Cushing's syndrome
B. Adiposogenital dystrophy
C. Simmonds disease
D. Pituitary dwarfism
E. * Cushing's disease

110. A 52-year-old male died from renal insufficiency. On microscopic examination of his organs,
the pancreas revealed lipomatosis and sclerosis with an atrophy of islets of Langerhans, the
kidneys had hyalinosis of the mesangium and glomeruli (Kimmelstiel-Wilson syndrome) and
a glycogenic infiltration of the epithelium of the tubules, the liver was characterized by fatty
degeneration. Which of the diagnoses listed below was the most probable?
A. Arterial nephrosclerosis
B. Amyloid wrinkled kidneys
C. Chronic glomerulonephritis
D. Goodpasture's syndrome
E. * Diabetic glomerulosclerosis
Назва наукового напрямку (модуля): Семестр: 6
2019 СРС англ
Опис:
2019 СРС англ
Перелік питань:
1. An experimental animal received a subcutaneous dose of an antigen preceded by sensitization. At the
place of the injection, some fibrinous inflammation developed with an alteration of the vascular
walls, the main substance and fibrous structures of the connective tissue in the form of a mucoid and
fibrinoid swelling, a fibrinoid necrosis. Which of the diagnoses listed below was the most probable?
A. Delayed hypersensitivity
B. Transplantation immunoreaction
C. Normergy
D. Granulomatosis
E. * Immediate hypersensitivity
2. A histological examination of the lungs if a male, who suffered for many years from atopic bronchial
asthma and died of asphyxia, revealed much mucus with an admixture of eosinophils in the lumens of
the bronchioles and small bronchi, sclerosis of interalveolar septa, dilation of alveolar lumens. Which
of the mechanisms in the development of a hypersensitivity reaction took place when a fit of
asphyxia developed?
A. Cytotoxic reaction
B. Immunocomplex reaction
C. Cytolysis owing to lymphocytes
D. Granulomatosis
E. * Reaginic reaction
3. A study of the thymus of a 5-year-old child, who died from acute destructive staphylococcal
pneumonia, revealed a decrease in the weight of the gland down to 3.0 g. On histological
examination, a smaller size of the lobules of the gland with a collapse of the stroma, an inversion of
the layers, and cyst-like Hassal’s bodies were found out. Which of the diagnoses listed below was the
most probable?
A. Thymomegaly
B. Hypoplasia of the thymus
C. Dysplasia of the thymus
D. Agenesia of the thymus
E. * Accidental reaction
4. An examination of a pregnant woman with a rhesus-negative group of blood revealed a high level of
antierythrocyte antibodies; in order to decrease it, a skin flap of her rhesus-positive husband was
grafted to her. Two weeks later the flap was rejected; its microscopic examination revealed
disturbances of circulation, an oedema, a cellular infiltration mostly by lymphocytes, neutrophils and
macrophages. Which of the pathological processes listed below was the most probable?
A. Immediate hypersensitivity
B. Delayed hypersensitivity
C. Granulomatous inflammation
D. Interstitial inflammation
E. * Transplantation immunity
5. An experimental animal received a subcutaneous dose of an antigen preceded by sensitization. At the
place of the injection, some fibrinous inflammation developed with an alteration of the vascular
walls, the main substance and fibrous structures of the connective tissue in the form of a mucoid and
fibrinoid swelling, a fibrinoid necrosis. Which of the diagnoses listed below was the most probable?
A. Delayed hypersensitivity
B. Transplantation immunoreaction

C. Normergy
D. Granulomatosis
E. * Immediate hypersensitivity
6. A histological examination of the lungs if a male, who suffered for many years from atopic bronchial
asthma and died of asphyxia, revealed much mucus with an admixture of eosinophils in the lumens of
the bronchioles and small bronchi, sclerosis of interalveolar septa, dilation of alveolar lumens. Which
of the mechanisms in the development of a hypersensitivity reaction took place when a fit of
asphyxia developed?
A. Cytotoxic reaction
B. Immunocomplex reaction
C. Cytolysis owing to lymphocytes
D. Granulomatosis
E. * Reaginic reaction
7. A study of the thymus of a 5-year-old child, who died from acute destructive staphylococcal
pneumonia, revealed a decrease in the weight of the gland down to 3.0 g. On histological
examination, a smaller size of the lobules of the gland with a collapse of the stroma, an inversion of
the layers, and cyst-like Hassal’s bodies were found out. Which of the diagnoses listed below was the
most probable?
A. Thymomegaly
B. Hypoplasia of the thymus
C. Dysplasia of the thymus
D. Agenesia of the thymus
E. * Accidental reaction
8. An examination of a pregnant woman with a rhesus-negative group of blood revealed a high level of
antierythrocyte antibodies; in order to decrease it, a skin flap of her rhesus-positive husband was
grafted to her. Two weeks later the flap was rejected; its microscopic examination revealed
disturbances of circulation, an oedema, a cellular infiltration mostly by lymphocytes, neutrophils and
macrophages. Which of the pathological processes listed below was the most probable?
A. Immediate hypersensitivity
B. Delayed hypersensitivity
C. Granulomatous inflammation
D. Interstitial inflammation
E. * Transplantation immunity
9. On supersonic examination of a 48-year-old male patient, a hepatic neoplasm was diagnosed and a
puncture biopsy was made. Microscopically, the tumour consisted of atypical hepatocytes which
formed trabeculae, acini or tubules. The tumour stroma was poor and had thin-walled blood vessels.
Which of the kinds of tumours listed below was the most probable?
A. Hepatocellular adenoma
B. Metastasis of adenocarcinoma
C. Cholangiocellular carcinoma
D. Solid carcinoma
E. * Hepatocellular carcinoma
10. An encapsulated tumour, 2 cm in diameter, surgically removed from an amputation stump of a lower
extremity, microscopically consists of spindle cells of the monomorphous kind with rod-shaped
nuclei which form "fence-like" structures together with fibres. Which of the tumours listed below is
the most probable?
A. Neurofibroma
B. Malignant neurilemmoma
C. Soft fibroma

D. Fibrosarcoma
E. * Benign neurilemmoma
11. A microscopic examination of a biopsy from a deformed mucous membrane of a lobar bronchus of a
45-year-old male, who smoked for many years, revealed a carcinoma consisting of atypical epithelial
cells with hyperchromatic nuclei and numerous pathological mitoses. The growth of the tumour did
not spread to the basal membrane of the epithelium. Name the histological form of carcinoma.
A. Squamous cell carcinoma
B. Adenocarcinoma
C. Solid carcinoma
D. Small-cell carcinoma
E. * Carcinoma in situ
12. A 45-year-old male underwent surgical removal of a tumour, 4 x 3 cm in size, from the lateral
ventricle of his brain; the tumour surface had small papillae, and it was connected with a vascular
plexus. Microscopically, the tumour consisted of villus-like vegetations covered with epithelial cells
of the cubical and columnar shape and the monomorphous kind. Which of the tumours listed below
was the most probable?
A. Ependymoma
B. Ependymoblastoma
C. Choriocarcinoma
D. Glioblastoma
E. * Choriopapilloma
13. An encapsulated tumour, 2 cm in diameter, surgically removed from an amputation stump of a lower
extremity, microscopically consists of spindle cells of the monomorphous kind with rod-shaped
nuclei which form "fence-like" structures together with fibres. Which of the tumours listed below is
the most probable?
A. Neurofibroma
B. Malignant neurilemmoma
C. Soft fibroma
D. Fibrosarcoma
E. * Benign neurilemmoma
14. A microscopic examination of a biopsy from a deformed mucous membrane of a lobar bronchus of a
45-year-old male, who smoked for many years, revealed a carcinoma consisting of atypical epithelial
cells with hyperchromatic nuclei and numerous pathological mitoses. The growth of the tumour did
not spread to the basal membrane of the epithelium. Name the histological form of carcinoma.
A. Squamous cell carcinoma
B. Adenocarcinoma
C. Solid carcinoma
D. Small-cell carcinoma
E. * Carcinoma in situ
15. A microscopic examination of a gastrobiopsy from a tumour of the pyloroduodenal portion of the
stomach revealed layers of atypical epithelial cells with a large number of mitoses; the tumour
architectonics is characterized by prevalence of the parenchyma over the stroma. Which of the
histological forms of carcinoma listed below was the most probable?
A. Adenocarcinoma
B. Solid carcinoma
C. Mucinous carcinoma
D. Small-cell carcinoma
E. * Medullary carcinoma

16. On supersonic examination of a 48-year-old male patient, a hepatic neoplasm was diagnosed and a
puncture biopsy was made. Microscopically, the tumour consisted of atypical hepatocytes which
formed trabeculae, acini or tubules. The tumour stroma was poor and had thin-walled blood vessels.
Which of the kinds of tumours listed below was the most probable?
A. Hepatocellular adenoma
B. Metastasis of adenocarcinoma
C. Cholangiocellular carcinoma
D. Solid carcinoma
E. * Hepatocellular carcinoma
17. A tumour was found in the locus of a pathological fracture of a rib in a male patient. The case history
contained information about persistent proteinuria with presence of abnormal proteins of
Bence-Jones type, as well as presence of osteolytic foci in the bones of the spine, skull and pelvis.
Histologically, the tumour cells were represented by plasmablasts and plasmacytes. What is your
diagnosis?
A. Primary macroglobulinaemia
B. Heavy-chain disease
C. Osteosarcoma
D. Fibrosarcoma
E. * Multiple myeloma
18. An autopsy of a male, who died from chronic renal insufficiency, revealed numerous nodes with soft
elastic consistency in the ribs, bones of the vault of the skull and the breastbone. The osseous
substance was decalcified according to the nodes. The kidneys were enlarged, light grey, dense, their
section had some greasy lustre. What is your diagnosis?
A. Primary amyloid nephropathy
B. Parathyroid osteodystrophy
C. Osteoma
D. Osteosarcoma
E. * Multiple myeloma
19. Some tumour, which was mobile and clearly delimited from the surrounding tissues, was revealed in
the skin. On section, the tumour tissue was white and fibrous. Microscopically, the tumour consisted
of chaotically interlaced collagenous fibres and a small number of connective tissue cells. Name the
tumour.
A. Soft fibroma
B. Histiocytoma
C. Dermatofibroma
D. Desmoid
E. * Hard fibroma
20. An enlarged dense tuberous prostate has been sent for a histological examination. On section, there
were tumour nodes, 1-2 cm in diameter, surrounded by connective-tissue layers. Microscopically,
against a background of fibrosis there were glandular complexes with atypical epithelial cells,
hyperchromatic nuclei and pathological mitoses. Which of the tumours listed below was the most
probable?
A. Solid carcinoma
B. Adenoma
C. Fibroma
D. Fibrosarcoma
E. * Adenocarcinoma

21. A thick node without any clear borders, about 10 cm in diameter, is contoured on the outer surface of
a thigh. Microscopically, the tumour consists of immature fibroblast-like cells with pathological
mitoses and collagenous fibres. The tumour cells grow among the muscular fibres. Indicate the
diagnosis which was the most probable one of those listed below.
A. Malignant histiocytoma
B. Hard fibroma
C. Soft fibroma
D. Dermatofibroma
E. * Fibrosarcoma
22. A newborn baby has some red-blue flattened tumor, 5 x 4 x 0.3 cm in size, in a capsule on the skin of
its face. Microscopically, the tumour consists of large thin-walled vascular cavities which have an
endothelial lining and are filled with blood. Name the tumour.
A. Venous haemangioma
B. Capillary haemangioma
C. Hemangiopericytoma
D. Lymphangioma
E. * Cavernous haemangioma
23. A histological examination of a thyroid gland revealed small cysts, which were lined with atypical
epithelium and filled with papillae, the latter originating from the walls of the cysts and growing into
their capsules. Name the tumour.
A. Papillary adenoma
B. Follicular carcinoma
C. Solid carcinoma
D. Carcinoma simplex
E. * Papillary carcinoma
24. A histological examination of some spherical neoplasm located under the surface of the skin,
revealed papilliform vegetations of the epithelium with phenomena of acanthosis and
hyperkeratinization. The tumour stroma consisted of a large amount of the connective tissue and
vessels. What tumour took place?
A. Keratoacanthoma
B. Carcinoma in situ
C. Keratinizing squamous cell carcinoma
D. Nonkeratinizing squamous cell carcinoma
E. * Papilloma
25. A bronchoscopy of the mucous membrane of the main bronchus revealed some tumour. A
microscopic examination of the tumour biopsy showed that it consisted of lymphocyte-like cells with
hyperchromatic nuclei growing in the form of layers or bands and involving the submucous layer.
The tumour had many pathological mitoses. Which of the histological forms of carcinoma listed
below was the most probable?
A. Squamous cell carcinoma
B. Adenocarcinoma
C. Adenoacanthoma
D. Scirrhous carcinoma
E. * Small-cell carcinoma
26. Examination of a patient revealed a dense, movable skin tumour that is standing out distinctly from
the surrounding tissues. Its section is found to be white and composed of fibrous tissue. Microscopic
examination revealed interlacing collagen fibers and few cells. What tumor is it?
A. Myoma
B. Histiocytoma

C. Dermatofibroma
D. Desmoid
E. * Fibroma
27. A 50-year-old man has felt vague abdominal discomfort within past 4 months. Physical examination
revealed no lymphadenopathy, and no abdominal masses or organomegaly at palpation. Bowel
sounds are heard. An abdominal CT scan shows a 20 cm retroperitoneal soft tissue mass obscuring
the left psoas muscle. A stool specimen tested for occult blood is negative. Which of the following
neoplasms is this man most likely to have?
A. Myoma
B. Histiocytoma
C. Dermatofibroma
D. Desmoid
E. * Fibroma
28. A 40-year-old woman has had a feeling of abdominal discomfort for the past 8 months. On pelvic
examination, there is the right adnexal mass. Abdominal CT scan demonstrates a 7 cm cystic mass
involving the right ovary with small areas of calcification. The uterus is normal in size. The right
fallopian tube and ovary have been removed surgically. Grossly, the mass on sectioning is filled with
abundant hair and sebum. Microscopically, the mass has glandular spaces lined by columnar
epithelium, squamous epithelium with hair follicles, cartilage, and dense connective tissue. What
type of tumour is it?
A. Squamous cell carcinoma of ovary
B. Melanoma
C. Sarcoma of ovary
D. Metastase of cervical carcinoma
E. * Teratoma
29. An autopsy of a 76-year-old male, who smoked for a long period of time, lived sedentary life and had
redundant weight, revealed in the intima of the aorta some grey-yellow spots and stripes, fibrous
plaques, calcified areas with haemorrhages and calcinosis. What disease do these changes indicate?
A. Nonspecific aortoarteritis
B. Hypertensive disease
C. Systemic lupus erythematosus
D. Visceral syphilis
E. * Atherosclerosis
30. An autopsy of a 27-year-old male, who died suddenly, revealed in the intima of the abdominal aorta
some yellow foci in the form of spots and stripes, which did not rise above the surface of the intima
but after staining with sudan III became orange. What stage in the morphogenesis of atherosclerosis
was revealed?
A. Atherocalcinosis
B. Liposclerosis
C. Atheroma
D. Prelipid
E. * Lipoidosis
31. A microscopic examination of the wall of an aorta revealed a focal infiltration of the intima by lipids
and proteins. The lipids impregnated the intima and accumulated in the muscle cells and
macrophages. Determine the stage of atherosclerosis.
A. Prelipid
B. Liposclerosis
C. Atheromatosis
D. Atherocalcinosis

E. * Lipoidosis
32. A 75-year-old male was hospitalized complaining of a sharp pain in the abdominal cavity, weakness,
filiform pulse. During an operation it was found that the paraaortic fat was imbibed with blood. The
abdominal aorta had a sac-like protrusion, its wall was thinned and had an area of rupture. What
disease caused the complication?
A. Coronary disease
B. Hypertensive disease
C. Cardiomyopathy
D. Systemic vasculitis
E. * Atherosclerosis
33. A 59-year-old patient receiving chemotherapy with the anthracycline Adriamycin develops severe
heart failure. Sections from an endocardial biopsy specimen reveal vacuolization of the endoplasmic
reticulum of the myocytes. Adriamycin therapy most frequently causes what type of
cardiomyopathy?
A. Restrictive cardiomyopathy
B. Hyperplastic cardiomyopathy
C. Hypertrophic cardiomyopathy
D. Obliterative cardiomyopathy
E. * Dilated cardiomyopathy
34. A 3-month-old girl is being evaluated for feeding difficulty and failure to thrive. Physical
examination finds pallor, peripheral cyanosis, tachypnea, and fine expiratory wheezing. Chest x-ray
shows cardiac enlargement. She is admitted to the hospital, quickly develops severe cardiac failure,
and dies 3 days after admission. At the time of autopsy the endocardium is found to have a “cream
cheese” gross appearance. Histologic sections from this area reveal thickening of the endocardium
due to a proliferation of fibrous and elastic tissue. Which of the following is the most likely
diagnosis?
A. Dilated cardiomyopathy
B. Hypertrophic cardiomyopathy
C. Infective endocarditis
D. Libman-Sachs endocarditis
E. * Restrictive cardiomyopathy
35. Shortly before death a patient got an electrocardiographically based diagnosis of acute myocardial
infarction. Autopsy revealed that the myocardial cavity contained 200 ml of liquid blood and 400 g of
clots; the posterior wall of the left ventricle had a perforation up to 2 cm long. What complication of
myocardial infraction is it?
A. Haemorrhagic pericarditis
B. Exudative pericarditis
C. Idiopathic myocarditis
D. Stone heart
E. * Myocardial rupture with cardiac tamponade
36. Dystrophic changes in the heart muscle are accompanied by an enlargement of the heart cavity, a
decrease in the strength of the heart, an increase in the volume of blood that remains during systole in
the cavity of the heart, overflow of the veins. For what heart condition is this characteristic?
A. Tamponade of heart
B. Tonic dilatation
C. Emergency stage of hyperfunction and hypertrophy
D. cardiosclerosis
E. * Myogenic dilatation

37. An autopsy of a male patient, who died from heart failure, revealed an enlarged heart weighing 550 g,
fibrinous pericarditis, as well as contracted dense kidneys weighing 50 g each and having a
fine-grained surface. Microscopically, the kidneys were characterized by an expressed hyalinosis of
arteioles and glomeruli. Name the basic disease.
A. Atherosclerosis
B. Rheumatism
C. Pericarditis
D. Cardiomyopathy
E. * Hypertensive disease
38. A 67-year-old male patient was suffering from hypertensive disease during 20 years. He died from
chronic renal insufficiency. What appearance did his kidneys have on autopsy?
A. Large, motley
B. Large, red
C. Large, white, dense
D. Small, dense, macrotuberous
E. * Small, dense, with a fine-grained surface
39. Against a background of hypertensive crisis, a male patient with hypertensive disease developed
acute renal insufficiency which caused his death. What morphological changes in the renal arteioles
were the most probable?
A. Stenosing atherosclerosis
B. B. Hyperelastosis
C. Hyalinosis
D. Sclerosis
E. * Fibrinoid necrosis
40. A 56-year-old male patient with elevated blood pressure (250/120 mm Hg) died from an impairment
of his cerebral circulation. An autopsy of the brain revealed a red focus in the thalamus, 2.5 cm in
diameter, which sank on section. Microscopically, there was fibrinoid necrosis of the vascular walls
and impregnation of the necrotized brain tissue with blood. Which of the diagnoses listed below was
the most probable?
A. Cerebral haematoma
B. Anaemic infarct of brain
C. Mixed infarct
D. Atheromatosis
E. * Haemorrhagic infarct of brain
41. A 2-month-old girl is being examined for a routine checkup. She was born at term, and there were no
problems or complications during the pregnancy. The baby appeared normal at birth and has been
asymptomatic. Physical examination at this time finds a soft systolic murmur with a systolic thrill.
No cyanosis is present, and her peripheral pulses are thought to be within normal limits. An ECG
reveals slight left ventricular hypertrophy. Which of the following is the most likely diagnosis?
A. Coarctation of the aorta
B. Patent ductus arteriosus
C. Persistent truncus arteriosus
D. Tetralogy of Fallot
E. * Ventricular septal defect
42. In a clinical study of tetralogy of Fallot, patients are examined before surgery to determine predictors
observed on echocardiography that correlate with the severity of the disease and the need for more
careful monitoring. A subset of patient is found to have more severe congestive heart failure, poor
exercise tolerance, and decreased arterial oxygen saturation levels. Which of the following is most
likely to predict a worse clinical presentation for these patients?

A. Size of the left ventricle


B. Size of the ventricular septal defect
C. Diameter of the tricuspid valve
D. Presence of an atrial septal defect
E. * Degree of pulmonary stenosis
43. A 7-year-old child has thick, painless nodules 1-2 mm in size on the extensor surfaces of the elbow
and knee joints. In the biopsy of the nodules - large foci of fibrinoid necrosis of the connective tissue
with lymphocytes and macrophages on the periphery. Specify the disease in which such nodules are
observed?
A. Systemic lupus erythematosus
B. Rheumatoid arthritis
C. Scleroderma
D. Periarteritis nodosa
E. * Rheumatism
44. In a patient who died of heart failure, during a pathological examination, it was discovered: the mitral
valve leaflets are deformed, thickened, joined at the edges; in the connective tissue of the
myocardium there are diffusely located nodules, which consist of areas of fibrinoid necrosis, around
which are clusters of macrophages resembling giant multinucleated cells. The foci are surrounded by
lymphocytes and single plasma cells. Which of the following granulomas takes place in this patient?
A. Leprous
B. Tuberculous
C. Actinomycotic
D. Syphilitic
E. * Rheumatic
45. The disease that is characterized by the systolic murmur without signs of mitral valve failure is called
as:
A. Rheumatic heart disease
B. Congenitally bicuspid aortic valve
C. Mitral annular calcification
D. Degenerative calcific aortic valve stenosis
E. * Mitral valve prolapse
46. Etiologically and pathogenetically, rheumatic fever and rheumatic heart disease are characterized by
all of the following, except:
A. Initial attack of illness some weeks after streptococcal infection
B. Elevated serum titers of antibodies to streptolysin and hyaluronidase
C. Sterile tissue lesions not resulting from direct bacterial invasion
D. Recurrent acute illness following the streptococcal infection
E. * Decreased serum protein levels
47. Heart lesions in acute rheumatic fever are called as which of the following:
A. Tuberculoma
B. Mitral stenosis
C. Foreign bodies
D. Gumma
E. * Aschoff bodies
48. The Anitschkow cells are characterized by all of the following, except:
A. Abundant amorphophilic cytoplasm
B. Central round-to-ovoid nucleus
C. Central disposed chromatin (caterpillar cells)

D. Monocytic cytogenesis
E. * B-lymphocytic cytogenesis
49. Child 3 month old, which has been not vaccinated, has died because of suppurative peritonitis. The
perforative ulcer of small intestine has been found in autopsy. The mesenteric lymph nodule have
been dense, their cut surface had caseous mass. Diagnosis of tuberculosis has been determined. Call
the rout of spreading of tuberculosis.
A. Child has primary tuberculosis
B. The primary tuberculose complex is found in autopsy
C. Tuberculosis is transmitted with food
D. The exudative and necrotic reactions are predominate
E. * Disease appears due to transplacental passway
50. During the histological examination of the biopsy the pathologist has found out granulomas
within the livers. They contain mainly T-lymphocytes and epithelioid cells, and solitary giant
Langhan’s cells. In the center of granulomas there was a small area of caseous necrosis. What
pathologic process is characterized by those changes?
A. Coagulative necrosis.
B. Liquefactive necrosis.
C. Neoplasm.
D. Exudative inflammation.
E. * Proliferative inflammation.
51. Autopsy of man aged 60 demonstrated numerous whitish millet-grain-sized nodes in the
lungs and liver. Microscopy demonstrated granulomas with foci of necrosis in the center and
epithelioid, lymphoid, plasmatic cells on the periphery as well as macrophages and
numerous Pirogov-Langhans cells in the infiltrations. Which of the listed granulomas is most
A. probable?
Phagocytic
B. Epithelioidcellular
C. Macrophage
D. Foreign-body granuloma
E. * Giant-cell
52. During the microscopic examination of bioptic fragment of the skin the granulomas
containing epithelioid cells, surrounded with T-lymphocytes were found out. Between the
epithelioid cells the solitary giant polynuclear Langhan’s cells located. There were areas of
caseous necrosis in the center of some granulomas. Blood vessels were absent. What disease do such
A. changes characterize?
Syphilis
B. Leprosy
C. Rhinoscleroma
D. Hodgkin’s disease
E. * Tuberculosis
53. The disease in a male hunter began with an elevation of his body temperature up to 37-38(C,
increased reflex excitability, a disturbance of sleep and hydrophobia. Later these signs were
accompanied by spasms of the muscles of the larynx and pharynx, as well as those of respiration. The
patient’s death was caused by arrest of respiration. On autopsy, an oedema and plethora of the brain,
as well as small haemorrhages in the region of the myelencephalon were found. On histological
examination of the brain, its stem part, walls of the 3rd ventricle and hippocampus revealed necrosis
of the nerve cells which were surrounded (as well as small vessels) by nodules consisting of clusters
of microglial and lymphoid cells. The cytoplasm of the nerve cells of the hippocampus contained
some rounded eosinophilic inclusions (Babes-Negri bodies). What disease is characterized by the
picture described?
A. Typhoid fever
B. Epidemic typhus

C. Poliomyelitis
D. Tick-borne encephalitis
E. * Rabies
54. The disease in a 67-year-old woman acutely began with an expressed oedema and tenderness of the
skin and soft tissues of the neck. A phlegmon of neck and mediastinitis were diagnosed. The patient
died under the increasing phenomena of intoxication. On autopsy, the left tonsil was slightly enlarged
and dense; on section, it was yellowish-greenish and had a lot of small cavities which imparted a
honeycomb structure to it. The soft tissues of the neck and the fat of the anterior mediastinum had
signs of purulent melting. Microscopically, the tissue had a lot of small abscesses, their centres
having intensive basophilic formations, which consisted of short rod-like elements connected with
their one end to the common centre. What is your diagnosis?
A. Giardiasis
B. Leishmaniasis
C. Brucellosis
D. Amoebiasis
E. * Actinomycosis
55. A male patient, who came from the Central Asia, had persistent diarrhoeae, a loss of body weight and
signs of intoxication against whose background he died. An autopsy revealed numerous hepatic
abscesses, the caecum was characterized by dingy green areas of necrosis of its mucous membrane,
these areas slightly rose above its surface and penetrated into the muscular layer. The ulcers resulting
from the necrosis were characterized by undermined edges which hung over their bottom. The
inflammatory reaction in the intestinal wall was poorly expressed. What was the most probable
disease in that case?
A. Salmonellosis
B. Cholera
C. Typhoid fever
D. Bacterial dysentery
E. * Amoebiasis
56. An autopsy of a 45-year-old male, who had had a fever with signs of intoxication during his life-time,
revealed an enlarged dense spleen (500 g); on section, its pulp had numerous grey-white and
white-yellow miliary necroses of follicles, and there were infarct-like foci of necrosis under the
capsule. A histological examination revealed hyperplasia of the follicles with breakdown of
leukocytes and accumulation of neutrophils, and numerous thrombi in the vessels. Which of the
diagnoses listed below was the most probable?
A. Plague
B. Haematogenous general miliary tuberculosis
C. Tularaemia
D. Typhoid fever
E. * Relapsing fever
57. The examination of the child with measles showed the non-clear border edematous fluctuated areas
of red-black color in the soft tissues of the cheeks and perineum. What complication did develop in
the child?
A. Dry gangrene
B. Gas gangrene
C. Bedsore
D. Trophic ulcer
E. * Wet gangrene (noma)

58. A 8 year-old child was ill acutely with clinical signs of vomiting, headache and severe intoxication.
After two days of the disease he has died. In autopsy the pathologist has found out: meninges
thickened, yellowish color on basal surface, edema and hyperemia. Meningococcus was detected
from liquor fluid. Diagnose this disease.
A. Scarlet fever
B. Pertussis
C. Diphtheria
D. Measles
E. * Meningococcal infection
59. A 5 year-old girl has died because of asphyxia owing to true croup. In the autopsy it was established;
mucosa of larynx, trachea and bronchi dwarfed, edematous, dull, coated by grayish fibrinous plaques,
which were easily removed. Described morphological changes are characteristic for…:
A. Flu
B. Measles
C. Pertussis
D. Scarlet fever
E. * Diphtheria
60. A 6 year-old child, was ill acutely with signs of intoxication. In 2 day the patient has died. In autopsy
the pathologist has found out: meninges of brain with edema, hyperemia, yellow-grey exudate.
Tissue of brain was edematous. Microscopic investigation: there were neutrophils, hyperemia,
hemorrhages and edema in meninges. Described changes are most typical for:
A. Flu
B. Pertussis
C. Diphtheria
D. Measles
E. * Meningococcal meningitis
61. Patient has suffered from cholera. Clinical dates are dehydratating, cyanosis and convulsions. In the
result of massive infusion therapy the exicosis has been diminished, but anuria has been remained.
Patient has dead because of uremia. What morphological features in kidney have been found out?
A. Choleric typhoid is developed
B. Development of uremia is connected with acute glomerulonephritis
C. Fibrinous colitis is found in autopsy
D. Exicosis is due to action of virus exotoxin
E. * Necrotic nephrosis with cortical necrosis takes place in the kidneys
62. An autopsy of a 45-year-old male, who suffered from numerous pathological fractures during his
lifetime, revealed changes in his long tubular bones: the bones of the thigh and shin were bent, in
some places they resembled spirals, their surface was tuberous, a section revealed an obliterated
medullary channel and a change in the compact structure of the cortical layer by the spongy type.
Microscopically, there was a mosaic type of the bone structures: against a background of a disordered
thin-fibrous or lamellar structure of the bone fragments there were numerous cavities of sinusal
resorption combined with signs of new formation of the osseous tissue. The arteries, which supplied
the bone tissue, were dilated and convoluted. Name a diagnosis.
A. Osteopetrosis
B. Parathyroid osteodystrophy
C. Fibrous dysplasia
D. Chronic osteomyelitis
E. * Deforming osteodystrophy

63. A 20-year-old girl developed complaints about an expressed fatiguability of her ocular, masticatory,
speech and deglutitive groups of muscles, when the normal contraction of the muscles after great
activity absolutely discontinued, but after some rest the functioning of the muscles was restored
again. Some time later the pathological process involved the muscles of the extremities and
intercostal ones. An inadequate ventilation of the lungs resulted in development of the secondary
lobular pneumonia which caused the patient’s death. An autopsy revealed an atrophy of the striated
muscles, their dystrophy with focal clusters of the lymphocytes in the interstice. An enlarged thymus
was characterized by follicular hyperplasia. What was the most probable diagnosis?
A. Pseudohypertrophic muscular dystrophy
B. Amyotrophic lateral sclerosis
C. Werdnig-Hoffman spinal amyotrophy
D. Erb’s muscular dystrophy
E. * Myasthenia
64. An autopsy of a male, who died from uraemia, revealed deformity of the spinal column with a sharp
limitation of mobility. The articular cartilages of small joints of the spinal column were destroyed,
there were some expressed signs of a prolonged chronic inflammation in the articular tissues, the
cavities of the joints were filled with the connective tissue, but in some places with the osseous one
together with formation of ankyloses. The aorta, heart and lungs revealed a chronic inflammation and
focal sclerosis. The kidneys were characterized by amyloidosis. What diagnosis was the most
probable in this case?
A. Rheumatoid arthritis
B. Paget’s disease (deforming osteosis)
C. Parathyroid osteodystrophy
D. Osteopetrosis (marble bone disease)
E. * Bekhterev’s disease
65. On autopsy of a male, who died from uraemia, it was found that the pancreas was reduced in size, his
contracted kidneys had a fine-grained surface, the liver was enlarged, yellow and flaccid.
Microscopically, the pancreatic tissue revealed an atrophy of the parenchyma, including islets of
Langerhans, the atrophied parenchyma was substituted for hyperplastic connective and fatty tissues.
The kidneys were characterized by sclerosis and hyalinosis of the glomeruli, as well as by a
glycogenic infiltration of the tubules; there was a fatty degeneration in the liver and a fibrinous
inflammation in the mucous coats of the trachea, bronchi and stomach. What disease did the died
person suffer from?
A. Chronic glomerulonephritis
B. Hypertensive disease
C. Chronic indurative pancreatitis
D. Steatosis
E. * Diabetes mellitus
66. An autopsy of a male, who died from chronic renal insufficiency, revealed atherosclerosis of the
aorta and large arteries, small and dense kidneys with a fine-grained surface, an enlarged
yellow-brown and flaccid liver, the pancreas was reduced in size. Microscopically, there was
atherocalcinosis of the aorta and arteries, an atrophy of the parenchyma, sclerosis and lipomatosis of
the pancreas; the kidneys were characterized by hyalinosis of the mesangium and glomeruli, a
glycogenic infiltration of the epithelium of the tubules, with large-drop adiposis in the hepatocytes.
What pathological process took place in the kidneys?
A. Arterial nephrosclerosis
B. Chronic pancreatitis
C. Chronic glomerulonephritis
D. Steatosis
E. * Diabetic nephrosclerosis

67. A 52-year-old male died from renal insufficiency. On microscopic examination of his organs, the
pancreas revealed lipomatosis and sclerosis with an atrophy of islets of Langerhans, the kidneys had
hyalinosis of the mesangium and glomeruli (Kimmelstiel-Wilson syndrome) and a glycogenic
infiltration of the epithelium of the tubules, the liver was characterized by fatty degeneration. Which
of the diagnoses listed below was the most probable?
A. Arterial nephrosclerosis
B. Amyloid shrunk kidneys
C. Chronic glomerulonephritis
D. Goodpasture’s syndrome
E. * Diabetic glomerulosclerosis
68. A 53-year-old male patient, who suffered from peptic ulcer of the stomach for more than 25 years,
was admitted to a surgical department with complaints about frequent vomiting after taking food,
progressing loss of weight, severe thirst. At the hospital, the signs of oliguria and later anuria
developed. The patient died. An autopsy revealed a cicatricial stenosis of the pylorus and a sharp
enlargement of the stomach which practically reached the pelvic region. Which of the complication
of peptic ulcer listed below caused the patient’s death?
A. Peritonitis
B. Penetration of ulcer
C. Erosive haemorrhage
D. Malignancy
E. * Chlorhydropenic uraemia
69. A 47-year-old woman underwent radical mastectomy for a neoplasm. A histological examination of
the mammary gland revealed an eczematous lesion of the nipple and areola, a cancerous lesion of the
ducts of the gland and presence of large light cells in the epidermis of the nipple and areola. Make a
diagnosis.
A. Intralobular carcinoma in situ
B. Acneiform carcinoma
C. Papillary carcinoma
D. Fibrous carcinoma
E. * Paget's disease
70. A histological express examination of a tumour node of a mammary gland revealed some
encapsulated formation with proliferation of alveoli and intralobular ducts; the interstitial connective
tissue grew either around or inside the ducts. Which of the tumours took place?
A. Foliaceous tumour
B. Noninfiltrating intralobular carcinoma
C. Infiltrating intralobular carcinoma
D. Paget's disease
E. * Fibroadenoma
71. During an operation on a woman, her cyst-like changed ovary was removed; it was a thin-walled
cavity filled with some yellowish transparent fluid and having a smooth inner surface. Histologically,
the cavity wall was lined with the cubical epithelium. Name the kind of the tumour.
A. Mucinous cystadenoma
B. Serous cystadenocarcinoma
C. Pseudomucinous cystocarcinoma
D. Granulosa cell tumour
E. * Serous cystadenoma

72. A histological examination of a biopsy from a uterine cervix revealed that its tissue was covered with
a wide layer of the stratified squamous epithelium having foci of proliferation of atypical cells with
pathological mitoses, but the basal membrane of the epithelium was not affected. What is your
diagnosis?
A. Nonkeratinizing squamous cell carcinoma
B. Keratinizing squamous cell carcinoma
C. Leukoplakia
D. Epithelial dysplasia
E. * Carcinoma in situ
73. An autopsy of a male, who died from chronic renal insufficiency, revealed atherosclerosis of the
aorta and large arteries, small and dense kidneys with a finegrained surface, an enlarged
yellow-brown and flaccid liver, the pancreas was reduced in size. Microscopically, there was
atherocalcinosis of the aorta and arteries, an atrophy of the parenchyma, sclerosis and lipomatosis of
the pancreas; the kidneys were characterized by hyalinosis of the mesangium and glomeruli, a
glycogenic infiltration of the epithelium of the tubules, with large-drop adiposis in the hepatocytes.
What pathological process took place in the kidneys?
A. Arterial nephrosclerosis
B. Chronic pancreatitis
C. Chronic glomerulonephritis
D. Steatosis
E. * Diabetic nephrosclerosis
74. On autopsy of a male, who died from uraemia, it was found that the pancreas was reduced in size, his
contracted kidneys had a fine-grained surface, the liver was enlarged, yellow and flaccid.
Microscopically, the pancreatic tissue revealed an atrophy of the parenchyma, including islets of
Langerhans, the atrophied parenchyma was substituted for hyperplastic connective and fatty tissues.
The kidneys were characterized by sclerosis and hyalinosis of the glomeruli, as well as by a
glycogenic infiltration of the tubules; there was a fatty degeneration in the liver and a fibrinous
inflammation in the mucous coats of the trachea, bronchi and stomach. What disease did the died
person suffer from?
A. Chronic indurative pancreatitis
B. Chronic glomerulonephritis
C. Hypertensive disease
D. Steatosis
E. * Diabetes mellitus
75. An autopsy of a male revealed a tumour in the anterior lobe of the hypophysis, enlarged adrenal
glands, a reduction of the gonads in size, a hypertrophy of the left cardiac ventricle, the pancreas was
reduced in size and thickened. Histologically, there was a pituitary basophil adenoma and a
hyperplasia of the cortical layer in the adrenal glands. The pancreas was characterized by a
moderately expressed atrophy of the parenchyma, including islets of Langerhans. What disease did
the patient suffer from?
A. Diabetes mellitus
B. Adiposogenital dystrophy
C. Cushing's syndrome
D. Simmonds disease
E. * Cushing's disease
76. For a histological examination, a lobe and a part of the isthmus of the thyroid gland were received.
The tissue of the gland was dense and tuberous, on section it was pale brown and had grey-whitish
foci. Microscopically, against a background of an atrophy of the follicles of the gland, there was
some diffuse lymphoplasmacytic infiltration of the stroma with formation of lymphoid follicles.
What pathological process were these changes typical for?

A. Toxic goiter
B. Thyroid adenoma
C. Colloid goiter
D. Sporadic goiter
E. * Allergic thyroiditis
77. A histological examination of a thyroid gland revealed follicles of various size and shape which were
lined with the columnar epithelium; the latter proliferated and formed papillae of various size. The
follicular lumens contained some liquid and vacuolized colloid. The stroma of the gland was
characterized by a lymphoplasmacytic infiltration, in some places with formation of lymphatic
follicles having light centres. Which of the diagnoses was the most probable?
A. Colloid goiter
B. Nodular goiter
C. Hashimoto's disease
D. Ligneous thyroiditis
E. * Toxic goiter
78. An autopsy of a 48-year-old male, who died from vascular collapse, revealed an increased
pigmentation of the skin, the adrenal glands were reduced in size, the brown-yellow liver was
enlarged. On histological examination, foci of necrosis with a tuberculous granulation tissue were
found in the adrenal glands. The liver was characterized by phenomena of fatty degeneration. Which
of the diagnoses was the most probable?
A. Steatosis
B. Primary aldosteronism
C. Cushing's syndrome
D. Lipofuscinosis
E. * Addison's disease
79. An autopsy of a 45-year-old male, who suffered from numerous pathological fractures during his
lifetime, revealed changes in his long tubular bones: the bones of the thigh and shin were bent, in
some places they resembled spirals, their surface was tuberous, a section revealed an obliterated
medullary channel and a change in the compact structure of the cortical layer by the spongy type.
Microscopically, there was a mosaic type of the bone structures: against a background of a disordered
thin-fibrous or lamellar structure of the bone fragments there were numerous cavities of sinusal
resorption combined with signs of new formation of the osseous tissue. The arteries, which supplied
the bone tissue, were dilated and convoluted. Name a diagnosis.
A. Osteopetrosis
B. Parathyroid osteodystrophy
C. Fibrous dysplasia
D. Chronic osteomyelitis
E. * Deforming osteodystrophy
80. An autopsy of a male, who suffered from right-sided pneumonia in the lower lobe during his
life-time and for a long period of time expectorated sputum of a purulent character, revealed some
cavity with dense edges that was located in the 9th-10th segments of the lung and was filled with
yellowish cream-like masses. There was some whitish path from the cavity to the root of the lung.
Microscopically, the cavity was separated from the intact pulmonary tissue with a membrane which
consisted of a fibrous connective tissue from the outside and a granulation one from inside. Which of
the diagnoses was the most probable?
A. Bronchiectatic disease
B. Pulmonary gangrene
C. Acute pulmonary abscess
D. Chronic pneumonia
E. * Chronic abscess

81. For a histological examination, a vermiform process (appendix) was sent. Its size is increased, the
serous membrane is dim, plethoric and covered with greyish films, the wall is thickened and some
pus is discharged from the lumen. Microscopically, a plethora of the vessels, an oedema of all the
layers and their diffuse infiltration by leukocytes are observed. Name the kind of inflammation in the
vermiform process.
A. Catarrhal
B. Putrid
C. Mixed
D. Fibrinous
E. * Phlegmonous
82. An examination of a 7-year-old child, who was referred to infectious department with complaints
about a sharp pain in his throat, difficult swallowing, an elevated body temperature up to 39cc, an
oedema of his neck, revealed that the tonsils were enlarged, their mucosa was plethoric and covered
with a large number of yellow-whitish films which were closely adjacent to the mucosa. an attempt to
remove a film results in a deep bleeding defect. what kind of inflammation takes place?
A. Suppurative
B. Serous
C. Croupous
D. Haemorrhagic
E. * Diphtheritic
83. A male was treated for purulent otitis. On the 9th day of his staying at an inpatient department he
died from a brain oedema. On autopsy, the temporal region of the left hemisphere revealed a cavity
with uneven rough inner edges which was filled with some yellowish-greenish thick dull fluid. The
outer wall of the cavity was represented with the cerebral tissue. What pathological process was it?
A. Colliquative necrosis
B. Phlegmon
C. Empyema
D. Chronic abscess
E. * Acute abscess
84. An autopsy revealed 0.5 I of some yellowish transparent fluid with small white crumble clots in the
right pleural cavity. The parietal and visceral pleurae were covered with a white crumble coat. What
kind of exudative inflammation was it?
A. Suppurative
B. Serous
C. Putrid
D. Catarrhal
E. * Croupous
85. A 6-year-old girl fell ill with diphtheria and three days later died of asphyxia resulting from
membranous croup. On autopsy, the mucous membranes of the larynx, trachea and bronchi were
thickened, oedematous and covered with greyish films which were easily separated. What kind of
inflammation did the morphological changes in the larynx indicate?
A. Serous
B. Haemorrhagic
C. Diphtheritic
D. Croupous
E. * Catarrhal
86. An autopsy of a 77-year-old male, who died from dysentery, revealed some grey-yellow films which
were closely connected with the underlying tissues in the colon and separated with formation of
ulcers. Name the kind of inflammation.

A. Serous
B. Catarrhal
C. Croupous
D. Suppurative
E. * Diphtheritic
87. A microscopic examination of the tissue dissected from some postoperative infiltrate revealed
granulomata with giant multinucleate cells around the suture material. What kind of granulomata did
they belong to?
A. Tuberculous
B. Rheumatic
C. Lepromatous
D. Mycotic
E. * Foreign-body
88. An examination of a renal biopsy revealed some mostly perivascular and periglomerular
lymphocytic, plasmacytic and macrophagal infiltration of the interstice against a background of its
sclerosis. Name the most probable kind of inflammation.
A. Productive diffuse
B. Granulomatous
C. Exudative diffuse
D. Exudative focal
E. * Productive focal
89. A microscopic examination of the myocardium in a male, who died from cardiac decompensation,
revealed sclerosis of the perivascular connective tissue and its diffuse infiltration by lymphocytes,
macrophages, plasmacytes and solitary neutrophils. Which of the listed kinds of inflammation was
the most probable?
A. Granulomatous productive
B. Alterative
C. Exudative diffuse
D. Exudative focal
E. * Interstitial productive
90. A little girl died of asphyxia resulting from membranous croup. A girl illed with diphtheria. On
autopsy, the mucous membranes of the larynx, trachea and bronchi were thickened, oedematous and
covered with greyish films which were easily separated. What type of inflammation
pathomorpholodist investigated?
A. Serous
B. Haemorrhagic
C. Diphtheritic
D. Croupous
E. * Catarrhal
91. A surgeon removed a tumour in the liver of a 47-year-old male patient. It was macroscopically
revealed that the wall of the cavity was formed by a dense fibrous connective tissue; the cavity
contained some yellow-greenish dull thick fluid which had an unpleasant odour and microscopically
consisted mainly of polymorphonuclear leukocytes. What pathological process did such
morphological changes correspond to?
A. Phlegmon
B. Acute abscess
C. Empyema
D. Colliquative necrosis
E. * Chronic abscess

92. A children died from asphyxia at the diphtheria. On autopsy were identified membranous plates in
space of respiratory ways, the mucous membranes of the larynx, trachea and bronchi were thickened,
oedematous and covered with greyish films which were easily separated. What kind of inflammation
did the morphological changes in the larynx indicate?
A. Serous
B. Haemorrhagic
C. Diphtheritic
D. * Croupous
E. Catarrhal
93. An autopsy of a 58-year-old male, who suffered from croupous pneumonia during his life-time and
died of cardiopulmonary insufficiency, revealed 900 ml of some yellow-greenish dull fluid in his
right pleural cavity. The pleural leaves were dull and plethoric. Name the clinical-morphological
form of the inflammation in the pleural cavity.
A. Dry pleurisy
B. Phlegmon
C. Chronic abscess
D. Acute abscess
E. * Empyema
94. An autopsy of a woman, who suffered from the right-sided purulent otitis during her life-time,
revealed a cavity 4 x 3 cm in size in the region of the right temporal lobe that contained some
yellow-green dull viscous fluid. The inner layer of the wall was represented with a yellowish crumble
tissue, the outer layer consisted of a whitish dense tissue. What process were the described changes in
the brain characteristic of?
A. Acute abscess
B. Empyema
C. Grey softening of the brain
D. Haemorrhage
E. * Chronic abscess
95. Microscopic analysis of tissue sampling from patient's skin reveals granulomas that consist of
epithelioid cells surrounded mostly by T-lymphocytes. Among epithelioid cells there are solitary
giant multinuclear cells of Pirogov-Langhans type. In the centre of some granulomas there are areas
of caseous necrosis. Blood vessels are absent. What disease are the described granulomas typical for?
A. * Tuberculosis
B. Syphilis
C. Leprosy
D. Rhinoscleroma
E. Glanders
96. A pathology-histology laboratory received a vermiform appendix up to 2,0 cm thick. Its serous
membrane was pale, thick and covered with yellowish-green films. The wall was flaccid, of
grayish-red colour. The appendix lumen was dilated and filled with yellowish-green substance.
Histological examination revealed that the appendix wall was infiltrated with neutrophils. Specify the
appendix disease:
A. * Acute phlegmonous appendicitis
B. Acute gangrenous appendicitis
C. Acute superficial appendicitis
D. Acute simple appendicitis
E. Chronic appendicitis

97. Autopsy of a 50-year-old man revealed the following changes: his right lung was moderately compact
in all parts, the dissected tissue was found to be airless, fine-grained, dryish. Visceral pleura had
greyish-brown layers of fibrin. What is the most likely diagnosis?
A. * Croupous pneumonia
B. Tuberculosis
C. Bronchopneumonia
D. Interstitial pneumonia
E. Pneumofibrosis
98. A patient with android-type obesity had been suffering from arterial hypertension, hyperglycemia,
glycosuria for a long time and died from the cerebral haemorrhage. Pathologic examination revealed
pituitary basophil adenoma, adrenal cortex hyperplasia. What is the most likely diagnosis?
A. * Itsenko-Cushing's syndrome
B. Diabetes mellitus
C. Acromegalia
D. Pituitary nanism
E. Adiposogenital dystrophy
99. Mucous membrane of the right palatine tonsil has a painless ulcer with smooth lacquer fundus and
accurate edges of cartlaginous consistency. Microscopically: inflammatory infiltrate that consists of
lymphocytes, plasmocytes, a small number of neutrophils and epithelioid cells; endovasculistis and
perivasculitis. What disease is in question?
A. * Syphilis
B. Actinomycosis
C. Tuberculosis
D. Pharyngeal diphtheria
E. Necrotic (Vincent's) tonsillitis
100. On the 5th day of illness a 12 year old child who was treated in the infectious department on account
of influenza felt severe headache, sickness, dizziness, got meningeal signs. The child died 24 hours
later from increasing brain edema. Dissection of cranial cavity revealed that pia maters of brain are
edematic, plethoric, saturated diffusively with bright red liquid. Convolutions and sulci of brain are
flattened. What influenza complication is in question?
A. * Hemorrhagic meningitis
B. Cerebral hemorrhage
C. Venous hyperemia of brain membranes
D. Suppurative leptomeningitis
E. Serous meningitis
Назва наукового напрямку (модуля): Семестр: 6
ТЕСТИ АНГЛ екзамен мед
Опис:
ЕКЗЕМ медичний
Перелік питань:
1. During autopsy approximately 2,0 liters of pus have been found in the abdominal cavity of the body.
Peritoneum is dull and of grayish shade, serous tunic of intestines has grayish-colored coating that is
easily removable. Specify the most likely type of peritonitis in the patient:
A. Hemorrhagic peritonitis
B. Serous peritonitis
C. Tuberculous peritonitis
D. Necrosis
E. * Fibrinopurulent peritonitis
2. The most characteristic manifestation of malignant tumor is which of the following:
A. Cellular atypia and pleomorphism
B. Compression of surrounding tissue
C. Large size
D. Necrosis
E. * Metastases
3. The most important feature to distinguish the malignant tumor from a benign one is which of the
following:
A. Lack of encapsulation
B. High mitotic rate
C. Necrosis
D. Nuclear pleomorphism
E. * Metastases
4. All of the following neoplasms are malignant, except:
A. Adenocarcinoma
B. Melanoma
C. Seminoma
D. Chorionepithelioma
E. * Papillary cystadenoma
5. Cytokines secreted by tumors that induce angiogenesis and assist the tumor in establishing its blood
supply include all of the following, except:
A. Fibroblast growth factor
B. Transforming growth factor - a
C. Transforming growth factor - b
D. Platelet-derived growth factor
E. * Tumor necrosis factor
6. All of the following genes (known as " tumor suppressor genes ") provide the negative control over
cell proliferation, except:
A. The p53gene
B. The DCC ("Deleted in Colon Cancer") gene
C. The Rb gene
D. The WT-1 gene
E. * The bcl-2 gene
7. The factor assessed in the histological grading of a malignant tumor is which of the following:
A. The number of lymph node metastases

B. The size (diameter) of the primary tumor


C. The extent of invasion of the primary tumor into surrounding structures
D. The presence or absence of liver metastases
E. * The degree of cytological differentiation of the primary tumor
8. A malignant tumor is characterized by all of the following, except:
A. Increased abnormal tissue mass
B. Uncoordinated invasive growth
C. Relatively autonomous growth
D. Metastases
E. * Decreased abnormal tissue mass
9. The growth of neoplasms is critically dependent on which of the following:
A. Localization
B. Neutrophil immigration
C. Lymphatic rainage
D. Inflammatory reaction
E. * Tumor stroma
10. Dysplasia is characterized by all of the following, EXCEPT:
A. Abnormal organization of cells;
B. Loss in the uniformity of individual cells;
C. Loss in cell architectural organization;
D. Variation of cells in size and shape
E. * Replacement of one adult cell type by another adult cell type;
11. Hyperplasia is characterized by which of the following:
A. Increase in the size of cells;
B. Increase in the number of nuclei in cells;
C. Shrinkage in the size of cells;
D. Atypia of cells.
E. * Increase in the number of cells;
12. Hypertrophy as a process is characterized by which of the following:
A. Shrinkage in the size of cells and of the organ;
B. Increase in the number of cells;
C. Abnormal organization of cells;
D. Variation of cells in size and shape
E. * Increase in the size of cells and of the organ;
13. Hypertrophy as an adaptive response is charactirized by which of the following:
A. Pathologic hypertrophy of the breast duringlactation;
B. Pathologic hypertrophy of the uterus duringpregnancy;
C. Hypertrophy of the skeletal muscle cells in apatient with immobilized broken limb;
D. Hypertrophy of the endometrium due toovarian tumor.
E. * Hypertrophy of the skeletal muscle cells in a body-builder;
14. Hypoplasia is characterized by all of the following, EXCEPT
A. Incomplete development of an organ;
B. Decreased number of cells;
C. Underdevelopment of an organ;
D. Decreased function of an organ.
E. * Increased number of cells;

15. Metaplasia is characterized by which of the following:


A. Reversible increase in the size of cells;
B. Irreversible change in which one adult celltype isreplaced by another adult cell type;
C. Reversible abnormal organization of cells;
D. Reversible increase in the number of cells.
E. * Reversible change in which one adult celltype is replaced by another adult cell type;
16. One of the variants of physiologic atrophy is:
A. Atrophy of skeletal muscle by the immobi lized broken limb;
B. Kidney atrophy from pressure (hydronephrosis) ;
C. Atrophy of the endometrium by ovarian tumor;
D. Atrophy of the brain in atherosclerosis
E. * Atrophy of uterus after parturition;
17. Pressure-overloaded (concentric) cardiac hypertrophy is characterized by all of the following,
EXCEPT:
A. Hypertrophy of left ventricle;
B. Increased wall thickness;
C. Normal left cavity diameter;
D. Reduced left cavity diameter.
E. * Dilated left cavity diameter;
18. Simple endometrial hyperplasia is characterized by all of the following, EXCEPT:
A. Increase in the number and size of endometrial glands;
B. Complex endometrial glands;
C. Increase in gland-to-stroma ratio;
D. Dilated endometrial glands.
E. * Atypia of gland cells;
19. The causes of hypertrophy are all of the following, EXCEPT:
A. Mechanical triggers;
B. Trophic triggers;
C. Vasoactive agents.
D. Polypeptide growth factors;
E. * Nervous triggers;
20. The hydronephrosis is characterized by all of the following, EXCEPT:
A. Thinning of the renal parenchyma;
B. Dilatation of the renal pelvis;
C. Dilatation of the renal calyces;
D. Progressive atrophy of the kidney.
E. * Thickening of the renal parenchyma
21. The hyperplasia of hepatic cells that occurs after partial hepatoectomy is an example of:
A. Pathologic hyperplasia;
B. Hormonal hyperplasia;
C. Hormonal hypertrophy;
D. Compensatory hypertrophy.
E. * Compensatory hyperplasia;
22. The hypertrophic and dilated (eccentric) heart is characterized by all of the following, EXCEPT:
A. Decreased mass and diminished left wallthickness;
B. Increased mass and normal left wallthickness;

C. Normal mass and left wall thickness;


D. Normal mass and diminished left wallthickness;
E. * Increased mass and diminished left wallthickness.
23. The nomenclature of tumors is based on which of the following:
A. Stromal component
B. Localization
C. Inflammatory changes
D. Vascular component
E. * Parenchymal component histogenesis
24. Malignant tumors arising from mesenchymal tissue are referred to as:
A. Adenocarcinomas
B. Papillomas
C. Cystadenomas
D. Polyps
E. * Sarcomas
25. The important factors associated with the increasing incidence of tumors are au of the following,
except:
A. Age
B. Diet
C. Environment
D. Genetic makeup
E. * Acute inflammation
26. Morphologic atypia is characterized by all of the following, except:
A. Cell pleomorphism
B. Alteration of parenchyma/ stroma ratio
C. Atypical mitoses
D. Enlarged hyperchromatic nuclei
E. * Metaplasia
27. Anaplasia is characterized by all of the following, except:
A. Cellular and nuclear pleomorphism
B. Formation of tumor giant cells
C. Formation of atypical mitotic figures
D. Hyperchromatic nuclei
E. * Formation of Langhans giant cells
28. Benign tumor arising from smooth muscle cells is called:
A. Myoma
B. Chondroma
C. Rhabdomyoma
D. Fibroma
E. * Leiomyoma
29. Tumor cell invasion into the extracellular matrix can be characterized by all of the following, except:
A. Detachment of the tumor cells from each other
B. Attachment to matrix components
C. Degradation of extracellular matrix
D. Migration of tumor cells
E. * Intravasation

30. Character of growth of high differentiated tumors in relation to surrounding tissues:


A. Exophytic
B. Endophytic
C. Infiltrating
D. Invasive
E. * Expansive
31. Character of growth of malignant tumors in relation to surrounding tissues.
A. Exophytic
B. Expansive
C. Endophytic
D. Apposition
E. * Invasive
32. Secondary changes in tumors are all of the following, except:
A. Necrosis
B. Inflammation
C. Calcification
D. Hemorrhages
E. * Metastases
33. Desmoid is located more often in:
A. Legs
B. Hands
C. Skin of back
D. Langs
E. * Anterior abdominal wall
34. Rhabdomyoma is formed from:
A. Collagen fibers
B. Smooth muscles
C. Histiocytes
D. Fibroblasts
E. * Striated muscles
35. Leiomyoma is formed from:
A. Collagen fibers
B. Histiocytes
C. Myoblast
D. Fibroblasts
E. * Smooth muscles
36. Capillary Hemangioma more frequently occur in:
A. Mans
B. Young women
C. Old age people
D. Women in menopause
E. * Children
37. Osteosarcoma is a malignant tumour which arising from:
A. Chondrocytes
B. Lypoblasts
C. Histiocytes

D. Chondroblast
E. * Atypical cells of osteoblastic type with a lot of mitosises
38. Astroblastoma metastasing by:
A. Blood
B. Lymph
C. Contact|
D. Implantation
E. * Cerebrospinal fluid
39. Ependymoma is glioma connected with:
A. Mesencephalon
B. Cerebelli
C. Spinal cord
D. Medulla oblongate
E. * Ventricular ependyma
40. Name of high differentiated tumor of the peripheral nervous system:
A. Hybernoma
B. Leiomyoma
C. Fibroma
D. Glioma
E. * Schwannoma
41. Name differentiated tumor from fatty tissue:
A. Fibroma
B. Desmoid
C. Histiocytoma
D. Myxoma
E. * Lipoma
42. Thyroid gland with small cysts, lined with atypical epithelium and filled with papillae, the latter
originating from the walls of the cysts and growing into their capsules. Choose rihgt diagnosis:
A. Papillary adenoma
B. Follicular carcinoma
C. Solid carcinoma
D. Carcinoma simplex
E. * Papillary carcinoma
43. Uterine cervix tissue covered with a wide layer of the stratified squamous epithelium having
roliferation of atypical cells with pathological mitoses, but the basal membrane of the epithelium was
not affected. Choose rihgt diagnosis:
A. Nonkeratinizing squamous cell carcinoma
B. Keratinizing squamous cell carcinoma
C. Leukoplakia
D. Epithelial dysplasia
E. * Carcinoma in situ
44. Deformed mucouse membrane of a lobar bronchus consistinyof hyperchromatic nuclei and numerous
pathological mitoses, the growth of the tumour did not spread to the basal membrane of the
epithelium. Choose rihgt diagnosis:
A. Squamous cell carcinoma
B. Adenocarcinoma
C. Solid carcinoma

D. Small-cell carcinoma
E. * Carcinoma in situ
45. The epithelial tumors without specific localization develop from:
A. Mesotheliocytes
B. Fibroblastes
C. Epithelioidcells
D. Epitheliocytes
E. * Squamous epithelium
46. The quiet formed benign tumor with squamous epithelium is known as:
A. Sarcoma
B. Fibroma
C. Lipoma
D. Mioma
E. * Papilloma
47. Not formed malignant tumors are named:
A. Fibroma
B. Adenoma
C. Mioma
D. Papilloma
E. * Carcinoma
48. The quiet formed benign tumor with glandular cells is known as:
A. Fibroma
B. Sarcoma
C. Mioma
D. Papilloma
E. * Adenoma
49. The papilloma looks like:
A. A noddle with narrow flat
B. Cyst
C. Stria
D. Scar
E. * A noddle with papilly flat
50. Adenoma is the quiet formed tumor with:
A. Transitional epithelium
B. Squamous epithelium
C. Epymisium
D. Perimisium
E. * Glandular epithelium
51. All of the following neoplasms are malignant, EXCEPT:
A. Adenocarcinoma
B. Sarcoma
C. Seminoma
D. Chorionepithelioma
E. * Mioma
52. All of the following neoplasms are malignant, EXCEPT:
A. Adenocarcinoma

B. Sarcoma
C. Seminoma
D. Chorionepithelioma
E. * Fybroma
53. A tumor that tends to spread over the surfaces of organs or body cavities rather than metastastring via
blood vessels or lymphatics is which of the following:
A. Colon carcinoma
B. Thyroid carcinoma
C. Renal cell carcinoma
D. Hepatocellular carcinoma
E. * Mesothelioma
54. A sequence of metaplasia-dysplasia-neoplasia typically is known to occur in the oncogenesis of all of
the following cancers, EXCEPT:
A. Squamous cell carcinoma of the bladder
B. Squamous cell carcinoma of the lung
C. Adenocarcinoma of the esophagus
D. Squamous cell carcinoma of the endocervix
E. * Carcinoid tumor of the small bowel
55. Poor prognosis in breast cancer is associated with all of the following, EXCEPT:
A. Extensive angiogenesis and presence of aneuploidy
B. Absence of estrogen receptors
C. Presence of aneuploidy
D. Extensive angiogenesis
E. * Overexpression of the N-myc oncogene
56. The most common benign tumor of the female breast is which of the following:
A. Cystadenoma
B. Sarcoma
C. Fibroma
D. Adenocarcinoma
E. * Fibroadenoma
57. A black tumour, 2 cm in diameter, from the skin of his thigh, microscopically, the tumour consisted
of polymorphous cells, the cytoplasm of most of them includs some brown pigment (with a positive
reaction to DOPA), large number of pathological mitoses was registered. Choose rihgt diagnosis:
A. Carcinoma
B. Sarcoma
C. Carcinosarcoma
D. Nevus
E. * Melanoma
58. Massive exophytic carcinoma on the lesser curvature of the stomach with metastases to the ovaries.
What kind of metastatic spreading took place?
A. Haematogenous
B. Lymphogenous orthograde
C. Implantation
D. Perineural
E. * Lymphogenous retrograde
59. What type of metaplasia may occur in the respiratory tract in habitual cigarette smoker?
A. Epithelial metaplasia: squamous to columnar

B. Metaplasia to undifferentiated mesenchymal cells


C. Connective tissue metaplasia
D. Compensatory metaplasia
E. * Epithelial metaplasia columnar to squamous
60. Carcinoid syndrome development results from producing of which the following substances:
A. Melanin
B. Bilirubin
C. Ferritin
D. Hemosiderin
E. * Serotonin
61. The most common malignant epithelial tumor of the stomach is the following:
A. Gastric lymphoma
B. Gastric myoma
C. Gastric leiomyoma
D. Adenomatous polyp
E. * Gastric carcinoma
62. Invasive breast adenocarcinomas of the female breast include all subtypes, EXCEPT:
A. Scirrhous adenocarcinoma
B. Adenocarcinoma
C. Medullary carcinoma
D. Tubular carcinoma
E. * Carcinoma in situ
63. A main morphologic features of Graves disease include all of the following, EXCEPT:
A. Tall and crowded follicular epithelial cells;
B. Small papillae into the follicular lumen;
C. Lack of fibrovascular cores in papillae;
D. Pale colloid with scalloped margins;
E. * Large leukocytic aggregates in the stroma.
64. A most common cause of the glomeruli damage in systemic lupus erythematosus is which of the
following:
A. Local anaphylaxis;
B. Immunologic tolerance;
C. Genetic deficiency of themonocyte/macrophage system;
D. Genetic deficiency of the complement system.
E. * Immune complex injury;
65. At morphologically exam, delayed-type hypersensitivity is characterized by all of the following,
EXCEPT:
A. Accumulation of the mononuclear cells around venules;
B. Dermal edema;
C. Deposition of fibrin in the interstitium of derma;
D. Accumulation of the mononuclear cellsaround small vein;
E. * Deposition of hemosiderin in the derma.
66. Name the macrophage cytokines which are characterized by all of the following, EXCEPT:
A. Mediate natural immunity;
B. Regulate lymphocytes growth, activation anddifferentiation;
C. Activate inflammatory cells;
D. Affect leukocyte movements.

E. * Inhibit hematopoesis;
67. Organs most commonly and seriously damaged in systemic lupus erythematosus are all of the
following, EXCEPT:
A. Skin;
B. Heart and vessels
C. Joints;
D. Serosal surfaces;
E. * Eye;
68. The all of the following diseases result from Type III hypersensithity (immune complex disorders),
EXCEPT:
A. Glomerulonephritis;
B. Farmer's lung;
C. Systemic lupus erythematosus
D. Hemolitic anemia;
E. * Myocardial infarction;
69. The autoimmune diseases involving a single organ or cell type are all of the following, EXCEPT:
A. Hashimoto thyroiditis;
B. Goodpasture syndrome;
C. Insulin-dependent diabetes mellitus;
D. Myasthenia gravis.
E. * Myocardial infarction;
70. The causes of pathologic atrophy are all of the following, EXCEPT:
A. Loss of endocrine stimulation;
B. Loss of innervation;
C. Diminished blood supply;
D. Decreased workload;
E. * Increased workload
71. The causes of pathologic atrophy are all of the following, EXCEPT:
A. Aging;
B. Pressure;
C. Inadequate nutrition;
D. Denervation
E. * Intracellular fat accumulation;
72. The CD4+ T cells influence the function of all of the following immune system cells, EXCEPT:
A. B lymphocytes;
B. Natural killer cells;
C. Macrophages;
D. CD8+ T cells.
E. * Fibroblasts;
73. The cells affected by HIV are all of the following, EXCEPT:
A. Macrophages;
B. Dendritic cells;
C. Monocytes;
D. CD4+ T cells.
E. * CD8+ T cells;
74. The cells of immune system which have numerous fine dendritic cytoplasmic processus are called:
A. Fibroblasts;

B. Macrophages;
C. Leucocytes;
D. CD4+ T cells
E. * Dendritic cells;
75. The cells which compose the granuloma in type IV hypersensitivity reactions are all of the following,
EXCEPT:
A. Lymphocytes;
B. Macrophages;
C. Epithelioid cells;
D. Giant cells.
E. * Erythrocytes;
76. The complement activation gives rise for all of the following proinflammatory effects, EXCEPT:
A. Release of C3b;
B. Production of chemotactic factors;
C. Release of anaphylatoxins;
D. Formation of membrane attack complex.
E. * Formation of antigens
77. The Disorders of the immune system include all of the following, EXCEPT:
A. Hypersensitivity reactions;
B. Autoimmune diseases;
C. Immunologic deficiency syndromes;
D. Amyloidosis.
E. * Hyalinosis
78. Atrophy is characterized by which of the following:
A. Variation of cells in size and shape;
B. Increase in the size of cells;
C. Abnormal organisation of cells.
D. Increase in the number of cells;
E. * Shrinkage in the size of cells by loss of cell substance;
79. Conditions leading to endometrial hyperplasia include all of the following, EXCEPT:
A. Polycystic ovarian disease;
B. Functioning granulosa cell tumors of the ovary;
C. Prolonged administration of estrogenic substances;
D. Excessive ovarian cortical function;
E. * Prolonged administration of analgesic substances
80. The local factors that influence wound healing are all of the following, EXCEPT:
A. Wound infection;
B. Mechanical factors;
C. Foreign bodies;
D. Size, location and type of the wound
E. * Hormones (glucocorticoids);
81. The massive growth of the gravid uterus with large plump cells is an example of:
A. Pathologic hypertrophy;
B. Pathologic hyperplasia;
C. Dysplasia;
D. Metaplasia.

E. * Hormone induced physiologic hypertrophy


82. The most common clinical manifestation of endometrial hyperplasia is:
A. Menses stopping;
B. Pains;
C. Purulent discharges;
D. Mucous discharges
E. * Abnormal uterine bleeding;
83. What pigment can be found in the cytoplasm of heart and muscle cells in aging atrophy?
A. Melanin;
B. Hemosiderin;
C. Bilirubin;
D. Ferritin.
E. * Lipofuscin
84. Which pathologic process results from dysplasia?
A. Aplasia;
B. Hypoplasia;
C. Hyperpigmentation;
D. Calcification;
E. * Neoplasia
85. Which type of metaplasia may occur in the respiratory tract in habitual cigarette smoker?
A. Epithelial metaplasia: squamous to columnar;
B. Metaplasia to undifferentiated mesenchymalcells;
C. Connective tissue metaplasia;
D. None of these.
E. * Epithelial metaplasia: columnar to squamosus
86. Give the determination of gynecomastia
A. Non-cancerous cystous dysplasia of mammary gland
B. Non-cancerous hyperplasia of mammary gland
C. Malignant dysplasia of mammary gland
D. Non-cancerous sclerotic dysplasia of mammary gland
E. * Non-cancerous dysplasia of breast in men
F. Is compatible with a normal life span if treated
87. Increased blood viscosity (hyperviscosity syndrome) is major complication of all of the following
disorders, EXCEPT:
A. Polycythemia vera
B. Immunoglobulin A myeloma
C. * "Hairy-cell" leukemia
D. Waldenstrom's macroglobulinemia
E. Heavy-chain disease
88. Factor that provides for a definitive diagnosis of hemolytic anemia is which of the following:
A. Red blood cell antibodies
B. * Red blood cell destruction
C. Red blood cell enzyme deficiency
D. Bone marrow erythroid hyperplasia
E. Abnormal hemoglobin
89. Nonimmune hemolytic anemia occurs in patients with which of the following disease:

A. Systemic lupus erythematosus


B. * Malaria
C. Chronic lymphocytic leukemia
D. Hodgkin's disease
E. All named
90. The disorder that most commonly causes iron deficiency anemia is which of the following:
A. Liver cirrhosis
B. Cardiomyopathy
C. * Peptic ulcer
D. Pancreatitis
E. Hemorrhagic stroke
91. The similarity between thalassemia major and thalassemia minor is which of the following:
A. Severity of anemia
B. Incidence of infection
C. Life spans
D. * Characterised by elevated HbF level and reduced HbA2 level.
E. Homozygous form
92. Name causes when the volume of stored blood transfused exceeds their normal blood volume, it
results in dilutional thrombocytopenia and dilution of coagulation factors.
A. * Massive transfusion
B. Transmission of infection
C. Air embolism
D. Thrombophlebitis
E. Circulatory overload
93. An increase in the amount of erythrocytes is the highest in:
A. Patients with erythromia
B. After transfusion of erythrocytes
C. In patients with emphysema of the lungs
D. In patients with heart failure
E. * People who live in mountains
94. Of the following anemias hyperchromic is:
A. B-12 deficiency
B. Metaplasmatic
C. Hemolytic
D. Post-hemmorhagic anemia
E. * Iron-deficiency anemia
95. Megalowblastic hyperplasia of bone marrow arises because of hypovitaminosis of:
A. A
B. В-1
C. C
D. PP
E. * B-12
96. The most characteristic cells of blood in bone marrow during B-12 deficient anemia are:
A. Poikilocyte
B. Megalowblasts
C. Hypochromatic erythrocytes

D. Normoblasts
E. * Anyzocytes
97. Leucosis-systemic tumor diseases:
A. Lymphoid tissue
B. Mesenchymal tissue
C. Endothelial tissue
D. Epithelial tissue
E. * Hemopoetic tissue
98. Bone marrow is red juicy, sometimes a gray tint during:
A. Undifferentiated form of leukosis
B. Jewler’s form of leucosis
C. Lymphogranulamatosis
D. During a child’s form of leucosis
E. * Early form of leucosis
99. All of the following statements correctly describe hereditary spherocytosis, EXCEPT:
A. * Red blood cells have the membrane-associated protein spectrin
B. Mutation in the ankyrin gene is present in most cases
C. Anaplastic crisis
D. Splenectomy is invariably therapeutic
E. -
100. Features of megaloblastic anemias include all of the following, EXCEPT:
A. Hypersegmented neutrophils
B. * Lack of the membrane-associated protein spectrin
C. Increased intramedullary hemolysis
D. Increased extramedullary hemolysis
E. Pancytopenia
101. All of the following statements correctly describe aplastic anemia, EXCEPT:
A. Production of all hematopoietic bone marrow elements is reduced
B. Chemical drug exposure is the most common cause
C. Fanconi's anemia represents an inherited form of the disease
D. No underlying etiology is evident in 50% of cases
E. * Splenomegaly is a characteristic clinical finding
102. Osmotic fragility is characteristic of the erythrocytes in which of the following diseases:
A. Fanconi's anemia
B. Sickle cell anemia
C. Glucose-6-phosphate dehydrogenase deficiency
D. * Hereditary spherocytosis
E. Pernicious anemia
103. Myelophthisic anemia can occur in patients with any of the following conditions, EXCEPT:
A. Miliary tuberculosis
B. Carcinomatosis
C. Myelofibrosis
D. Multiple myeloma
E. * Uremia
104. During chronic lympho leucosis the liver is
A. Shrunken

B. Atrophic
C. Infiltrated by scar tissue
D. Cirrhotic
E. * Enlarged, gray-brown color
105. To the group of paraprotein leukosis belong disease of
A. Hodgkins
B. Waldenstrom’s
C. Langerhans’
D. Letterer-Siwe
E. * Mielomic
106. The well developed atheromatous plaque results from all of the following processes, except:
A. Endothelial injury with adhesion of monocytes and platelets
B. Migration of monocytes from the lumen and smooth muscle cells from the media into the intima
C. Smooth muscle cell proliferation, collagen deposition
D. Lipid absorption by macrophages and smooth muscle cells
E. * Action of membrane-bound tissue factor
107. The chronic endothelial injury in atherosclerosis may be caused by all of the following factors,
except:
A. Hyperlipidemia
B. Hypertension
C. Smoking
D. Immune reactions
E. * Genetic defects
108. The chronic endothelial injury in atherosclerosis includes all pathologic processes, except:
A. Increased endothelial permeability
B. Adhesion of blood monocytes to the endothelium
C. Adhesion of platelets to the intima
D. Insudation of lipoproteins into the vessel wall
E. * Regeneration of endothelial cells
109. The special stain used to identify "foam cells" in atherosclerotic plaque is which of the following:
A. PAS-reaction
B. Picrofuchsin
C. Hematoxylin and eosin
D. Congo red stain
E. * SudanIII
110. The major nonmodifiable risk factors for atherosclerosis are all of the following, except:
A. Increasing age
B. Genetic abnormalities
C. Male gender
D. Family history
E. * Increased water resorption
111. The potentially controllable risk factors for atherosclerosis are all of the following, except:
A. Hyperlipidemia
B. Hypertension
C. Diabetes mellitus
D. Cigarette smoking

E. * Genetic abnormalities
112. What vessels are struck by atherosclerosis?
A. Capillaries
B. Aorta
C. V. cava inferior
D. Veins
E. * Arteries
113. Aschoff giant cells are large mesenchymal cells and are believed to be derived from which of the
following:
A. Fibroblasts
B. Lymphocytes
C. Eosinophils
D. Erythrocytes
E. * Macrophages
114. Atherosclerosis commonly affects the arteries supplying all of the following organs, except:
A. Lower extremities
B. Heart
C. Brain
D. Kidney
E. * Upper extremities
115. Blood vessels affected in atherosclerosis are which of the following:
A. Aorta only
B. Large arteries and large veins
C. Large veins
D. Aorta and arterioli
E. * Aorta and large arteries
116. Cholesterol-laden macrophages and smooth muscle cells in atheromatous plaque are also referred to
as:
A. Giant cells
B. "Popcorn" -cells
C. Epithelioid cells
D. Vacuolated cells
E. * "Foam" cells
117. Material accumulated in macrophages and smooth muscle cells in atherosclerotic plaque is which of
the following:
A. Glycogen
B. Lipohyaline
C. Triglycerides
D. Lipoproteids
E. * Cholesterol esters
118. Type of uncomplicated macroscopic changes of intima arteries in atherosclerosis.
A. Postural thrombosis
B. Plaque detachment
C. Fibrous plaque with ulceration
D. Intramural hematoma
E. * Fat stains and stripes
119. Macroscopic characteristic of fibrous plaques in atherosclerosis

A. A Intimacy is similar to "shagreen skin"


B. Localized in the media
C. Do not rise above intimacy
D. Localized in adventitia
E. * Rise above intimacy
120. A type of calcinosis in atherosclerotic "plaques".
A. Metabolic
B. Primary
C. Metastatic
D. Secondary
E. * Dystrophic
121. Nature foam cells in atherosclerotic plaques
A. Granulocytes
B. Eosinophils
C. Lymphocytes
D. Plasmocytes
E. * Macrophages
122. Morphogenetic factors of parietal thrombosis at atherosclerosis.
A. Laminar flow of blood
B. Changing the rheological properties of the blood
C. Sclerosis
D. Deposition of calcium into a plaque
E. * Turbulent blood flow in the plaque area
123. Coronary artery supplying blood to the anterior part of the inter-ventricular septum is which of the
following:
A. Left main coronary artery
B. Left anterior descending artery
C. Left circumflex coronary artery
D. Proximal marginal coronary artery
E. * Right coronary artery
124. Coronary artery suppying blood to the posterior portion of the interventricular septum is which of the
following:
A. Left main coronary artery
B. Left anterior descending artery
C. Left circumflex coronary artery
D. Proximal marginal coronary artery
E. * Right coronary artery
125. Macroscopic characteristic of fatty spots and strips in atherosclerosis.
A. Rising over intimacy
B. Covered with ulcers
C. Localized in the media
D. Localized in adventitia
E. * Do not rise above intimacy
126. Changes in the aging heart are all of the following, except:
A. Epicardial coronary artery tortuosity
B. Decreased left ventricular cavity size
C. Decreased subepicardial fat

D. Brown atrophy of myocardium


E. * Increased number of inflammatory cells in myocardium
127. Blood vessels affected in systemic hypertension are which of the folowing:
A. Aorta and large arteries only
B. Aorta only
C. Large arteries and large veins
D. Venules
E. * Arterioli
128. The antihypertensive substances produced in the kidney are all of the following, except:
A. Prostaglandines
B. Urinary kallikrein-kinin system
C. Platelet-activating factor
D. Nitric oxide
E. * Thromboxane
129. What is coronary heart disease:
A. Myocardial disease due to exogenous intoxication
B. Myocardial disease due to endogenous intoxication
C. Myocardial disease due to exposure to infections
D. Myocardial disease due to metabolic disorders
E. * Myocardial disease due to absolute or relative failure of the coronary circulation
130. Specify the type of heart attack, depending on the timing of the development since the first signs of
ischemia:
A. Dystrophic
B. Necrotizing
C. Metabolic
D. Chronic
E. * Acute
131. What fatal complication may develop chronic thrombosis of the aneurysm of the heart:
A. Gangrene
B. Adrenal insufficiency
C. Chronic renal failure
D. Renal insufficiency
E. * Cerebral infarction
132. Name form of acute ischemic heart disease:
A. Metabolic myocardial injury
B. Fatty degeneration of the myocardium
C. Interstitial myocarditis
D. Productive myocarditis
E. * Acute ischemic dystrophy
133. Name stage of a myocardial infarction
A. Late
B. Metabolic
C. Compensatory
D. Early
E. * Necrotic
134. Name the morphological expression of chronic the ischemic heart disease:

A. Fatty degeneration of the myocardium


B. Heart obesity
C. Myocardial hypertrophy
D. Heart defect
E. * Diffuse small-focal cardiosclerosis
135. How deadly complications can occur in patients with left side chronic heart aneurism with parietal
thrombosis?
A. Pulmonary infarction
B. Anasarca
C. Fatty degeneration of the myocardium
D. Pulmonary embolism
E. * Spleen infarction
136. The favorable outcome of myocardial infarction:
A. Cyst rupture
B. Purulent fusion
C. Fusion of myocardium
D. A heart attack
E. * Scarring
137. Changes in muscle cells at the stage of acute ischemia
A. Lysis of nuclei
B. Coagulation cytoplasm
C. Lysis of the cytoplasm
D. Increased amount of glycogen
E. * Disappearance of glycogen
138. The most common cause of death in patients with chronic ischemic heart disease:
A. Hemopericardium
B. Cardiogenic shock
C. Asystole
D. Acute heart failure
E. * Chronic heart failure
139. The type of myocardial infarction, depending on the developmental date since the first signs of
ischemia.
A. Chronic
B. Hemorrhagic, ischemic
C. Acute
D. Subacute
E. * Recurrent
140. Specify the type of heart attack, depending on the developmental date since the first symptoms of
ischemia.
A. Focal
B. Necrotic
C. Ischemic
D. Transmural
E. * Primary
141. Macroscopic changes in the heart when acute myocardial infarction.
A. In a myocardium a necrosis of a triangular shape
B. In the myocardium, many small sclerosis centers

C. In the myocardium, the center of a sclerosis of irregular shape


D. The heart has a tiger's look
E. * In a myocardium a necrosis center of irregular shape
142. The morphological changes in the kidney during malignant form of hypertension.
A. Acute nephritis
B. Tubulointerstitial nephrosis
C. Cysts
D. Purulent inflammation
E. * Fibrinoid necrosis of arterioles and capillary loops of the kidney’s glomeruli
143. The cause of death in malignant nephrosclerosis
A. Acute heart failure
B. Chronic heart failure
C. Acute liver failure
D. Chronic renal failure
E. * Acute renal failure
144. Morphology changes in renal hypertension benign course.
A. Heart attack
B. Regeneration glomerular hypertrophy
C. Glomeruli hypertrophy
D. Fibrinoid necrosis of arterioles and glomeruli
E. * Glomeruli sclerosis
145. Morphology preclinical stages of hypertension.
A. Compensatory hypertrophy of the left ventricle
B. Spasm of arterioles
C. Hypertrophy of the muscle layer of arteries
D. Hypertrophy elastic structures arterioles
E. * All the above named
146. Macroscopic characteristics of renal hypertension benign course.
A. Greasy kidney
B. The kidneys are enlarged in size
C. Red
D. Pale
E. * The surface of fine-grained
147. Clinical expression kidney form hypertension.
A. Acute heart failure
B. Chronic heart failure
C. Acute renal failure
D. * Chronic renal failure
E. Amyloidosis
148. Changes in large vessels in hypertension in gross examination.
A. Arteriosclerosis
B. Liposclerosis
C. Calcification
D. Sclerosis
E. * Fatty spots and stripes
149. The clinical syndrome of malignant hypertension includes all ofthe following symptoms, except:

A. Papilledema
B. Retinal hemorrhages
C. Diastolic pressure >120 mmHg
D. Renal failure
E. * Systolic pressure >120 mmHg
150. The humoral constriction factors that lead to the peripheral resistence in hypertension are all of the
following, except:
A. Angiotensin II
B. Catecholamines
C. Leukotriens
D. Thromboxanes
E. * Prostaglandines
151. All of the following factors may contribute to vasoconstriction, except:
A. Increased sensitivity of vascular smooth muscles to constricting agent
B. Behavioral factors
C. Neurogenic factors
D. Increased release of vasoconstric-tive agents
E. * Acute coagulation factor
152. The malignant hypertension can be identified microscopically by all of the following features,
except:
A. Onion-skin concentric laminated thickening of the arteriolar walls
B. Progressive narrowing of the arteriolar lumina
C. Perivascular fibrosis
D. Fibrinoid necrosis of the arteriolar walls
E. * Cholesterol-laden "foam cells" in the arteriolar walls
153. On gross examination, the kidneys in systemic chronic hypertension are characterized by all of the
following features, except:
A. Reduced in size and weight
B. Fine granularity on cortical surface
C. Grain leather-like cortical surface
D. Narrowed cortical surface on section
E. * Hemorrhages on cortical surface
154. The pathologic process in arterioles in malignant hypertension is referred to as:
A. Endothrombovasculitis
B. Thromboangitis obliterans
C. Necrotizing atherosclerosis
D. Productive arteriolitis
E. * Necrotizing arteriolitis
155. Nonbacterial thrombotic endocarditis is characterized by all of the following, except:
A. Mitral and/or aortic valve involvement
B. Systemic emboli development
C. Small (1 to 5 mm), sterile vegetations
D. Vegetations along the line of valve closure
E. * Tricuspid valve involvement
156. Microscopically, most cases of acute myocarditis are characterized by all of th following, except:
A. Nonspecific inflammatory infiltrate
B. Variable amount of interstitial edema

C. Variable degree of myofiber degeneration


D. Variable degree of myofiber necrosis
E. * Marked fibrosis of interstitium
157. The correct order of histologic phases in the development of Aschoff bodies is which of the
following:
A. Early — pathognomonic — intermediate
B. Granulomatous — late — exudative
C. Acute — chronic — healed
D. Nonspecific — specific
E. * Exudative - proliferative – fibrous
158. The valvular effects of chronic rheumatic heart disease are all of the following, except:
A. Thickened and deformed valve leaflets
B. Valvular calcification
C. Thickened and shortened chordae tendineae
D. "Fish-mouth" appearance of mitral valve
E. * Valvular abscess formation
159. Left-sided heart failure is most often caused by all of the following, except:
A. Ischemic heart disease
B. Nonischemic heart disease
C. Hypertension
D. Aortic and mitral valvular diseases
E. * Brown atrophy of myocardium
160. The nonrheumatic degenerative calcific aortic stenosis is characterized by all of the following,
except:
A. Heaped-up calcified masses within aortic cusps
B. Primary involvement of the valvular fibrosa
C. Distorted cuspal architecture
D. Not involved free cuspal edges
E. * Lipid deposition and cellular proliferation
161. The mitral annular calcification is characterized by all of the following, except:
A. Absence of inflammatory changes
B. Irregular, stony hard nodule appearance
C. Possible thromboembolic complications
D. Infective endocarditis
E. * Severely impaired valvular function
162. The Aschoff bodies consist of all of the following, except:
A. Lymphocytes (primarily T cells)
B. Anitschkow cells
C. Plasma cells
D. Fibrinoid degeneration
E. * "Foam cells"
163. Infective endocarditis is characterized by all of the following, except:
A. Microbic invasion into the heart valves
B. Destructive vegetation formation
C. Mural thrombosis
D. Destruction of the underlying cardiac tissues
E. * Amyloid mass deposition

164. Systemic embolism in infective endocarditis may cause infarcts in all of the following organs, except:

A. Brain
B. Kidneys
C. Heart
D. Spleen
E. * Lungs
165. Microscopycally the vegetations of subacute infective endocarditis are characterized by all of the
following, except:
A. Fibrosis
B. Microbic body calcifications
C. Chronic inflammatory infiltrate
D. Granulation tissue at vegetation's bases
E. * Granuloma formation
166. Microscopically, chronic rheumatic heart disease is characterized by all of the following, except:
A. Diffuse fibrosis of leaflets
B. Neovascularization of leaflets
C. Aschoff bodies in perivascular connective tissue
D. Destruction of original leaflet architecture
E. * Acute purulent inflammation
167. Fibrosis bridging across the valvular commissures and calcification in chronic rheumatic heart
disease cause which of the following:
A. Granulomatosis
B. "Tigered effect"
C. Fibrofatty atheromas
D. Starry sky appearance
E. * Fish mouth or buttonhole stenosis
168. The morphological and clinical effects of pure (isolated) right-sided heart failure are all of following,
except:
A. Anasarca
B. Congestive hepatomegaly
C. Ascites
D. Pleural and pericardial effusions
E. * Cerebral hematoma
169. Etiologically and pathogenetically, rheumatic fever and rheumatic heart disease are characterized by
all of the following, except:
A. Initial attack of illness some weeks after streptococcal infection
B. Elevated serum titers of antibodies to streptolysin and hyaluronidase
C. Sterile tissue lesions not resulting from direct bacterial invasion
D. Recurrent acute illness following the streptococcal infection
E. * Decreased serum protein levels
170. Heart lesions in acute rheumatic fever are called as which of the following:
A. Tuberculoma
B. Foreign bodies
C. Gumma
D. Mitral stenosis
E. * Aschoff bodies

171. Nonembolic complications of infective endocarditis include all of the following, except;
A. Valvular insufficiency or stenosis with cardiac failure
B. Myocardial ring abscess
C. Suppurative pericarditis
D. Endocarditis of artificial valves
E. * Acute myocardial infarction
172. Disturbance of connective tissue in rheumatic diseases includes:
A. cell reaction (granulematosis) and sclerosis
B. hyaline droplets dystrophy
C. amyloidosis
D. hydropic dystrophy
E. * Hyalinosis
173. In rheumatism sensitizing factor antibodies:
A. B-hemolytic streptococcus, type B
B. B-hemolytic streptococcus type AB
C. B-hemolytic staphylococcus type D
D. B-hemolytic staphylococcus type C
E. * B-hemolytic streptococcus typeA
174. What antibodies are sensitizing factor in rheumatic conditions:
A. B-hemolytic streptococcus, type B
B. B-hemolytic streptococcus type AB
C. B-hemolytic streptococcus type D
D. B-hemolytic staphylococcus type C
E. * B-hemolytic staphylococcus type A
175. B-hemolytic streptococci type A has affinity with antigens antibodies against
A. liver
B. kidney
C. tonsils
D. connective tissue of mediastinum
E. * connective tissue of heart
176. Kinship antibodies against B-hemolytic streptococci type A antigens from connective tissue of the
heart explains the predominant lesion in patients with rheumatism:
A. liver
B. only epikard
C. only endocard
D. more pericardiocentesis
E. * heart
177. In rheumatoid arthritis in patients with damage mainly to the connective tissue:
A. heart
B. kidney
C. renal capsule
D. liver
E. * capsule of the joints
178. Connective tissue structures are damaged in rheumatoid arthritis:
A. heart
B. kidney

C. renal capsule
D. liver
E. * capsule of the joints
179. In the pathogenesis of rheumatoid arthritis are important:
A. none ANSWER among following is correct
B. immune complexes, where there are different classes of immunoglobulins (Ig N, Ig Q, Ig B)
C. complexes, where there are different classes of immunoglobulins (Ig Y, Ig H, Ig L)
D. immune complexes, where the antigen is a globulin of different classes (Ig M, Ig G, Ig A)
E. * immune complexes, where the antibodies are different classes of immunoglobulins (Ig M, Ig G, Ig A)

180. In systemic lupus erythematosus is violated:


A. exchange of DNA and antigens to components formed the nucleus and cytoplasm
B. exchange of RNA and formation of antibodies to components of the nucleus and cytoplasm
C. exchange of RNA and formation of antibodies
D. exchange of DNA and antigens to components formed the nucleus and cytoplasm
E. * exchange of DNA and formation of antibodies to components of the nucleus and cytoplasm
181. What disease does not belong to a group of rheumatic diseases:
A. Periarthritis nodosa
B. Rheumatic fever
C. Dermatomyositis
D. Scleroderma
E. * Hypertension
182. Bekhterev disease - a chronic rheumatic disease with the damage:
A. Articular apparatus
B. Large joints
C. Small and large joints
D. Heart
E. * The small joints of the spine
183. What disease refers to a group of rheumatic:
A. Atherosclerosis
B. Obesity
C. Myocardial infarction
D. Amyloidosis
E. * SLE
184. What type of necrosis can be found in tuberculous granuloma?
A. Coagulation necrosis
B. Liquefactive necrosis
C. Enzymatic fat necrosis
D. Fibrinoid necrosis
E. * Caseous necrosis
185. Mycobacterium tuberculosis is characterized by all of the following features, EXCEPT:
A. Red colored in acid-fast staining
B. Aerobic
C. Non-spore-forming
D. Nonmotile
E. * Pili-forming

186. Named the least frequent location for miliary extrapulmonary seeding:
A. Bone marrow
B. Kidneys
C. Liver
D. Spleen
E. * Uterus
187. Miliary tuberculosis is associated with which of the following pathologic conditions:
A. Reinfection
B. Localized caseation in the lungs
C. Localized caseation in the lymph node
D. Primary infection
E. * Hematogenous tuberculosis
188. Secondary tuberculosis is characterized by all of the following features, EXCEPT:
A. Caseous necrosis and cavities in the lung
B. Progression directly into the disseminated disease
C. Reinfection with Mycobacteria
D. Reactivation of dormant disease
E. * Primary focus in the lung
189. Caseous focus in tuberculosis may progress into a cavity in which of the following pathologic
conditions:
A. Reactivation of dormant disease
B. Reinfection
C. Caseous pneumonia
D. Lobar exudate consolidation
E. * Erosion into the bronchiole (drainage)
190. Growth and multiplication of the tubercle bacilli in cavernous fibrocaseous tuberculosis are favored
by which of the following pathologic conditions:
A. Lymphatic drainage
B. Progressive hypoxia
C. Increased perfusion
D. Sludging of blood in alveolar capillaries
E. * Increased oxygen tension
191. The cavity in cavernous fibrocaseous tuberculosis is characterized by all of the following features,
EXCEPT:
A. Localized in the apex of the lung
B. Lined by yellow-grey caseous material
C. Walled by fibrous tissue
D. Drained by bronchus
E. * Filled with suppurative exudate
192. Complications of tuberculosis osteomyelitis include all of the following, EXCEPT:
A. Tuberculosis arthritis
B. Sinus tract formation
C. Cold abscess formation
D. Amyloidosis
E. * Caseation in the lung
193. The intestine may be affected in which of the following forms of tuberculosis:
A. Secondary tuberculosis

B. Cavernous fibrocaseous tuberculosis


C. Dormant tuberculosis
D. Miliary tuberculosis
E. * Primary tuberculosis complex
194. Healed lesions in primary tuberculosis include all of the following, EXCEPT:
A. Fibrous incapsulation
B. Focal pleural adhesions
C. Fibrocalcific scar
D. Foci of ossification
E. * Caseous pneumonia
195. Tuberculoma is macroscopically characterized by all of the following features, EXCEPT:
A. Intraparenchymal single mass
B. Greyish-white appearance
C. Well-circumscribed mass
D. Several centimeters in diameter
E. * Several millimeters in diameter
196. Tuberculous salpingitis can be found in which of the following forms of tuberculosis:
A. Secondary tuberculosis
B. Cavernous fibrocaseous tuberculosis
C. Dormant lesions
D. Primary tuberculosis
E. * Miliary tuberculosis
197. The cells playing the most important role in chronic tuberculous inflammation are which of the
following:
A. Leukocytes
B. Eosinophils
C. Erythrocytes
D. Plasma cells
E. * Macrophages
198. The miliary lung tuberculosis is characterized by which type of inflammation:
A. Serous
B. Fibrinous
C. Suppurative
D. Hemorrhagic
E. * Granulomatous
199. Complications of primary tuberculosis:
A. Heart disease
B. Acute pyelonephritis
C. Scleroderma
D. Pneumosclerosis
E. * Meningitis
200. Influenza virus of type A infects all of the following, EXCEPT:
A. Horses
B. Humans
C. Pigs
D. Birds

E. * Dogs
201. The incubation period of the flu continues:
A. 7-10 days
B. 10-20 days
C. 30 days
D. 12-24 years
E. * 2-4 days
202. The electron micrograph of epithelial cells infected by influenza virus is characterized by all of the
following, EXCEPT:
A. Presense of influenza virus particles
B. Hyperchromatic and enlarged nuclei
C. Dilation of cysterns of endoplasmic reticulum
D. Large vacuoles
E. * Displasia of epithelial cells
203. Influenza tracheitis is characterized by all of the following, EXCEPT:
A. Hyperemic and swollen mucosa
B. Narrowing of trachea lumen
C. Pinpoint hemorrhages
D. Grey-yellow mucosa covering films within trachea
E. * Purulent exudate
204. Morphology of influenza pneumonia is characterized by all of the following features, EXCEPT:
A. Diffuse neutrophil infiltration of the bronchiolar walls
B. Dense fibrin-rich neutrophil exudate into adjacent alveoli
C. Foci of necrosis and hemorrhages in lung tissue
D. Foci of necrosis and ulceration in bronchiolar mucosa
E. * Granuloma formation in bronchiolar walls
205. Figuratively the influensa pneumonia is often referred to as:
A. Hemosiderosis of the lung
B. Silicosis
C. Lung infarct
D. Farmer's lung
E. * Multicolored lung
206. Morphologically, measles pneumonia is characterized by all of the following, EXCEPT:
A. Diffuse neutrophil infiltration of the bronchiolar walls
B. Destruction of some fragments of bronchiolar walls and squamous cell metaplasia
C. Exudate with giant cells in the adjacent alveoli
D. Interstitial lung inflammation
E. * Large areas of hemorrhage in lung tissue
207. A histological specimen shows terminal secretory parts of glands made by conic cells with basophilic
cytoplasm and a roundish nucleus in the centre. Specify the type of terminal secretory parts by the
type of secretion:
A. Seromucous
B. Sebaceous
C. Combined
D. Mucous
E. * Serous

208. A 35-year-old patient complains of pain in the upper jaw, bleeding, a slight loosening of teeth. He has
been diagnosed with periodontitis. What is a typical pathological process in this case?
A. Bleeding
B. Caries
C. Pain
D. Redness
E. * Inflammation
209. B-cells secreting IgA in gastrointestinal infections are localized in which of the following parts of the
gastrointestinal tract:
A. Muscle layer
B. Submucosa
C. Serosa
D. Regional lymphatic nodes
E. * Mucosa-associated lymphoid tissue (MALT)
210. Haw many stages of changes of Peyer's patches does the pathogeny of enteric fever include?
A. 2
B. 3
C. 4
D. Hasn't stage development
E. * 5
211. Haw long does everyone of the stage of enteric fever continue mainly?
A. 3 days
B. 1 month
C. 2 weeks
D. 3 weeks
E. * 1 week
212. Haw many days does the incubative period of enteric fever continue mainly?
A. to 2 years
B. 2-3 weeks
C. 1 month
D. 1.5-2 months
E. * 10-14 days
213. Where do the main changes situated at the typhoid fever?
A. in retroperitoneal space
B. in submucous layer
C. in adventitia membrane
D. in serosal membrane (peritoneum)
E. * in mucous membrane
214. Which stage of changes between enumerated at the dysentery is the third?
A. fibrinous colitis
B. catarrhal colitis
C. no indicated here
D. healing of the wound
E. * ulcer formation
215. The preventive measures against gastrointestinal infections are all of the following, EXCEPT:
A. Disposal of waste and vermin
B. Cooking food

C. Cleaning drinking water


D. Hand washing
E. * Killing the parasites which are the transmitters of these infections
216. Normal defenses against ingested gut pathogens include all of the following, EXCEPT:
A. Lytic pancreatic enzymes and bile detergents
B. Secreted IgA antibodies
C. Acid gastric juice
D. Viscous mucous layer covering the gut
E. * Activated C3 complement
217. The term "dysentery" refers to diarrhea associated with all of the following symptoms, EXCEPT:
A. Abdominal cramping
B. Tenesmus
C. Stools containing blood
D. Stools containing pus and mucus
E. * "Rice-water" stools
218. Shigellae cause all of the following pathologic features, EXCEPT:
A. Hemorrhagic colitis
B. Hemolytic-uremic syndrome
C. Damage of endothelial cells in the colon microvasculature
D. Stools containing blood, pus and mucus
E. * Acute tubular necrosis
219. The pathogenic mechanisms of Vibrio cholera infection are all of the following, EXCEPT:
A. Adherence of enterotoxigenic organisms to the enterocyte
B. Changes in the underlying cell cytoplasm
C. Effacement of the apical enterocyte membrane
D. Destruction of the microvillus brush border
E. * Ability to synthesize prostaglandin
220. Measles virus is transmitted by which of the following:
A. Milk
B. Food
C. Respiratory droplets
D. Feces
E. * Blood
221. Measles virus multiplies inside all of the following cells, EXCEPT:
A. Upper respiratory epithelial cells
B. lymphocytes
C. Macrophages
D. T lymphocytes
E. * Hepatocytes
222. "Black measles" is characterized by which of the following:
A. Multiple nevi
B. Hypermelanosis
C. Icterus
D. Hyperchromatosis
E. * Hemorrhages
223. The measles rash results from which of the following:
A. Nonimmune inflammation
B. Delayed — type hypersensitivity
C. Systemic immune complex reaction
D. Antibody-mediated cellular dysfunction
E. * T-cell - mediated immunity
224. Ulcerated mucosal lesions in the oral cavity near the opening of Stensen ducts are referred to as:
A. Vegetations
B. Luschka spots
C. Burkitt spots
D. Paget spots
E. * Koplik spots
225. The term "Koplik spots" referes to which of the following:
A. Ulcerated gastric mucosal lesions
B. Foci of granulomatous inflammation of the oral cavity
C. Foci of purulent inflammation in the skin
D. Ulcerated esophageal mucosal lesions
E. * Ulcerated mucosal lesions in the oral cavity
226. Pathological feature that can be found in the heart in diphtheria is which of the following:
A. Fibrinous pericarditis
B. Heart aneurysm
C. Bacterial endocarditis
D. Myocardial infarction
E. * Toxic myocarditis
227. Nervous system pathology in diphtheria includes which of the following:
A. Intracerebral hemorrhage
B. Ischemic necrosis
C. Hydrocephalus
D. Encephalitis
E. * Polyneuritis
228. The most common cause of death in patients with diphtheria is which of the following:
A. Intracerebral hemorrhage
B. Chronic heart failure
C. Chronic lung failure
D. Acute tubular necrosis
E. * Acute heart failure
229. The disease causing predominant derangement of the upper airways is which of the following:
A. Bacterial pneumonia
B. Tuberculosis
C. Meningococcal nasopharyngitis
D. Cholera
E. * Diphtheria
230. Diphtheria damage of the larynx is characterized by which of the following types of inflammation:
A. Serous
B. Catarrhal
C. Hemorrhagic
D. Granulomatous
E. * Fibrinous
231. Consequences of diphtheria include all of the following, EXCEPT:
A. Hyperplasia of the spleen
B. Fatty myocardial changes
C. Focal necroses of the parenchyma of the organs
D. Polyneuritis
E. * Systemic hemosiderosis
232. Exotoxin in diphtheria damages all of the following organs, EXCEPT:
A. Thyroid gland
B. Nervous system
C. Adrenals
D. Kidneys
E. * Heart
233. Pathological feature in the skin found in meningococcemia is which of the following:
A. Erythematous rash
B. Koplic spots
C. Furuncle
D. Impetigo
E. * Hemorrhagic rash
234. Diphtheria is characterized by the derangement of all of the following organs, EXCEPT:
A. Nasopharynx
B. Oropharynx
C. Larynx
D. Trachea
E. * Esophagus
235. Microscopic features in diphtheria include all of the following, EXCEPT:
A. Neutrophilic infiltration
B. Vascular congestion
C. Interstitial edema
D. Fibrin exudation
E. * Hyaline deposition
236. Meningococcal meningitis is most commonly characterized by which of the following types of
inflammation:
A. Serous
B. Catarrhal
C. Fibrinous
D. Granulomatous
E. * Purulent
237. The stain used to identify the causative organism in pyogenic meningitis is which of the following:
A. Hematoxylin and eosin
B. PAS-reaction
C. Weigert's
D. Congo red
E. * Gram
238. Complications of pyogenic meningitis include all of the following, EXCEPT:
A. Leptomeningeal fibrosis

B. Hydrocephalus
C. Adhesive arachnoiditis
D. Focal encephalitis
E. * Intracerebral hemorrhage
239. The complications of scarlet fever include all of the following, EXCEPT:
A. Poststreptococcal glomeru-lonephritis
B. Retropharyngeal abscess
C. Purulent mastoiditis
D. Phlegmon of the neck
E. * Acute poststreptococcal hepatitis
240. All of the following pathologic processes can be found in the second period of scarlet fever,
EXCEPT:
A. Vasculitis
B. Acute glomerulonephritis
C. Arthritis
D. Endocarditis
E. * Amyloidosis
241. Renal pathology that can be found in patients with scarlet fever is which of the following:
A. Lipoid nephrosis
B. Pyelonephritis
C. Amyloidosis
D. Glomerulosclerosis
E. * Glomerulonephritis
242. The causes of death in meningococcemia include all of the following, EXCEPT:
A. Acute adrenal insufficiency
B. Bacterial shock
C. Acute tubular necrosis
D. Acute heart failure
E. * Cerebral cachexia
243. Acute adrenal insufficiency syndrome is also referred to as:
A. Budd-Chiari syndrome
B. Kimmelstill-Wilson syndrome
C. Hamman-Rich syndrome
D. Zollinger-Ellison syndrome
E. * Waterhouse - Friderichsen syndrome
244. The causative agent of scarlet fever is:
A. Greening streptococcus
B. Diplococcus
C. Staphylococcus aureus
D. Intestinal sticks
E. * Streptococcus group A
245. All of the following clinical features are likely to be found in nephrotic syndrome, EXCEPT:
A. Proteinuria
B. Hypoalbuminemia
C. Hyperlipidemia
D. Edema

E. * Hematuria
246. All of the following statements correctly describe renal artery stenosis, EXCEPT:
A. It is an uncommon form of hypertension
B. It is the most common curable form of hypertension
C. It is usually caused by atherosclerotic plaque
D. It produces high renin levels in the venous blood of the ischemic kidney
E. * It is treated by hemodialysis
247. All of the following statements correctly describe analgesic abuse nephropathy, EXCEPT:
A. It is characterized by tubulo-interstitial component
B. It is often caused by phenacetin
C. It causes inability to concentrate urine
D. It often improves with drug withdrawal
E. * It predisposes to the development of renal cell carcinoma
248. All of the following statements regarding Goodpasture's syndrome are true, EXCEPT:
A. Patients present with hemoptysis and hematuria
B. Death occurs due to uremia and pulmonary hemorrhage
C. Electron microscopy shows the absence of electron-dense deposits
D. Immunofluorescence reveals linear deposits of IgG in the glomeruli
E. * Immunofluorescence reveals granular deposits of IgG in the glomeruli
249. All of the following statements regarding postinfectious glomerulonephritis are true, EXCEPT:
A. The disease follows streptococcus infection
B. Electron microscopy shows large subendothelial immune-type deposits
C. The histologic picture is that of diffuse proliferative glomerulonephritis
D. The clinical picture is characteristic of acute nephritis
E. * Most affected children develop chronic renal failure
250. An immunofluorescence-stained kidney specimen from a patient with poststreptococcal
glomerulonephritis is likely to show which of the following:
A. Linear deposits of Ig G
B. Granular deposits of IgA
C. Linear deposits of streptococcal antigen
D. Granular deposits of streptococcal antigen
E. * Granular deposits of Ig G
251. At extracapillar productive glomerulonephritis half moons are formed by
A. Red corpuscles
B. Neutrophilic leucocytes
C. Red corpuscles and neutrophilic leucocytes
D. Exfoliated endothelium and fibrin
E. * Exfoliated renal epithelium and fibrin
252. Benign nephrosclerosis is characterized by all of the following, EXCEPT:
A. Narrowing of the lumen of the arterioles and small arteries
B. Thickening and hyalinization of the vessels' walls
C. Foci of tubular atrophy
D. Deposition of collagen within the Bowman space
E. * Deposition of amyloid within the Bowman space
253. Changes in a colon at uremia are
A. Granulomatous inflammation

B. Stricture|
C. Polyposis|
D. Ulcerous colitis
E. * Fibrinous [diphtheritic] colitis
254. Diabetes mellitus is associated with all of the following renal disorders, EXCEPT:
A. Diffuse glomerulosclerosis
B. Nodular glomerulosclerosis
C. Benigh nephrosclerosis
D. Acute pyelonephritis
E. * Urate nephropathy
255. Ultrastructural changes in children primary nephrotie syndrome involve which of the following
glomerular elements:
A. Endothelium
B. Mesangium
C. Bloodvessels
D. Basement membrane
E. * Podocytes
256. Uremia is associated with all of the following abnormalities, EXCEPT:
A. Peripheral neuropathy
B. Gastritis
C. Pericarditis
D. Diffuse alveolar damage
E. * Polycythemia
257. What enumerate illness from the listed below relates to the large mottled kidney
A. Amiloidosis of kidney
B. Chronic pyelonephritis
C. Nephrolithiasis
D. Chronic glomerulonephritis
E. * Subacute glomerulonephritis
258. What disease can be complicated by amyloidosis of kidneys:
A. Atherosclerosis
B. Heart ischemic disease
C. Croupous pneumonia
D. Hypertension
E. * Fibrous cavernous tuberculosis of lungs
259. Most forms of chronic renal failure produce increased serum levels of all of the following substances,
EXCEPT:
A. Renin
B. Aldosterone
C. Phosphate
D. Parathormone
E. * Calcium
260. Mesangial cells can be characterized by all of the following properties, EXCEPT:
A. Ingestion of macromolecules
B. Connection with Lacis cells
C. Ability to contract
D. Production of basement membrane proteins

E. * Production of renin
261. In immunologicahy mediated glomerulonephritis all of the following cells contribute to the
glomerular injury, EXCEPT:
A. Macrophages
B. Platelets
C. Neutrophils
D. Mesangial cells
E. * Mast cells
262. Systemic lupus erythematosus gives rise to all of the following patterns of glomerular injury,
EXCEPT:
A. Focal proliferative glomerulonephritis
B. Diffuse membranous glomerulonephritis
C. Diffuse proliferative glomerulonephritis
D. Mesangial proliferative glomerulonephritis
E. * Lipoid nephrosis
263. Renal diseases producing systemic hypertension include all of the following, EXCEPT:
A. Acute glomerulonephritis
B. Chronic glomerulonephritis
C. Chronic pyelonephritis
D. Renal vasculitis
E. * Renal amyloidosis
264. Histologic features of malignant nephrosclerosis include all of the following, EXCEPT:
A. Fibrinoid necrosis of arterioles
B. Medial thickening of arterioles
C. Renal artery thrombosis
D. Focal renal parenchymal infarction
E. * Fibromuscular dysplasia of the renal artery
265. Which statement correctly characterizes membranous glomerulopathy?
A. It is the most common cause of nephrotie syndrome in children
B. Patients usually present with acute renal failure
C. It is characterized by diffuse proliferative glomerulonephritis
D. It is characterized by mesangial interposition phenomenon
E. * Electron microscopy demonstrates numerous subepithelial immunetype deposits
266. Subendothelial granular electron-dense deposits can be found in which of the following diseases:
A. Rapidly progressive glomerulonephritis
B. Side cell nephropathy
C. Membranous glomerulonephritis
D. Gouty nephropathy
E. * Systemic lupus erythematosus
267. Hydronephrosis is characterized by all of the following, EXCEPT:
A. Thinning of the renal parenchyma
B. Dilatation of the renal pelvis
C. Dilatation of the renal calyces
D. Progressive atrophy of the kidney
E. * Kidney infarct
268. Hydronephrosis is caused by all of the following, EXCEPT:
A. Large uterine leiomyoma

B. Renal calculi
C. Benign prostatic hypertrophy
D. Papillary transitional cell carcinoma of the ureter
E. * Chronic renal vein thrombosis
269. In nonobstructive chronic pyelonephritis the most common way for bacteria to gain entrance into the
kidney is which of the following:
A. Arterial bloodstream
B. The lymphatics
C. Venous bloodstream
D. Aberrant arteriovenous shunts
E. * Vesicoureteral reflux
270. All of the following statements correctly describe chronic pyelonephritis, EXCEPT:
A. It causes asymmetrically scarred kidneys
B. It is associated with vesicoureteral reflux in most cases
C. It may produce thyroidization of tubules
D. It is an important cause of secondary nephrosclerosis
E. * It spares the calyces and pelvis
271. All of the following conditions predispose to urolithiasis, EXCEPT:
A. Hyperparathyroidism
B. Gout
C. Proteus pyelonephritis
D. Enteric hyperoxaluria
E. * Sickle cell nephropathy
272. The factor least likely to cause acute pyelonephritis is which of the following:
A. Pregnancy
B. Nephrolithiasis
C. Catheterization of the bladder
D. Prostatic hypertrophy
E. * Septicemia
273. What pathologic condition of the kidneys is caused by mercury poisoning?
A. Renal papillary necrosis
B. Crescentic glomerulonephritis
C. Acute interstitial nephritis
D. Renal cell carcinoma
E. * Acute tubular necrosis
274. What develops in case of prolong obstruction of ureters by stone in a kidney
A. Heart attack
B. Gangrene
C. Glomerulonephritis
D. Amiloidosis
E. * Hydronephrosis
275. The tubular epithelial cells in acute tubular necrosis are characterized by all of the following
pathologic features, EXCEPT:
A. Karyolysis
B. Plasmolysis
C. Plasmorrhexis
D. Plasmocoagulation

E. * Tubulorrhexis
276. Hematuria is a characteristic clinical feature of all of the following diseases, EXCEPT:
A. Glomerulonephritis
B. Nephrolithiasis
C. Renal cell carcinoma
D. Bladder papilloma
E. * Malakoplakia
277. In the urinary tract obstruction all pathologic processes can be found, EXCEPT:
A. Dilatation of the pelvis and calyces;
B. Glomerular and tubular atrophy.
C. Interstitial fibrosis;
D. Interstitial inflammation;
E. * Acute tubular necrosis;
278. Microscopic examination revealed- necrosis of renal convoluted tubule epithelium, tubuloreksys,
stromal edema, ischemia cortical layer and the plethora of medulla. Name this pathological process.
A. Glomerulonephritis
B. Pyelonephritis
C. Policystosis
D. Chronic lung disease
E. * Necrotizing nephrosis
279. An autopsy of a male revealed a prostatic adenoma and large kidneys with sharply enlarged pelves
and calyces filled with some transparent fluid. Name the process in the kidneys.
A. Glomerulonephritis
B. Polycystic kidneys
C. Tuberculosis
D. Pyelonephritis
E. * Hydronephrosis
280. Name the disease of kidneys, which can be attributed to the group of tubulopathies:
A. Amyloidosis of kidneys
B. Chronic kidney insufficiency
C. Hydronephrosis
D. Pyelonephritis
E. * Acute kidney insufficiency
281. An autopsy has revealed that kidneys are enlarged, surface is large-granulary because of multiple
cavities with smooth wall, which are filled with clear fluid. What kidney disease did the patient have?
A. Infarction
B. Glomerulonephritis
C. Pyelonephritis
D. Necrotic nephrosis
E. * Polycystic kidney
282. In the urinary tract obstruction all pathologic processes can be found. EXCEPT:
A. Dilatation of the pelvis and calyces
B. * Ischemic tubular necrosis
C. Interstitial inflammation
D. Interstitial fibrosis
E. Glomerular and tubular atrophy

283. Indicate the localization of metastases of the uterus cancer?


A. Ovaries
B. Intestine
C. Lungs
D. Peritoneum
E. * Lymph nodes of the pelvis
284. What form of mastopathy transforms into cancer?
A. Tubular
B. Cyst
C. Fibrotic
D. Nonproliferative
E. * Proliferative
285. The most common tumor that occurs in the uterus
A. Rhabdomyoma
B. Abrikosov's tumor
C. Rabdomiosarcoma
D. lipoma
E. * Leiomyoma
286. The most common tumor that occurs in the breast
A. Rhabdomyoma
B. Leiomyoma
C. Liposarcoma
D. lipoma
E. * Fibromyoma
287. Advanced cervical carcinoma can extend by direct continuity to all of the following, EXCEPT:
A. Urinary bladder
B. Ureters
C. Rectum
D. Peritoneum
E. * Colon transversum
288. All of the following morphologic features characterize the ovary adenocarcinoma cells, EXCEPT:
A. Enlarged nucleoli
B. Atypical mitoses
C. Variation in size and shape
D. Hyperchromatic nuclei
E. * Nucleus inclusions
289. All statements concerning chorion-carcinoma are true, EXCEPT:
A. The tumor has a dimorphic pattern
B. Hemorrhages and necrosis are present
C. The tumor is malignant
D. The tumor is composed of cytotrophoblast and syncytiotrophoblast cells
E. * The tumor produces chorionic villi
290. All statements concerning leiomyomas are true, EXCEPT:
A. They regress or calcify after castration or menopause
B. They may undergo rapid increase in size during pregnancy
C. Their cause is unknown

D. They are found in 25% of reproductive women


E. * They do not respond to estrogens
291. An important factor in cervical oncogenesis is which of the following:
A. Herpes simplex
B. Herpes zoster
C. Human immunodeficiency virus
D. Respiratory syncytial virus
E. * HPV-infection
292. Chorioncarcinoma is commonly associated with which of the following:
A. Systemic hypertension
B. Obesity
C. Oral contraceptive steroid use
D. Diabetes mellitus
E. * Pregnancy
293. Conditions leading to endometrial hyperplasia include all of the following, EXCEPT:
A. Polycystic ovarian disease
B. Functioning granulosa cell tumors of the ovary
C. Excessive cortical function (cortical stroma hyperplasia)
D. Estrogen replacement therapy
E. * Endometriosis
294. Criteria to differentiate between benign and malignant tumors are all of the following, EXCEPT:
A. Maturity
B. Rate and character of growth
C. Metastases
D. Local invasion
E. * Edema
295. In a biopsy of сervix of a 26-year-old woman the diagnosis following was established:
pseudo-erosion. What microscopical changes has the pathologist revealed?
A. Cell-atypia of an epithelium of an mucosal epithelium
B. Keratinization of an epithelium
C. “Carcinomatous pearls”
D. Local inflammation and necrosis in mucosa
E. * Local changes of a stratified squamous epithelium on single-layer prismatic one
296. A 30-year-old woman had ectopic tubal pregnancy which finished with a location of a fetus in the
tubal cavity with bleeding. Call this pathology of pregnancy
A. Complete tubal abortion
B. Spontaneous abortion
C. Induced abortion
D. Criminal abortion
E. * Incomplete tubal abortion
297. The pathologist has found out in histological examination of a remote uterus a lot of glandular
formations with single cysts in myometrium, endometrium had usual structure. Described changes
are characteristic for …
A. Glandular hyperplasia of endometrium.
B. Adenocarcinoma of uterus.
C. Adenomatosis of a mucosa of a uterus
D. Leyomyosarcoma of uterus.

E. * Endometriosis
298. Distant hematogenic metastases of invasive cervical carcinoma occur in all of the following,
EXCEPT:
A. Liver
B. Lungs
C. Bone marrow
D. Kidney
E. * Lymph nodes
299. Endometrial hyperplasia is associated with which of the following:
A. High estrogenic stimulation with normal progestational activity
B. Normal estrogenic stimulation with increased progestational activity
C. High estrogenic stimulation and progestational activity
D. Normal estrogenic stimulation and progestational activity
E. * High estrogenic stimulation with diminished progestational activity
300. First hematogenous metastases of the endometrial carcinoma can be found in which of the following
organs:
A. Bones
B. Regional lymph nodes
C. Liver
D. Central nervous system
E. * Lungs
301. First lymphogenous metastases of the papillary mucinous cystadnocarcinoma of the ovary can be
found in which of the following organs:
A. Aortic lymph nodes
B. Liver
C. Lungs
D. Bones
E. * Pelvic lymph nodes
302. First metastases of the chorioncarcinoma can be found in which of the following organs:
A. Liver
B. Kidney
C. Bones
D. Brain
E. * Lung
303. Higher frequency of carcinoma of the endometrium is commonly associated with all of the following,
EXCEPT:
A. Obesity
B. Diabetes mellitus
C. Hypertension
D. Infertility (anovulatory cycles)
E. * Young age
304. The most common benign tumor of the ovary is which of the following:
A. Adenocarcinoma
B. Cystadenocarcinoma
C. Papilloma
D. Fibroadenoma
E. * Cystadenoma

305. The tumor composed of clusters of cuboid cytotrophoblast cells separated by streaming masses of
syncytiotrophoblast giant cells with marked atypia is referred to as:
A. Adenoma
B. Adenocarcinoma
C. Cystadenoma
D. Fibroadenoma
E. * Chorioncarcinoma
306. What pathological process the hypertrophy of prostatic gland is related to?
A. Chronic prostatitis
B. Acute prostatitis
C. Hyperfunction of sexual glands
D. Disorder of the urine outflow
E. * Hypofunction of sexual glands
307. Benign tumor arising from osteoblasts is called:
A. Chondroma
B. Adenoma
C. Fibroma
D. Papilloma
E. * Osteoma
308. All of the following genes (known as " tumor suppressor genes ") provide the negative control over
cell proliferation, except:
A. The p53gene
B. The DCC ("Deleted in Colon Cancer") gene
C. The Rb gene
D. The WT-1 gene
E. * The bcl-2 gene
309. Benign tumor arising from fibroblastic cells is called:
A. Chondroma
B. Adenoma
C. Osteoma
D. Papilloma
E. * Fibroma
310. Benign tumor arising from cartilaginous tissue is called:
A. Adenoma
B. Osteoma
C. Fibroma
D. Papilloma
E. * Chondroma
311. Dysplasia is characterized by all of the following, except:
A. Loss of cell uniformity
B. Hyperchromatic enlarged nuclei
C. Appearance of mitotic figures
D. Loss of cell architectural orientation
E. * Formation of tumor giant cells
312. Benign tumors composed of either cavernous spaces or serpentine capillary-like channels containing
blood or lymph are all the following, except:
A. Capillary lymphangiomas

B. Cavernous hemangiomas
C. Cavernous lymphangiomas
D. Capillary hemangiomas
E. * Capillary telangiectases
313. Type of Hemangioma:
A. Microcirculatory
B. Fibrous
C. Arterial
D. Granular-cell
E. * Venous
314. Differentiated synoviom arises from:
A. Capillries
B. Venous capillary
C. Arteriolas
D. Bone tissues
E. * Synovial elements of tendons
315. Schwannoma is formed:
A. Mallory’s bodies
B. Giant cells
C. Paccionian bodies
D. Melanin-producing cells
E. * Spinder-like cells
316. In the initial part of the upper lobe bronchus of the right lung some polyp-like formation, 1.0 cm in
diameter, with a superficial ulcer, consists of lymphocyte-like cells with hyperchromatic nuclei; the
cells grew in layers and bands. Choose rihgt diagnosis:
A. Undifferentiated large-cell carcinoma
B. Squamous cell carcinoma
C. Adenocarcinoma
D. Glandular squamous cell carcinoma
E. * Undifferentiated small-cell carcinoma
317. Аll of the following morphologic features characterize the adenocarcinoma cells, EXCEPT:
A. Variation in size and shape
B. Hyperchromatic nuclei
C. Enlarged nucleoli
D. Atypical mitoses
E. * Hypochromatic nuclei
318. The most significant histogenetic sign of well-differentiated squamosus carcinoma is which of the
following:
A. Necrotic foci
B. Areas of inflammation
C. Hemorrhages
D. Abscesses
E. * Keratinization
319. A loss of CD4+ T cells resulting from HIV infection leads to the decrease of all of the following,
EXCEPT:
A. Response to soluble antigens;
B. Specific cytotoxicity;

C. Killing of tumor cells;


D. Lymphokine secretion;
E. * Spontaneous secretion of IL-1.
320. A most common cause of endogenous hyperthyroidism is which of the following:
A. Hashimoto thyroiditis;
B. Subacute (granulomatous) thyroiditis;
C. Sporadic goiter.
D. Tuberculosis;
E. * Graves disease;
321. A point out the mechanism which is the basis for classification of the hypersensitivity diseases.
A. Byochemical;
B. Non-immunologic;
C. Chemical;
D. Physiologic.
E. * Immunologic;
322. A replacement of the normal secretory columnar epithelium by the nonfunctioning stratified
squamous epithelium may occur in all organs of the following, EXCEPT:
A. Bile ducts of the liver;
B. Excretory ducts of the salivary glands;
C. Excretory ducts of the pancreas;
D. Respiratory epithelium of the bronchi;
E. * Tubular epithelium of the kidney.
323. A type I hypersensitivity (anaphylactk type) is characterized by all of the following, EXCEPT:
A. Occurs in humans previously sensitized to theantigen;
B. Develops rapidly (within minutes);
C. Mediates by IgE antibodies in humans;
D. Mast cell or basophil degranulation.
E. * Develops slowly (within days);
324. Classical example of delayed type hypersensitivity is which of the following:
A. Arthus reaction;
B. Local anaphylaxis;
C. Complement-dependent reaction;
D. Systemic anaphylaxis
E. * Tuberculin reaction;
325. Predominant cells aggregating in the form of follicles in the spleen are which of the following:
A. CD4+T cells;
B. CD8+ T cells;
C. Macrophages;
D. Natural killer cells.
E. * B lymphocytes;
326. Secondary mediators released during Type I hypersensitivity (anaphylactic type) are all of the
following, EXCEPT:
A. Leukotriens;
B. Cytokines;
C. Prostaglandin D2.
D. Platelet-activating factor;
E. * Biogenic amines;

327. The all of the following cells are involved in immune response, EXCEPT:
A. Macrophages;
B. Natural killer cells;
C. T lymphocytes;
D. B lymphocytes.
E. * Eeosinophils;
328. The antinuclear antibodies in systemic lupus erythematosus are directed against all of the following,
EXCEPT:
A. DNA;
B. Histones;
C. Nonhiston proteins bound to RNA;
D. Nucleolar antigens.
E. * Nuclear membrane;
329. The cells which play an important role both in the induction and in the effector phase of immune
response are which of the following:
A. CD4+T cells;
B. CD8+ T cells;
C. B lymphocytes;
D. Natural killer cells.
E. * Macrophages;
330. The electron micrograph of a renal glomerular capillary loop from a patient with systemic lupus
erythematosus reveals which of the following:
A. Dense deposits in epithelial cells;
B. Dense deposits in capillary lumen;
C. Laminated deposits in subendothelial location;
D. Laminated deposits in epithelial cells.
E. * Dense deposits in subendothelial location;
331. Cardiac hypertrophy is characterized by all pathologic changes, EXCEPT:
A. Increased mass and size of the heart;
B. Increased protein synthesis;
C. Interstitial cardiac fibrosis;
D. Inadequate vasculature.
E. * Increased number of myocardial cells;
332. The proliferation of the glandular epithelium of a female during pregnancy is an example of:
A. Compensatory hyperplasia;
B. Pathologic hyperplasia;
C. Compensatory hypertrophy;
D. Hormonal hypertrophy.
E. * Hormonal hyperplasia;
333. Which type of wound healing can be termed as "keloid"?
A. Atrophic scar;
B. Scar with dysplasia;
C. Hyperplastic scar;
D. Hypotrophic scar;
E. * Hypertrophic scar
334. Polycythemia vera is a proliferative disorder of stem cells that:
A. Has an X-linked recessive mode of transmission

B. Is associated with high levels of erythropoietin


C. Produces abnormalities in the red cell series only
D. * Is rapidly fatal if untreated
E. Is compatible with a normal life span if treated
335. A deficiency of the red cell membrane component spectrin causes which of the following diseases:
A. Pernicious anemia
B. * Hereditary spherocytosis
C. Sickle cell anemia
D. Thalassemia major 5.
E. Iron - deficiency anemia
336. Acute leucosis is characterized by prolipheration of:
A. Developed healthy cells
B. Undifferentiated or patially differentiated blast cells
C. Erythrocytes
D. Leucocytes
E. * Endothelium
337. Acute leucosis is more commonly seen in
A. children
B. women
C. older people
D. young people
E. * Men
338. Iron-deficiency anemia is commonly associated with all of the following factors, EXCEPT:
A. Colon cancer
B. * Polycythemia
C. Gastrectomy
D. Normal menses
E. Pregnancy
339. Patients with polycythemia vera show all of the following characteristic features, EXCEPT:
A. High platelet count (thrombocytosis)
B. High hematocrit
C. * High erythropoietin level
D. High mean age at presentation
E. Higher than average chance of stroke or myocardial infarction
340. All of the following are stages of the atherosclerotic plaque development, except:
A. Fibrous plaque
B. Fibrofatty plaque
C. Atheromatous plaque
D. Fibrolipid plaque
E. * Fibrinous plaque
341. All of the following components can be found in the necrotic center of atheromatous plaque, except:
A. Cell debris
B. Cholesterol crystals
C. "Foam" cells
D. Calcium crystals
E. * Glycogen granules

342. Type of complicated macroscopic changes of intima arteries in atherosclerosis.


A. Fatty spots
B. Hyalineized plaques
C. Fibrosis plaques
D. Fatty stripes
E. * Intramural hematoma
343. Macroscopic characteristic of fatty spots and strips in atherosclerosis.
A. Rising over intimacy
B. Covered with ulcers
C. Localized in the media
D. Localized in adventitia
E. * Do not rise above intimacy
344. Material deposited in arteriolar walls in systemic hypertension is which of the following:
A. Amyloid
B. Glycogen
C. Cholesterol or cholesterol esters
D. Lipoprotein
E. * Hyaline
345. Specify the cause of death in patients with chronic ischemic heart disease:
A. Coma
B. Cardiogenic shock
C. Acute vascular insufficiency
D. Chronic obstructive pulmonary disease
E. * Chronic heart failure
346. Choose the reason of myocardial infarction:
A. Tumor compression of the coronary veins
B. Fatty degeneration of the myocardium
C. An inflammation of the coronary arteries
D. An inflammation of the coronary veins
E. * Coronary artery thrombosis
347. Name the morphological expression of chronic ischemic heart disease:
A. Myocardial infarction
B. Fatty degeneration of the myocardium
C. Heart defect
D. Obesity heart
E. * Chronic cardiac aneurysm
348. The cause of death in acute coronary heart disease:
A. Asphyxia
B. Chronic heart failure
C. Coma
D. Acute pulmonary insufficiency
E. * Cardiogenic shock
349. Name the morphological expression of chronic the ischemic heart disease:
A. Fatty degeneration of the myocardium
B. Heart obesity
C. Myocardial hypertrophy

D. Heart defect
E. * Diffuse small-focal cardiosclerosis
350. Name the morphological expression of acute ischemic heart disease:
A. Small-focal cardiosclerosis
B. Chronic aneurysm
C. Myocardial infarction
D. Heart defect
E. * Myocardial infarction
351. What complication can develop in patients with acute heart aneurysm:
A. A heart attack
B. Myocardial infarction
C. Cardiac hypertrophy
D. Cardiosclerosis
E. * Rupture of the ventricle
352. What are the stages of myocardial infarction?
A. Dystrophic
B. Metabolic
C. All of the above
D. Ischemic
E. * Necrotic
353. Name renal changes in hypertension.
A. Acute nephritis
B. Secondary wrinkled kidney
C. Atherosclerotic nephrosclerosis
D. Atrophy of the kidneys
E. * Primary wrinkled kidney
354. Name synonyms hypertension.
A. Symptomatic hypertension
B. Primary hypertension
C. Secondary hypertension '
D. Neurogenic hypertension
E. * Idiopathic hypertension
355. What are the pathological conditions can lead to symptomatic hypertension?
A. The disease of the CNS
B. Vascular disease
C. Kidney disease
D. Endocrine diseases
E. * All of the above
356. The leading factor in the pathogenesis of hypertension.
A. Hepatic factor
B. Kidney
C. Metabolic
D. Reception lipoprotein
E. * Hereditary
357. Symptoms of cardiac dysfunction include all of the following, except:
A. Failure of the pump itself

B. Obstruction of blood flow


C. Regurgitant blood flow
D. Disorders of cardiac conduction
E. * Disorders of blood coagulation
358. The renin-angiotensin system consists of all of the following components, except:
A. Renin
B. Angiotensin I
C. Plasma angiotensinogen
D. Angiotensin II
E. * Tissue angiotensinogen
359. The typical pathologic proces in arterioles and small arteries in systemic hypertension is referred to
as:
A. Hyaline atrophy
B. Hyaline hyperplasia
C. Fibroelastic hyalinosis
D. Hyaline atherosclerosis
E. * Hyaline arteriolosclerosis
360. The gene defects in enzymes involved in aldosterone metabolism in systemic hypertension lead to all
of the following effects, except:
A. Adaptive increase in aldosterone secretion
B. Increased salt resorption
C. Increased water resorption
D. Plasma volume expansion
E. * Increased lipoprotein resorption
361. Resulting from left-sided heart failure are all of these pathologic changes, except:
A. Brown induration of the lung
B. Hypoxic encephalopathy
C. Pulmonary edema
D. Prerenal azotemia
E. * Secondary amyloidosis
362. Secondary hypertension may be caused by all of the following diseases, except:
A. Acute glomerulonephritis
B. Gushing's syndrome
C. Pheochromocytoma
D. Coarctation of aorta
E. * Lobar pneumonia
363. Acute rheumatic carditis is characterized by all of the following, except:
A. Unremarkable gross appearance of the heart
B. The Aschoff bodies in myocardium
C. Diffuse nonspecific myocarditis
D. The Aschoff bodies in joints
E. * "Tigered effect" gross appearance
364. Non embolic complications of infective endocarditis include all of the following, except;
A. Valvular insufficiency or stenosis with cardiac failure
B. Myocardial ring abscess
C. Suppurative pericarditis
D. Endocarditis of artificial valves

E. * Acute myocardial infarction


365. The cardinal anatomic changes of the mitral (or tricuspid) valve in chronic rheumatic heart disease
are all of the following, except:
A. Fusion of the tendinous cords
B. Leaflet thickening
C. Commisural fusion
D. Shortening and thickening of the tendinous cords
E. * Leaflet softening
366. The clinical consequences of valvular dysfunction depend on all of the following, except:
A. Valve involved
B. Rate and quality of compensatory mechanisms
C. Degree of impairment
D. Rate of dysfunction development
E. * Sex of the patient
367. Systemic emboli in infective endocarditis may occur because of all of the following, except:
A. Involvement of aortic and mitral valves
B. Severe valvular damage
C. Numerous amount of vegetations
D. Friable nature of vegetations
E. * Chronic heart insufficiency
368. The Anitschkow cells are characterized by all of the following, except:
A. Abundant amorphophilic cytoplasm
B. Central round-to-ovoid nucleus
C. Central disposed chromatin (caterpillar cells)
D. Monocytic cytogenesis
E. * B-lymphocytic cytogenesis
369. What explains the predominant lesion in patients with rheumatism affinity antibodies against
B-hemolytic streptococci type A antigens from connective tissue of the heart:
A. liver
B. only epikard
C. only endocard
D. more pericardiocentesis
E. * heart
370. The disease is related to a group of rheumatic:
A. Atherosclerosis
B. Tuberculosis
C. Amyloidosis
D. Arthrosis
E. * Scleroderma
371. What is rheumatic disease - (a group of chronic diseases characterized by systemic lesion):
A. Joints
B. The circulatory system
C. Liver
D. Kidney
E. * Connective tissue and blood vessels
372. The decisive factor for the development of scleroderma is a violation of the synthesis:
A. Reticular fibers

B. Fibrin
C. Fibrinogen
D. Keloid
E. * Collagen
373. What disease refers to a group of rheumatic:
A. Atherosclerosis
B. Obesity
C. Myocardial infarction
D. Amyloidosis
E. * Dermatomyositis
374. Typical tyberculous granuloma is characterized by all of the following, EXCEPT:
A. Area of central necrosis
B. Epithelioid cells
C. Langhans-type giant cells
D. Lymphocytes
E. * Plasma cells
375. The most common sites of skeletal tuberculosis involvement are all of the following, EXCEPT:
A. Hips
B. Thoracic vertebrae
C. Lumbar vertebrae
D. Knees
E. * Skull bones
376. Substances that prevent complete phagocytosis of Mycobacterium tuberculosis by macrophages and
induce delayed type hypersensitivity are all of the following, EXCEPT:
A. Cord factor
B. Activated complement
C. Lipoarabinomanan (LAM)
D. Heart-shock protein
E. * Interleukin
377. A calcified focus (fibrocalcific scar) forming in the lung parenchyma and in the hilar lymph node
after the primary tuberculosis infection is also referred to as:
A. Keloid
B. Granuloma
C. Ghon focus
D. Simon focus
E. * Aschoff-Pule focus
378. All of these cells participate in immune response in primary lung tuberculosis, EXCEPT:
A. Type I pneumocytes
B. Double negative T-cells
C. CD4+ helper T-cells
D. CD8+ suppressor T-cells
E. * Alveolar macrophages
379. Severe destruction of vertebrae in spine tuberculosis may result in all of the following pathologic
conditions, EXCEPT:
A. Permanent compression fractures
B. Scoliotic deformities
C. Neurologic deficits

D. Kyphotic deformities
E. * Drainage tract (sequestrum) forming
380. The liver may be affected in which of the following forms of tuberculosis:
A. Secondary tuberculosis
B. Cavernous fibrocaseous tuberculosis
C. Dormant tuberculosis
D. Primary tuberculosis complex
E. * Miliary tuberculosis
381. The granuloma in tuberculosis is composed predominantly of which of the following cells:
A. Fibroblasts
B. Eosinophils
C. Plasma cells
D. Neutrophils
E. * Epithelioid cells
382. The possible causes of chronicity of inflammation in tuberculosis are all of the following, EXCEPT:
A. Persistence of certain microorganisms
B. Prolonged exposure to toxic agents
C. Incomplete phagocytosis
D. Resistance of etiologic agent
E. * Complete phagocytosis
383. Impairment of bronchociliary function in influenza results in which of the following:
A. Fungal superinfection
B. Chlamydial superinfection
C. Protozoal superinfection
D. Mycoplasmal superinfection
E. * Bacterial superinfection
384. Peribronchial pneumonia at measles often results in which of the following complications:
A. Peripheral lung tumor
B. Central lung tumor
C. Hemosiderosis of the lung
D. Lung gangrene
E. * Bronchiectasis
385. The severe form of measles pneumonia is characterized by which of the following:
A. Foci of hemorrhagic inflammation
B. Catarrhal bronchitis
C. Granulomatous inflammation
D. Caseous necrosis
E. * Destructive panbronchitis
386. Which stage of changes of Peyer's patches is the second in typhoid between enumerated?
A. no indicated here
B. ulcer formation
C. healing
D. medullar swelling
E. * necrosis
387. Uterine cervix tissue covered with a wide layer of the stratified squamous epithelium having
roliferation of atypical cells with pathological mitoses, but the basal membrane of the epithelium was
not affected. Choose rihgt diagnosis:

A. Nonkeratinizing squamous cell carcinoma


B. Keratinizing squamous cell carcinoma
C. Leukoplakia
D. Epithelial dysplasia
E. * Carcinoma in situ
388. Deformed mucouse membrane of a lobar bronchus consistinyof hyperchromatic nuclei and numerous
pathological mitoses, the growth of the tumour did not spread to the basal membrane of the
epithelium. Choose rihgt diagnosis:
A. Squamous cell carcinoma
B. Adenocarcinoma
C. Solid carcinoma
D. Small-cell carcinoma
E. * Carcinoma in situ
389. The epithelial tumors without specific localization develop from:
A. Mesotheliocytes
B. Fibroblastes
C. Epithelioidcells
D. Epitheliocytes
E. * Squamous epithelium
390. The quiet formed benign tumor with squamous epithelium is known as:
A. Sarcoma
B. Fibroma
C. Lipoma
D. Mioma
E. * Papilloma
391. What are the pathological conditions can lead to symptomatic hypertension?
A. The disease of the CNS
B. Vascular disease
C. Kidney disease
D. Endocrine diseases
E. * All of the above
392. The leading factor in the pathogenesis of hypertension.
A. Hepatic factor
B. Kidney
C. Metabolic
D. Reception lipoprotein
E. * Hereditary
393. Symptoms of cardiac dysfunction include all of the following, except:
A. Failure of the pump itself
B. Obstruction of blood flow
C. Regurgitant blood flow
D. Disorders of cardiac conduction
E. * Disorders of blood coagulation
394. The renin-angiotensin system consists of all of the following components, except:
A. Renin
B. Angiotensin I
C. Plasma angiotensinogen

D. Angiotensin II
E. * Tissue angiotensinogen
395. The typical pathologic proces in arterioles and small arteries in systemic hypertension is referred to
as:
A. Hyaline atrophy
B. Hyaline hyperplasia
C. Fibroelastic hyalinosis
D. Hyaline atherosclerosis
E. * Hyaline arteriolosclerosis
396. The gene defects in enzymes involved in aldosterone metabolism in systemic hypertension lead to all
of the following effects, except:
A. Adaptive increase in aldosterone secretion
B. Increased salt resorption
C. Increased water resorption
D. Plasma volume expansion
E. * Increased lipoprotein resorption
397. Resulting from left-sided heart failure are all of these pathologic changes, except:
A. Brown induration of the lung
B. Hypoxic encephalopathy
C. Pulmonary edema
D. Prerenal azotemia
E. * Secondary amyloidosis
398. Secondary hypertension may be caused by all of the following diseases, except:
A. Acute glomerulonephritis
B. Gushing's syndrome
C. Pheochromocytoma
D. Coarctation of aorta
E. * Lobar pneumonia
399. Acute rheumatic carditis is characterized by all of the following, except:
A. Unremarkable gross appearance of the heart
B. The Aschoff bodies in myocardium
C. Diffuse nonspecific myocarditis
D. The Aschoff bodies in joints
E. * "Tigered effect" gross appearance
400. Non embolic complications of infective endocarditis include all of the following, except;
A. Valvular insufficiency or stenosis with cardiac failure
B. Myocardial ring abscess
C. Suppurative pericarditis
D. Endocarditis of artificial valves
E. * Acute myocardial infarction
401. The cardinal anatomic changes of the mitral (or tricuspid) valve in chronic rheumatic heart disease
are all of the following, except:
A. Fusion of the tendinous cords
B. Leaflet thickening
C. Commisural fusion
D. Shortening and thickening of the tendinous cords
E. * Leaflet softening

402. The clinical consequences of valvular dysfunction depend on all of the following, except:
A. Valve involved
B. Rate and quality of compensatory mechanisms
C. Degree of impairment
D. Rate of dysfunction development
E. * Sex of the patient
403. Systemic emboli in infective endocarditis may occur because of all of the following, except:
A. Involvement of aortic and mitral valves
B. Severe valvular damage
C. Numerous amount of vegetations
D. Friable nature of vegetations
E. * Chronic heart insufficiency
404. Diabetes mellitus is associated with all of the following renal disorders, EXCEPT:
A. Diffuse glomerulosclerosis
B. Nodular glomerulosclerosis
C. Benigh nephrosclerosis
D. Acute pyelonephritis
E. * Urate nephropathy
405. Ultrastructural changes in children primary nephrotie syndrome involve which of the following
glomerular elements:
A. Endothelium
B. Mesangium
C. Bloodvessels
D. Basement membrane
E. * Podocytes
406. Uremia is associated with all of the following abnormalities, EXCEPT:
A. Peripheral neuropathy
B. Gastritis
C. Pericarditis
D. Diffuse alveolar damage
E. * Polycythemia
407. What enumerate illness from the listed below relates to the large mottled kidney
A. Amiloidosis of kidney
B. Chronic pyelonephritis
C. Nephrolithiasis
D. Chronic glomerulonephritis
E. * Subacute glomerulonephritis
408. What disease can be complicated by amyloidosis of kidneys:
A. Atherosclerosis
B. Heart ischemic disease
C. Croupous pneumonia
D. Hypertension
E. * Fibrous cavernous tuberculosis of lungs
409. Most forms of chronic renal failure produce increased serum levels of all of the following substances,
EXCEPT:
A. Renin
B. Aldosterone

C. Phosphate
D. Parathormone
E. * Calcium
410. Mesangial cells can be characterized by all of the following properties, EXCEPT:
A. Ingestion of macromolecules
B. Connection with Lacis cells
C. Ability to contract
D. Production of basement membrane proteins
E. * Production of renin
411. In immunologicahy mediated glomerulonephritis all of the following cells contribute to the
glomerular injury, EXCEPT:
A. Macrophages
B. Platelets
C. Neutrophils
D. Mesangial cells
E. * Mast cells
412. Systemic lupus erythematosus gives rise to all of the following patterns of glomerular injury,
EXCEPT:
A. Focal proliferative glomerulonephritis
B. Diffuse membranous glomerulonephritis
C. Diffuse proliferative glomerulonephritis
D. Mesangial proliferative glomerulonephritis
E. * Lipoid nephrosis
413. Renal diseases producing systemic hypertension include all of the following, EXCEPT:
A. Acute glomerulonephritis
B. Chronic glomerulonephritis
C. Chronic pyelonephritis
D. Renal vasculitis
E. * Renal amyloidosis
414. Histologic features of malignant nephrosclerosis include all of the following, EXCEPT:
A. Fibrinoid necrosis of arterioles
B. Medial thickening of arterioles
C. Renal artery thrombosis
D. Focal renal parenchymal infarction
E. * Fibromuscular dysplasia of the renal artery
415. Which statement correctly characterizes membranous glomerulopathy?
A. It is the most common cause of nephrotie syndrome in children
B. Patients usually present with acute renal failure
C. It is characterized by diffuse proliferative glomerulonephritis
D. It is characterized by mesangial interposition phenomenon
E. * Electron microscopy demonstrates numerous subepithelial immunetype deposits
416. Subendothelial granular electron-dense deposits can be found in which of the following diseases:
A. Rapidly progressive glomerulonephritis
B. Side cell nephropathy
C. Membranous glomerulonephritis
D. Gouty nephropathy
E. * Systemic lupus erythematosus

417. Hydronephrosis is characterized by all of the following, EXCEPT:


A. Thinning of the renal parenchyma
B. Dilatation of the renal pelvis
C. Dilatation of the renal calyces
D. Progressive atrophy of the kidney
E. * Kidney infarct
418. Hydronephrosis is caused by all of the following, EXCEPT:
A. Large uterine leiomyoma
B. Renal calculi
C. Benign prostatic hypertrophy
D. Papillary transitional cell carcinoma of the ureter
E. * Chronic renal vein thrombosis
419. Bone marrow is red juicy, sometimes a gray tint during:
A. Undifferentiated form of leukosis
B. Jewler’s form of leucosis
C. Lymphogranulamatosis
D. During a child’s form of leucosis
E. * Early form of leucosis
420. All of the following statements correctly describe hereditary spherocytosis, EXCEPT:
A. * Red blood cells have the membrane-associated protein spectrin
B. Mutation in the ankyrin gene is present in most cases
C. Anaplastic crisis
D. Splenectomy is invariably therapeutic
E. -
421. Features of megaloblastic anemias include all of the following, EXCEPT:
A. Hypersegmented neutrophils
B. * Lack of the membrane-associated protein spectrin
C. Increased intramedullary hemolysis
D. Increased extramedullary hemolysis
E. Pancytopenia
422. All of the following statements correctly describe aplastic anemia, EXCEPT:
A. Production of all hematopoietic bone marrow elements is reduced
B. Chemical drug exposure is the most common cause
C. Fanconi's anemia represents an inherited form of the disease
D. No underlying etiology is evident in 50% of cases
E. * Splenomegaly is a characteristic clinical finding
423. Osmotic fragility is characteristic of the erythrocytes in which of the following diseases:
A. Fanconi's anemia
B. Sickle cell anemia
C. Glucose-6-phosphate dehydrogenase deficiency
D. * Hereditary spherocytosis
E. Pernicious anemia
424. Myelophthisic anemia can occur in patients with any of the following conditions, EXCEPT:
A. Miliary tuberculosis
B. Carcinomatosis
C. Myelofibrosis

D. Multiple myeloma
E. * Uremia
425. During chronic lympho leucosis the liver is
A. Shrunken
B. Atrophic
C. Infiltrated by scar tissue
D. Cirrhotic
E. * Enlarged, gray-brown color
426. To the group of paraprotein leukosis belong disease of
A. Hodgkins
B. Waldenstrom’s
C. Langerhans’
D. Letterer-Siwe
E. * Mielomic
427. Metaplasia is characterized by which of the following:
A. Reversible increase in the size of cells;
B. Irreversible change in which one adult celltype isreplaced by another adult cell type;
C. Reversible abnormal organization of cells;
D. Reversible increase in the number of cells.
E. * Reversible change in which one adult celltype is replaced by another adult cell type;
428. One of the variants of physiologic atrophy is:
A. Atrophy of skeletal muscle by the immobi lized broken limb;
B. Kidney atrophy from pressure (hydronephrosis) ;
C. Atrophy of the endometrium by ovarian tumor;
D. Atrophy of the brain in atherosclerosis
E. * Atrophy of uterus after parturition;
429. Pressure-overloaded (concentric) cardiac hypertrophy is characterized by all of the following,
EXCEPT:
A. Hypertrophy of left ventricle;
B. Increased wall thickness;
C. Normal left cavity diameter;
D. Reduced left cavity diameter.
E. * Dilated left cavity diameter;
430. Simple endometrial hyperplasia is characterized by all of the following, EXCEPT:
A. Increase in the number and size of endometrial glands;
B. Complex endometrial glands;
C. Increase in gland-to-stroma ratio;
D. Dilated endometrial glands.
E. * Atypia of gland cells;
431. The causes of hypertrophy are all of the following, EXCEPT:
A. Mechanical triggers;
B. Trophic triggers;
C. Vasoactive agents.
D. Polypeptide growth factors;
E. * Nervous triggers;
432. The hydronephrosis is characterized by all of the following, EXCEPT:

A. Thinning of the renal parenchyma;


B. Dilatation of the renal pelvis;
C. Dilatation of the renal calyces;
D. Progressive atrophy of the kidney.
E. * Thickening of the renal parenchyma
433. The hyperplasia of hepatic cells that occurs after partial hepatoectomy is an example of:
A. Pathologic hyperplasia;
B. Hormonal hyperplasia;
C. Hormonal hypertrophy;
D. Compensatory hypertrophy.
E. * Compensatory hyperplasia;
434. The hypertrophic and dilated (eccentric) heart is characterized by all of the following, EXCEPT:
A. Decreased mass and diminished left wallthickness;
B. Increased mass and normal left wallthickness;
C. Normal mass and left wall thickness;
D. Normal mass and diminished left wallthickness;
E. * Increased mass and diminished left wallthickness.
435. The local factors that influence wound healing are all of the following, EXCEPT:
A. Wound infection;
B. Mechanical factors;
C. Foreign bodies;
D. Size, location and type of the wound
E. * Hormones (glucocorticoids);
436. The massive growth of the gravid uterus with large plump cells is an example of:
A. Pathologic hypertrophy;
B. Pathologic hyperplasia;
C. Dysplasia;
D. Metaplasia.
E. * Hormone induced physiologic hypertrophy
437. The most common clinical manifestation of endometrial hyperplasia is:
A. Menses stopping;
B. Pains;
C. Purulent discharges;
D. Mucous discharges
E. * Abnormal uterine bleeding;
438. What pigment can be found in the cytoplasm of heart and muscle cells in aging atrophy?
A. Melanin;
B. Hemosiderin;
C. Bilirubin;
D. Ferritin.
E. * Lipofuscin
439. The cells of immune system which have numerous fine dendritic cytoplasmic processus are called:
A. Fibroblasts;
B. Macrophages;
C. Leucocytes;
D. CD4+ T cells
E. * Dendritic cells;
440. The cells which compose the granuloma in type IV hypersensitivity reactions are all of the following,
EXCEPT:
A. Lymphocytes;
B. Macrophages;
C. Epithelioid cells;
D. Giant cells.
E. * Erythrocytes;
Назва наукового напрямку (модуля): Семестр: 6
ЗАДАЧІ АНГЛ екзамен мед
Опис:
Екзамен МЕДИЧНИЙ
Перелік питань:
1. In the pathomorphological laboratory sent a connective tissue with a large arterial vessel, in the
lumen of which a grayish-colored dry brittle mass, densely connected with the vessel wall. At the
histological examination, an arterial vessel has a mass of obliterating nature that contains fibrin,
platelets, and a small number of red blood cells with an overwhelming number of leukocytes with the
phenomenon of initial organization. What kind of process is there in this case?
A. Connective tissue in the lumen of blood vessels
B. Thromb-embolus in the lumen of the vessel
C. Hyaline thrombus in the lumen of the vessel
D. A blood vessel in the lumen of blood vessels
E. * Cladding white blood clot in the lumen of the arterial vessel
2. A man 56 years old, for 20 years has not suffered from chronic ischemic illness. Suddenly there were
pains in the heart area, sharply increased cynosiness of the face, there was a "feeling of fear of
death." Despite the onset of treatment, death has occurred. At the autopsy, a large left ventricular
heart attack has been detected. A large amount of pinkish-reddish foamy fluid flows from the surface
of the lung. In the liver a peculiar figure resembling muscat has been found, blood from dark cherry
drains from the surface of the cut. Morphological changes in the lungs and liver are manifestations:
A. local arterial hyperemia
B. general acute arterial hypertension
C. Local acute venous hyperemia
D. local chronic venous hyperemia
E. * general acute venous hyperemia
3. A 68-year-old woman developed acute myocardial infarction as a result of stenotic atherosclerosis of
the coronary arteries. During the section it was revealed: in the lumen of the anterior interventricular
branch of the left coronary artery there are brittle gray-yellow masses reminiscent of the
atheromatous detritus and completely lining the lumen of the vessel, in the thickness of the anterior
and lateral walls of the left ventricle - transmural portion of irregular shape, gray-yellow color,
periphery surrounded by a hemorrhagic crown. Diagnose the cause of a heart attack.
A. Tissue embolism
B. Embryos by extraneous bodies
C. Fatty embolism
D. Retrograde embolism
E. * Thrombosis
4. The section revealed: multiple hemorrhagic pulmonary heart attacks, some pulmonary vessels of a
lilac-colored dense mass, varicose veins of the lower extremities, in which the presence of fragile
dark-brown colored masses. What kind of pathological process is being discussed?
A. Fatty pulmonary artery embolism.
B. Tissue embolism of the vessels of the pulmonary artery.
C. Congestive thrombosis of the vessels of the pulmonary artery.
D. Hemorrhagic bronchopneumonia.
E. * Trombembolia of the vessels of the pulmonary artery
5. At the autopsy, an enlarged liver of a colorful species, with a pattern of nutmeg in the section, was
discovered. In the lumen of the liver veins, the wall clots are found. What kind of disruption of blood
circulation in the liver?
A. General venous plethora
B. Anemia

C. Hemorrhage
D. Bleeding
E. * Local venous hyperemia
6. In a pilot who died as a result of depressurizing the cockpit, a large number of vesicles were detected
in the histological examination of the internal organs in the blood vessels, and in the liver it was fecal
dystrophy. In the brain and spinal cord - multiple small ischemic centers of gray softening. Indicate
the most likely cause of such changes.
A. Fatty embolism
B. Air embolism
C. Thromboembolism
D. Tissue embolism
E. * Gas embolism
7. A man 57 years old, for 20 years suffered from emphysema of the lungs. He died of heart failure. At
the intersection: cirrhosis, mucous membranes and cerebellar cysts. Expressed swelling of the dermis
and subcutaneous tissue, hydrothorax, hydropericardium, ascites. The liver has a "muscat" ("heart")
cirrhosis. The described changes are a manifestation:
A. local acute arterial hyperemia
B. general acute arterial hyperemia
C. general acute venous hyperemia
D. local chronic venous hyperemia
E. * general chronic venous hyperemia
8. Name the mechanism lying in the basis of the pathogenesis of organ-nonspecific autoimmune
diseases.
A. Disturbance of the physiological isolation of organs and tissues to which there is no physiological
tolerance
B. Appearance of new nonshared antigens in the organism
C. Atrophy of the lymphoid system
D. * Disturbance of the immune homeostasis in the lymphoid system
E. Cellular immunodeficiency
9. Name the mechanism lying in the basis of the pathogenesis of organ-specific autoimmune diseases.
A. * Disturbance of the physiological isolation of organs and tissues to which there is no physiological
tolerance
B. Primary disturbance in the immunocompetent system
C. Appearance of new nonshared antigens in the organism
D. Disturbance in the control of immune homeostasis
E. Atrophy of the lymphoid system
10. At the patient, 29 years old, with a split-fold fracture of the right thigh for the 3rd day after injury,
complaints of pain in the chest appeared to the right, difficult breathing. A day later, with the
background of progressive cardio-respiratory failure, death has come. At histological examination in
the blood vessels of the lungs and brain were found sudanophilic drops of orange color, which
completely blocked the lumen of the vessels of the microcirculation bed. What complication is the
death of a patient?
A. Gas embolism.
B. Medication embolism.
C. Microbial embolism.
D. Thromboembolism.
E. * Fat embolism.

11. A man 21 years old died in the phenomena of acute pulmonary edema. At autopsy revealed: skin is
pale with pronounced acrocyanosis. Blood in vessels is thick, dark cherry color. Microscopically in
the vessels of the microcirculatory channel, small thrombi and phenomena of "slag phenomenon" are
detected. Which of the following processes are covered?
A. general chronic venous plethora
B. general acute venous plethora
C. local chronic venous glorification
D. local acute venous hypertension
E. * Condensation of blood
12. The mountaineers on the sixth day of ascension to Everest during a control examination in peripheral
blood revealed that the amount of erythrocytes reaches 10 million in 1 ml of blood. A slight increase
in blood pressure, redness of the skin of the face and visible mucous membranes is noted. What
disorder of blood circulation arose in climbers?
A. DIC-syndrome
B. Blood Condensation
C. general venous plethora
D. local arterial hypertension
E. * general arterial hypertension
13. A woman 23 years old, for several years suffering from fibrous-cavernous pulmonary tuberculosis.
Suddenly there was a pallor of the skin and mucous membranes, dizziness, and fainted. A few
minutes later there was death. At autopsy paid attention to the sharp pallor of the skin, visible mucous
membranes, serous membranes, tissues of the internal organs. Cavities of the heart and large vessels
are empty, spleen is small, wrinkled. Point and spotty hemorrhage under the left ventricular
endocardium of the heart (Minakov spots) was detected. What is a circulatory disorder out of the
following, found in this case?
A. bleeding
B. hemorrhage
C. general chronic anemia
D. Blood congestion
E. * general acute anemia
14. A woman is 29 years old, a worker of a mercury plant, with a marked pallor and light jaundice of the
skin, mucous membranes, and the presence of diapedetic hemorrhages. Complaints about fatigue,
weakness, decreased ability to work, dizziness, unconscious condition. Significant dystrophic
changes were detected in liver biopsy, and the presence of hemosiderin in Kupffer cells. What
general disorder of blood circulation from the below mentioned is in this case?
A. general acute anemia
B. Blood Condensation
C. hemohromatosis
D. local chronic anemia
E. * general chronic anemia
15. At the autopsy of a corpse of a young man who died after a bite of a snake from multiple organ
failure, a slag phenomenon and numerous thrombi in the microvessels of the lungs, kidneys, liver,
adrenal glands, pituitary gland, brain, gastrointestinal tract, skin were revealed. Along with this there
were multiple hemorrhages in the lungs, the brain, in combination with dystrophy and necrosis of
organs and tissues. What kind of blood circulation disorder is in this case?
A. thrombosis
B. general venous plethora
C. Blood Congestion
D. general acute anemia
E. * DIC-syndrome

16. A man 45 years old died of liver failure. At the intersection revealed thrombosis of the liver veins
(Kiri disease), muscat cirrhosis of the liver, ascites. At the anterior abdominal wall, the vessels are
sharply enlarged, filled with dark cherry blossoms, with a characteristic pattern of "jellyfish heads".
The above changes are due to:
A. general venous hypertension
B. a stasis of blood
C. compressive venous hyperemia
D. collateral venous congestion
E. * obstructive venous hyperemia
17. A woman 23 years old was taken to a surgical department diagnosed with "ectopic pregnancy" and a
picture of general acute anemia. During surgery, the surgeon found elastic clots in the cavity of the
small pelvis and about 1000 ml of liquid blood. What type of disorder of blood circulation on the
mechanism of its formation revealed a surgeon?
A. haemorrhagia over rhexin
B. hemorrhagia through diapedesin
C. petechiae autchichems
D. suggilatio
E. * haemorrhagia through diabrosin
18. A 57-year-old man suffering from a trophic ulcer of the left shin has pain, redness, a feeling of
warmth, swelling in the affected shin. During a surgery in the lumen of the veins, a dry clot of blood,
which is easily crumbly, layered, with a rough surface, is detected. The described changes are a
manifestation:
A. phlebothrombosis
B. DIC-syndrome
C. slag phenomenon
D. Blood stasis
E. * thrombophlebitis
19. A 32-year-old woman suffering from subacute septic endocarditis suddenly lost her vision in the right
eye. During examination, an ophthalmologist showed a sharp increase in the lumen of the central
artery of the eye and the presence of a blood clot in it. The detected changes are a manifestation:

A. phlebothrombosis
B. DIC-syndrome
C. slag phenomenon
D. Blood stasis.
E. * thromboembolism
20. A histological examination of the lungs if a male, who suffered for many years from atopic bronchial
asthma and died of asphyxia, revealed much mucus with an admixture of eosinophils in the lumens of
the bronchioles and small bronchi, sclerosis of interalveolar septa, dilation of alveolar lumens. Which
of the mechanisms in the development of a hypersensitivity reaction took place when a fit of
asphyxia developed?
A. Cytotoxic reaction
B. Immunocomplex reaction
C. Cytolysis owing to lymphocytes
D. Granulomatosis
E. * Reaginic reaction

21. A study of the thymus of a 5-year-old child, who died from acute destructive staphylococcal
pneumonia, revealed a decrease in the weight of the gland down to 3.0 g. On histological
examination, a smaller size of the lobules of the gland with a collapse of the stroma, an inversion of
the layers, and cyst-like Hassal’s bodies were found out. Which of the diagnoses listed below was the
most probable?
A. Thymomegaly
B. Hypoplasia of the thymus
C. Dysplasia of the thymus
D. Agenesia of the thymus
E. * Accidental reaction
22. An examination of a pregnant woman with a rhesus-negative group of blood revealed a high level of
antierythrocyte antibodies; in order to decrease it, a skin flap of her rhesus-positive husband was
grafted to her. Two weeks later the flap was rejected; its microscopic examination revealed
disturbances of circulation, an oedema, a cellular infiltration mostly by lymphocytes, neutrophils and
macrophages. Which of the pathological processes listed below was the most probable?
A. Immediate hypersensitivity
B. Delayed hypersensitivity
C. Granulomatous inflammation
D. Interstitial inflammation
E. * Transplantation immunity
23. An autopsy of a 43-year-old female, who suffered from attacks of expiratory dyspnoea during her
life-time and died from asphyxia, revealed some dense glass-like mucus in the lumens of the bronchi,
their walls were thick, the lungs had foci of an emphysema and atelectases. A histological
examination of the pulmonary tissue revealed some mucus with an admixture of eosinophils in the
lumens of small bronchi, sclerosis of the peribronchial connective tissue and interalveolar septa,
dilation of the lumens in the alveoli. What mechanism of hypersensitivity formed the basis for the
development of asphyxia?
A. Cytotoxic reaction
B. Immunocomplex reaction
C. Cytolysis owing to lymphocytes
D. Granulomatosis
E. * Reaginic reaction
24. An 8-year-old child was done an intracutaneous tuberculin (Mantoux) test with a diagnostic purpose.
Forty-eight hours following the injection of tuberculin, a dense hyperaemic papule, 20 cm in
diameter, with necrosis in its centre formed. Name the mechanism of hypersensitivity which lay in
the basis of the above changes.
A. Anaphylaxis
B. Antibody-dependent cytotoxicity
C. Immunocomplex cytotoxicity
D. Granulomatosis
E. * Cellular cytotoxicity
25. A 16-year-old youth developed oedemata of his face, oliguria and an increased blood pressure 20
days after he recovered from scarlet fever. A urinalysis revealed an increase of relative density,
haematuria, proteinuria. On microscopic examination of a renal biopsy, a picture of intracapillary
proliferative glomerulonephritis was found out, while an electron microscopy revealed deposits on
the basal membranes. Which of the mechanisms listed below lay in the basis of this disease?
A. Anaphylactic reaction
B. Antibody-dependent cytotoxic reaction
C. Cell-dependent cytolysis
D. Granulomatosis

E. * Immunocomplex mechanism
26. Three years ago in the open skin areas of a female worker of a shop which manufactures synthetic
detergents appeared eruptions in the form of papules, vesicles and small weeping erosions, some of
them were covered with crusts. The patient notices that in case of a direct contact with detergents her
skin manifestations intensify. Skin tests with several chemical substances manufactured at the shop
are positive. A histological examination of an area of the affected skin revealed acanthosis and
hyperkeratosis, an oedema of the derma, a vascular plethora, as well as a perivascular lymphocytic
infiltration. Clinically, occupational eczema was diagnosed. Which of the immunological
mechanisms lies in the basis of this disease?
A. Anaphylactic reaction
B. Antibody-dependent cytotoxic reaction
C. Granulomatosis
D. Immunocomplex mechanism
E. * Cellular cytotoxicity
27. In a 10-year-old child, eating of strawberries was followed by appearance of some disseminated
monomorphous and severely itching urticaria (red blisters of the round and oval form), elevation of
body temperature up to 37.8(C and a gastrointestinal disturbance. A blood analysis revealed
eosinophilia. After taking of antihistamine agents the above manifestations were rapidly controlled.
Which of the immunological mechanisms lay in the basis of this disease?
A. Granulomatosis
B. Antibody-dependent cytotoxic reaction
C. Cellular cytotoxicity
D. Immunocomplex mechanism
E. * Anaphylactic reaction
28. An autopsy of a 23-year-old female, who died from puerperal sepsis, revealed an enlarged plethoric
spleen whose section gave an abundant scrape. Microscopically, hyperplasia and a plasmacytic
infiltration of both the red pulp and splenic follicles were found out; the red pulp was rich in
macrophages. Which of the immunopathological mechanisms most probably lay in the basis of the
changes in the spleen?
A. Hereditary insufficiency of the peripheral lymphoid tissue
B. Immediate hypersensitivity reaction
C. Delayed hypersensitivity reaction
D. Autoimmunization
E. * Antigenic stimulation of the organism
29. In a child, 48 hours after a tuberculin (Mantoux) test, a papule up to 10 cm in diameter formed at the
place of an injection of tuberculin. What mechanism of hypersensitivity lay in the basis of the above
changes?
A. Anaphylaxis
B. Antibody-dependent cytotoxicity
C. Immunocomplex cytotoxicity
D. Granulomatosis
E. * Cellular cytotoxicity
30. A 23-year-old male patient developed the urinary syndrome (haematuria, proteinuria, leukocyturia)
after having angina. A puncture biopsy of the kidneys revealed a picture of intracapillary proliferative
glomerulonephritis, while on electron microscopy some large subepithelial deposits were found out.
What was the pathogenesis of this disease?
A. Anaphylactic reaction
B. Antibody-dependent cytotoxic reaction
C. Cell-dependent cytolysis
D. Granulomatosis

E. * Immunocomplex mechanism
31. Nine days after receiving large doses of the antitetanic serum a female patient developed urticaria
with an elevated body temperature, skin itch, an oedema of mucosae. Indicate the immunological
mechanism of the disease.
A. Reaginic anaphylactic reaction
B. Pathogenic effect of sensitized lymphocytes
C. Granulomatosis
D. Cytotoxic effect of antibodies
E. * Immunocomplex mechanism
32. This disease develops in newborns and manifests itself in the form of persistent diarrhoeae, general
cachexia and a clinical picture of sepsis. The children die at the age of 2-3 years. On postmortem
examination, the thymus is represented with a reticular stroma, no epithelial reticulum, thymus
corpuscles and lymphocytes are present. The lymphoid organs are characterized by a severe stage of
hypoplasia. What type of immunopathological processes does the above disease belong to?
A. Autoimmune disease
B. Syndrome of insufficiency of cellular immunity
C. Antibody deficiency syndrome
D. AIDS-related syndrome
E. * Combined immunodeficiency syndrome
33. A histological examination of an increased cervical lymph node in a male patient, who worked at an
enterprise with occupational hazards, revealed a plethora and swelling of the cortical layer, the light
centre of follicles and the medullary substance contained a large amount of plasma cells, the number
of lymphocytes was reduced, there was an active proliferation of sinus cells and a significant
macrophage response. Name the character of changes in the lymph node.
A. Lymphoma
B. Acute lymphadenitis
C. Lymphogranulomatosis
D. Insufficiency of peripheral lymphoid tissue
E. * Antigenic stimulation of lymphoid tissue
34. A histological examination of a skin graft in a male patient, who underwent dermatoplasty revealed a
diffuse lymphohistiocytic infiltration with an admixture of macrophages and neutrophils, an oedema
and haemorrhages. Which of the diagnoses listed below was the most probable?
A. Delayed hypersensitivity reaction
B. Immediate hypersensitivity reaction
C. Interstitial inflammation
D. Arthus phenomenon
E. * Graft rejection reaction
35. A 45-year-old woman has had a chronic, non-productive cough for 3 months, along with intermittent
fever. She has a chest radiograph that reveals multiple small parenchymal nodules along with hilar
and cervical lymphadenopathy. A cervical lymph node biopsy is performed. Microscopic
examination of the biopsy shows noncaseating granulomatous inflammation. Cultures for bacterial,
fungal, and mycobacterial organisms are negative. Which of the following chemical mediators is
most important in the development of this inflammation?
A. Bradykinin
B. Complement C5a
C. Histamine
D. Prostaglandin E2
E. * Interferon gamma

36. At histological research of a biopsy specimen from an auricle of a heart of a patient with rheumatic
disease the foci of a mucoid swelling, fibrinoid necrosis of a connective tissue has been found out.
What immune response has developed in tissues of the auricle of the heart?
A. Hypersensitivity of a delayed type
B. Reaction of the transplantative immunity
C. Normergic reaction
D. Exudative reaction
E. * Hypersensitivity of an immediate type
37. The 30-year-old man has had for two months lacrination, pruritic palpebras, and rhinitis with mucus.
All symptoms disappeared after treatment by desensebilizators. What type of hypersensitivity
occurred in patient?
A. The type II
B. The type III
C. The type IV
D. The type V
E. * The type I
38. Histologic investigation of thyroid gland has showed destruction and atrophy of follicles, diffuse
lymphoid infiltration with formation of lymphoid follicles in the stroma. call the group of diseases
with respect to this thyroiditis.
A. autoimmune nonspecific
B. bacterial
C. infectious-allergic
D. viruses infectious
E. * autoimmune specific
39. In biopsy of stomach in patient with autoimmune gastritis it was found out: infiltration by
lymphocytes and macrophages in mucous layer. Which type of hypersensitivity is connected with
these morphologic changes?
A. The type II
B. The type V
C. The type I
D. The type III
E. * The type IV
40. In biopsy of lymph node it was found out a lot of lymphoid follicles with large centers of duplication,
increasing of mitoses. Which process is characterized by these morphologic changes?
A. Atrophy of lymphoid tissue
B. Lymphosarcoma
C. Hodgkin’s disease
D. Metastases of cancer
E. * Antigenic stimulation with follicular hyperplasia
41. In biopsy of transplantanted kidney it was found out: diffuse infiltration of stroma by lymphocytes,
plasma cells, lymphoblasts, plasmablasts and necrotic arteriitis. Which pathologic process was
appeared in organ?
A. Acute glomerulonephritis
B. Ischemic infarction
C. Tuberculosis
D. Acute pyelonephritis
E. * Immune mutilation

42. In the 30-year-old woman it was found: cough, sputum with blood, fever, increased blood pressure,
decreased urine output, edema of low extremities. All symptoms have developed for 6 weeks.
Diagnostic renal biopsy showed Goodpasture’s syndrome. Which pathologic process is characteristic
for this syndrome?
A. Autoantibodies to mitochondrias of the lungs and kidneys
B. Autoantibodies to DNA
C. Cytotoxic reaction against epithelium of renal tubules
D. Appearance of immune complexes in glomeruli of kidneys and lungs
E. * Autoantibodies to basement membrane of the lungs and kidneys
43. The 30-year-old patient with transplanted kidney has received prolonged immunosupressive therapy
and he has died because of intoxication. Microscopic examination showed giant cells with large
nuclei encircled by rings-like brightening, which looked as “owl-eye”, located in the kidneys, liver,
pancreas, lungs. Call this disease.
A. Tuberculosis
B. Syphilis
C. Leprosy
D. Bubonic plaque
E. * Cytomegalovirus infection
44. Local lymph nodules enlarged near the infected wound. Increased amount of macrophages,
lymphocytes, lymphatic follicles in the cortical layer and large amount of plasma cells were revealed
on histological examination. What process in the lymphatic nodules represents these histological
changes?
A. Tumour transformation
B. Innate insufficiency of the lymphoid tissue
C. Hypersensibility reaction
D. Acquired insufficiency of the lymphoid tissue
E. * Antigen stimulation
45. A 12-year-old boy often suffers from virus and bacterial infections and eczematous skin lesions.
Enlargement of T-lymphocytes and IgM with normal IgA and IgG was revealed on examination.
What type of immune system pathology is presented in the patient?
A. Hereditary immundeficiency of the complement system
B. Composite immunedefficiency
C. Hypoplasia of thymus
D. Turner's syndrome
E. * Bruton's hypogammaglobulinemia
46. The specimens present sections of haemopoetic and immunogenetic organs. Organ has lymph tissue
forming different structures (lymph nodes, lobules, bars). In what organ does antigen-independent
proliferation and differentiation take place?
A. Lymphatic nodes
B. Hemolymph nodes
C. Tonsil
D. Spleen
E. * Thymus
47. In the patient with drug abuse (narcomania) it was diagnosed AIDS. What type of hypersensitivity
occurred in-patient?
A. The type II
B. The type V
C. The type I
D. The type III

E. * The type IV
48. Ten years ago a male patient’s right lung was removed because of a tumour, since then the capacity
of his left lung has increased by 50 %. What process has developed in the left lung?
A. Neurohumoral hypertrophy
B. Vicarious hypertrophy
C. Work hypertrophy
D. Hypertrophic vegetations
E. * Vicarious hypertrophy
49. An autopsy of a male patient, who died from hypertensive disease, revealed an enlarged heart
weighing 600 g, with a thickened left ventricular wall up to 2 cm and a dilated cavity of the left
ventricle. Name the kind of an adaptive reconstruction in the heart.
A. Concentric hypertrophy
B. Vicarious hypertrophy
C. Eccentric atrophy
D. Vicarious hypertrophy
E. * Eccentric hypertrophy
50. As a result of falling down, a small abrasion formed of the knee of a child and some time later it
epithelialized completely without formation of any scar. What form of regeneration took place in this
case?
A. Substitution
B. Physiological
C. Pathological
D. Intracellular
E. * Restitution
51. A 74-year-old male died from chronic heart failure. On autopsy, an old postinfarction scar was found
in the heart. A histological examination revealed a focus of fibrosis and hypertrophy of
cardiomyocytes. What regeneration do the described changes manifest?
A. Physiological
B. Pathological
C. Restitution
D. Intracellular
E. * Substitution
52. Following a traumatic injury of the liver, there was a complete restoration of its structure and
functions with time. What is the name for such a kind of regeneration?
A. Substitution
B. Pathological
C. Physiological
D. Intracellular
E. * Restitution
53. A 20-year-old male patient with a posttraumatic variceal dilation and thrombosis of the subcutaneous
vein in the middle third part of the shin underwent its surgical removal. Histologically, an obstructive
thrombus was found in the lumen of the vein with growing of a connective tissue into the thrombus
from the side of the vascular wall. What process did the changes in the thrombus result from?
A. Reconstruction
B. Canalization
C. Revascularization
D. Repair
E. * Organization

54. A male patient with chronic cystitis revealed, along with the transitional epithelium, foci of the
stratified squamous one without keratinization in a biopsy of the mucosa of his bladder. What
pathological process does it indicate?
A. Leukoplakia
B. Hyperplasia
C. Dysplasia
D. Hyperkeratosis
E. * Metaplasia
55. A 30-year-old male patient, who 10 years before had undergone removal of his traumatized left
kidney, against a background of good health revealed a twice-fold enlargement of the right kidney in
comparison with the norm on prophylactic medical examination. Which of the listed processes was
the most probable one in the kidney?
A. Work hypertrophy
B. Hyperplasia
C. Neurohumoral hypertrophy
D. Pseudohypertrophy
E. * Vicarious hypertrophy
56. A microscopic examination of a myocardium revealed postinfarction transmural cardiosclerosis
surrounded by enlarged cardiomyocytes with large hyperchromatic nuclei rich in DNA. Which of the
listed morphological processes in the cardiomyocytes was the most probable?
A. Complete reparative regeneration
B. Physiological regeneration
C. Pathological regeneration
D. Work hypertrophy
E. * Regenerative hypertrophy
57. An autopsy of a 75-year-old male patient, who suffered from a prostatic adenoma and died of renal
insufficiency, revealed enlarged kidneys; on section, there was an atrophy of the parenchyma and a
lot of thin-walled cavities filled with urine. Which of the listed kinds of an atrophy in the kidney was
the most probable?
A. Dysfunctional
B. Caused by physical factors
C. Neurotic
D. Caused by insufficiency of blood circulation
E. * Caused by pressure
58. In a postoperative wound of a 10-year-old child, who was operated for acute appendicitis; 2-3 days
later appeared some granulation tissue and on the 10th day a small thin scar was formed. Name the
kind of healing.
A. Under a crust
B. Simple
C. By second intention
D. –
E. * By first intention
59. A histological examination of a scrape from the mucous membrane of the uterus was made in a
50-year-old female patient who complained of a disorder in the ovariomenstrual cycle manifested by
irregular significant haemorrhages. A cystoglandular hyperplasia of the endometrium was diagnosed.
Name the kind of the pathological process in the endometrium.
A. Hypertrophic vegetation
B. Vicarious hypertrophy
C. Regenerative hypertrophy

D. Vicarious hypertrophy
E. * Neurohumoral hyperplasia
60. A biopsy of a bronchus of a 50-year-old male patient, who suffered from chronic bronchitis for 20
years, revealed foci of substitution of the stratified squamous epithelium for the columnar one. Which
of the pathological processes listed below took place?
A. Hyperplasia
B. Heterotopia
C. Heteroplasia
D. Dysplasia
E. * Metaplasia
61. For examination, a scrape from the mucous coat of the uterus of a 45-year-old woman suffering from
dysfunctional uterine bleedings was sent. Histologically, an increased number of endometrial glands
and their intensified branching with a cyst-like dilation of the lumens were found. Which of the
pathological processes listed below was the most probable?
A. Hypertrophy
B. Metaplasia
C. Dysplasia
D. Aplasia
E. * Hyperplasia
62. An autopsy of a 57-year-old male patient, who suffered from hypertensive disease and died of cardiac
decompensation, revealed an enlarged heart with dilated cavities. Microscopically, the
cardiomyocytes were significantly enlarged and had fatty degeneration with hyperchromatic
barrel-like nuclei. Which of the listed morphological processes in the heart was the most probable?
A. Hypertrophic vegetations
B. Brown atrophy
C. Concentric hypertrophy
D. Vicarious hypertrophy
E. * Eccentric hypertrophy
63. A 4-years-old girl was operated because of an acute appendicitis. During the operation in the
retroperitoneal space it was found that the right kidney is less by 1/3 in comparison with the left one.
The diameter of the right renal artery was 0,3 cm, the left one was 0,4 cm. What pathologic process
did occur in that case?
A. Pathologic atrophy
B. Physiologic atrophy
C. Aplasia
D. Agenesia
E. * Hypoplasia
64. A 23-years-old patient got a lesion of the liver because of trauma. In time a structure and functions of
that organ was restored completely. What kind of regeneration did occur in that case?
A. Pathologic regeneration
B. Physiologic regeneration
C. Metaplasia
D. Substitution
E. * Restitution
65. A child fell down the tree and got a simple fracture of the one of hand’s bones. In a time healing
occurred. Call the kind of regeneration and cells, which taking part in restoration.
A. Physiological regeneration, octeoclasts
B. Pathological regeneration, osteoclasts

C. Intracellular regeneration, osteocytes


D. Cellular regeneration, osteoclasts and osteoclasts
E. * Reparative regeneration, osteoclasts and osteoblasts
66. A patient has died as a result of cardiac insufficiency. In anamnesis he had got a pulmonectomy in
account of cyst of the right lung. During the autopsy enlarged left one was found out. Call the
pathologic process in the left lung.
A. Neurohumoral hypertrophy
B. Dyscirculatory atrophy
C. Dysfunctional atrophy
D. Physiologic hypertrophy
E. * Replacement hypertrophy
67. A patient has undergone to amputation of lower extremity. In a time a painful nodules appeared in a
stump. Amputatious neuromas were found out during the microscopical examination. What kind of
pathological processes do those formation relate to?
A. Dystrophy
B. Inflammation
C. Hyperemia
D. Metaplasia
E. * Regeneration
68. After deep burns of the skin a patient has got a keloid scarring. What kind of pathologic processes do
those formations relate to?
A. Complete regeneration (restitution)
B. Atrophy
C. Hypertrophy
D. Metaplasia
E. * Incomplete regeneration (substitution)
69. Under microscopic investigation the postinfarction cardiosclerosis has been found out. Around
cardiosclerotic area myocardiocytes were enlarged in size and had large hyperchromic nuclei riched
in DNA. What process taking place in myocardiocytes is more probable?
A. Physiologic regeneration
B. Complete reparative regeneration
C. Pathologic regeneration
D. Hypertrophy because of increased workload
E. * Regenerative hypertrophy
70. Dystrophic changes of the heart muscle are accompanied with cardiac cavity enlargement, decrease
of the strength of heart contraction, increased amount of blood, which remains in the heart during
systolic phase, overfilled veins. For what state of heart is it characteristic?
A. Tamponage of the heart
B. Tonogenic dilatation
C. Cardiosclerosis
D. Emergency stage of hyperfunction and hypertrophy
E. * Myogenic dilatation
71. Decreased blood supply to the organs causes hypoxia that activates fibroblasts function. Volume of
what elements is increased in this case?
A. Lymphatic vessels
B. Parenchymatous elements of the organ
C. Vessels of microcircular stream
D. Nerve elements

E. * Intercellular substance
72. At autopsy of a patient died because of a cerebral hemorrhage, strongly enlarged dense and anemic
kidneys (size: 6х3х2см, weight 60.0 g) with a uniformly small-granulated surface and with
uniformly thinned cortex on a cut-surface have been found out. Call adaptive-compensative
process in this case?
A. Hyperplasia
B. Complete regeneration (restitution)
C. Hypertrophy
D. Metaplasia
E. * Atrophy
73. During surgery in a 17-year-old patient it was revealed the tumour of 4,5х5,0х3,5 sm in size on the
lower surface of the liver with subserose localization, of dark-red color. On the section
tumour has cavities with marked amount of blood. What is preliminary diagnosis?
A. Capillar hemangioma
B. Hemangiopericytoma
C. Hemangioendothelioma
D. Lymphangioma
E. * Cavernous hemangioma
74. Examination of the anterior abdominal wall of a pregnant woman revealed a tumour-like formation
that arose on the spot of a tumour that was removed two years ago. The neoplasm was well-defined,
dense, 2х1 cm large. Histological examination revealed that the tumour was composed of
differentiated connective tissue with prevailing collagen fibres. What tumour might be
A. suspected?
Lipoma
B. Fibrosarcoma
C. Hibernoma
D. Leiomyoma
E. * Desmoid
75. A 50-year-old man has felt vague abdominal discomfort within past 4 months. Physical examination
revealed no lymphadenopathy, and no abdominal masses or organomegaly at palpation. Bowel
sounds are heard. An abdominal CT scan shows a 20 cm retroperitoneal soft tissue mass obscuring
the left psoas muscle. A stool specimen tested for occult blood is negative. Which of the following
neoplasms is this man most likely to have?
A. Melanoma
B. Hamartoma
C. Adenocarcinoma
D. Lymphoma
E. * Lipoma
76. A woman suffering from dysfunctional metrorrhagia was made a diagnostic abortion. Histologically
in the scrape there were a lot of small stamped glandulars covered with multirowed epithelium. The
lumens of some glandulars were cystically extended. Choose the variant of general pathologic
process in the endometrium.
A. Atrophy of endometrium
B. Metaplasia of endometrium
C. Neoplasm of endometrium
D. Hypertrophic growth
E. * Glandular-cystic hyperplasia of endometrium

77. A 75 year old male patient consulted a surgeon about a brown nonhealing ulcer of shin. Examination
of biopsy material revealed diffuse growth of polymorphic atypic cells with brown pigment in their
cytoplasm. Pearls reaction was negative. There were also a lot of pathological mitoses and foci of
tissue necrosis. What is the most probable diagnosis?
A. Local hemosiderosis
B. Intradermal nevus
C. Trophic ulcer
D. Skin cancer
E. * Melanoma
78. A 69-year-old patient got a small plaque with subsequent ulceration on the skin of the lower eyelid.
The formation was removed. Microscopic examination of dermis revealed complexes of atypical
epitelial cells arranged perpendicularly to the basal membrane on the periphery. The cells were dark,
of polygonal prismatic shape with hyperchromic nuclei with frequent mitoses. What is the
histological form of carcinoma in this patient?
A. Keratinizing squamous cell carcinoma
B. Nonkeratinizing squamous cell carcinoma
C. Adenocarcinoma
D. Undifferentiated
E. * Basal cell carcinoma
79. Histological study of the bronchial wall and adjacent lung segments revealed sheets and strands of
squamous epithelium. The cells have moderately expressed symptoms of atypia: polymorphism,
nuclear hyperchromatism, mitoses. In the center of the complex there are concentric pink formations.
What is the most likely diagnosis?
A. Non-keratinizing squamous cell carcinoma
B. Adenocarcinoma
C. Scirrhus
D. Undifferentiated carcinoma
E. * Keratinizing squamous cell carcinoma
80. Histologically, the internal wall of a cyst localized on the upper jaw is lined with stratified squamous
epithelium with underlying granulation tissue infiltrated by lymphocytes. The external layer is
represented by loose fibrous connective tissue surrounded by cicatrical fibrous tissue. What diagnosis
can be made?
A. Simple granuloma
B. Epithelial granuloma
C. Keratocyst
D. Ameloblastoma
E. * Cystic granuloma
81. Microscopic examination of a skin tumor revealed that it invaded the underlying tissue, destroyed it
and formed nests and cords of atypical epithelium which included some pearl-like formations.
Specify the tumor:
A. Squamous cell non-keratinizing carcinoma
B. Solid carcinoma
C. Adenocarcinoma
D. Medullary carcinoma
E. * Keratinizing squamous cell carcinoma
82. Histologic analysis of uterus mucous membrane revealed twisting glands, serrated and spinned, they
were extended by stroma growth with proliferation of its cells. Formulate a diagnosis:
A. Acute endometritis
B. Leiomyoma

C. Cystic mole
D. Placental polyp
E. * Glandular hyperplasia of endometrium
83. Microscopy of colonic biopsy material revealed a tumour made up of prismatic epithelium and
forming atypical glandular structures of various shapes and sizes. The basal membrane of glands was
destroyed. Tumour cells were polymorphic, with hyperchromatic nuclei and a large number of
pathological mitoses. What is the most likely diagnosis?
A. Basal cell carcinoma
B. Solid carcinoma
C. Mucosal carcinoma
D. Undifferentiated carcinoma
E. * Adenocarcinoma
84. The man of 46 years old has a dark patch on a skin which towered and did not disturb. In course of
time a spot began to increase, pain appeared, a color became blackly-brown and a it was easy to feel
the knot. Fusiform and polymorphic cells the cytoplasm of which contained the pigment of brown
color concerned on histological research of remote fabric. What tumour is the question about?
A. Basalioma
B. Gemangioma
C. Haematoma
D. Karsinoid
E. * Melanoma
85. At the patient of 21 year the tumour of frontal part of right hemisphere is remote head brain by a
diameter 5 см, which was unclear delimited from near-by fabric. On a cut - homogeneous
kind, histological — consists of cells the numerous sprouts of which form thick
interlacements. What tumour took place in a cerebrum?
A. Oligodendroglioma
B. Ganglioneuroma
C. Epsidimoma
D. Chorionpapiloma
E. * Astrocytoma
86. At a patient on the skin of person gradual a name-plate developed with necrosis and ulcer in a center.
At pathohistologic research of bioptate excrescence of atypical ephithelial cells is exposed with
plenty of pathological mitosises. What is the most reliable diagnosis?
A. Sarcoma
B. Villoma
C. Trophic ulcer
D. Fibroma
E. * Shrines of skin
87. A dense tumour mobile is macroscopically found in a skin. At a microscopy she is presented by the
chaotically located bunches of collogens fibres with the two-bit of fusiform cells. What tumour is
remote?
A. Leiomyoma
B. Melanoma
C. Lipoma
D. angioneoplasm
E. * Hard fibroma

88. At a patient on small curvature in pyloric department found out formation of crateriform form. From
the regional area of education a biopsy is taken.Information of histological research: tumour with the
glandlike structures of different form and size, ingrowing in surrounding fabric, by expressed
атипизмом of cells. Name the histological variant of this tumour.
A. Flatcell crawfish
B. Scirrhus of stomach
C. Mucous crawfish of stomach
D. Solid crawfish of stomach
E. * Adenocarcinoma
89. Tumour in a capsule by a diameter 2 see operatively remote from amputation cults of lower
extremity, microscopically consists of fusiform cells of monomorphic kind with sticklike kernels
which form "palisades" structures together with fibres. What from the transferred types of tumours
наиболее reliable?
A. Neurofibroma
B. Malignant neurolimoma
C. Soft fibroma
D. Fibrosarcoma
E. * Of high quality neurolimoma
90. At a patient 55 years the relapsic uterine bleeding appeared. The diagnostic endometrectomy is
executed. In соскобе of endometrium among the elements of blood evidently the ferrous
elements of different size and form, formed by atypical cells with hyperchromic kernels, with
numerous mitosises (including pathological). About what process is it possible to think?
A. Ferrous hyperplasia of endometrium
B. Chorionepithelioma
C. Adenomatous polypus
D. Signs of the interrupted pregnancy
E. * Adenocarcinoma
91. At the autopsy of a deceased man who died in the result of intoxication there were found following
signs: cachexia, muscle atrophy, wrinkled skin, reduced weight of internal organs, stenotic tumor of
antral part of stomach with metastases in the liver and regional lymph nodes. What type of cachexia
is the most likely?
A. Alimentary
B. Pituitary
C. Cerebral
D. One that accompanies chronic infectious disease
E. * Cancerous
92. At microscopy study of skin neoplasm of brown colour there was found that the tumor consists of
nevus cells that are located in the dermis as conglomerations and chords. Cytoplasm of cells contains
brown pigment, which gives a negative Perl’s reaction. Which pigment is the most probably found in
the cytoplasm of cells?
A. Hematoidin
B. Hemosiderin
C. Bilirubin
D. Hemomelanin
E. * Melanin
93. On bronchoscopy, an exophytic tumour was found; it was localized in the bronchus and significantly
narrowed its lumen. Histologically, the tumour consisted of complexes of polymorphous epithelial
cells with hyperchromatic nuclei and pathological mitoses. Among the tumour cells there were
eosinophilic concentric structures. Make a diagnosis of the tumour.

A. Nonkeratinizing squamous cell carcinoma


B. Large-cell carcinoma
C. Small-cell carcinoma
D. Adenoacanthoma
E. * Keratinizing squamous cell carcinoma
94. For a histological examination, an eyeball was sent; some black tumour, 1 x 0.4 cm in size, was
revealed in its vascular membrane. Microscopically, the tumour consisted of large polymorphous
cells grouped in alveolar structures. The cytoplasm of the cells contained some brown pigment. What
is your diagnosis?
A. Neurilemmoma
B. Angiosarcoma
C. Neuroblastoma
D. Ganglioneuroblastoma
E. * Melanoma
95. A 65-year-old woman underwent removal of some tumour, 1.0 x 1.0 x 0.8 cm in size, localized under
the skin of her thigh. Macroscopically, the tumour had a connective-tissue capsule and was
represented on section with a yellowish lobate tissue. Microscopically, there were large cells, which
had the sudanophilic cytoplasm and formed lobules separated with connective-tissue layers. Name
this tumour.
A. Hibernoma
B. Liposarcoma
C. Fibroma
D. Desmoid
E. * Lipoma
96. A thick encapsulated node, 2.0 cm in diameter, was surgically removed from the mammary gland of a
female patient. On section, the tissue of the node was white-pink and fibrous. Microscopically, the
tumour consisted of glandular structures, which had no signs of cellular atypism and were
compressed with a connective tissue vegetating around. In the tumour, the stroma prevailed over the
glandular parenchyma. What is your diagnosis?
A. Adenoma
B. Nonproliferative mastopathy
C. Proliferative mastopathy
D. Adenocarcinoma
E. * Fibroadenoma
97. A male patient, who suffered from chronic bronchitis for a long period of time, revealed a pulmonary
tumour, which was closely connected with the bronchial wall and grew in the form of a polyp.
Microscopically, the tumour consisted of complexes of polymorphous epithelial cells with a large
number of mitoses. Among the tumour cells there were stratified concentric oxyphilic structures.
Name the histological type of the tumour.
A. Mucinous carcinoma
B. Solid carcinoma
C. Nonkeratinizing squamous cell carcinoma
D. Adenocarcinoma
E. * Keratinizing squamous cell carcinoma
98. A newborn baby has some red-blue flattened tumor, 5 x 4 x 0.3 cm in size, in a capsule on the skin of
its face. Microscopically, the tumour consists of large thin-walled vascular cavities which have an
endothelial lining and are filled with blood. Name the tumour.
A. Venous haemangioma
B. Capillary haemangioma

C. Hemangiopericytoma
D. Lymphangioma
E. * Cavernous haemangioma
99. On bronchoscopy in the initial part of the upper lobe bronchus of the right lung some polyp-like
formation, 1.0 cm in diameter, with a superficial ulcer was found. A histological examination
revealed a tumour consisting of lymphocyte-like cells with hyperchromatic nuclei; the cells grew in
layers and bands. Indicate the most probable tumour.
A. Undifferentiated large-cell carcinoma
B. Squamous cell carcinoma
C. Adenocarcinoma
D. Glandular squamous cell carcinoma
E. * Undifferentiated small-cell carcinoma
100. An autopsy of a female who died from cachexia, revealed some massive exophytic carcinoma on the
lesser curvature of the stomach with metastases to the ovaries. What kind of metastatic spreading
took place?
A. Haematogenous
B. Lymphogenous orthograde
C. Implantation
D. Perineural
E. * Lymphogenous retrograde
101. A histological examination of a thyroid gland revealed small cysts, which were lined with atypical
epithelium and filled with papillae, the latter originating from the walls of the cysts and growing into
their capsules. Name the tumour.
A. Papillary adenoma
B. Follicular carcinoma
C. Solid carcinoma
D. Carcinoma simplex
E. * Papillary carcinoma
102. A 47-year-old woman underwent radical mastectomy for a neoplasm. A histological examination of
the mammary gland revealed an eczematous lesion of the nipple and areola, a cancerous lesion of the
ducts of the gland and presence of large light cells in the epidermis of the nipple and areola. Make a
diagnosis.
A. Intralobular carcinoma in situ
B. Acneiform carcinoma
C. Papillary carcinoma
D. Fibrous carcinoma
E. * Paget's disease
103. A histological express examination of a tumour node of a mammary gland revealed some
encapsulated formation with proliferation of alveoli and intralobular ducts; the interstitial connective
tissue grew either around or inside the ducts. Which of the tumours took place?
A. Foliaceous tumour
B. Noninfiltrating intralobular carcinoma
C. Infiltrating intralobular carcinoma
D. Paget's disease
E. * Fibroadenoma
104. During an operation on a woman, her cyst-like changed ovary was removed; it was a thin-walled
cavity filled with some yellowish transparent fluid and having a smooth inner surface. Histologically,
the cavity wall was lined with the cubical epithelium. Name the kind of the tumour.
A. Mucinous cystadenoma

B. Serous cystadenocarcinoma
C. Pseudomucinous cystocarcinoma
D. Granulosa cell tumour
E. * Serous cystadenoma
105. A histological examination of a biopsy from a uterine cervix revealed that its tissue was covered with
a wide layer of the stratified squamous epithelium having foci of proliferation of atypical cells with
pathological mitoses, but the basal membrane of the epithelium was not affected. What is your
diagnosis?
A. Nonkeratinizing squamous cell carcinoma
B. Keratinizing squamous cell carcinoma
C. Leukoplakia
D. Epithelial dysplasia
E. * Carcinoma in situ
106. A histological examination of some spherical neoplasm located under the surface of the skin,
revealed papilliform vegetations of the epithelium with phenomena of acanthosis and
hyperkeratinization. The tumour stroma consisted of a large amount of the connective tissue and
vessels. What tumour took place?
A. Keratoacanthoma
B. Carcinoma in situ
C. Keratinizing squamous cell carcinoma
D. Nonkeratinizing squamous cell carcinoma
E. * Papilloma
107. A microscopic examination of a gastrobiopsy from a tumour of the pyloroduodenal portion of the
stomach revealed layers of atypical epithelial cells with a large number of mitoses; the tumour
architectonics is characterized by prevalence of the parenchyma over the stroma. Which of the
histological forms of carcinoma listed below was the most probable?
A. Adenocarcinoma
B. Solid carcinoma
C. Mucinous carcinoma
D. Small-cell carcinoma
E. * Medullary carcinoma
108. A 26-year-old male patient underwent surgical removal of a tumour, 4 x 5 cm in size, which was
surrounded by a capsule and located in the white matter of his brain. Microscopically, the tumour
consisted of the stellate and glia cells having various size and located among the glial fibres. Name
the tumour.
A. Oligodendroglioma
B. Astroblastoma
C. Glioblastoma
D. Ependymoma
E. * Astrocytoma
109. A 45-year-old male underwent surgical removal of a tumour, 4 x 3 cm in size, from the lateral
ventricle of his brain; the tumour surface had small papillae, and it was connected with a vascular
plexus. Microscopically, the tumour consisted of villus-like vegetations covered with epithelial cells
of the cubical and columnar shape and the monomorphous kind. Which of the tumours listed below
was the most probable?
A. Ependymoma
B. Ependymoblastoma
C. Choriocarcinoma
D. Glioblastoma

E. * Choriopapilloma
110. An encapsulated tumour, 2 cm in diameter, surgically removed from an amputation stump of a lower
extremity, microscopically consists of spindle cells of the monomorphous kind with rod-shaped
nuclei which form "fence-like" structures together with fibres. Which of the tumours listed below is
the most probable?
A. Neurofibroma
B. Malignant neurilemmoma
C. Soft fibroma
D. Fibrosarcoma
E. * Benign neurilemmoma
111. A microscopic examination of a biopsy from a deformed mucous membrane of a lobar bronchus of a
45-year-old male, who smoked for many years, revealed a carcinoma consisting of atypical epithelial
cells with hyperchromatic nuclei and numerous pathological mitoses. The growth of the tumour did
not spread to the basal membrane of the epithelium. Name the histological form of carcinoma.
A. Small-cell carcinoma
B. * Carcinoma in situ
C. Squamous cell carcinoma
D. Adenocarcinoma
E. Solid carcinoma
112. A bronchoscopy of the mucous membrane of the main bronchus revealed some tumour. A
microscopic examination of the tumour biopsy showed that it consisted of lymphocyte-like cells with
hyperchromatic nuclei growing in the form of layers or bands and involving the submucous layer.
The tumour had many pathological mitoses. Which of the histological forms of carcinoma listed
below was the most probable?
A. Squamous cell carcinoma
B. Adenocarcinoma
C. Adenoacanthoma
D. Scirrhous carcinoma
E. * Small-cell carcinoma
113. On supersonic examination of a 48-year-old male patient, a hepatic neoplasm was diagnosed and a
puncture biopsy was made. Microscopically, the tumour consisted of atypical hepatocytes which
formed trabeculae, acini or tubules. The tumour stroma was poor and had thin-walled blood vessels.
Which of the kinds of tumours listed below was the most probable?
A. Hepatocellular adenoma
B. Metastasis of adenocarcinoma
C. Cholangiocellular carcinoma
D. Solid carcinoma
E. * Hepatocellular carcinoma
114. A patient has hoarseness of voice. Duri-ng laryngoscopy a gray-white larynx tumor with papillary
surface has been detected.Microscopic investigation has shown the following: growth of connective
tissue covered with multilayer, strongly kerati-nized pavement epithelium, no cellular atypia. What is
the most likely diagnosis?
A. Fibroma
B. Polyp
C. Angioma
D. Angiofibroma
E. * Papilloma

115. A tumour removed from the white matter of the right hemisphere of the brain is some soft "motley"
node, 4 cm in diameter, without any clear borders with the substance of the brain. Microscopically,
the tumour consists of polymorphous cells with numerous pathological mitoses, and it also reveals
foci of necrosis and haemorrhages which occurred at different time. Name the tumor.
A. Oligodendroglioma
B. Oligodendroglioblastoma
C. Astrocytoma
D. Astroblastoma
E. * Glioblastoma
116. Autopsy of a man who died of intoxication revealed cachexia, muscular atrophy, wrinkled skin,
decreased mass of the inner organs, stenosing tumor of the stomach with metastases to the liver and
regional nodes. Which type of cachexia is most probable?
A. Alimentary
B. Hypophyseal
C. Cerebral
D. In chronic infectious disease
E. * Cancerous
117. In a 37-year-old female patient, an enlarged dense mammary gland was revealed, the nipple with the
areola of the mammary gland were oedematous, the skin had an appearance of an "intradermal bleb".
On microscopic examination, the gland tissues revealed layers of tumour cells with polymorphous
nuclei and a large number of pathological mitoses. The tumour stroma was poorly expressed. Make
the diagnosis.
A. Adenofibroma
B. Scirrhous carcinoma
C. Paget's disease
D. Adenoma
E. * Medullary carcinoma
118. A patient with acute myeloblast leukemia has developed liver and spleen enlargement, anemia,
myeloblasts in peripheral blood. What principal morphological sign allows differing myeloblast
leukemia from chronic one?
A. * Blast cells in peripheral blood
B. Thrombocytopenia
C. Pancytopenia
D. Anemia
E. Leukemic collapse
119. In course of an operation on account of a granuloma in the area of the right upper incisor a patient
began to bleed. The hemorrhage was stopped just only 3 hours later. The patient’s anamnesis contains
information about chronic lymphatic leukemia. What is the most probable cause of hemorrhage?

A. * Thrombocytopenia
B. Thrombocytopathia
C. Lymphocytosis
D. Leukopenia
E. Eosinophilia
120. Fale patient, who worked for a long period of time with benzene, develops progressing anaemia and
the haemorrhagic syndrome. A biopsy of his breastbone reveals prevalence of a fatty tissue, and there
are some small islets of haemopoiesis with solitary cells of myelopoiesis. What is your diagnosis?
A. Chronic myeloleukaemia
B. Pernicious anaemia

C. Haemolytic anaemia
D. Hypoplastic anaemia
E. * Aplastic anaemia
121. The patient who long worked with benzene, progressing anemia and hemorrhagic syndrome. In the
biopsy of sternum predominant adipose tissue, revealed a few small foci with isolated blood cells
myelopoesis. Your diagnosis.
A. Chronic myeloid leukemia
B. Pernicious anemia
C. Hemolytic anemia
D. Iron deficiency anemia
E. * Aplastic anemia
122. An autopsy of a 76-year-old male, who smoked for a long period of time, lived sedentary life and had
redundant weight, revealed in the intima of the aorta some grey-yellow spots and stripes, fibrous
plaques, calcified areas with haemorrhages and calcinosis. What disease do these changes indicate?
A. Nonspecific aortoarteritis
B. Hypertensive disease
C. Systemic lupus erythematosus
D. Visceral syphilis
E. * Atherosclerosis
123. An autopsy of a 27-year-old male, who died suddenly, revealed in the intima of the abdominal aorta
some yellow foci in the form of spots and stripes, which did not rise above the surface of the intima
but after staining with sudan III became orange. What stage in the morphogenesis of atherosclerosis
was revealed?
A. Atherocalcinosis
B. Liposclerosis
C. Atheroma
D. Prelipid
E. * Lipoidosis
124. A microscopic examination of the wall of an aorta revealed a focal infiltration of the intima by lipids
and proteins. The lipids impregnated the intima and accumulated in the muscle cells and
macrophages. Determine the stage of atherosclerosis.
A. Prelipid
B. Liposclerosis
C. Atheromatosis
D. Atherocalcinosis
E. * Lipoidosis
125. A microscopic examination of the wall of an aorta revealed a focal infiltration of the intima by lipids
and proteins whith enlargement of the connective tissue around. Determine the stage of
atherosclerosis.
A. Prelipid
B. Lipoidosis
C. Atheromatosis
D. Atherocalcinosis
E. * Liposclerosis
126. Аt autopsy of a 38-year-old male, who died in a car accident, revealed in his aorta some
yellow-grey spots and stripes which did not rise above the surface of the intima.
Microscopically, the intima had an abundant deposition of proteins, plasma, fibrin, GAG,
cholesterol, low-density lipoproteins; the endothelium had foci of affection. Name the stage
A. of morphogenesis of atherosclerosis.
Prelipid

B. Lipoidosis
C. Atheromatosis
D. Atherocalcinosis
E. * Liposclerosis
127. An autopsy of a 70-year-old male, who died of cardiovascular insufficiency mill during his life-time
suffered from angina pectoris, hypercholesterolaemia and obesity, revealed a chronic venous plethora
of the organs, hypertrophy of the left ventricle of the heart with microfocal cardiosclerosis,
voluminous yellow plaques in the intima of the aorta with fine-grained masses in their centre and
these masses went deep into the thickness of the wall. Which of the stages of atherosclerosis listed
below was the most probable?
A. Prelipid
B. Lipoidosis
C. Liposclerosis
D. Atherocalcinosis
E. * Atheromatosis
128. A 75-year-old male was hospitalized complaining of a sharp pain in the abdominal cavity, weakness,
filiform pulse. During an operation it was found that the paraaortic fat was imbibed with blood. The
abdominal aorta had a sac-like protrusion, its wall was thinned and had an area of rupture. What
disease caused the complication?
A. Coronary disease
B. Hypertensive disease;
C. Cardiomyopathy
D. Systemic vasculitis
E. * Atherosclerosis
129. An autopsy of a 70-year-old male patient, who died from cardiac failure, revealed deformed and
narrowed coronary arteries. On section, the inner surface of the arteries was tuberous, the wall was
whitish, fragile and stony in consistency. Which of the diagnoses listed below was the most
probable?
A. Ulceration
B. Liposclerosis;
C. Atheromatosis
D. Lipoidosis
E. * Atherocalcinosis
130. An autopsy of a 48-year-old male, who died in a car accident, revealed in his aorta some yellow-grey
spots and stripes which did not rise above the surface of the intima. Microscopically, the intima had
an abundant deposition of proteins, plasma, fibrin, GAG, cholesterol, low-density lipoproteins; the
endothelium had foci of affection. Name the stage of morphogenesis of atherosclerosis.
A. Prelipid
B. Liposclerosis
C. Atheromatosis
D. Atherocalcinosis
E. * Lipoidosis
131. A microscopic examination of the wall of an aorta revealed a focal infiltration of the intima by lipids
and proteins. The lipids impregnated the intima and accumulated in the muscle cells and
macrophages. Determine the stage of atherosclerosis.
A. Prelipid;
B. Liposclerosis;
C. Atheromatosis;
D. Atherocalcinosis;

E. * Lipoidosis
132. The patient, who has long suffered from intermittent claudication, gangrene developed dry foot. Add
a disease that led to such complications.
A. Nodular arteritis;
B. Arteriolosklerosis;
C. Arteriosclerosis
D. Coronary arteritis;
E. * Atherosclerosis
133. An autopsy of a male, who suffered from arterial hypertension during his life_time, revealed oedema
of the brain substance, arterial walls at the base were thickened, with white-yellowish plaques in the
intima, the left hemisphere had a focus, 5 x 4 x 3 cm in size, representing a cavity filled with liquid
blood and its clots. Define the pathological process in the brain.
A. Transitory ischaemia
B. Haemorrhagic infarct
C. Anaemic infarct
D. Mixed infarct
E. * Haematoma
134. Autopsy of a 75-year-old patient who had been suffering from disseminated atherosclerosis and died
under chronic cardiac failure revealed constriction and deformation of coronary arteries, tuberous
intima whose section appeared to be white and petrosal. Specify the stage of atherosclerosis
morphogenesis:
A. Atheromatosis
B. Lipoidosis
C. Bilipid
D. Liposclerosis
E. * Atherocalcinosis
135. Morphological examination of an amputated gangrenous extremity revealed that the lumen of
femoral artery was constricted due to stony, partly ulcerated plaques with obturating thrombi. What is
the most likely diagnosis?
A. Obliterating endartheriitis
B. Nodular periarthritis
C. Obliterating thromboangiitis
D. Non-specific aortoartheriitis
E. * Atherosclerosis
136. Obliterating atherosclerosis causes changes in the vessels of the lower extermities. A histological
specimen of such a vessel evidently presents both internal and external elastic membranes, middle
membrane contains a lot of myocytes. What vessel is affected in case of this disease?
A. Vein with strongly developed muscles
B. Artery of elastic type
C. Artery of mixed type
D. Lymph node
E. * Artery of muscular type
137. On autopsy of a 68-year-old male, who died from cardiac decompensation, the myocardium of the
anterior wall in left ventricle of his heart contained an irregular grey focus, 5 x 4 cm in size, with a
dense consistency, fibrous structure and clear borders. What pathological process in the myocardium
did the pathologist reveal?
A. Myocarditis
B. Microfocal cardiosclerosis
C. Infarction
D. Rheumatism
E. * Postinfarction cardiosclerosis
138. A male patient developed substernal pains at 8 a.m., and at 9 a.m. myocardial infarction was
diagnosed by ECG data at the admission department. Ten minutes later the patient died. What most
reliable sign of myocardial infarction will be found on histological examination?
A. Vacuole dystrophy of cardiomyocytes;
B. Fat infiltration of cardiomyocytes;
C. Relaxation of myofibrils in cardiomyocytes
D. Reduced activity of dehydrogenases in fibroblasts;
E. * Disappearance of glycogen in cardiomyocytes;
139. Three weeks following a myocardial infarction, a 54-year-old man presents with fever, productive
cough, and chest pain. The pain is worse with inspiration, better when he is sitting up, and not
relieved by nitroglycerin. Physical examination finds a friction rub along with increased jugular
venous pressure and pulsus paradoxus (excess blood pressure drop with inspiration). Which of the
following is the most likely explanation for these findings?
A. Caplan’s syndrome;
B. Dressler’s syndrome;
C. Ruptured papillary muscle;
D. Ventricular aneurysm.
E. * Ruptured ventricular wall;
140. On autopsy of a 66-year-old male, who died from acute cardiac failure, an acute venous plethora of
the internal organs was found. The cardiac cavities were dilated; a myocardial section revealed some
dim yellowish focus, 3.5 x 4 cm in size, in the anterior wall of the left ventricle. The coronary arteries
had stenosing atheromatous plaques. Which of the diagnoses listed below was the most probable?

A. Myocarditis
B. Microfocal cardiosclerosis
C. Macrofocal cardiosclerosis
D. Fatty degeneration of myocardium
E. * Myocardial infarction
141. A 3-month-old girl is being evaluated for feeding difficulty and failure to thrive. Physical
examination finds pallor, peripheral cyanosis, tachypnea, and fine expiratory wheezing. Chest x-ray
shows cardiac enlargement. She is admitted to the hospital, quickly develops severe cardiac failure,
and dies 3 days after admission. At the time of autopsy the endocardium is found to have a “cream
cheese” gross appearance. Histologic sections from this area reveal thickening of the endocardium
due to a proliferation of fibrous and elastic tissue. Which of the following is the most likely
diagnosis?
A. Dilated cardiomyopathy;
B. Hypertrophic cardiomyopathy;
C. Infective endocarditis;
D. Libman-Sachs endocarditis;
E. * Restrictive cardiomyopathy
142. A 49-year-old man 7 days after being admitted to the hospital for an inferior wall, transmural
myocardial infarction suddenly becomes short of breath. Physical examination reveals hypotension,
elevated jugular venous pressure, and muffled heart sounds. His systemic blood pressure drops 13
mmHg with inspiration. Which one of the following pathologic processes produced these clinical
findings?
A. Acute inflammation of the pericardium due to an autoimmune reaction
B. Acute mitral regurgitation due to rupture of a papillary muscle
C. Acute suppurative inflammation of the pericardium due to bacterial infection

D. Serous fluid accumulation in the pericardial cavity due to congestive heart failure
E. * Blood accumulation in the pericardial cavity due to rupture of the ventricular wall
143. A 78-year-old patient suffering from atherosclerosis has been delivered to a surgical ward with signs
of acute abdomen. Laparoscopy revealed blackened and flaccid small intestine loops; demarcation
line is not clear. Diagnose the changes that occurred in the patient’s small intestine:
A. Hemorrhagic infarction complicated with dry gangrene
B. Ischemic stroke complicated with humid gangrene
C. Ischemic stroke complicated with dry gangrene
D. -
E. * Hemorrhagic infarction complicated with wet gangrene
144. A 65-year-old patient had been treated for 3 days in a resuscitation unit for a cardiac pathology.
Suddenly he developed ventricular fibrillation that became the immediate cause of death of this
patient. Microscopy of the left ventricular myocard revealed a large focus of cardiomyocyte
karyolysis demarcated by the zone of hyperaemia. What cardiac pathology was the cause of death?
A. Ischemic myocardial degeneration
B. Acute myocarditis
C. Diffuse cardiosclerosis
D. Postinfarction cardiosclerosis
E. * Acute myocardial infarction
145. In the dead from complications of hypertension, during the section detected small, dense kidneys
with grainy surface, atrophied parenchyma and cortical tissue. Define such kidneys.
A. Amyloid-wrinkled
B. Atrophic
C. Secondary wrinkled
D. Pyelonephrotic wrinkled
E. * Primary wrinkled
146. At autopsy the patient died of heart failure, enlarged heart discovered a mass of 550 g, fibrinous
pericarditis and wrinkled, dense buds weighing 50 grams, with a fine surface and pronounced
Hyalinosis arterioles and glomeruli. What are the underlying disease.
A. Atherosclerosis
B. Rheumatism
C. Pericarditis
D. Cardiomyopathy
E. * Hypertension
147. At autopsy of the dead men, who suffered from hypertension, in the brain revealed a cavity whith
rusty color walls. What preceded these phenomena?
A. Diapedesis hemorrhage
B. Abscess
C. Ischemic heart attack
D. Plasmorrhage
E. * Hematoma
148. Three weeks following a myocardial infarction, a 54-year-old man presents with fever, productive
cough, and chest pain. The pain is worse with inspiration, better when he is sitting up, and not
relieved by nitroglycerin. Physical examination finds a friction rub along with increased jugular
venous pressure and pulsus paradoxus (excess blood pressure drop with inspiration). Which of the
following is the most likely explanation for these findings?
A. Caplan’s syndrome
B. Dressler’s syndrome

C. Ruptured papillary muscle


D. Ventricular aneurysm
E. * Ruptured ventricular wall
149. In a patient with hypertension disease, in the context of a hypertensive crisis, acute renal failure, from
which he died, developed. What are the most likely morphological changes in kidney arterioles?

A. Constrictive atherosclerosis
B. Hyperelastosis
C. Sclerosis
D. Hyalinosis
E. * Fibrinoid necrosis
150. At the autopsy of the deceased who suffered from hypertension, a cavity of round shape of 4 ? 5 cm
with a rusty wall, filled with a yellowish transparent liquid, was found in the left hemisphere of the
brain. What is the pathology that has developed in the brain of the patient?
A. Hematoma
B. Hemorrhagic leakage
C. Ischemic heart attack
D. Abscess
E. * Cyst
151. Autopsy has revealed shrunken kidneys weighing 50 mg, with finegrained surface and uniformly
thinned substance. Microscopic investigation has shown the thickening of arteriole walls due to
accumulation of homogeneous anhistic pink-colored masses in them. Glomerules were undersized,
sclerotic, with atrophied tubules. What disease are these changes characteristic of?
A. Membranous nephropathy
B. Pyelonephritis with kidney shrinkage
C. Renal amyloidosis
D. Acute glomerulonephritis
E. * Essential hypertension
152. At autopsy the occipital lobe of brain was found to have a cavity 2,5x1,5 cm large filled with a
transparent liquid. The cavity had smooth brownish walls. What process had developed in the brain?
A. A cyst on the site of the softening of the cerebrocortical grey matter
B. Softening of the cerebrocortical grey matter
C. Brain abscess
D. Paracephalia
E. * Cyst on the site of a hemorrhage
153. An autopsy of a male patient, who died from heart failure, revealed an enlarged heart weighing 550 g,
fibrinous pericarditis, as well as contracted dense kidneys weighing 50 g each and having a
fine-grained surface. Microscopically, the kidneys were characterized by an expressed hyalinosis of
arteioles and glomeruli. Name the basic disease.
A. Pericarditis
B. Rheumatism
C. Atherosclerosis
D. Cardiomyopathy
E. * Hypertensive disease
154. Against a background of hypertensive crisis, a male patient with hypertensive disease developed
acute renal insufficiency which caused his death. What morphological changes in the renal arteioles
were the most probable?
A. Sclerosis

B. Hyperelastosis
C. Stenosing atherosclerosis
D. Hyalinosis
E. * Fibrinoid necrosis
155. A 52-year-old male patient with elevated blood pressure (250/120 mm Hg) died from an impairment
of his cerebral circulation. An autopsy of the brain revealed a red focus in the thalamus, 2.5 cm in
diameter, which sank on section. Microscopically, there was fibrinoid necrosis of the vascular walls
and impregnation of the necrotized brain tissue with blood. Which of the diagnoses listed below was
the most probable?
A. Mixed infarct
B. Cerebral haematoma
C. Anaemic infarct of brain
D. -
E. * Haemorrhagic infarct of brain
156. An autopsy of a male, who suffered from arterial hypertension during his life-time, revealed oedema
of the brain substance, arterial walls at the base were thickened, with white-yellowish plaques in the
intima, the left hemisphere had a focus, 5 x 4 x 3 cm in size, representing a cavity filled with liquid
blood and its clots. Define the pathological process in the brain.
A. Mixed infarct
B. Haemorrhagic infarct
C. Anaemic infarct
D. Transitory ischaemia
E. * Haematoma
157. At autopsy the diminished kidneys with weight of 50.0 have been found out, the surface has been
closed-grained, cortex has been uniformly thinned. At microscopic examination a wall of arterioles
has been considerably thickened because of deposition of homogeneous unstructured pink colored
masses, the lumen has sharply narrowed down, the glomeruli have been reduced, with sclerosis and
atrophy of tubules. What disease the described changes are typical for?
A. Acute necrotic nephrosis
B. Chronic glomerulonephritis
C. Amyloidosis of kidney
D. Pyelonephritis with shrinkage of kidneys
E. * Hypertensive disease
158. The patient, aged 74, with the history of hypertensive syndrome, lost consciousness suddenly and
died of increasing disturbance of respiration and heart activity. The autopsy has demonstrated a
dark-red focus in the trunk of the brain measuring 2x1x5 cm. The weight of the heart is 550 g. the
thickness of left ventricle wall is 2,5 cm. The vessels of the brain base are considerably thickened,
whitish-yellow, the lumen is narrowed. What is the diagnosis?
A. Chronic bronchitis
B. Ischemic heart disease
C. Glomerulonephritis
D. Atherosclerosis
E. * Hypertensive disease
159. An autopsy of a 9-year-old child, who suffered from rheumatism and died of heart failure, revealed
dilatation of cavities in the ventricles of his heart. Microscopically, the myocardial stroma was
characterized by a plethora, oedema, diffuse infiltrations of histiocytes, lymphocytes, neutrophils and
eosinophils. What diagnosis was the most probable one?
A. Focal interstitial exudative myocarditis
B. Granulomatous productive myocarditis

C. Interstitial productive myocarditis


D. Alterative myocarditis
E. * Diffuse interstitial exudative myocarditis
160. An autopsy of a female revealed morphological manifestations of chronic heart failure in the right
ventricle, stenosis of the left atrioventricular aperture, insufficiency of the mitral valve.
Histologically, a connective-tissue disorganization in the form of some mucoid and fibrinoid swelling
was found with presence of blooming Aschoff s bodies against a background of focal cardiosclerosis
in the myocardium. Which of the diagnoses listed below was the most probable?
A. Scleroderma
B. Dermatomyositis
C. Polyarteritis nodosa
D. Systemic lupus erythematosus
E. * Rheumatism
161. A room for dissections received the body of a 56-year-old male who was ill with rheumatism during 8
years and died from cardiovascular insufficiency. An autopsy revealed the rheumatic defect of the
heart -mitral incompetence. A histological examination revealed oedema of the interstice, its diffuse
infiltration by lymphocytes, histiocytes, neutrophilic and eosinophilic leukocytes, as well as
parenchymatous protein and fatty degeneration of the cardiomyocytes. The left atrial auricle had foci
of fibrinoid necrosis surrounded by large macrophages which were located in the form of a veil.
Which of the diagnoses listed below was the most probable?
A. Diffuse isolated myocarditis
B. Focal isolated myocarditis
C. Acute serous myocarditis
D. Acute purulent myocarditis
E. * Productive granulomatous myocarditis
162. Two weeks after angina, a 14-year-old child developed pains in the heart, cyanosis, dyspnoea,
oedemata of his lower extremities. The death was caused by cardiac arrest. On autopsy, the heart was
enlarged, its cavities were dilated, the myocardium was flaccid. A microscopic examination revealed
an acute oedema and plethora of the interstice, an expressed infiltration by lymphocytes, histiocytes,
neutrophils and eosinophils, foci of dystrophy of cardiomyocytes. Which of the diagnoses listed
below was the most probable?
A. Ischaemic dystrophy of myocardium
B. Nodular productive myocarditis
C. Idiopathic myocarditis
D. Focal interstitial exudative myocarditis
E. * Diffuse interstitial exudative myocarditis
163. A room for dissections received the body of a 56-year-old male who was ill with rheumatism during 8
years and died from cardiovascular insufficiency. An autopsy revealed the rheumatic defect of the
heart -mitral incompetence. A histological examination revealed oedema of the interstice, its diffuse
infiltration by lymphocytes, histiocytes, neutrophilic and eosinophilic leukocytes, as well as
parenchymatous protein and fatty degeneration of the cardiomyocytes. The left atrial auricle had foci
of fibrinoid necrosis surrounded by large macrophages which were located in the form of a veil
(Aschoff’s granulomA.. Which of the diagnoses listed below was the most probable?
A. Diffuse isolated myocarditis
B. Focal isolated myocarditis
C. Acute serous myocarditis
D. Acute purulent myocarditis
E. * Productive granulomatous myocarditis
164. What form of rheumatism myocardial lesions observed more frequently in adults?
A. Eosinophilic myocarditis

B. Diffuse acute serous myocarditis


C. Exudative focal myocarditis
D. Alterative myocarditis
E. * Poductive (granulomatous) myocarditis
165. During autopsy of the body of a patient, who had died due to heart failure, the following has been
detected: myogenic dilatation of the heart left ventricle, microfocal cardi- osclerosis, vasculitis,
Aschoff bodies with disorganization of connective tissue, myocardosis. Make the diagnosis:
A. Rheumatic exudative myocarditis
B. Cardiac infarction
C. Systemic lupus erythematosus
D. Myocardial ischemic dystrophy
E. * Rheumatic productive myocarditis
166. Postmortem examination of a patient with a long history of rheumatism revealed thickening and
shortening of the mitral valve leaflets with abundant thrombotic deposits. Histological examnation of
the valve leaflets confirmed sclerosis and revealed multiple foci of connective tissue disorganization
in form of mucoid and fibrinoid swelling, as well as deendothelization foci. Endothelium defects
were covered with thrombotic deposits of 1-2 mm. What type of valvular endocarditis is the case?
A. Acute verrucous endocarditis
B. Fibroplastic endocarditis
C. Diffuse valvulitis
D. Polypous-ulcerative endocarditis
E. * Recurrent verrucous endocarditis
167. Examination of a patient who had been suffering from rheumatism for a long time revealed stenosis
of mitral orifice, death was caused by cardiac and pulmonary insufficiency. Autopsy has shown
brown induration of lungs. What type of circulation disturbance provokes such changes in lungs?
A. Chronic right ventricular insufficiency
B. Acute left ventricular insufficiency
C. Acute right ventricular insufficiency
D. Portal hypertension
E. * Chronic left ventricular insufficiency
168. A 36-year-old female suffers from an expressed deformity of joints of her lingers und Iocs.
Histologically, the periarticular connective tissue reveals some mucoid swelling, foci of fibrinoid
necrosis, clusters of macrophages and areas of sclerosis, the synovial membrane has an oedema of
villi, as well as their mucoid and fibrinoid swelling, the synovial cavity contains "rice bodies". Make
a diagnosis of the disease.
A. -
B. Rheumatism
C. Bekhterev's disease
D. Infectious polyarthritis
E. * Rheumatoid arthritis
169. An autopsy of a 45-year-old female, who died from cardiac failure, revealed on the lateral surfaces of
her both cheeks some reddish-brownish spots which fused on the bridge of the nose. The heart was
enlarged, the myocardium was flaccid in consistency, the cusps of the aortic valve were thickened
and had thrombotic superpositions. The kidneys were motley and had focal haemorrhages. The
pyramids of the medullary layer were dark red, the cortical layer was greyish and had red specks. A
microscopic examination of the kidneys revealed haematoxylin bodies in the tubular epithelial nuclei,
the basal membranes of capillaries of the glomeruli were thickened and gave an appearance of wire
loops, somewhere the walls of the capillaries contained hyaline thrombi and foci of fibrinoid
necrosis. Which diagnosis was the most probable?
A. Rheumatic defect of heart

B. Atherosclerotic defect of heart


C. Septic endocarditis
D. Subacute glomerulonephritis
E. * Systemic lupus erythematosus
170. Histological investigation of skeletal muscle detected lymphocytic infiltration of the walls arteries,
sometimes circular and segmental fibrinoid necrosis, cell proliferation segments of the outer shell of
a transition sclerosis and the formation of small plots thickening of the walls of arteries. Define
pathological process.
A. Wegener's granulomatosis
B. Takayasu's disease
C. Horton disease
D. Syphilitic vasculitis
E. * Polyarteritis nodosa
171. A clinical examination of a 41-year-old male patient revealed some deformity in small joints of his
extremities. A microscopic examination of a biopsy of the synovial coat revealed foci of mucoid and
fibrinoid swelling and fibrinoid necrosis in the stroma of villi and vascular walls, proliferation of
synoviocytes, a perivascular infiltration by lymphocytes, plasmacytes and neutrophils; there was IgG
in the plasmacytes. Which of the diagnoses listed below was the most probable?
A. Polyarthritic form of rheumatism
B. Arthritis in systemic lupus erythematosus
C. Rheumatoid arthritis (stage II)
D. Rheumatoid arthritis (stage III)
E. * Rheumatoid arthritis (stage I)
172. Male 47 years died of a heart attack. Histologically in the wall of the aortic arch and the vessels
departing from it revealed granulomatous arteritis with a mononuclear infiltrate predominance in a
small number of giant multi-cell destruction of elastic fibers, smooth myocyte necrosis and parietal
thrombosis. What disease in this case is most likely?
A. Nonspecific aortitis
B. Rheumatism
C. C Polyarteritis nodosa
D. Wegener's granulomatosis
E. * Takayasu's disease
173. Histological examination of a kidney patient 26 years old who died from renal failure revealed
capillary membranes of glomeruli in the form of wire loops, hyaline thrombi in capillaries, centers of
fibrinoidal necrosis. What is the most likely diagnosis?
A. Nodular periarthritis
B. Rheumatism
C. Scleroderma
D. Rheumatoid arthritis
E. * Systemic lupus erythematosus
174. In a patient with systemic connective tissue disease, the dryness of the conjunctiva and oral cavity is
noted. Puncture biopsy revealed the immune destruction of the salivary glands. Which disease is
most probable?
A. Rheumatism
B. Scleroderma
C. Calculous sialoadenitis
D. Systemic lupus erythematosus
E. * Shegren's syndrome

175. The histological examination of the deformed mitral valve revealed a pronounced basophilic reaction
in the connective tissue of the valve when colored with hematoxylin and eosin, and when the
toluidine blue colored, the reaction was metachromasia. What changes in connective tissue reveal
these reactions?
A. Amyloidosis
B. Fibrinoid swelling
C. Gyalinosis
D. Fibrinoid necrosis
E. * Mucoid swelling
176. In a patient after supercooling, deformity of the joints, pain and limitation of movements in the
fingers of the extremities developed; small dense nodules appeared around the joints. At the biopsy,
nodules found tricus of fibrinous necrosis surrounded by histiocytes. Your diagnosis?
A. Deforming arthrosis
B. Dermatomyositis
C. Rheumatism
D. Gout
E. * Rheumatoid arthritis
177. A patient with high-titer antinuclear antibodies died from progressing renal impairment. Autopsy
revealed mesangioproliferative glomerulonephritis and abacterialpolypous endocarditis. Periarterial
bulbar sclerosis was detected in spleen and productive proliferative vasculitis in skin. What is the
most likely diagnosis?
A. Periarteritisnodosa
B. Nephrotic syndrome
C. Rheumatism
D. Dermatomyositis
E. * Systemic lupus erythematosus
178. A 70-year-old male patient died from acute coronary insufficiency. He had knee joint swelling,
gonycampsis and gonalgia during his lifetime. Pathomorphologic examination of the deformed joints
and synovial membranes revealed membrane hyperaemia with multiple perivascular inflammatory
infiltrations made by lymphocytes, plasmocytes and macrophagocytes. There was an accumulation of
organized fibrin covering some areas of synovium membrane and looking like rice grains in the
articular liquid. What is the most likely diagnosis?
A. Deforming arthrosis
B. Periarteritis nodosa
C. Ankylosing spondylitis
D. Tuberculous arthritis
E. * Atrophic arthritis
179. In autopsy of the 40-year-old woman suffering from rheumatic arthritis, the enlarged solid spleen was
revealed. On section its tissue is of the mahogany color with enlarged follicles, which look like
semitransparent grayish-whitish grains. What pathological process is the most likely?
A. * Sago spleen
B. Glaze spleen
C. Hyaline spleen
D. Porphyric spleen
E. Waxy spleen
180. A 45-year-old patient has died because of uremia in a systemic lupus erythematosus. Formation of
what antigens is most probable in the mechanism of development of this disease?
A. * To native DNA
B. To myocytes

C. To a myosin
D. To endothelial cells
E. Of the rheumatoid factor
181. A patient ill with tuberculosis died from progressing cardiopulmonary decompensation. Autopsy in
the area of the right lung apex revealed a cavity 5 cm in diameter communicating with lumen of a
segmental bronchus. On the inside cavity walls are covered with caseous masses with epithelioid and
Langhans cells beneath them. What morphological form of tuberculosis is it?
A. Tuberculoma
B. Caseous pneumonia
C. Infiltrative tuberculosis
D. Acute focal tuberculosis
E. * Acute cavernous tuberculosis
182. A male patient is 28 years old. Histological study of a cervical lymph node revealed a change of its
pattern due to the proliferation of epithelioid, lymphoid cells and macrophages having nuclei in form
of a horseshoe. In the center of some cell clusters there were non-structured light-pink areas with
fragments of nuclei. What disease are these changes typical for?
A. Hodgkin’s disease
B. Actinomycosis
C. Tumor metastasis
D. Syphilis
E. * Tuberculosis
183. A 63-year-old man, who has been suffering from chronic fibrous-cavernous pulmonary tuberculosis
for 24 years, has been delivered to a nephrology department with uremia. Intravital diagnostic test for
amyloid in the kidneys was positive. What amyloidosis is it in this case?
A. Primary systemic
B. Localized (focal)
C. Hereditary (genetic)
D. Senile
E. * Secondary systemic
184. A 3-year-old child with meni-ngeal symptoms died. Postmortem macroscopy of the pia matter
revealed miliary nodules which were microscopically represented by a focus of caseous necrosis with
masses of epithelioid and lymphoid cells with some crescent-shaped large cells inbetween having
peripheral nuclei. Specify the type of meningitis in the child:
A. Syphilitic
B. Brucellar
C. Grippal
D. Meningococcal
E. * Tuberculosis
185. A 40-year-old prisoner died of tuberculosis in the corrective labor camp. Autopsy of the body
revealed deformation and diminishing of both lung apices; in the both upper lobes there are multiple
cavities with dense walls 2-3 mm thick; in the lower lung lobes there are disseminated foci of caseous
necrosis varying from 5 mm to 2 cm in diameter. Diagnose the type of tuberculosis:
A. Secondary fibrous-focal tuberculosis
B. Hematogenous macrofocal pulmonary tuberculosis
C. Primary tuberculosis, primary affect development
D. Secondary cirrhotic tuberculosis
E. * Secondary fibro-cavitary tuberculosis

186. Autopsy of a man who had tuberculosis revealed a 3-2 cm large cavity in the superior lobe of the
right lung. The cavity was interconnected with a bronchus, its wall was dense and consisted of three
layers: the internal layer was pyogenic, the middle layer was made by tuberculous granulation tissue
and the external one was made by connective tissue. What is the most likely diagnosis?
A. Fibrous focal tuberculosis
B. Tuberculoma
C. Acute focal tuberculosis
D. Acute cavernous tuberculosis
E. * Fibrous cavernous tuberculosis
187. Autopsy of a young man revealed some lung cavities with inner walls made up of granulation tissue
with varying degrees of maturity; pronounced pneumosclerosis and bronchiectasis. Some cavities had
caseation areas. What is your presumptive diagnosis?
A. Infiltrative tuberculosis
B. Caseous pneumonia
C. Acute cavernous tuberculosis
D. Bronchiectasis
E. * Fibrous cavernous tuberculosis
188. A patient was suffering from primary tuberculosis 5 years ago. Radiography has revealed a sharply
marginated nodular shadow with diameter of 4 cm in the 2nd segment of the right lung. Focus was
surgically removed. Histological study has revealed the following: the focus of caseous necrosis
surrounded by the thick capsule of connective tissue. What kind of secondary tuberculosis has
occurred in patient?
A. Cirrhotic tuberculosis
B. Acute cavernous tuberculosis
C. Fibro-cavernous tuberculosis
D. Caseous pneumonia
E. * Tuberculoma
189. A 4-year-old child with meningeal symptoms died. Postmortem macroscopy of the pia matter
revealed miliary nodules which were microscopically represented by a focus of caseous necrosis with
masses of epithelioid and lymphoid cells with large cells containing crescent-shaped peripheral
nuclei situated between them. Specify the type of meningitis in the child:
A. Syphilitic
B. Brucellar
C. Grippal
D. Meningococcal
E. * Tuberculosis
190. A male patient is 20 years old. Histological study of a cervical lymph node revealed a change of its
pattern due to the proliferation of epithelioid, lymphoid cells and macrophages having nuclei in form
of a horseshoe. In the center of some cell clusters there were non-structured light-pink areas with
fragments of nuclei. What disease are these changes typical for?
A. Hodgkin’s disease
B. Actinomycosis
C. Tumor metastasis
D. Syphilis
E. * Tuberculosis
191. Autopsy of a 18 year old girl who died from pulmonary failure revealed a small area of caseous
necrosis in the inferior lobe of the right lung, and occurrences of caseous necrosis in the
bronchopulmonary, bronchial and bifurcational lymph nodes. What is the most probable postmortem
diagnosis?

A. Hematogenous progression of primary tuberculosis


B. Hematogenous tuberculosis with predominant lung affection
C. Tuberculoma
D. Caseous pneumonia under secondary tuberculosis
E. * Primary tuberculosis
192. A patient with tuberculosis died from progressing cardiopulmonary decompensation. Autopsy in the
region of the right lung apex revealed a cavity 5 cm in diameter communicating with lumen of a
segmental bronchus. On the inside cavity walls are covered with caseous masses with epithelioid and
Langhan's cells beneath them. What morphological form of tuberculosis is it?
A. Tuberculoma
B. Caseous pneumonia
C. Infiltrative tuberculosis
D. Acute focal tuberculosis
E. * Acute cavernous tuberculosis
193. A man is 28 years old. Histological investigation of the cervical lymph node revealed a change of its
pattern due to proliferation of epithelioid, lymphoid cells and macrophages with horseshoe-shaped
nuclei. In the center of some cell clusters there were non-structured light-pink areas with fragments
of nuclei. What disease are these changes typical of?
A. Hodgkin’s disease
B. Actinomycosis
C. Tumor metastasis
D. Syphilis
E. * Tuberculosis
194. 48 hours after performing tuberculin test (Mantoux test) to a child a 10 mm papule appeared on the
spot of tuberculin introduction. What hypersensitivity mechanism underlies these changes?
A. Anaphylaxis
B. Antibody-dependent cytotoxicity
C. Immune complex cytotoxicity
D. Granulomatosis
E. * Cellular cytotoxicity
195. Autopsy of a 17 year old girl who died from pulmonary failure revealed a small area of caseous
necrosis in the inferior lobe of the right lung, and occurences of caseous necrosis in the
bronchopulmonary, bronchial and bifurcational lymph nodes. What is the most probable postmortem
diagnosis?
A. Hematogenous progression of primary tuberculosis
B. Hematogenous tuberculosis with predominant lung affection
C. Tuberculoma
D. Caseous pneumonia under secondary tuberculosis
E. * Primary tuberculosis
196. A 20 year old patient died from intoxication 8 days after artificial illegal abortion performed in her
14-15th week of pregnancy. Autopsy of the corpse revealed yellowish colour of eye sclera and of
skin, necrotic suppurative endometritis, multiple pulmonary abscesses, spleen hyperplasia with a big
number of neutrophils in its sinuses. What complication after abortion was developed?
A. Chroniosepsis
B. Hemorrhagic shock
C. Septicemia
D. Viral hepatitis type A
E. * Septicopyemia

197. A man with a wound of his limb that had been suppurating for a long time di-ed from intoxication.
Autopsy revealed extreme emaciation, dehydration, brown atrophy of liver, myocardium, spleen and
cross-striated muscles as well as renal amyloidosis. What diagnosis corresponds with the described
presentations?
A. Septicopyemia
B. Septicemia
C. Chernogubov’s syndrome
D. Brucellosis
E. * Chroniosepsis
198. A 71-year-old man had been presenting with diarrhea for 10 days. The feces had admixtures of blood
and mucus. He was delivered to a hospital in grave condition and died 2 days later. Autopsy of the
body revealed the following: diphtheritic colitis with multiple irregularly-shaped ulcers of different
depth in both sigmoid colon and rectus. Bacteriological analysis revealed Shigella.What was the main
disease?
A. Typhoid fever
B. Salmonellosis
C. Nonspecific ulcerous colitis
D. Yersiniosis
E. * Dysentery
199. Autopsy of a 42-year-old man revealed a distinctly dilated lumen of small intestine filled with
rice-water-like liquid. The intestine wall was edematic with lots of petechial haemorrhages on the
mucosa.What infectious disease is the described enteritis typical for?
A. Dysentery
B. Salmonellosis
C. Amebiasis
D. Typhoid fever
E. * Cholera
200. A worker of a cattle farm consulted a surgeon about fever up to 40oC, headache, weakness. Objective
examination of his back revealed hyperaemia and a dark red infiltration up to 5 cm in diameter with
black bottom in the centre and some pustules. What disease are these presentations typical for?

A. Plaque
B. Tularemia
C. Furuncle
D. Abscess
E. * Anthrax
201. A 42-year-old man died with symptoms of severe intoxication and respiratory failure. A slide of lung
tissue was heterogenous, with multiple microfocal hemorrhages and foci of emphysema. Histological
examination of lungs revealed hemorrhagic abscessing bronchopneumonia; eosinophilic and
basophilic granules in the cytoplasm of epithelial cells of bronchi. What is the most likely diagnosis?
A. Parainfluenza
B. Adenovirus infection
C. Respiratory syncytial virus infection
D. Staphylococcal bronchopneumonia
E. * Influenza
202. Autopsy of a man who died from intraintestinal hemorrhage revealed necrosis of grouped and
solitary follicles, dead tissues imbibed with bile and blood in the ileum; sequestration and rejection of
necrotic masses with defect formation in the lower segment of the intestine. Which of the following
diagnoses is most likely?

A. Typhoid fever, "clean ulcer"stage


B. Typhoid fever, necrosis stage
C. Abdominal typhoid salmonellosis
D. Crohn’s disease
E. * Typhoid fever, ulcerative stage
203. On the base of the clinical data a child was diagnosed with atypical pneumonia resistant to the effects
of beta-lactam antibiotics. The patient’s sputum was cultured and incubated in a special medium,
which resulted in growth of microorganisms formingmicroscopic colonies with a dense center
(looking like fried eggs). What microorganism caused the disease?
A. Klebsiella pneumoniae
B. Streptococcus pneumoniae
C. Legionella pneumophila
D. Chlamidia pneumoniae
E. * Mycoplasma pneumoniae
204. A patient with marked manifestations of exsicosis died in the infectious disease hospital. Postmortem
examination results: the corpse with contracted muscles, dry skin and mucous membranes, thick and
dark blood in veins, edematous plethoric mucosa, distended bowel loops, the lumen contains about 4
liters of ricewater fluid. What is the most likely diagnosis?
A. Enteric fever
B. Dysentery
C. Anthrax, intestinal form
D. Yersiniosis
E. * Cholera
205. A 47-year-old patient with symptoms of severe intoxication and respiratory failure died. A section of
lung tissue had a mottled pattern with multiple small focal hemorrhages and foci of emphysema.
Histological examination revealed hemorrhagic bronchopneumonia accompanied by abscess; the
cytoplasm of bronchial epithelial cells had eosinophil and basophil inclusions. According to the
section analysis, make your diagnosis:
A. Adenovirus infection
B. Parainfluenza
C. Respiratory syncytial
D. Staphylococcal bronchopneumonia
E. * Influenza
206. A 47-year-old male patient consulted a dentist about difficult mouth opening (lockjaw). The patient
has a history of a stab wound of the lower extremity. What infection can be manifested by these
symptoms?
A. Brucellosis
B. Whooping cough
C. Anaerobic wound infection
D. Tularemia
E. * Tetanus
207. A patient, who works as a milkmaid, has made an appoinment with a dentist with complaints of
aphtha-shaped rash on the mucosa of oral cavity. The doctor detected rash on her hands in the area of
nail plates. What agent causes this disease?
A. Herpesvirus
B. Vesicular stomatitis virus
C. Cytomegalovirus
D. Coxsackie A virus
E. * Foot-and-mouth disease virus

208. Autopsy of the body of a 46-year-old man, who had been suffering from typhoid fever and died of
intestinal hemorrhage, has revealed sequestration areas, tissue rejection in the areas of lymphoid
follicle clusters. What stage of typhoid fever is it?
A. Healing
B. Arain-like swelling of the follicles
C. Necrosis
D. Clean ulcers
E. * Dirty ulcers
209. During autopsy approximately 2,0 liters of pus have been found in the abdominal cavity of the body.
Peritoneum is dull and of grayish shade, serous tunic of intestines has grayish-colored coating that is
easily removable. Specify the most likely type of peritonitis in the patient:
A. Hemorrhagic peritonitis
B. Serous peritonitis
C. Tuberculous peritonitis
D. Necrosis
E. * Fibrinopurulent peritonitis
210. A patient, having suffered a thermal burn, developed painful boils filled with turbid liquid in the skin.
What morphological type of inflammation has developed in the patient?
A. Proliferative
B. Croupous
C. Granulomatous
D. Diphtheritic
E. * Serous
211. A 7-year-old child has acute onset of disease: temperature rise up to 38oC, rhinitis, cough,
lacrimation, and large-spot rash on the skin. Pharyngeal mucosa is edematous, hyperemic, with
whitish spots in the buccal area. What kind of inflammation caused the changes in the buccal
mucosa?
A. Suppurative inflammation
B. Fibrinous inflammation
C. Hemorrhagic inflammation
D. Serous inflammation
E. * Catarrhal inflammation
212. During autopsy of a man, who died of acute transmural cardiac infarction, the following has been
detected on the pericardium surface: fibrous whitish-brown deposit connecting parietal and visceral
pericardial layers. What kind of inflammation occurred in the pericardium?
A. Diphtheritic
B. Serous
C. Suppurative
D. Granulomatous
E. * Croupous
213. A 39-year-old man who had been operated for the stomach ulcer died 7 days after the surgery.
Autopsy revealed that peritoneal leaves were dull, plethoric, covered with massive yellow-greenish
films, the peritoneal cavity contained about 300 ml of thick yellow-greenish liquid. What pathologic
process was revealed in the peritoneal cavity?
A. Serous peritonitis
B. Fibrinous serous peritonitis
C. Peritoneal commissures
D. Fibrinoushaemorrhagic peritonitis

E. * Fibrinoussuppurative peritonitis
214. A patient has undergone surgical removal of a cavitary liver lesion 2 cm in diameter. It was revealed
that the cavity wall was formed by dense fibrous connective tissue; the cavity contained murky thick
yellowish-green fluid with an unpleasant odor. Microscopically the fluid consisted mainly of
polymorphonuclear leukocytes. What pathological process are these morphological changes typical
for?
A. Acuteabscess
B. Phlegmon
C. Empyema
D. Furuncle
E. * Chronicabscess
215. Granulomas containing lymphocytes and macrophages were detected during analysis of skin biopsy
material. Among macrophages there are large cells with fat inclusions, which contain
microorganisms in spherical packages (Virchow’s cells). The following disease is based on the
described type of hypersensitivity:
A. Syphilis
B. Tuberculosis
C. Rhinoscleroma
D. Epidemic typhus
E. * Leprosy
216. As a result of careless handling of an iron, a 34-year-old female patient has got acute pain, redness,
swelling of her right index finger. A few minutes later, there appeared a blister filled with a
transparent liquid of straw-yellow color. The described changes are a manifestation of the following
pathological process:
A. Traumatic edema
B. Alternative inflammation
C. Proliferative inflammation
D. Vacuolar degeneration
E. * Exudative inflammation
217. A 7-year-old boy got ill with diphtheria. On the third day he died of asphyxiation. At autopsy the
mucosa of the larynx, trachea and bronchi had thickened, edematous, lusterless appearance and was
covered with gray films which could be easily removed. Specify the type of laryngeal inflammation:
A. Diphtheritic
B. Purulent
C. Catarrhal
D. Intermediate
E. * Croupous
218. There is a 7-year-old child with complains of cough, lacrimation, rhinitis, skin rash, photophobia and
three-day-long fever as high as 38°N. Physical examination has revealed the following:
conjunctivitis; bright red maculopapular rash covering the skin of face, neck and torso; hyperemic
pharynx; serous purulent secretions from the nose; dry rales in the lungs. What is the most probable
diagnosis?
A. Adenovirus infection
B. Scarlet fever
C. Chicken pox
D. Rubella
E. * Measles

219. Autopsy of a dead 6-year-old child revealed a marked edema of the soft tissues of neck and enlarged
tonsils. Pharyngeal mucosa was covered with numerous dense whitish-yellow pellicles exposing deep
ulcers after their removal. Histological examination of the pharyngeal mucosa revealed necrosis of
the upper epithelial layers, impregnation of the mucous memrane with the fibrinous exudate and
moderate leukocyte infiltration. What infectious disease caused the death of the child?
A. Parainfluenza
B. Scarlet fever
C. Whooping cough
D. Measles
E. * Diphtheria
220. A 4-year-old girl died suddenly with symptoms of asphyxia. Autopsy revealed white spots on the
buccal mucosa; large blotches of rash on the skin of face, trunk and extremities; conjunctivitis, edema
with foci of necrosis on the laryngeal mucosa; giant-cell pneumonia on microscopy. What is the most
likely diagnosis?
A. Scarlet fever
B. Influenza
C. Meningococcal infection
D. Typhus
E. * Measles
221. Such presentations as catarrhal conjunctivitis, pharyngitis, laryngotracheobronchitis, white spots on
the buccal mucosa in the region of lower premolar teeth, maculopapular rash on face, body and
extremities are typical for the following disease:
A. Spotted fever
B. Scarlet fever
C. Meningococcal infection
D. Influenza
E. * Measles
222. A child is 10 years old. The followi-ng presentations have developed: sharp pain during swallowing,
swollen neck, body temperature rise up to 39, 0oC , bright-red fi-nely papular rash all over the body.
Pharynx and tonsils are sharply hyperemic ("flaming pharynx"), "crimson tongue". On the tonsi-ls
surface there are isolated greyish necrosis focuses. What disease it might be?
A. Meningococcal nasopharyngitis
B. Diphtheria
C. Influenza
D. Measles
E. * Scarlet fever
223. Autopsy of a Middle-Eastern woman, who had been suffering from wasting fever for a long time,
revealed enlarged blackened liver and spleen. Bone marrow was hyperplastic and black-colored as
well. Cerebral cortex was smoky grey. What disease is it characteristic of?
A. AIDS
B. Epidemic typhus
C. Sepsis
D. Hepatitis
E. * Malaria
224. A child is 7 years old. The following presentations have developed: sharp pain during swallowing,
swollen neck, body temperature rise up to 39,0°C, bright and red finelypapular rash all over the body.
Pharynx and tonsils are sharply hyperemic ("flaming pharynx"), "crimson tongue". On the tonsils
surface there are isolated grayish necrosis focuses. What disease it might be?
A. Meningococcal nasopharyngitis

B. Diphtheria
C. Influenza
D. Measles
E. * Scarlet fever
225. A 9-year-old boy has acute onset of disease: sore throat, body temperature rise up to 39, 5oC; on the
second day diffuse skin rash was detected all over his skin except for nasolabial triangle. On
examination of oral cavity: crimson tongue, "flaming pharynx", necrotic tonsillitis. What diagnosis is
the most likely?
A. Measles
B. Diphtheria
C. Influenza
D. Meningococcemia
E. * Scarlet fever
226. During autopsy the following has been revealed: the meninges of the upper cerebral hemispheres are
extremely plethoric, of yellow-green color and are soaked with purulent effluent. What kind of
meningitis is characterized by such clinical presentations?
A. Tuberculous meningitis
B. Grippal meningitis
C. Anthrax-induced
D. Epidemic typhus-induced
E. * Meningococcal meningitis
227. A 10-year-old child has painful swallowing, neck edema, temperature rise up to 39,0oC, the whole
body is covered with bright-red petechial rash. Back of the throat and tonsils are hyperemic, the
tongue is crimson colored. Tonsillar surface is covered with isolated grayish-colored necrosis nidi.
What disease is it?
A. Meningococcal nasopharyngitis
B. Diphtheria
C. Influenza
D. Measles
E. * Scarlet fever
228. On the 24th day since the onset of disease, a male patient diagnosed with typhoid fever and
undergoing treatment in an infectious diseases hospital has suddenly developed clinical presentations
of acute abdomen leading to the death of the patient. During autopsy peritonitis has been revealed,
with numerous ulcers covering the colon mucosa and reaching as deep as muscular and, in places,
serous tunic. The ulcers have smooth edges and even floor. The intestinal wall is perforated. What
stage of typhoid fever has the lethal complication arisen at?
A. Medullary swelling
B. Necrosis
C. Dirty ulcer
D. Regeneration
E. * Clean ulcer
229. A diseased child has a high fever, sore throat, swelling of submandibular lymph nodes. Objectively:
pharyngeal mucosa is edematous, moderately hyperemic, the tonsils are enlarged, covered with
grayish membrane tightly adhering to the tissues above. Attempts to remove the membrane produce
the bleeding defects. What disease are these presentations typical for?
A. Catarrhal tonsillitis
B. Scarlet fever
C. Meningococcal disease
D. Measles

E. * Diphtheria
230. A 60-year-old male patient was suffering from chronic tibial osteomyelitis during 10 years. Three
years ago he developed the nephrotic syndrome. His death was caused by uraemia. On autopsy, the
kidneys were moderately reduced in size, white, dense and had some cicatrices in their cortical layer.
Indicate the renal pathology which developed.
A. Primary amyloidosis
B. Senile amyloidosis
C. Periodic disease
D. Acute glomerulonephritis
E. * Secondary amyloidosis
231. A 23-year-old male patient acutely fell ill after supercooling. An increased blood pressure,
haematuria and oedemata on his face were observed. The treatment was not effective. Six months
later he died from uraemia. On autopsy, the kidneys were enlarged, flaccid, their cortical layer was
yellow-grey, dim, with some red specks, broad, oedematous and well delimited from the dark red
medullary substance. Microscopically, there was proliferation of the epithelium of the glomeruli,
podocytes and macrophages with formation of (crescents(. Make a diagnosis.
A. Chronic glomerulonephritis
B. Acute glomerulonephritis
C. Acute pyelonephritis
D. Amyloid shrunk kidneys
E. * Subacute glomerulonephritis
232. In a 52-year-old female patient, whose case history had arterial hypertension, phenomena of uraemia
and oedemata of her face began to develop during the past year. A renal biopsy revealed a diffuse
affection of all the studied glomeruli, whose main part had signs of focal and total sclerosis and
hyalinosis, and only in solitary intact glomeruli there was dilation of the mesangium with
proliferation of the mesangial cells. Diagnose the kind of glomerulonephritis.
A. Chronic mesangial proliferative
B. Subacute extracapillary
C. Acute intracapillary
D. Chronic mesangial capillary
E. * Chronic fibroplastic
233. An autopsy of a 62-year-old male patient revealed that his skin was grey-sallow with microfocal
haemorrhages, his face was as if covered with some whitish powder, the patient had
fibrinous-haemorrhagic laryngitis, tracheitis, fibrinous pericarditis, gastritis, enterocolitis. What
syndrome is characterized by this complex of morphological changes?
A. Cushing’s
B. Acute renal failure
C. Chronic cardiac insufficiency
D. Chronic hepatic insufficiency
E. * Chronic renal insufficiency
234. An autopsy of a male patient, who suffered from hypertensive disease and died of an intracerebral
haemorrhage, revealed small dense kidneys with a fine-grained surface, the parenchyma and cortical
substance were atrophic. Indicate the kind of changes in the kidneys.
A. Secondary shrunk
B. Pyelonephritic
C. Amyloid shrunk
D. Nephritic
E. * Primary shrunk

235. A 30-year-old male patient was hospitalized complaining of a headache and faints. During the
previous year, high indices of blood pressure were observed. A renal biopsy revealed an
extracapillary proliferation of the epithelium of the glomerular capsule with formation of (crescents(.
What disease is characterized by such signs?
A. Acute glomerulonephritis
B. Lupus erythematosus glomerulonephritis
C. Membranous nephropathy
D. Chronic glomerulonephritis
E. * Subacute glomerulonephritis
236. A 63-year-old male patient, who was suffering from infectious polyarthritis for a long period of time,
died under the phenomena of renal insufficiency. Macroscopically, his kidneys were significantly
enlarged, dense and waxy. A histological examination of the kidneys revealed sclerosis of the
intermedial region and malpighian pyramids, a large number of lipids and much congophilic
substance in the stroma of the tubules. What pathological process complicated the course of
infectious polyarthritis?
A. Chronic glomerulonephritis
B. Nephrosclerosis
C. Nephrotic syndrome
D. Chronic pyelonephritis
E. * Amyloid shrunk kidneys
237. Soon after she had acute pharyngitis, a 3-year-old girl developed diffuse oedemata (anasarca),
massive proteinuria, hypoalbuminaemia and hyperlipidaemia which were controlled by corticosteroid
preparations. An electron microscopy of a renal biopsy revealed absence of small processes of the
podocytes. What was the most probable diagnosis?
A. Subacute glomerulonephritis
B. Acute glomerulonephritis
C. Focal segmental glomerulosclerosis
D. Membranous nephropathy
E. * Lipoid nephrosis
238. Fourteen days following angina, a 15-year-old child developed oedemata on the face and an increased
blood pressure, the urine resembled (broth slops(. An immunohistological examination of a renal
biopsy revealed deposition of immune complexes on the basal membranes of the tubules and in the
glomerular mesangium. What disease developed in the patient?
A. Lipoid nephrosis
B. Acute pyelonephritis
C. Necrotic nephrosis
D. Subacute glomerulonephritis
E. * Acute glomerulonephritis
239. A microscopic examination of a renal biopsy revealed proliferation of the nephrothelium of the
Bowman’s capsule, podocytes and macrophages with formation of (crescent( structures which
squeezed the glomeruli. The glomerular capillaries were characterized by necrosis, some of them by
sclerosis or hyalinization. An oedema and a lymphoplasmacytic infiltration of the renal stroma were
observed. Name the renal pathology.
A. Chronic glomerulonephritis
B. Chronic pyelonephritis
C. Tubulointerstitial nephritis
D. Acute glomerulonephritis
E. * Subacute glomerulonephritis

240. On macroscopic examination, the kidneys are enlarged and motley because of alternation of light
areas with foci of petechial haemorrhages, their surface is smooth. Microscopically, some vascular
loops of the glomeruli were in the state of fibrinoid necrosis, the basal membranes of the capillaries
were thickened and appeared like (wire loops(, the lumens of some capillaries contained hyaline
thrombi. Make a diagnosis.
A. Chronic glomerulonephritis
B. Necronephrosis
C. Acute glomerulonephritis
D. Lipoid nephrosis
E. * Lupus erythematosus nephritis
241. A 12-year-old boy, who had had streptococcal tonsillitis before, developed oliguria, an increased
blood pressure and oedemata in the facial region. A renal biopsy revealed diffuse affection of all the
glomeruli studied: hyperaemia, solitary fibrin microthrombi in the lumens of the capillaries, dilation
of the mesangium and its infiltration by segmented leukocytes. Indicate the kind of
glomerulonephritis.
A. Chronic mesangial proliferative
B. Chronic mesangial capillary
C. Chronic fibroplastic
D. Subacute extracapillary
E. * Acute exudative
242. A 29-year-old male was admitted to a nephrological department complaining of oedemata, headache,
vomiting and nausea. His death was caused by uraemia. On autopsy, his kidneys were significantly
reduced in size and dense, their capsule was removed with difficulty exposing a fine-grained surface,
the cortical layer was thinned on section. A microscopic examination of a renal biopsy revealed that
most of the glomeruli were scleroid, some of them were characterized by dilation of the mesangium
and proliferation of the mesangial cells. What disease should be suspected?
A. Amyloid shrunk kidneys
B. Chronic pyelonephritis
C. Necrotic nephrosis
D. Fulminating glomerulonephritis
E. * Chronic glomerulonephritis
243. On autopsy of a male, who suffered from hypertensive disease during his lifetime, the kidneys were
reduced in size and dense, their capsule was removed with difficulty, and the surface was
fine-grained. On section, there was an atrophy of the renal parenchyma, particularly that of the
cortical layer, the blood vessels were in the form of thick tubules which did not collapse. A
histological examination revealed hyalinosis and sclerosis of small arteries, hyalinosis and sclerosis
of the larger part of the glomeruli, a reduced number of tubules, a vegetation of the connective tissue
in the stroma. Some glomeruli were characterized by compensatory hypertrophy and their tubules
were dilated. What is your diagnosis?
A. Amyloid shrunk kidneys
B. Secondary shrunk kidneys
C. Fulminating glomerulonephritis
D. Chronic glomerulonephritis
E. * Primary shrunk kidneys
244. On autopsy of a 32-year-old female, who died from renal insufficiency, it was found that the kidneys
were enlarged and flaccid, their capsule was easily removed exposing a motley surface. On section,
the cortical layer was oedematous and definitely delimited from the medullary one. A histological
examination revealed an expressed plethora of vessels, enlarged and oedematous glomeruli, with
clusters of erythrocytes and accumulation of some serous fluid in the lumen of the Bowman’s
capsule. Some vascular loops of the glomeruli were necrotized. An expressed peritubular and
periglomerular lymphoidocytic infiltration was observed. What is your diagnosis?

A. Acute productive glomerulonephritis


B. Acute tubulopathy
C. Acute tubulointerstitial nephritis
D. Acute fibrinous glomerulonephritis
E. * Acute haemorrhagic glomerulonephritis
245. An autopsy of a 70-year-old female, who died from renal insufficiency, revealed dense kidneys which
were reduced in size (each weighing 90 g). Their fibrous capsule was removed with difficulty,
exposing a rough-grained surface; on section, the cortical substance was thinned, the tissue was dim.
Histologically, some glomeruli developed a proliferation of the epithelium in the Bowman’s capsule
with crescent formations, while other glomeruli were characterized by hyalinization. The stroma
contained some lymphoid infiltrates, the vascular walls were thickened, their lumens were narrowed.
What is your diagnosis?
A. Atherosclerotic nephrosclerosis
B. Chronic glomerulonephritis
C. Acute glomerulonephritis
D. Arterial nephrosclerosis
E. * Subacute glomerulonephritis
246. On autopsy of a female, who was suffering from bronchiectatic disease during 15 years and died of
renal insufficiency, it was found that the kidneys were reduced in size, dense and (sebaceous( by
appearance, their surface was fine-grained. Microscopically, the glomeruli were atrophied and
substituted for a connective tissue. There was accumulation of congophilic masses under the intima
and adventitia of the vessels, as well as in the basal membranes of the tubules and in the renal stroma.
Which of the diagnoses listed below was the most probable?
A. Chronic glomerulonephritis
B. Subacute glomerulonephritis
C. Chronic tubulointerstitial nephritis
D. Malignant nephrosclerosis
E. * Amyloid shrunk kidneys
247. Urinalyses of a 44-year-old male patient, who suffers from a chronic renal disease, demonstrate
persistent proteinuria. A microscopic examination of a renal biopsy reveals some diffuse and even
thickening of the basal membranes in the glomerular capillaries with formation of their processes in
the direction of the podocytes ((small thorns(); the proliferation of the mesangiocytes was absent or
poorly expressed; there were small foci of sclerosis in the cortical layer and a fatty degeneration of
the epithelium of the proximal tubules. Make a diagnosis.
A. Mesangial capillary glomerulonephritis
B. Mesangial proliferative glomerulonephritis
C. Lipoid nephrosis
D. Focal segmental glomerular sclerosis
E. * Membranous glomerulonephritis
248. A microscopic examination of a renal biopsy revealed an expressed proliferation of the mesangial
cells with expulsion of their processes to the periphery of the capillary loops (mesangial
interposition), a diffuse thickening and doubling of the basal membranes in the capillaries, and an
accumulation of the mesangial matrix. Indicate the diagnosis which was the most probable.
A. Mesangial proliferative glomerulonephritis
B. Membranous glomerulonephritis
C. Mesangial glomerulonephritis
D. Acute intracapillary glomerulonephritis
E. * Mesangial capillary glomerulonephritis
249. A 38-year-old male patient complained of dyspnoea, a cough with production of a large amount of
sputum, an elevated body temperature up to 38(C, and a reduced urination. An X-ray examination
revealed some focal shadow in the lungs, an increased amount of urea and uric acid in the blood. The
urine was characterized by its low specific weight, presence of proteinuria and casts. The patient died
two weeks later. On autopsy, fibrinous-necrotic laryngotracheobronchitis, fibrinous-haemorrhagic
pneumonia and contracted kidneys were found. Which of the diagnoses listed below was the most
probable?
A. Fibrinous-haemorrhagic pneumonia
B. Influenza with pneumonic complications
C. Chronic glomerulonephritis
D. Chronic destructive bronchitis
E. * Chronic renal insufficiency
250. An electron microscopy of a renal biopsy of a 10-year-old boy with expressed oedemata and
proteinuria revealed absence of small processes of the podocytes in the glomeruli, the basal
membranes of the capillaries were thickened, the mesangium was insignificantly dilated. The lumens
of dilated proximal tubules contained hyaline and granular casts, the epithelium of the tubules had
phenomena of hyaline-drop, hydropic and fatty degeneration. The interstice contained a lot of lipids
and lipophages. Which of the diagnoses listed below was the most probable?
A. Subacute glomerulonephritis
B. Acute glomerulonephritis
C. Membranous glomerulonephritis
D. Mesangial capillary glomerulonephritis
E. * Lipoid nephrosis
251. An autopsy of a 60-year-old female, who had suffered from bronchiectatic disease for 15 years and
died of renal insufficiency, revealed that the kidneys were reduced in size, dense and (sebaceous( by
appearance, their surface was fine-grained. Microscopically, the glomeruli were atrophied and
substituted for a connective tissue. There was accumulation of congophilic masses under the intima
and adventitia of the vessels, as well in the basal membranes of the tubules and in the renal stroma.
Name the most probable diagnosis.
A. Lipoid nephrosis
B. Chronic pyelonephritis
C. Chronic tubulointerstitial nephritis
D. Postnecrotic nephrosclerosis
E. * Amyloid shrunk kidneys
252. Three weeks following a severe supercooling, a young male developed an elevated body temperature,
a pain in his lumbar region, oedemata, proteinuria and haematuria. A renal biopsy demonstrated
enlargement of the glomeruli, a plethora of their capillaries, some eosinophilic fluid, fibrin and
erythrocytes in the cavity of the Bowman’s capsule, the mesangium was infiltrated by
polymorphonuclear leukocytes. What diagnosis was the most probable?
A. Subacute glomerulonephritis
B. Acute renal failure
C. Acute productive glomerulonephritis
D. Acute pyelonephritis
E. * Acute exudative glomerulonephritis
253. In a 25 year-old female patient with severe edemas, hyperproteinuria, hyperlipidemia the biopsy of
kidney established: well-developed thickening of the glomerular capillary wall with the presence of
electron-dense immunoglobulin-containing deposits along the epithelial surface of the basement
membrane. Epithelial cells lost their foot processes. Described changes are characteristic for…
A. Acute Poststreptococcal glomerulonephritis
B. Crescentic glomerulonephritis
C. Chronic glomerulonephritis
D. Acute pyelonephritis
E. * Membranous glomerulonephritis
254. A 12-year-old child has developed nephritic syndrome (proteinuria, hematuria, cylindruria) after
angina of 2 weeks ago. In a biopsy of kidney it was established: diffuse proliferation of endothelial
and mesangial, epithelial cells in glomeruli, infiltration with leukocytes, both neutrophils and
monocytes, interstitial edema. Described changes are characteristic for…
A. Membranous glomerulonephritis
B. Chronic glomerulonephritis
C. Acute necrotic nephrosis
D. Crescentic glomerulonephritis
E. * Acute poststreptococcal glomerulonephritis
255. A 55 year-old man has died after chronic glomerulonephritis, chronic renal insufficiency. In the
autopsy the pathologist has found out characteristic changes in kidneys for this disease, also fibrinous
pericarditis, pleuritis, and bronchitis. Call the cause of the fibrinous inflammation in serosal and
mucosal layers.
A. Hypolipidemia
B. Hyperlipidemia
C. Arterial hypertension
D. Arterial plethora
E. * Uremia
256. For a long time a 49-year-old woman was suffering from glomerulonephritis which caused death.On
autopsy it was revealed that kidneys size was 7x3x2.5 sm, weight is 65,0 g, they are dence and
small-grained. Microscopically: fibrinogenous inflammation of serous and mucous capsules,
dystrophic changes of parenchymatous organs, brain edema. What complication can cause such
changes of serous capsules and inner organs?
A. Thrombopenia
B. Anemia
C. Sepsis
D. DIC-syndrome
E. * Uremia
257. On macroscopic examination of the kidneys in a male, who died from renal insufficiency, a
pathologist found some asymmetrical corticomedullary cicatrices and dilation of the pelvicocaliceal
system. A microscopic examination revealed atrophic tubules which had cyst-like dilations in some
parts, were filled with eosinophilic masses and resembled the thyroid gland, as well as there was
periglomerular sclerosis. Make a diagnosis.
A. Chronic tubulointerstitial nephritis
B. Amyloid shrunk kidneys
C. Acute tubulointerstitial nephritis
D. Necronephrosis
E. * Chronic pyelonephritis
258. On autopsy of a male patient, who died from ethylene glycol poisoning, the kidneys were somewhat
enlarged and oedematous, their capsule was easily removed, the cortical substance was pale grey and
wide, the medullary one was dark red. Microscopically, there were diffuse necroses of the tubules
with ruptures of their basal membranes. What renal pathology developed in the patient?
A. Acute pyelonephritis
B. Acute glomerulonephritis
C. Acute tubulointerstitial nephritis
D. Acute glomerulosclerosis

E. * Necrotic nephrosis
259. In a male patient, who died from chronic renal insufficiency against a background of a chronic renal
disease, the kidneys are symmetrically reduced in size and dense in consistency, their capsule is
removed with difficulty exposing a grained surface. On section, the cortical and medullary layers are
thin, the renal tissue is grey, dry and ischaemic. Name the pathological process in the kidneys.
A. Arteriolosclerotic nephrosclerosis
B. Atherosclerotic nephrosclerosis
C. Amyloid shrunk kidneys
D. Pyelonephritic kidneys
E. * Secondary shrunk kidneys
260. A histological examination of a 56-year-old male, who died from chronic renal insufficiency,
revealed the following changes: focal sclerosis and a lymphoplasmacytic infiltration of the interstice,
mucous membranes of the pelves and calyces, a metaplasia of the transitional epithelium into the
stratified squamous one, numerous encapsulated abscesses. The epithelium of the tubules was in the
state of dystrophy and necrosis. The lumens of solitary tubules were dilated and filled with some
colloid contents, the epithelium was flattened (the tissue resembled the thyroid gland). The glomeruli
were focally scleroid. Name the pathological process.
A. Chronic tubulointerstitial nephritis
B. Necronephrosis
C. Acute pyelonephritis
D. Chronic glomerulonephritis
E. * Chronic pyelonephritis
261. An autopsy of a male revealed a prostatic adenoma and large kidneys with sharply enlarged pelves
and calyces filled with some transparent fluid. Name the process in the kidneys.
A. Polycystic kidneys
B. Glomerulonephritis
C. Tuberculosis
D. Pyelonephritis
E. * Hydronephrosis
262. An autopsy of a male, who died from chronic renal insufficiency, enlarged kidneys with large tubera
on their surface were found. On section, the kidneys revealed numerous smooth-walled cysts filled
with some transparent fluid. Name the pathological process in the kidneys.
A. Hydroureteronephrosis
B. Hydronephrosis
C. Pyonephrosis
D. Hydrocalycosis
E. * Polycystic kidneys
263. On autopsy of a male, who died from burn disease, enlarged swollen kidneys were revealed; their
fibrous capsule was easily removed and tense. On section, the cortical layer was pale grey, wide and
sharply delimited from the dark red medullary one. Microscopically, there was necrosis of the renal
tubules, in some places with a destruction of the basal membranes, an oedema of the interstice, a
leukocytic infiltration and haemorrhages. Which of the diagnoses listed below was the most
probable?
A. Acute pyelonephritis
B. Acute tubulointerstitial nephritis
C. Chronic tubulopathy
D. Chronic pyelonephritis
E. * Acute tubulopathy

264. An autopsy of a male, who died from renal insufficiency, revealed enlarged kidneys with a tense
fibrinous capsule which was easily removed. The cortical layer was pale grey, the malpighian
pyramids were dark red; there were microfocal haemorrhages in the intermediary area of the kidneys
and in the renal pelves. A histological examination revealed dystrophy and necrosis in the epithelium
of the convoluted tubules, in some places with a destruction of the basal membranes mostly in the
distal tubules (tubulorrhexis), there were casts in the lumens of the tubules at different levels of the
nephrones, an oedema of the interstice with its leukocytic infiltration and haemorrhages. The lumens
of the glomerular capsules had an accumulation of the ultrafiltrate. Which of the diagnoses listed
below was the most probable?
A. Acute glomerulonephritis
B. Necronephrosis, the shock stage
C. Acute pyelonephritis
D. Necronephrosis, the stage of restoration of diuresis
E. * Necronephrosis, the oligoanuric stage
265. An autopsy of a 55-year-old female, who died from uraemia, revealed that her kidneys were unevenly
reduced in size, had a dense consistency, large tubera on their surface and whitish cicatrices in the
cortical layer. On section, the renal pelves were dilated, their mucous membrane was whitish and
thickened. Microscopically, the renal interstice was characterized by a focal inflammation, an atrophy
and sclerosis of the renal tissue; the tubules were dilated and had hyaline casts in their lumens. The
arteries and veins were scleroid. Which of the pathological processes in the kidneys was the most
probable?
A. Diabetic nephrosclerosis
B. Acute glomerulonephritis
C. Primary shrunk kidneys
D. Amyloid shrunk kidneys
E. * Pyelonephritic kidneys
266. An autopsy of a male with a syndrome of prolonged compression revealed enlarged swollen kidneys
whose fibrous capsule was easily removed exposing a pale grey cortical layer sharply delimited from
dark red malpighian pyramids. Histologically, there was necrosis in the epithelium of the tubules
with phenomena of tubulorrhexis, an obstruction of the tubules with detritus and crystals of
myoglobin, an oedema of the interstice and its infiltration by heterophilic leukocytes. Which of the
diagnoses was the most probable?
A. Acute pyelonephritis
B. Acute glomerulonephritis
C. Tubulointerstitial nephritis
D. Haemolytic kidneys
E. * Acute renal failure
267. On autopsy of a female, who died from uraemia, it was found that her kidneys differed in size and
their surface had large tubera; there were dense adhesions between the renal surface and capsule.
Microscopically, the renal tissue had foci of the following changes: numerous encapsulated
abscesses, an expressed lymphohistiocytic infiltration of the interstice and a vegetation of the
connective tissue. The mucous membrane of the pelves had foci of metaplasia of the transitional
epithelium into the stratified one. The tubules were partly atrophied, partly stretched and filled with
some colloid-like contents. Which of the diagnoses listed below was the most probable?
A. Chronic glomerulonephritis
B. Acute pyelonephritis
C. Acute glomerulonephritis
D. Tubulointerstitial nephritis
E. * Chronic pyelonephritis

268. A patient suffered with urolithiasis has died as a result uremia. Autopsy has showed an enlargement
of right kidney, its parenchyma has been thinned; the pelvis and calices have been extended, filled
with fluid. In the orifice of ureter there has been a calculus. Call the pathologic process in the right
kidney.
A. Replacement hypertrophy
B. Hyperplasia
C. Neoplasm of kidney
D. Pyelonephritis
E. * Hydronephrosis
269. A 4 year-old child has died because of an uremia. In autopsy the pathologist has found out: the
kidneys have been increased and consisted of large multiple cavities with thin walls contained fluid.
Call this pathological process in kidneys.
A. Necrotic nephrosis
B. Pyelonephritis
C. Glomerulonephritis
D. Infarction
E. * Policystosis
270. On autopsy it is revealed that kidneys are enlarged, surface is large-granular because of multiple
cavities with smooth wall, which are filled with clear fluid. What kidney disease did the patient have?

A. Infarction
B. Necrotic nephrosis
C. Glomerulonephritis
D. Pyelonephritis
E. * Polycystic kidney
271. An autopsy has revealed that kidneys are enlarged, surface is large-granulary because of multiple
cavities with smooth wall, which are filled with clear fluid. What kidney disease did the patient have?

A. Infarction
B. Glomerulonephritis
C. Pyelonephritis
D. Necrotic nephrosis
E. * Polycystic kidney
272. Autopsy of a man who died from ethylenglycol poisoning revealed that his kidneys are a little bit
enlarged, edematic; their capsule can be easily remove. Cortical substance is broad and light-grey.
Medullary substance is dark-red. What pathology had this man?
A. Acute tubular-interstitial nephritis
B. Lipoid nephrosis
C. Acute glomerulonephritis
D. Acute pyelonephritis
E. * Necrotic nephrosis
273. Autopsy of a man who died from burn disease revealed brain edema, liver enlargement as well as
enlargement of kidneys with wide light-grey cortical layer and plethoric medullary area.Microscopic
examination revealed necrosis of tubules of main segments along with destruction of basal
membranes, intersticium edema with leukocytic infiltration and haemorrhages. What is the most
probable postmortem diagnosis?
A. Myeloma kidney
B. Tubulointerstitial nephritis
C. Pyelonephritis

D. Gouty kidney
E. * Necrotic nephrosis
274. A 39-year-old female with a clinical picture of acute abdomen underwent surgical removal of an
enlarged uterine tube. On examination, the serous coat of the uterine tube was dark purple, the lumen
contained some blood clots. A histological examination of the wall of the tube revealed that the
mucous membrane had layers of the decidual cells, and there were villi of the chorion among the
blood clots. What is the most probable diagnosis?
A. Placental polyp
B. Choriocarcinoma
C. Haematosalpinx
D. Salpingitis
E. * Tubal pregnancy
275. A histological examination of a scrape from the uterine cavity of a 45-year-old woman with clinical
manifestations of a disordered ovariomenstrual cycle revealed that the number of the endometrial
glands was increased, the lumens in some of them were significantly dilated and rounded, the
epithelium of the glands had morphological signs of the phase of proliferation. What is your
diagnosis?
A. Atypical hyperplasia of endometrium
B. Glandular polyp of endometrium
C. Glandular hyperplasia of endometrium
D. Endometrial adenocarcinoma
E. * Glandulocystic hyperplasia of endometrium
276. A histological examination of the vaginal portion of the uterine cervix revealed substitution of the
stratified squamous epithelium for the simple columnar one with numerous underlying glands
without any signs of their new formation. Which of the diagnoses listed below was the most
probable?
A. Papillary endocervicosis
B. Progressing endocervicosis
C. Healing endocervicosis
D. Adenosis
E. * Simple endocervicosis
277. A 34-year-old female took medical advice of a gynaecologist complaining of a uterine haemorrhage
for 10 days after her menses. A scrape from the uterine cavity revealed a large number of the
endometrial glands which were spirally convoluted, dichotomically divided and ramified in a
tree-like manner. The stroma of the endometrium was hyperplastic and had a great number of
predecidual cells. Which of the diagnoses was the most probable?
A. Mixed hyperplasia of endometrium
B. Atypical hyperplasia of endometrium
C. Glandulocystic hyperplasia of endometrium
D. Remnants of abortion
E. * Glandular hyperplasia of endometrium
278. A mammary gland of a 25-year-old female revealed a slightly tender node, 3 cm in diameter; a
sectorial resection of the mammary gland was made. Macroscopically, the node was whitish and had
round hollows 0.1-0.3 cm in diameter. Microscopically, there was a vegetation of the connective
tissue with foci of hyalinosis, an atrophy of the glandular lobules and dilation of the lumens in some
places. What disease are the above morphological changes typical for?
A. Fibrous mastopathy
B. Sclerosing adenosis
C. Fibroadenoma of mammary gland

D. Acute mastitis
E. * Fibrocystic mastopathy
279. A woman suffered with dysfunctional metrorrhagia was made a diagnostic abortion. Histologically in
the scrape there were a lot of small stamped glandulas covered by multirowed epithelium. The
lumens of some glandulas were cystically extended. Call the variant of general pathologic process in
the endometrium.
A. Atrophy of endometrium
B. Metaplasia of endometrium
C. Neoplasm of endometrium
D. Hypertrophic growth
E. * Glandular-cystic hyperplasia of endometrium
280. During histological investigation of prostate gland, that has been resected in 72-year-old patient with
symptoms of difficult urination the following signs were found: enlargement of glandular and
muscle’s elements with disturbance of glandular structure. What process is more probable?
A. Muscle-fibrotic hyperplasia
B. Adenocarcinoma
C. Mixed variant of prostatopathy
D. Prostatitis
E. * Glandular hyperplasia
281. At examination of 35-year-old woman it was found out red area about 1 cm in diameter with uneven
edges in cervix. Microscopic examination showed accumulation of numerous glands, look like
uterina ones, under epithelium layer. Call this pathologic process.
A. Endocervicosis
B. Vaginitis
C. Adenomatosis
D. Candidosis
E. * Endometriosis
282. A 25-year-old woman palpates a left breast "lump" on self-examination. Her physician palpates an
ill-defined mass. There is no pain or tenderness. No axillary lymphadenopathy is noted. Fine needle
aspiration is performed and cytologic examination shows cells that appear benign. The lesion
persists, and 6 months later another biopsy is taken and shows ductal epithelial proliferation with
ductal apocrine metaplasia, stromal fibrosis, and sclerosing adenosis. Which of the following is the
most likely diagnosis?
A. Fat necrosis
B. Ductal carcinoma in situ
C. Lobular carcinoma in situ
D. Intraductal papilloma
E. * Fibrocystic changes
283. A clinical study is peformed involving subjects who women are ranging in age from 15 to 45 years
who palpated breast "lumps" on self-examination. The presence of breast mass lesions in these
subjects was subsequently confirmed by physical examination and by mammography. All subjects
had a biopsy or excision of their lesion performed, with a definitive pathologic diagnosis made.
Which of the following diagnoses is likely to be the most frequent in these subjects?
A. Abscess
B. Fibroadenoma
C. Lobular carcinoma in situ
D. Infiltrating ductal carcinoma
E. * Fibrocystic changes

284. A 19-year-old woman gave birth to a healthy male infant at term following an uncomplicated
pregnancy. She has now been breast feeding the baby for a month, but notes that her left breast has
gradually become swollen and painful to touch over the past week. On physical examination her
temperature is 38.2 C. Which of the following is the most likely diagnosis?
A. Fibrocystic disease
B. Fat necrosis
C. Intraductal papilloma
D. Galactocele
E. * Acute mastitis
285. A 35-year-old woman has noted a palpably firm, irregular mass in her right breast for the past 3
months. On physical examination there is no tenderness or swelling. By mammography there is an
irregular 2 cm density that demonstrates scattered microcalcifications. Biopsy of this mass reveals
extensive fat necrosis. Which of the following is the most likely cause for this breast lesion?
A. Pregnancy
B. Prolactinoma
C. Fibrocystic changes
D. Lobular carcinoma in situ
E. * Trauma
286. A 29-year-old woman has been taking oral contraceptives for the past 12 years. She now has a
palpable "lump" in the left breast that has persisted for the past 3 months. Which of the following
breast lesions is most likely to be associated with her oral contraceptive use?
A. Acute mastitis
B. Fat necrosis
C. Hypertrophy
D. Cyst formation
E. * Galactocele
287. A 20-year-old woman gives birth to a term girl infant following an uncomplicated pregnancy. She
breast feeds the infant. Six weeks later, her left breast becomes painful and slightly swollen. On
physical examination there is a tender 3-cm mass in the left breast beneath a nipple that shows several
painful fissures. Which of the following pathologic findings is most likely to be present in this
breast?
A. Infiltrating ductal carcinoma
B. Numerous plasma cells
C. Sclerosing adenosis
D. Fat necrosis
E. * Staphylococcus aureus infection
288. A 30-year-old woman is claiming in a civil lawsuit that her husband has abused her for the past year.
A workup by her physician reveals a 2 cm left breast mass. There is no lymphadenopathy. No skin
lesions are seen, other than a bruise to her upper arm. An excisional biopsy of the breast mass is
performed. On microscopic examination, the biopsy shows fat necrosis. This biopsy result is most
consistent with which of the following etiologies?
A. Physiologic atrophy
B. Lactation
C. Radiation injury
D. Hypoxic injury
E. * Breast trauma

289. A scrape from the uterine cavity revealed in the blood some particles of a neoplasm which had an
organoid structure and consisted of the columnar epithelial cells which formed glandular structures;
the stroma of the neoplasm was significantly developed, the area of its (pedicle( had glomi of
thick-walled vessels. Which of the tumours was the most probable?
A. Glandular hyperplasia of endometrium
B. Uterine fibroadenoma
C. Endometriosis
D. Uterine adenocarcinoma
E. * Endometrial polyp
290. On gynaecological examination of a 36-year-old female, a red focus, which was 0.8 x 0.5 cm in size
and had uneven contours, was found in the mucous coat of the vaginal portion of the uterine cervix
on its border with the mouth of the womb. A microscopical examination revealed substitution of the
stratified squamous epithelium for the columnar one, as well as accumulation and new formation of
glands from the cambial elements of the columnar epithelium in the cervical channel were observed
under the integmentary epithelium. Which of the diagnoses listed below was the most probable?
A. Adenomatosis of uterine cervix
B. Healing endocervicosis
C. Simple endocervicosis
D. Dysplasia of epithelium of uterine cervix
E. * Proliferative endocervicosis
291. In a biopsy of сervix of a 26-year-old woman the diagnosis following was established:
pseudo-erosion. What microscopical changes has the pathologist revealed?
A. Cell-atypia of an epithelium of an mucosal epithelium
B. Keratinization of an epithelium
C. “Carcinomatous pearls”
D. Local inflammation and necrosis in mucosa
E. * Local changes of a stratified squamous epithelium on single-layer prismatic one
292. Diagnostic scraping was performed to the woman with dysfunctional uterine bleeding. Multiple
convoluted glands, ganglially dilated cavities of some glands were revealed histologically in the
scrape. Name the type of general pathological process.
A. Hypertrophic excrescence
B. Metaplasia
C. Displasia
D. Atrophy
E. * Glandular- cystic hyperplasia
293. A patient with gastric juice hypersecretion has been recommended to exclude from the diet rich
broths and vegetable infused water. A doctor recommended it, because these food products stimulate
production of the following hormone:
A. * Gastrin
B. Secretin
C. Cholecystokinin
D. Somatostatin
E. Neurotensin
294. A 49-year-old patient was found to have a disproportionate enlargement of hands, feet, nose, ears,
superciliary arches and cheek bones. Blood test revealed hyperglycemia, impaired glucose tolerance.
What is the most likely cause of this pathology development?
A. * Hypersecretion of growth hormone
B. Posterior pituitary hormone hypersecretion
C. Insulin hyposecretion

D. Vasopressin hyposecretion
E. Glucocorticoid hypersecretion
295. An autopsy of the body of an elderly man, who was suffering from acute intestinal disorder during
his last 2 weeks of life, has revealed the following change in the rectum and sigmoid colon: brown
and green film covering the mucosa is detected. The itestinal wall is thickened; the cavi- ty sharply
narrows down. Microscopy reveals mucosa necrosis of varying depth, necrotic tissue is pierced
through with fibrin threads, leucocytic infiltrati- on is observed. What diagnosis is the most probable?

A. * Fibrinous colitis
B. Catharrhal colon
C. Ulcerative colitis
D. Follicular colitis
E. -
296. A 43-year-old woman complains of weight loss, hyperhidrosis, low-grade fever, increased irritability.
She has been found to have hyperfunction of the sympathetic-adrenal system and basal metabolism.
These disorders can be caused by hypersecretion of the followi- ng hormone:
A. * Thyroxine
B. Somatotropin
C. Corticotropin
D. Insulin
E. Aldosterone
297. Specify the type of jaundice, during which there is no direct bilirubin in blood, and urine
urobilinogen level is high:
A. * Suprahepatic
B. Hepatic
C. Subhepatic
D. Mechanical
E. -
298. A 35-year-old woman complai- ns of swollen neck. Subtotal thyreoidectomy is performed. On
histological examination of the removed part of the thyroid gland the following was detected: atrophy
of parenchyma, moderate sclerosis development, duffuse infiltration by lymphocytes and plasma
cells leading to formation of lymphatic follicles. What pathology has developed in the thyroid gland?
A. * Hashimoto’s thyroiditis
B. Follicular adenoma
C. Riedel’s thyroiditis
D. Papillary carcinoma of the thyroid gland
E. Diffuse toxic goiter
299. A 55-year-old man had been suffering from chronic glomerulonephritis. He died from chronic renal
failure. Macroscopical examination revealed on the surface of epicardium and pericardium some
greyish-white villous depositions. After their removal dilated and plethoric vessels were uncovered.
What process took place in the pericardium?
A. Organization
B. Haemorrhagic inflammation
C. Proliferative inflammation
D. Arterial hyperemia
E. * Fibrinous inflammation

300. Preventive examination of a 55-year-old patient revealed type II diabetes mellitus. An endocrinologist
revealed an increase in body weight and liver enlargement. The man is non-smoker and doesn't abuse
alcohol but likes to have a good meal. Histological examination by means of diagnostic liver
puncture revealed that the hepatocytes were enlarged mostly on the lobule periphery, their cytoplasm
had transparent vacuoles showing positive reaction with sudan III. What liver pathology was
revealed?
A. Acute viral hepatitis
B. Chronic viral hepatitis
C. Alcohol hepatitis
D. Portal liver cirrhosis
E. * Fatty hepatosis
301. A female patient with a tumour of pancreas has developed mechanic jaundice resulting from
compression of a bile-excreting duct. Which duct is compressed?
A. Ductus hepaticus dexter
B. * Ductus choledochus
C. Ductus hepaticus sinister
D. Ductus cysticus
E. Ductus hepaticus communis
302. In a young male, an abundant quantity of the somatotropic hormone and enlargement of the nose,
lips, ears, loWER jaw, hands and feet were revealed. What is your diagnosis?
A. Pituitary dwarfismB.
B. shing's disease
C. Addison's disease
D. Adiposogenital dystrophy
E. * Acromegaly
303. In a male patient with an increased level of the parathormone, a histological examination in the area
of a pathological fracture of his femur revealed foci of a lacunar resolution of the osteoid beams and
new formation of a fibrous tissue. What is your diagnosis?
A. Multiple myeloma
B. Osteoblastoclastoma
C. Paget's disease
D. Osteopetrosis
E. * Parathyroid osteodystrophy
304. A male patient with phenomena of hypothyroidism died from heart failure. On histological
examination, his thyroid gland revealed a diffuse infiltration of the gland by lymphocytes and
plasmacytes, an atrophy of the parenchyma and a vegetation of the connective tissue. What disease
was it?
A. Acute nonsuppurative thyroiditis
B. Ligneous thyroiditis
C. Nodular goiter
D. Toxic goiter
E. * Hashimoto's disease
305. An autopsy of a 45-year-old female, who was suffering from arterial hypertension, diabetes mellitus
and ovarian dysfunction during past 15 years, revealed obesity by the upper type, a pituitary basophil
adenoma in the anterior lobe of the hypophysis, hyperplasia of the adrenal cortex. Which of the
diagnoses listed below was the most probable?
A. Cushing's syndrome
B. Hypertensive disease
C. Addison's disease

D. Adiposogenital dystrophy
E. * Cushing's disease
306. A 50-year-old female took medical advice complaining of excretion of a large amount of urine and
excessive thirst. On examination, her nourishment was reduced, the skin was dry, density of the urine
ranged from 1001 to 1010, data of an ultrasound examination and computed tomography of the brain
revealed a tumour in the posterior lobe of the hypophysis. Indicate the most probable disease.
A. Acromegaly
B. Babinsky-Frelich disease
C. Simmonds disease
D. Cushing's disease
E. * Diabetes insipidus
307. A 46-year-old male patient, who suffered from bulimia, polydipsia, polyuria with glucosuria and
albuminuria, died of renal insufficiency. On autopsy, the kidneys were reduced in size, dense and had
a fine-grained surface. The pancreas was reduced and partially substituted for a fatty tissue.
Microscopically, islets of Langerhans were fine, in some places they were substituted for a
connective tissue, solitary ones were hypertrophic. The kidneys reveal intracapillary
glomerulosclerosis. Which of the diagnoses was the most probable?
A. Subacute glomerulonephritis
B. Chronic indurative pancreatitis
C. Diabetes insipidus
D. Chronic glomerulonephritis
E. * Diabetes mellitus
308. A 36-year-old female patient underwent resection of the both lobes of her thyroid gland; each of them
was 5 x 6 cm in size, pink-yellow, moderately dense and had a tuberous surface. A microscopic
examination revealed follicles of various size, some of them were dilated like cysts and filled with
some colloid; the follicular walls were lined with the smoothed cuboidal epithelium; the stroma of the
gland was redundantly developed owing to the connective tissue, there were foci of calcinosis. Which
of the diseases listed below corresponded most to the changes found?
A. Parenchymatous goiter
B. Toxic goiter
C. Hashimoto's disease
D. Ligneous thyroiditis
E. * Colloid goiter
309. An autopsy of a 24-year-old female (from her case history it is known that a year before the woman
had given birth to a child) revealed a sharp decrease of the body weight down to 38 kg, the skin was
dry and thin, the weight of the internal organs was loWER ed. Also, there was a sharp decrease in the
weight of the adenohypophysis, the latter had cicatrices; there were foci of dystrophy, necrobiosis
and hyalinosis in the diencephalon. The ovaries, thyroid and adrenal glands had phenomena of
hypotrophy, the mucous membrane of the intestines was atrophied. Which of the diagnoses was the
most probable?
A. Nutritional dystrophy
B. Suprarenal cachexia
C. Cachexia associated with chronic amoebiasis
D. Cachexia associated with pellagra
E. * Cerebrohypophysial cachexia

310. An autopsy of a 45-year-old female patient, who suffered from obesity by the upper type, steroid
diabetes mellitus, arterial hypertension and secondary ovarian dysfunction, revealed hypertrichosis,
hirsutism, striae on the skin of the thighs and abdomen. The anterior lobe of the hypophysis
contained a white-pink encapsulated tumour, 2.5 cm in diameter (microscopically, it was a pituitary
basophil adenoma); the adrenal glands were characterized by bilateral hyperplasia of the fascicular
layer. Which of the diagnoses was the most probable?
A. Cushing's syndrome
B. Adiposogenital dystrophy
C. Simmonds disease
D. Pituitary dwarfism
E. * Cushing's disease
311. An autopsy of a male, who died from chronic renal insufficiency, revealed atherosclerosis of the
aorta and large arteries, small and dense kidneys with a finegrained surface, an enlarged
yellow-brown and flaccid liver, the pancreas was reduced in size. Microscopically, there was
atherocalcinosis of the aorta and arteries, an atrophy of the parenchyma, sclerosis and lipomatosis of
the pancreas; the kidneys were characterized by hyalinosis of the mesangium and glomeruli, a
glycogenic infiltration of the epithelium of the tubules, with large-drop adiposis in the hepatocytes.
What pathological process took place in the kidneys?
A. Arterial nephrosclerosis
B. Chronic pancreatitis
C. Chronic glomerulonephritis
D. Steatosis
E. * Diabetic nephrosclerosis
312. On autopsy of a male, who died from uraemia, it was found that the pancreas was reduced in size, his
contracted kidneys had a fine-grained surface, the liver was enlarged, yellow and flaccid.
Microscopically, the pancreatic tissue revealed an atrophy of the parenchyma, including islets of
Langerhans, the atrophied parenchyma was substituted for hyperplastic connective and fatty tissues.
The kidneys were characterized by sclerosis and hyalinosis of the glomeruli, as well as by a
glycogenic infiltration of the tubules; there was a fatty degeneration in the liver and a fibrinous
inflammation in the mucous coats of the trachea, bronchi and stomach. What disease did the died
person suffer from?
A. Chronic indurative pancreatitis
B. Chronic glomerulonephritis
C. Hypertensive disease
D. Steatosis
E. * Diabetes mellitus
313. An autopsy of a male revealed a tumour in the anterior lobe of the hypophysis, enlarged adrenal
glands, a reduction of the gonads in size, a hypertrophy of the left cardiac ventricle, the pancreas was
reduced in size and thickened. Histologically, there was a pituitary basophil adenoma and a
hyperplasia of the cortical layer in the adrenal glands. The pancreas was characterized by a
moderately expressed atrophy of the parenchyma, including islets of Langerhans. What disease did
the patient suffer from?
A. Diabetes mellitus
B. Adiposogenital dystrophy
C. Cushing's syndrome
D. Simmonds disease
E. * Cushing's disease

314. For a histological examination, a lobe and a part of the isthmus of the thyroid gland were received.
The tissue of the gland was dense and tuberous, on section it was pale brown and had grey-whitish
foci. Microscopically, against a background of an atrophy of the follicles of the gland, there was
some diffuse lymphoplasmacytic infiltration of the stroma with formation of lymphoid follicles.
What pathological process were these changes typical for?
A. Toxic goiter
B. Thyroid adenoma
C. Colloid goiter
D. Sporadic goiter
E. * Allergic thyroiditis
315. A histological examination of a thyroid gland revealed follicles of various size and shape which were
lined with the columnar epithelium; the latter proliferated and formed papillae of various size. The
follicular lumens contained some liquid and vacuolized colloid. The stroma of the gland was
characterized by a lymphoplasmacytic infiltration, in some places with formation of lymphatic
follicles having light centres. Which of the diagnoses was the most probable?
A. Colloid goiter
B. Nodular goiter
C. Hashimoto's disease
D. Ligneous thyroiditis
E. * Toxic goiter
316. An autopsy of a 48-year-old male, who died from vascular collapse, revealed an increased
pigmentation of the skin, the adrenal glands were reduced in size, the brown-yellow liver was
enlarged. On histological examination, foci of necrosis with a tuberculous granulation tissue were
found in the adrenal glands. The liver was characterized by phenomena of fatty degeneration. Which
of the diagnoses was the most probable?
A. Steatosis
B. Primary aldosteronism
C. Cushing's syndrome
D. Lipofuscinosis
E. * Addison's disease
317. An autopsy of a male, who suffered from right-sided pneumonia in the lower lobe during his
life-time and for a long period of time expectorated sputum of a purulent character, revealed some
cavity with dense edges that was located in the 9th-10th segments of the lung and was filled with
yellowish cream-like masses. There was some whitish path from the cavity to the root of the lung.
Microscopically, the cavity was separated from the intact pulmonary tissue with a membrane which
consisted of a fibrous connective tissue from the outside and a granulation one from inside. Which of
the diagnoses was the most probable?
A. Bronchiectatic disease
B. Pulmonary gangrene
C. Acute pulmonary abscess
D. Chronic pneumonia
E. * Chronic abscess
318. A male was treated for purulent otitis. On the 9th day of his staying at an inpatient department he
died from a brain oedema. On autopsy, the temporal region of the left hemisphere revealed a cavity
with uneven rough inner edges which was filled with some yellowish-greenish thick dull fluid. The
outer wall of the cavity was represented with the cerebral tissue. What pathological process was it?
A. Colliquative necrosis
B. Phlegmon
C. Empyema
D. Chronic abscess

E. * Acute abscess
319. An autopsy revealed 0.5 I of some yellowish transparent fluid with small white crumble clots in the
right pleural cavity. The parietal and visceral pleurae were covered with a white crumble coat. What
kind of exudative inflammation was it?
A. Suppurative
B. Serous
C. Putrid
D. Catarrhal
E. * Croupous
320. A 6-year-old girl fell ill with diphtheria and three days later died of asphyxia resulting from
membranous croup. On autopsy, the mucous membranes of the larynx, trachea and bronchi were
thickened, oedematous and covered with greyish films which were easily separated. What kind of
inflammation did the morphological changes in the larynx indicate?
A. Serous
B. Haemorrhagic
C. Diphtheritic
D. Croupous
E. * Catarrhal
321. An autopsy of a 77-year-old male, who died from dysentery, revealed some grey-yellow films which
were closely connected with the underlying tissues in the colon and separated with formation of
ulcers. Name the kind of inflammation.
A. Serous
B. Catarrhal
C. Croupous
D. Suppurative
E. * Diphtheritic
322. A microscopic examination of the tissue dissected from some postoperative infiltrate revealed
granulomata with giant multinucleate cells around the suture material. What kind of granulomata did
they belong to?
A. Tuberculous
B. Rheumatic
C. Lepromatous
D. Mycotic
E. * Foreign-body
323. An examination of a renal biopsy revealed some mostly perivascular and periglomerular
lymphocytic, plasmacytic and macrophagal infiltration of the interstice against a background of its
sclerosis. Name the most probable kind of inflammation.
A. Productive diffuse
B. Granulomatous
C. Exudative diffuse
D. Exudative focal
E. * Productive focal
324. A microscopic examination of the myocardium in a male, who died from cardiac decompensation,
revealed sclerosis of the perivascular connective tissue and its diffuse infiltration by lymphocytes,
macrophages, plasmacytes and solitary neutrophils. Which of the listed kinds of inflammation was
the most probable?
A. Granulomatous productive
B. Alterative
C. Exudative diffuse

D. Exudative focal
E. * Interstitial productive
325. A little girl died of asphyxia resulting from membranous croup. A girl illed with diphtheria. On
autopsy, the mucous membranes of the larynx, trachea and bronchi were thickened, oedematous and
covered with greyish films which were easily separated. What type of inflammation
pathomorpholodist investigated?
A. Serous
B. Haemorrhagic
C. Diphtheritic
D. Croupous
E. * Catarrhal
326. A surgeon removed a tumour in the liver of a 47-year-old male patient. It was macroscopically
revealed that the wall of the cavity was formed by a dense fibrous connective tissue; the cavity
contained some yellow-greenish dull thick fluid which had an unpleasant odour and microscopically
consisted mainly of polymorphonuclear leukocytes. What pathological process did such
morphological changes correspond to?
A. Phlegmon
B. Acute abscess
C. Empyema
D. Colliquative necrosis
E. * Chronic abscess
327. A children died from asphyxia at the diphtheria. On autopsy were identified membranous plates in
space of respiratory ways, the mucous membranes of the larynx, trachea and bronchi were thickened,
oedematous and covered with greyish films which were easily separated. What kind of inflammation
did the morphological changes in the larynx indicate?
A. Serous
B. Haemorrhagic
C. Diphtheritic
D. * Croupous
E. Catarrhal
328. An autopsy of a 58-year-old male, who suffered from croupous pneumonia during his life-time and
died of cardiopulmonary insufficiency, revealed 900 ml of some yellow-greenish dull fluid in his
right pleural cavity. The pleural leaves were dull and plethoric. Name the clinical-morphological
form of the inflammation in the pleural cavity.
A. Dry pleurisy
B. Phlegmon
C. Chronic abscess
D. Acute abscess
E. * Empyema
329. An autopsy of a woman, who suffered from the right-sided purulent otitis during her life-time,
revealed a cavity 4 x 3 cm in size in the region of the right temporal lobe that contained some
yellow-green dull viscous fluid. The inner layer of the wall was represented with a yellowish crumble
tissue, the outer layer consisted of a whitish dense tissue. What process were the described changes in
the brain characteristic of?
A. Acute abscess
B. Empyema
C. Grey softening of the brain
D. Haemorrhage
E. * Chronic abscess

330. A pathology-histology laboratory received a vermiform appendix up to 2,0 cm thick. Its serous
membrane was pale, thick and covered with yellowish-green films. The wall was flaccid, of
grayish-red colour. The appendix lumen was dilated and filled with yellowish-green substance.
Histological examination revealed that the appendix wall was infiltrated with neutrophils. Specify the
appendix disease:
A. * Acute phlegmonous appendicitis
B. Acute gangrenous appendicitis
C. Acute superficial appendicitis
D. Acute simple appendicitis
E. Chronic appendicitis
331. Autopsy of a 50-year-old man revealed the following changes: his right lung was moderately compact
in all parts, the dissected tissue was found to be airless, fine-grained, dryish. Visceral pleura had
greyish-brown layers of fibrin. What is the most likely diagnosis?
A. * Croupous pneumonia
B. Tuberculosis
C. Bronchopneumonia
D. Interstitial pneumonia
E. Pneumofibrosis
332. A patient with android-type obesity had been suffering from arterial hypertension, hyperglycemia,
glycosuria for a long time and died from the cerebral haemorrhage. Pathologic examination revealed
pituitary basophil adenoma, adrenal cortex hyperplasia. What is the most likely diagnosis?
A. * Itsenko-Cushing's syndrome
B. Diabetes mellitus
C. Acromegalia
D. Pituitary nanism
E. Adiposogenital dystrophy
333. Mucous membrane of the right palatine tonsil has a painless ulcer with smooth lacquer fundus and
accurate edges of cartlaginous consistency. Microscopically: inflammatory infiltrate that consists of
lymphocytes, plasmocytes, a small number of neutrophils and epithelioid cells; endovasculistis and
perivasculitis. What disease is in question?
A. * Syphilis
B. Actinomycosis
C. Tuberculosis
D. Pharyngeal diphtheria
E. Necrotic (Vincent's) tonsillitis
334. At patomorphological microscopic examination of the myocardium in a male, who died from cardiac
decompensation, revealed sclerosis of the perivascular connective tissue and its diffuse infiltration by
lymphocytes, macrophages, plasmacytes and solitary neutrophils. Which of the listed kinds of
inflammation was the most probable?
A. * Interstitial productive
B. Granulomatous productive
C. Alterative
D. Exudative diffuse
E. Exudative focal
335. A microscopic examination of some inflammatory infiltrate of skin revealed an accumulation of
purolekocytic exudate in several hair follicles. What type of inflammation is this?
A. Phlegmon
B. Furuncle
C. * Carbuncle

D. Wandering abscess
E. Abscess
336. A 77-year-old male died from dysentery. An autopsy revealed some grey-yellow films which were
closely connected with the underlying tissues in the colon and separated with formation of ulcers.
Name the kind of inflammation.
A. * Diphtheritic
B. Serous
C. Catarrhal
D. Croupous
E. Suppurative
337. A tumour in the liver was investigated. Macroscopically was revealed that the wall of the cavity was
formed by a dense fibrous connective tissue; the cavity contained some yellow-greenish dull thick
fluid which had an unpleasant odour and microscopically consisted mainly of polymorphonuclear
leukocytes. What pathological process did such morphological changes correspond to?
A. Phlegmon
B. Acute abscess
C. * Chronic abscess
D. Empyema
E. Colliquative necrosis
338. During patomorphological examination examination of a renal biopsy revealed some mostly
perivascular and periglomerular lymphocytic, plasmacytic and macrophagal infiltration of the
interstice against a background of its sclerosis. Name the kind of inflammation.
A. Productive diffuse
B. * Productive focal
C. Granulomatous
D. Exudative diffuse
E. Exudative focal
339. The myocardium in a male, who died from cardiac decompensation, revealed sclerosis of the
perivascular connective tissue and its diffuse infiltration by lymphocytes, macrophages, plasmacytes
and solitary neutrophils. What kind of inflammation is this most probable?
A. * Interstitial productive
B. Granulomatous productive
C. Alterative
D. Exudative diffuse
E. Exudative focal
340. A microscopic examination of some inflammatory infiltrate of skin revealed an accumulation of
purolekocytic exudate in several hair follicles. What is the name of such an inflammation?
A. Phlegmon
B. Furuncle
C. Abscess
D. Wandering abscess
E. * Carbuncle
341. During pathomorphological investigation of a male, who died from vascular collapse, revealed an
increased pigmentation of the skin, the adrenal glands were reduced in size, the brown-yellow liver
was enlarged. On histological examination, foci of necrosis with a tuberculous granulation tissue
were found in the adrenal glands. The liver was characterized by phenomena of fatty degeneration.
Which of the diagnoses was the most probable?
A. * Addison's disease
B. Steatosis

C. Primary aldosteronism
D. Cushing's syndrome
E. Lipofuscinosis
342. During pathomorphological investigation of a 50-year-old male, who suffered from numerous
pathological fractures during his lifetime, revealed changes in his long tubular bones: the bones of the
thigh and shin were bent, in some places they resembled spirals, their surface was tuberous, a section
revealed an obliterated medullary channel and a change in the compact structure of the cortical layer
by the spongy type. Microscopically, there was a mosaic type of the bone structures: against a
background of a disordered thin-fibrous or lamellar structure of the bone fragments there were
numerous cavities of sinusal resorption combined with signs of new formation of the osseous tissue.
The arteries, which supplied the bone tissue, were dilated and convoluted. Name a diagnosis.
A. * Deforming osteodystrophy
B. Osteopetrosis
C. Parathyroid osteodystrophy
D. Fibrous dysplasia
E. Chronic osteomyelitis
343. A death of a 7-year-old boy resulted from acute posthaemorrhagic anaemia caused by a profuse
bleeding from the gastrointestinal tract. A postmortem examination revealed: macroscopically - an
anaemia of the internal organs, an enlargement of lymph nodes in different groups, thymomegaly, a
moderately manifested hepatosplenomegaly, a bright red bone marrow; microscopically - a
hypercellular bone marrow with some monomorphous infiltrate of blast cells, diffuse-focal tumour
infiltrates in the liver, spleen, lymph nodes, meninges and substance of the brain. Make a diagnosis
for this form of leukaemia.
A. * Acute lymphoblastic
B. Acute myeloblastic
C. Acute stem cell
D. Acute monoblastic
E. Acute plasmablastic
344. An autopsy of 78-year-old male patient, who died from cardiopulmonary insufficiency, revealed
an enlarged right lung with massive fibrinous superpositions on the pleura. Histologically, the
alveolar lumen had accumulations of fibrin and neutrophils. The lymph nodes in the lung roots were
pale pink and somewhat enlarged. Name the stage of croupous pneumonia.
A. Red hepatization
B. Influx
C. * Grey hepatization
D. Resolution
E. Edema
345. An autopsy of a 45-year-old male patient, who had had double bronchopneumonia and died under the
phenomena of intoxication, revealed in the lower lobe of the right lung some thick-walled cavity, 4
cm in diameter, filled with liquid yellowish masses. What pathological process complicated the
course of pneumonia?
A. Tuberculoma
B. * Gangrene
C. Abscess
D. Sequester
E. Empyema
346. On autopsy, 380 ml of some yellow fluid with an unpleasant odour were found in the right pleural
cavity. Microscopically, the liquid contained a lot of neutrophilic granulocytes. What is your
diagnosis?
A. Pulmonary abscess

B. Phlegmon
C. * Pleural empyema
D. Pulmonary gangrene
E. Purulent catarrh of pleura
347. An autopsy of a 34-year-old male, who died from cardiopulmonary insufficiency, revealed an
enlarged grey dense air-free lower lobe of the left lung, the vincernl pleura of this lobe had
superpositions of fibrin. Microscopically, the lumens of alveoli revealed some fibrinous-leukocytic
exudate. Name the stage of croupous pneumonia.
A. Influx
B. Red hepatization
C. Resolution
D. * Grey hepatization
E. Edema
348. An autopsy of a male patient, who died from cardiopulmonary insufficiency, revealed some dim
liquid with grey flakes in the pleural cavities, the lungs were enlarged, air-free and had some
grey-whitish films on the pleura. On section, the lungs were white-greyish and dense, their surface
was granular. The peribronchial lymph nodes were enlarged and whitish-grey on section.
Microscopically, the alveoli contained some fibrinous-leukocytic exudate. Name the stage of
croupous pneumonia.
A. * Grey hepatization
B. Red hepatization
C. Influx
D. Resolution
E. Caseous pneumonia
349. During pathomorphological investigation of a male, who suffered from right-sided pneumonia,
revealed in the right lung some cavity 3.5 cm in diameter, that had uneven edges, communicated with
the bronchus and was filled with some purulent exudate. Microscopically, the wall of the cavity was
formed by the granulation tissue diffusely infiltrated by leukocytes. Which of the diagnoses, listed
below, was the most probable?
A. Acute pneumogenic abscess
B. * Acute bronchiogenic abscess
C. Chronic bronchiogenic abscess
D. Chronic pneumogenic abscess
E. Pulmonary echinococcosis
350. A 7 year old child was taken to the infectious disease hospital with complaints of acute pain during
swallowing, temperature rise up to 390С, neck edem. Objective signs: tonsills are enlarged,
their mucous membrane is plethoric and covered with a big number of whitish-yellowish
films that are closely adjacent to the mucous membran. After removal of these films the
deep bleeding defect remains. What type of inflammation is it?
A. Purulent
B. Crupous
C. Serous
D. Hemorrhagic
E. * Diphteritic
351. Histologic examination revealed in all layers of appendix a big number of polymorthonuclear
leukocytes; hyperemia, stases. What disease are these symptoms typical for?
A. Superficial appendicitis
B. * Phlegmonous appendicitis
C. Gangrenous appendicitis

D. Chronic appendicitis
E. Simple appendicitis
352. Post-mortem examination of a 5 year old boy who died from acute pulmonary and cardiac
insufficiency revealed the following: serohemorrhagic tracheobronchitis with some necrotic areas of
mucous membrane, multiple foci of hemorrhagic pneumonia in lungs. What disease is in question?
A. Diphtheria
B. Croupous pneumonia
C. Scarlet fever
D. * Influenza
E. Measles
353. At the operation the doctor opened a patient's abdominal cavity and revealed for about 2,0 L of
purulent fluid. Peritoneum is dull, greyish, serous tunic of intestines has grayish layers that can be
easily removed. It is most likely to be:
A. Serous peritonitis
B. Hemorrhagic peritonitis
C. Tuberculous peritonitis
D. * Fibrinopurulent peritonitis
E. -
354. During the histologic examination of thyroid gland of a man who died of cardiac insufficiency
together with hypothyroidism there was found the diffusive infiltration of gland by lymphocytes and
plasmocytes, parenchyma atrophy and growth of connective tissue. Formulate a diagnosis:
A. * Hashimoto's thyroiditis
B. Thyroid gland adenoma
C. Purulent thyroiditis
D. Thyrotoxic goiter
E. -
355. During pathomorphological investigation was revealed a big number of polymorphonuclear
leukocytes in all layers of appendix; hyperemia, stases. What disease are these symptoms typical for?
A. * Phlegmonous appendicitis
B. Gangrenous appendicitis
C. Superficial appendicitis
D. Simple appendicitis
E. Chronic appendicitis
356. During pathomorphological investigation of a 45 old-age man microscopic analysis of tissue
sampling from patient's skin reveals granulomas that consist of epithelioid cells surrounded mostly by
T-lymphocytes. Among epithelioid cells there are solitary giant multinuclear cells of
Pirogov-Langhans type. In the centre of some granulomas there are areas of caseous necrosis. Blood
vessels are absent. What disease are the described granulomas typical for?
A. * Tuberculosis
B. Syphilis
C. Leprosy
D. Rhinoscleroma
E. Glanders
357. Autopsy of a man who died of typhoid fever revealed ulcers along the ileum. These ulcers have even
sides, clean fundus formed by muscle layer or even by serous tunic of an intestine. What stage of
disease does the described presentation correspond with?
A. * Stage of "clean" ulcers
B. Stage of medullary swelling

C. Stage of necrosis
D. Stage of "dirty" ulcers
E. Stage of ulcer healing
358. Mucous membrane of the right palatine tonsil has a painless ulcer with smooth lacquer fundus and
accurate edges of cartlaginous consistency. Microscopically: inflammatory infiltrate that consists of
lymphocytes, plasmocytes, a small number of neutrophils and epithelioid cells; endovasculistis and
perivasculitis. What disease is in question?
A. * Syphilis
B. Actinomycosis
C. Tuberculosis
D. Pharyngeal diphtheria
E. Necrotic (Vincent's) tonsillitis
359. A 48 old-age years woman patient with android-type obesity had been suffering from arterial
hypertension, hyperglycemia, glycosuria for a long time and died from the cerebral haemorrhage.
Pathologic examination revealed pituitary basophil adenoma, adrenal cortex hyperplasia. What is the
most likely diagnosis?
A. Pituitary nanism
B. Diabetes mellitus
C. Acromegalia
D. * Itsenko-Cushing's syndrome
E. Adiposogenital dystrophy
360. Enzymatic jaundices are characterized by disbalanced activity of UDP-glucuronyl transferase. What
compound is accumulated in the blood serum in case of these pathologies?
A. * Indirect bilirubin
B. Direct bilirubin
C. Biliverdin
D. Mesobilirubin
E. Verdoglobin
361. A patient with apparent icteritiousness of skin, sclera and mucous membranes was admitted to the
hospital. The patient's urine was of brown ale colour, analysis revealed presence of direct bilirubin.
Feces had low concentration of bile pigments. What type of jaundice is it?
A. * Obturative
B. Parenchymatous
C. Haemolytic
D. Conjugated
E. Absorbtion
362. During pathomorphological investigation a doctor laboratory received a vermiform appendix up to
2,0 cm thick. Its serous membrane was pale, thick and covered with yellowish-green films. The wall
was flaccid, of grayish-red colour. The appendix lumen was dilated and filled with yellowish-green
substance. Histological examination revealed that the appendix wall was infiltrated with neutrophils.
Specify the appendix disease:
A. Acute gangrenous appendicitis
B. * Acute phlegmonous appendicitis
C. Acute superficial appendicitis
D. Acute simple appendicitis
E. Chronic appendicitis
363. A pathohistological examination of the thyroid gland detected a significant infiltration of its tissue by
lymphocytes, formation of lymphoid follicles, an atrophy of parenchymatous elements and a
significant vegetation of the connective tissue. What disease is characterized by this picture?

A. Colloid goiter
B. Endemic goiter
C. Diffuse toxic goiter
D. Parenchymatous goiter
E. * Hashimoto's disease
364. A young female died from adrenal insufficiency. An autopsy was performed in pathoanatomical
bureau detected diffuse hypermelanosis of the skin, hyperplasia of the cells in islets of Langerhans in
the pancreas, the adrenal glands were sharply reduced in size and their thinned cortical substance had
foci of necrosis, haemorrhages and sclerosis. What is your diagnosis?
A. Waterhouse-Friderichsen syndrome
B. Primary aldosteronism
C. Cushing's disease
D. Pheochromocytoma
E. * Addison's disease
365. A 50-year-old male died from vascular collapse. An autopsy was performed in the pathoanatomical
bureau and detected an increased pigmentation of the skin, the adrenal glands were reduced in size,
the brown-yellow liver was enlarged. On histological examination, foci of necrosis with a
tuberculous granulation tissue were found in the adrenal glands. The liver was characterized by
phenomena of fatty degeneration. Which of the diagnoses was the most probable?
A. Steatosis
B. Primary aldosteronism
C. Cushing's syndrome
D. Lipofuscinosis
E. * Addison's disease
366. A 54-year-old male suffered from numerous pathological fractures during his lifetime. After his death
an autopsy detected the changes in his long tubular bones: the bones of the thigh and shin were bent,
in some places they resembled spirals, their surface was tuberous, a section revealed an obliterated
medullary channel and a change in the compact structure of the cortical layer by the spongy type.
Microscopically, there was a mosaic type of the bone structures: against a background of a disordered
thin-fibrous or lamellar structure of the bone fragments there were numerous cavities of sinusal
resorption combined with signs of new formation of the osseous tissue. The arteries, which supplied
the bone tissue, were dilated and convoluted. Name a diagnosis.
A. Osteopetrosis
B. Parathyroid osteodystrophy
C. Fibrous dysplasia
D. Chronic osteomyelitis
E. * Deforming osteodystrophy
367. A 43-year-old male suffered from right-sided pneumonia in the lower lobe during his life-time and
for a long period of time expectorated sputum of a purulent character. After his death an autopsy
detected some cavity with dense edges that was located in the 9th-10th segments of the lung and was
filled with yellowish cream-like masses. There was some whitish path from the cavity to the root of
the lung. Microscopically, the cavity was separated from the intact pulmonary tissue with a
membrane which consisted of a fibrous connective tissue from the outside and a granulation one from
inside. Which of the diagnoses was the most probable?
A. Bronchiectatic disease
B. Pulmonary gangrene
C. Acute pulmonary abscess
D. Chronic pneumonia
E. * Chronic abscess

368. An autopsy of a male, who died from sepsis, revealed a purulent melting of the tissue of the neck and
the fat of the anterior mediastinum. Histologically, there were a lot of small abscesses with a
honeycomb structure, whose centres had basophilic homogenous formations to which some short
rod-like structures were connected with one end. What is your diagnosis?
A. * Actinomycosis
B. Tuberculosis
C. Diphtheria
D. Leishmaniasis
E. Scarlet fever
369. A 45-year-old female furrier died from brain oedema. On autopsy, the pia mater in the vault and base
of the skull were dark red and impregnated with blood, as if they were covered with a (red cap(; the
brain substance was oedematous, the stem was strangulated in the great foramen. Microscopically,
the pia mater had a haemorrhagic inflammation, the cerebral tissue was characterized by a
serous-haemorrhagic inflammation, necrosis of the walls of small vessels, numerous haemorrhages.
What disease was the patient most probably ill with?
A. * Anthrax
B. Meningococcal leptomeningitis
C. Meningococcal meningoencephalitis
D. Subarachnoid haemorrhage
E. Tuberculous leptomeningitis
370. An autopsy of a cachectic 36-year-old male, who during 10 years was suffering from femoral
osteomyelitis with periodic elevations of the body temperature up to 38(C, general weakness, a
headache, and manifestations of intoxication and died from a progressing renal failure, revealed
amyloid shrunk kidneys, a brown atrophy of the myocardium and liver, a sago spleen. What kind of
sepsis was the complication of osteomyelitis in this case?
A. * Chronic septicaemia
B. Fulminant sepsis
C. Septicopyaemia
D. Septicaemia
E. Septic endocarditis
371. An autopsy of a female, who died 4 days after a criminal abortion, revealed jaundice, an expressed
thrombohaemorrhagic syndrome, fibrinous-purulent endometritis, necronephrosis in the kidneys, a
septic hyperplasia of the spleen. Name the clinical-morphological form of sepsis.
A. * Septicaemia
B. Bacterial shock
C. Septicopyaemia
D. Chronic septicaemia
E. Septic endometritis
372. A 40-year-old male butcher died from sepsis. His right cheek had a dark red, conical and dense
infiltrate, 6 cm in diameter, with a black crust in its centre.. The right halves of the face and neck
were oedematous and dense. A microscopic examination revealed an acute serous-haemorrhagic
inflammation in the infiltrate, the centre of the latter had necrosis of the epidermis and underlying
tissues. What diagnosis did a pathologist make?
A. * Anthrax
B. Plague
C. Tularaemia
D. Phlegmon of neck
E. Furuncle

373. During an operation for elimination of phimosis, a round ulcer with a smooth bottom and dense edges
was found on the glans penis. On a microscopic examination of the tissue taken from an edge of the
ulcer a pathologist found a polymorphous infiltrate consisting of disorderly located plasma and
lymphoid elements with some admixture of heterophilic leukocytes and epithelioid cells; the infiltrate
was located mostly around small vessels. The vessels were characterized by phenomena of
endangiitis. What disease are the described changes typical for?
A. * Syphilis
B. Actinomycosis
C. Tuberculosis
D. Polyarteritis nodosa
E. Leprosy
374. On visual examination of a male, who was admitted in a severe state and with a high body
temperature, the physician noticed that the inguinal lymph nodes were enlarged up to 8 cm in
diameter, tightly united with the surrounding tissue and immobile, the skin above them was
hyperaemic and sharply tender. A microscopic examination revealed the most acute
serous-haemorrhagic inflammation in a lymph node. From his case history it was known that the
patient had spent some time before on a mountain pasture. What disease is characterized by the
changes described?
A. * Plague
B. Tularaemia
C. Anthrax
D. Brucellosis
E. –
375. A 40-year-old male hunter at first noticed appearance of some pustule on the index finger of his right
hand, later it turned into an ulcer with uneven edges. His right axillary lymph nodes were
significantly enlarged and tightly united among themselves. A biopsy of the finger skin was made. Its
microscopic examination revealed necrosis of the epithelium and underlying tissues, the derma had
granulomata consisting of epithelioid cells, lymphocytes, a large number of segmented leukocytes
and giant cells. There were areas of necrosis and suppuration in the centres of the granulomata. What
are these changes typical for?
A. * Tularaemia
B. Syphilis
C. Tuberculosis
D. Brucellosis
E. Actinomycosis
376. A 35-year-old milkmaid died from cardiovascular insufficiency. On autopsy, a pathologist diagnosed
polypous-ulcerous endocarditis of the aortic valve. Microscopically, the aortic valve revealed an
inflammatory infiltration by polymorphonuclear leukocytes, areas of ulceration and thrombi with
organization. The myocardial stroma was characterized by sclerosis and granulomata consisting of
disorderly located epithelioid, giant, plasma cells and eosinophils, the vessels had vasculitis. What
disease is characterized by the described changes in the heart?
A. * Brucellosis
B. Chronic sepsis
C. Rheumatism
D. Systemic lupus erythematosus
E. Systemic scleroderma

377. A 40-year-old male butcher died from sepsis. His right cheek had a dark red, conical and dense
infiltrate, 5 cm in diameter, with a black coal-like centre. The right halves of the face and neck were
oedematous and dense. The cervical lymph nodes were enlarged, on section they were dark red, the
surrounding fat was red and oedematous. A microscopic examination revealed the most acute
serous-haemorrhagic inflammation in the infiltrate, the centre of the latter had necrosis of the
epidermis and underlying layers, the lymph nodes and fat were characterized by a
serous-haemorrhagic inflammation. What disease should be suspected?
A. * Anthrax
B. Tularaemia
C. Plague
D. Carbuncle
E. Furuncle
378. The mucous membrane of the right tonsil revealed a painless ulcer with a smooth vanished bottom
and even edges of the chondroid density. Microscopically, a biopsy from the ulcer edge had a
perivascular inflammatory infiltrate consisting of lymphocytes, plasmacytes, some neutrophils and
epithelioid cells. The vessels were characterized by a proliferation of endotheliocytes up to the
absolute closure of the lumens. Name the disease.
A. * Syphilis
B. Actinomycosis
C. Ulceronecrotic angina
D. Candidiasis
E. Faucial diphtheria
379. In a male cattle-breeder, the disease began with appearance of some red spot on the skin of his right
arm; at first, a vesicle with some serous-haemorrhagic fluid formed in the centre of the spot, but later
the central part of the vesicle necrotized and became black. The axillary lymph nodes were sharply
enlarged and dark red on section. Microscopically, a biopsy of the skin and lymph nodes had a
picture of acute serous-haemorrhagic inflammation. What disease was the most probable?
A. * Anthrax
B. Streptococcal pyoderma
C. Plague
D. Actinomycosis of skin
E. Tularaemia
380. For a long time a 45-year-old countrywoman complained of weakness, insomnia, irritability, pains in
her muscles and joints, an elevation of the body temperature up to 37.1-37.3(C. Her death came under
the phenomena of multiorgan insufficiency. On autopsy, the organs revealed granulomata formed by
epithelioid cells, disorderly located Pirogov-Langhans giant cells and a small number of plasma cells
and eosinophils. The granulomata had a lot of vessels, there were no foci of necrosis, and systemic
productive-destructive vasculitis was revealed. What is your diagnosis?
A. * Brucellosis
B. Tularaemia
C. Plague
D. Tuberculosis
E. Anthrax
381. Three days after a criminal abortion a female developed an elevation of her body temperature up to
40(C, cloudiness of consciousness, petechial haemorrhages on her skin. Two days later she died
under increasing phenomena of intoxication. An autopsy revealed jaundice, petechial haemorrhages
in the serous and mucous membranes, a sharp enlargement of the spleen and lymph nodes.
Microscopically, the spleen and lymph nodes were characterized by a proliferation of the lymphoid
and reticular cells, as well as a large number of immature forms of haemopoiesis. Besides, there were
dystrophic changes and an interstitial inflammation in the liver, kidneys, heart, as well as
disseminated necrotic vasculititides. What was the most probable form of sepsis?

A. * Septicaemia
B. Chernogubov’s disease
C. Septicopyaemia
D. Chronic sepsis
E. Fulminant sepsis
382. An autopsy of 2-month-old infant revealed papules and pustules on the skin, white dense lungs (white
pneumonia), a brown dense liver (a brimstone liver), signs of osteochondritis. Indicate the most
probable diagnosis.
A. * Congenital syphilis
B. Congenital toxoplasmosis
C. Congenital cytomegalovirus infection
D. Congenital herpetic infection
E. Congenital mononucleosis
383. An autopsy of an 8-year-old child, who suffered from deafness during his life-time and died of
pneumonia, revealed signs of parenchymatous keratitis and twisted teeth with formation of crescent
grooves on the central incisors (Hutchinson’s triad). Microscopically, there was a chronic diffuse
interstitial inflammation in the liver, lungs and ovarian tissue, as well as Dubois’s abscesses in the
thymus. Which of the diagnoses listed below was the most probable?
A. * Congenital syphilis
B. Congenital toxoplasmosis
C. Congenital cytomegalovirus infection
D. Congenital herpetic infection
E. Congenital mononucleosis
384. A male furrier, who dressed coypu skins (nutrias), died from respiratory insufficiency. On autopsy,
his lung had dense foci whose microscopic examination revealed serous-haemorrhagic pneumonia,
coagulation necroses, granulomata consisting of epithelioid, lymphoid and giant cells, as well as foci
of suppuration. The enlarged regional lymph nodes had the same granulomata, necroses and
suppuration. Which of the diagnoses listed below was the most probable?
A. * Tularaemia
B. Plague
C. Tuberculosis
D. Sarcoidosis
E. Actinomycosis
385. A 32-year-old female died during a puerperal period. An autopsy revealed purulent endometritis,
purulent thrombophlebitis of the uterine veins, numerous pulmonary abscesses, abscesses of the
kidneys and spleen, apostematous myocarditis and purulent meningitis. Make a diagnosis of the
clinical-morphological form of sepsis.
A. * Septicopyaemia
B. Septicaemia
C. Chronic sepsis
D. Chernogubov’s disease
E. Fulminant sepsis
386. A female, who had a phlegmon of her thigh, died from intoxication. An autopsy revealed jaundice,
petechiae on the skin, haemorrhages in the serosae, mucosae and internals. Microscopically, the
spleen and lymph nodes had a proliferation of the lymphoid and reticular cells with accumulation of
immature cells of haemopoiesis in the marrow. The parenchymatous organs were characterized by a
diffuse interstitial inflammation, manifestations of cloudy swelling and fatty degeneration. Which of
the diagnoses listed below was the most probable?
A. * Septicaemia

B. Septicopyaemia
C. Chronic sepsis
D. Plague
E. AIDS
387. During his life-time, a 40-year-old male had a high body temperature, jaundice, clouded
consciousness, bacteraemia. On autopsy, his sclerae and skin were yellow, there were numerous
haemorrhages in the serous and mucous membranes, as well as enlarged lymph nodes and spleen.
Microscopically, the spleen and lymph nodes had a proliferation of the reticular cells. The heart, liver
and kidneys were characterized by an interstitial inflammation, a parenchymatous cloudy swelling;
the vascular walls had vasculititides and a fibrinoid swelling. Which of the diagnoses listed below
was the most probable?
A. * Septicaemia
B. Septicopyaemia
C. Acute lymphocytic leukaemia
D. Acute stem cell leukaemia
E. Chronic sepsis
388. An autopsy of a 40-year-old male, who worked as a sheep-breeder and died from a severe
intoxication, revealed on the skin some papulopustular haemorrhagic rash with necroses and
ulcerations, as well as enlarged immovable dark red lymph nodes in the inguinal region which were
united with one another and had a doughy consistency. The spleen was enlarged, flaccid and had an
abundant scrape. Microscopically, the lungs revealed a focal serous-haemorrhagic inflammation with
necroses; the heart, liver and kidneys were characterized by dystrophic and necrotic changes; the
lymph nodes had haemorrhagic-necrotic lymphadenitis with some purulent melting. Make a
diagnosis.
A. * Bubonic plague
B. Bubocutaneous form of plague
C. Primary pneumonic plague
D. Septicaemic plague
E. Septicaemia
389. A professional male hunter developed an acute elevation of body temperature and enlarged united
lymph nodes in his left axilla. A microscopic examination of the lymph nodes revealed granulomata
consisting of epithelioid, lymphoid, giant cells and polymorphonuclear leukocytes, as well as foci of
necrosis and purulent inflammation. Which of the diagnoses listed below was the most probable?
A. * Tularaemia
B. Plague
C. Tuberculosis
D. Anthrax
E. Brucellosis
390. On autopsy of a 65-year-old male, who for many years was ill with a rheumatic heart defect and died
from cardiovascular insufficiency, it was found that the weight of the heart was 900 g, the patient had
hypertrophy of the left ventricle of the heart, polypous-ulcerous endocarditis of the aortic and mitral
valves with their petrification and deformation, hyperplasia of the spleen, “old” renal and pulmonary
infarcts. A microscopic examination revealed diffuse vasculititides, proliferative isolated
myocarditis, nephritis, hepatitis, glomerulonephritis, the thrombohaemorrhagic syndrome. Which of
the diagnoses listed below was the most probable?
A. * Secondary bacterial endocarditis
B. Chernogubov’s disease
C. Fibroplastic endocarditis
D. Acute verrucous endocarditis
E. Aortomitral heart defect
391. An autopsy of a male revealed that the right upper extremity was oedematous, the skin in the middle
third part of the arm was dark red, dense and diffusely impregnated with blood, in the centre there
was a vesicle with some serous-haemorrhagic exudate (a “phlyctena”). The regional lymph nodes
were enlarged, united with one another and immobile. Microscopically, the lymph nodes were
characterized by a serous-haemorrhagic inflammation, a large number of microbes, a proliferation of
the reticular cells, foci of necrosis. There were dystrophic processes in the parenchymatous organs,
the spleen was enlarged and flaccid. Which of the diagnoses listed below was the most probable?
A. * Bubocutaneous plague
B. Bubonic plague
C. Primary pneumonic plague
D. Septicaemic plague
E. Secondary bubonic pneumonia
392. A male cattle-farm worker developed a dark red spot on the skin of his thigh; in the centre of the spot
there was formation of a vesicle filled with some serous-haemorrhagic fluid, a bit later the centre
necrotized and became dark. The inguinal lymph nodes were enlarged, dense and dark red on section.
Under the phenomena of severe intoxication the patient died. A microscopic examination of the skin
vesicles and lymph nodes revealed a diffuse haemorrhagic infiltration and a sharp oedema of the
surrounding tissues. Which of the forms of anthrax was the most probable?
A. * Cutaneous
B. Septicaemic
C. Enteric
D. Primary pneumonic
E. Mixed
393. An autopsy of a male, who had a prolonged suppuration of the wound following an injury of his
extremity and died under the phenomena of intoxication, revealed cachexia, dehydration, a brown
atrophy of the liver, myocardium, spleen and striated muscles, as well as renal amyloidosis. Which of
the diagnoses was the most probable?
A. * Chronic sepsis
B. Chernogubov’s disease
C. Septicaemia
D. Septicopyaemia
E. Brucellosis
394. A cattle worker has died with signs of severe intoxication in a 2nd day after beginning of disease.
Autopsy was showed the enlarged flabby spleen, on the surface of the cut of dark-cherry colored; the
scrape of the pulp is abundant. The leptomeninges are swelling, infiltrating by blood of dark-red
color (“a red cap”). It was defined a microorganism – bacillus anthracis – and diagnosed anthrax.
What kind of inflammation does occur in that disease?
A. * Hemorrhagic
B. Phlegmonous
C. Fibrinous
D. Purulent
E. Putrefactive
395. A 36-year-old farmer had been suffering from unknown disease for 3,5 months. Microscopical
investigation: there were granulomas in internal organs consisting of epithelioid, Langhans giant-
cells, plasma cells, eosinophils. In granulomas there are a lot of vessels with features of
productive-destructive vasculitis. Diagnose this disease
A. * Brucellosis
B. Malignant anthrax
C. Tuberculosis
D. Classical typhus
E. Leprosy
396. An experimental animal received a subcutaneous dose of an antigen preceded by sensitization. At the
place of the injection, some fibrinous inflammation developed with an alteration of the vascular
walls, the main substance and fibrous structures of the connective tissue in the form of a mucoid and
fibrinoid swelling, a fibrinoid necrosis. Which of the diagnoses listed below was the most probable?
A. Delayed hypersensitivity
B. Transplantation immunoreaction
C. Normergy
D. Granulomatosis
E. * Immediate hypersensitivity
397. The disease in a male hunter began with an elevation of his body temperature up to 37-38(C,
increased reflex excitability, a disturbance of sleep and hydrophobia. Later these signs were
accompanied by spasms of the muscles of the larynx and pharynx, as well as those of respiration. The
patient’s death was caused by arrest of respiration. On autopsy, an oedema and plethora of the brain,
as well as small haemorrhages in the region of the myelencephalon were found. On histological
examination of the brain, its stem part, walls of the 3rd ventricle and hippocampus revealed necrosis
of the nerve cells which were surrounded (as well as small vessels) by nodules consisting of clusters
of microglial and lymphoid cells. The cytoplasm of the nerve cells of the hippocampus contained
some rounded eosinophilic inclusions (Babes-Negri bodies). What disease is characterized by the
picture described?
A. Typhoid fever
B. Epidemic typhus
C. Poliomyelitis
D. Tick-borne encephalitis
E. * Rabies
398. The disease in a 67-year-old woman acutely began with an expressed oedema and tenderness of the
skin and soft tissues of the neck. A phlegmon of neck and mediastinitis were diagnosed. The patient
died under the increasing phenomena of intoxication. On autopsy, the left tonsil was slightly enlarged
and dense; on section, it was yellowish-greenish and had a lot of small cavities which imparted a
honeycomb structure to it. The soft tissues of the neck and the fat of the anterior mediastinum had
signs of purulent melting. Microscopically, the tissue had a lot of small abscesses, their centres
having intensive basophilic formations, which consisted of short rod-like elements connected with
their one end to the common centre. What is your diagnosis?
A. Giardiasis
B. Leishmaniasis
C. Brucellosis
D. Amoebiasis
E. * Actinomycosis
399. A male patient, who came from the Central Asia, had persistent diarrhoeae, a loss of body weight and
signs of intoxication against whose background he died. An autopsy revealed numerous hepatic
abscesses, the caecum was characterized by dingy green areas of necrosis of its mucous membrane,
these areas slightly rose above its surface and penetrated into the muscular layer. The ulcers resulting
from the necrosis were characterized by undermined edges which hung over their bottom. The
inflammatory reaction in the intestinal wall was poorly expressed. What was the most probable
disease in that case?
A. Salmonellosis
B. Cholera
C. Typhoid fever
D. Bacterial dysentery
E. * Amoebiasis

400. An autopsy of a 45-year-old male, who had had a fever with signs of intoxication during his life-time,
revealed an enlarged dense spleen (500 g); on section, its pulp had numerous grey-white and
white-yellow miliary necroses of follicles, and there were infarct-like foci of necrosis under the
capsule. A histological examination revealed hyperplasia of the follicles with breakdown of
leukocytes and accumulation of neutrophils, and numerous thrombi in the vessels. Which of the
diagnoses listed below was the most probable?
A. Plague
B. Haematogenous general miliary tuberculosis
C. Tularaemia
D. Typhoid fever
E. * Relapsing fever
401. The examination of the child with measles showed the non-clear border edematous fluctuated areas
of red-black color in the soft tissues of the cheeks and perineum. What complication did develop in
the child?
A. Dry gangrene
B. Gas gangrene
C. Bedsore
D. Trophic ulcer
E. * Wet gangrene (noma)
402. A 8 year-old child was ill acutely with clinical signs of vomiting, headache and severe intoxication.
After two days of the disease he has died. In autopsy the pathologist has found out: meninges
thickened, yellowish color on basal surface, edema and hyperemia. Meningococcus was detected
from liquor fluid. Diagnose this disease.
A. Scarlet fever
B. Pertussis
C. Diphtheria
D. Measles
E. * Meningococcal infection
403. A 5 year-old girl has died because of asphyxia owing to true croup. In the autopsy it was established;
mucosa of larynx, trachea and bronchi dwarfed, edematous, dull, coated by grayish fibrinous plaques,
which were easily removed. Described morphological changes are characteristic for…:
A. Flu
B. Measles
C. Pertussis
D. Scarlet fever
E. * Diphtheria
404. A 6 year-old child, was ill acutely with signs of intoxication. In 2 day the patient has died. In autopsy
the pathologist has found out: meninges of brain with edema, hyperemia, yellow-grey exudate.
Tissue of brain was edematous. Microscopic investigation: there were neutrophils, hyperemia,
hemorrhages and edema in meninges. Described changes are most typical for:
A. Flu
B. Pertussis
C. Diphtheria
D. Measles
E. * Meningococcal meningitis
405. Patient has suffered from cholera. Clinical dates are dehydratating, cyanosis and convulsions. In the
result of massive infusion therapy the exicosis has been diminished, but anuria has been remained.
Patient has dead because of uremia. What morphological features in kidney have been found out?
A. Choleric typhoid is developed

B. Development of uremia is connected with acute glomerulonephritis


C. Fibrinous colitis is found in autopsy
D. Exicosis is due to action of virus exotoxin
E. * Necrotic nephrosis with cortical necrosis takes place in the kidneys
406. An autopsy of a 45-year-old male, who suffered from numerous pathological fractures during his
lifetime, revealed changes in his long tubular bones: the bones of the thigh and shin were bent, in
some places they resembled spirals, their surface was tuberous, a section revealed an obliterated
medullary channel and a change in the compact structure of the cortical layer by the spongy type.
Microscopically, there was a mosaic type of the bone structures: against a background of a disordered
thin-fibrous or lamellar structure of the bone fragments there were numerous cavities of sinusal
resorption combined with signs of new formation of the osseous tissue. The arteries, which supplied
the bone tissue, were dilated and convoluted. Name a diagnosis.
A. Osteopetrosis
B. Parathyroid osteodystrophy
C. Fibrous dysplasia
D. Chronic osteomyelitis
E. * Deforming osteodystrophy
407. A 20-year-old girl developed complaints about an expressed fatiguability of her ocular, masticatory,
speech and deglutitive groups of muscles, when the normal contraction of the muscles after great
activity absolutely discontinued, but after some rest the functioning of the muscles was restored
again. Some time later the pathological process involved the muscles of the extremities and
intercostal ones. An inadequate ventilation of the lungs resulted in development of the secondary
lobular pneumonia which caused the patient’s death. An autopsy revealed an atrophy of the striated
muscles, their dystrophy with focal clusters of the lymphocytes in the interstice. An enlarged thymus
was characterized by follicular hyperplasia. What was the most probable diagnosis?
A. Pseudohypertrophic muscular dystrophy
B. Amyotrophic lateral sclerosis
C. Werdnig-Hoffman spinal amyotrophy
D. Erb’s muscular dystrophy
E. * Myasthenia
408. An autopsy of a male, who died from uraemia, revealed deformity of the spinal column with a sharp
limitation of mobility. The articular cartilages of small joints of the spinal column were destroyed,
there were some expressed signs of a prolonged chronic inflammation in the articular tissues, the
cavities of the joints were filled with the connective tissue, but in some places with the osseous one
together with formation of ankyloses. The aorta, heart and lungs revealed a chronic inflammation and
focal sclerosis. The kidneys were characterized by amyloidosis. What diagnosis was the most
probable in this case?
A. Rheumatoid arthritis
B. Paget’s disease (deforming osteosis)
C. Parathyroid osteodystrophy
D. Osteopetrosis (marble bone disease)
E. * Bekhterev’s disease
409. On autopsy of a male, who died from uraemia, it was found that the pancreas was reduced in size, his
contracted kidneys had a fine-grained surface, the liver was enlarged, yellow and flaccid.
Microscopically, the pancreatic tissue revealed an atrophy of the parenchyma, including islets of
Langerhans, the atrophied parenchyma was substituted for hyperplastic connective and fatty tissues.
The kidneys were characterized by sclerosis and hyalinosis of the glomeruli, as well as by a
glycogenic infiltration of the tubules; there was a fatty degeneration in the liver and a fibrinous
inflammation in the mucous coats of the trachea, bronchi and stomach. What disease did the died
person suffer from?
A. Chronic glomerulonephritis

B. Hypertensive disease
C. Chronic indurative pancreatitis
D. Steatosis
E. * Diabetes mellitus
410. An autopsy of a male, who died from chronic renal insufficiency, revealed atherosclerosis of the
aorta and large arteries, small and dense kidneys with a fine-grained surface, an enlarged
yellow-brown and flaccid liver, the pancreas was reduced in size. Microscopically, there was
atherocalcinosis of the aorta and arteries, an atrophy of the parenchyma, sclerosis and lipomatosis of
the pancreas; the kidneys were characterized by hyalinosis of the mesangium and glomeruli, a
glycogenic infiltration of the epithelium of the tubules, with large-drop adiposis in the hepatocytes.
What pathological process took place in the kidneys?
A. Arterial nephrosclerosis
B. Chronic pancreatitis
C. Chronic glomerulonephritis
D. Steatosis
E. * Diabetic nephrosclerosis
411. A 52-year-old male died from renal insufficiency. On microscopic examination of his organs, the
pancreas revealed lipomatosis and sclerosis with an atrophy of islets of Langerhans, the kidneys had
hyalinosis of the mesangium and glomeruli (Kimmelstiel-Wilson syndrome) and a glycogenic
infiltration of the epithelium of the tubules, the liver was characterized by fatty degeneration. Which
of the diagnoses listed below was the most probable?
A. Arterial nephrosclerosis
B. Amyloid shrunk kidneys
C. Chronic glomerulonephritis
D. Goodpasture’s syndrome
E. * Diabetic glomerulosclerosis
412. A 53-year-old male patient, who suffered from peptic ulcer of the stomach for more than 25 years,
was admitted to a surgical department with complaints about frequent vomiting after taking food,
progressing loss of weight, severe thirst. At the hospital, the signs of oliguria and later anuria
developed. The patient died. An autopsy revealed a cicatricial stenosis of the pylorus and a sharp
enlargement of the stomach which practically reached the pelvic region. Which of the complication
of peptic ulcer listed below caused the patient’s death?
A. Peritonitis
B. Penetration of ulcer
C. Erosive haemorrhage
D. Malignancy
E. * Chlorhydropenic uraemia
413. A woman with a clinical picture of acute abdomen underwent surgical removal of an enlarged uterine
tube. On examination, the serous coat of the uterine tube was dark purple, the lumen contained some
blood clots. A histological examination of the wall of the tube revealed that the mucous membrane
had layers of the decidual cells, and there were villi of the chorion among the blood clots. Which of
the following contributes to this disease?
A. antigenic incompatibility of tissues of woman and fetus
B. the age is over 30 years old
C. chromosomal aberrations in gametes
D. all answers are correct
E. * the use of intrauterine contraceptives

414. Microscopically, a scrape from the uterine cavity, taken in a 36-year-old female against a background
of uterine bleeding, revealed a neoplasm which consisted of a large number of light epithelial cells of
Langhans and multinuclear symplasts, the number of figures of mitosis was increased. The stroma
was absent, the vascular cavities were lined with the above cells. Make a diagnosis of the uterine
tumour.
A. Endometrial polyp
B. Endometrial adenocarcinoma
C. Simple hydatidiform mole
D. Invasive hydatidiform mole
E. * Choriocarcinoma
415. Within the first stage of labour, the blood pressure in a female elevated (220/110 mm Hg), she
developed convulsions and a loss of consciousness. Her death resulted from a sharp impairment of
cerebral circulation. An autopsy revealed jaundice, an intracerebral haemorrhage, a pulmonary
oedema with microfocal haemorrhages into the pulmonary parenchyma, an enlarged yellow liver
with numerous haemorrhages, enlarged flaccid kidneys with a swollen cortical layer, where on
microscopic examination symmetrical necroses were found. Which of the diagnoses listed below was
the most probable?
A. Viral hepatitis
B. Haemorrhagic insult
C. Necrotic nephrosis
D. Haemorrhagic pneumonia
E. * Eclampsia
416. A 30-year-old woman had ectopic tubal pregnancy which finished with a location of a fetus in the
tubal cavity with bleeding. Call this pathology of pregnancy
A. Complete tubal abortion
B. Spontaneous abortion
C. Induced abortion
D. Criminal abortion
E. * Incomplete tubal abortion
417. The body of a young woman after delivery was taken for autopsy procedure. In autopsy, the
following features were found: enlarged parti-colored dimmed liver with areas of necrosis.
Microscopically the following signs were found in the liver: hemorrhages, thrombosis of the vessels,
proteinous and fatty degeneration of hepatocytes; necrosis of epithelium of kidney’s canals with
fibrinoid necrosis of vessels’ walls and hemorrhages in interstitial tissue; the hemorrhages also can be
seen in brain, heart, lungs and serous membranes. The main cause of death – hepatocellular
insufficiency. What determines the expressed hemodynamic disorders in the liver?
A. specific angioarchitectonics
B. high responsiveness of the liver parenchyma to hypoxia
C. high regenerative capacity of hepatocytes
D. mechanical pressure of the uterus of pregnant woman on the liver
E. * antigenic familiarity of basal membranes of the liver and placenta
418. A woman, 40 weeks of pregnancy, had a cesarian section. From the uterine cavity there were
delivered symmetrically formed twins with split heads while their bodies were divided. What is the
variant of twin deformity?
A. heteropagus
B. homopagus
C. blastopagus
D. all of the terms listed are synonyms
E. * diplopagus

419. A woman, 42 weeks of pregnancy, had a cesarian section. From the uterine cavity there were
delivered symmetrically formed twins with split heads while their bodies were divided. In what
period of development such an anomaly occured?
A. fetogenesis
B. early neonatal period
C. late neonatal period
D. embryogenesis
E. * blastogenesis
420. At the examination of the child in pediatric hospital clinic-morphologic triad was found: congenital
heart defect, deafness and cataracts. Choose the teratogenic factor that was most likely to cause such
changes:
A. X-ray examination in the third trimester of pregnancy
B. alcohol abuse
C. smoking
D. taking thalidomide
E. * rubella in the first trimester of pregnancy
421. A patient W., born in 1960, has defects of the development of the upper limbs - shoulders and
forearms are absent, arms attach directly to the body, are widespread and resemble a walrus flippers.
Choose the teratogenic factor that was most likely to cause such changes:
A. X-ray examination in the second trimester of pregnancy
B. abuse of surrogate alcohol
C. smoking cannabis
D. influenza in the second trimester of pregnancy
E. * taking thalidomide
422. At the ultrasound examination were found multiple congenital malformations of the fetus. Abortion
has been done on medical indications. At the pathoanatomical investigation: the lungs of the fetus are
reduced, the volume of each is approximately ? of the volume of the heart. Histologically structural
elements of the pulmonary tissue are visualized. How to characterize such defect of the respiratory
system?
A. ectopia of the lungs
B. agenesis of the lungs
C. neonatal respiratory distress syndrome
D. aplasia of the lungs
E. * hypoplasia of the lungs
423. At the pathoanatomical examination of the stillborn it was revealed that the esophagus is represented
by two isolated fragments, which are connected by thin cord of connective tissue. The proximal
fragment of the esophagus is combined with the trachea in the region of its bifurcation. What
complication is typical for such congenital defect?
A. hemorragia per diabrosin
B. squamous cell carcinoma of the esophagus
C. such a defect does not affect the health status
D. neonatal respiratory distress syndrome
E. * aspiration pneumonia
424. During the section of the deceased newborn there was found an aperture in the left dome of the
diaphragm, through which the left lobe of the liver and the loop of the small intestine shifted to the
chest cavity, squeezing the left lung. The right lung is compressed by mediastinal organs, the size of
both lungs is reduced. Set diagnosis:
A. situs inversus
B. congenital myotonia

C. birth trauma
D. gastroschisis
E. * false congenital diaphragmatic hernia
425. A patient was hospitalized with blunt abdominal trauma and signs of intra-abdominal bleeding. The
surgeon discovered a spleen rupture, and at its lower pole – extra spleen. In what period of
development such an anomaly occured?
A. blastogenesis
B. fetogenesis
C. early neonatal period
D. late neonatal period
E. * embryogenesis
426. A lung of a premature infant is presented on electronic photomicrography of biopsy material.
Collapse of the alveolar wall caused by the deficiency of surfactant was revealed. Dysfunction of
what cells of the alveolar wall caused it?
A. Alveolar macrophages
B. Secretory cells
C. Alveocytes type I
D. Fibroblasts
E. * Alveocytes type II
427. Autopsy of a fetus who died of intranatal asphyxia due to acute disturbance of utero-placental
circulation revealed small perivascular punctuates hemorrhages in the pia mater, under the
epicardium and pleura. What is the most probable mechanism of the vascular wall damage?
A. Spasm
B. Erosion
C. Edema
D. Rupture
E. * Increased permeability
428. The Respiratory Distress Syndrome often takes place in immature newborns. What is the most
probable cause of this syndrome?
A. Aspiration of amniotic fluid
B. Intrauterine asphyxia
C. Imperfection of nervous regulation of respiration
D. Intrauterine hypercapnia
E. * Immaturity of alveolar-parenchyma connected with deficiency of surfactant
429. Autopsy of the newborn showed jaundice of the skin, signs of the bilirubin encephalopathy in the
brain substance, bilirubin infarctions in the kidneys, enlarged liver and spleen. His mother is
Rh-negative. The child died on the third day after birth. What is your diagnosis?
A. Birth injury
B. Pneumonia in newborn
C. Respiratory distress syndrome of newborn
D. Edematous hemorrhagic syndrome
E. * Hemolytic disease of newborn
430. Microscopic examination of the lungs of a dead 2-day-old newborn showed: pinkish masses that line
the respiratory bronchioles, alveoli. These masses are largely made up of fibrinogen and fibrin,
admixed with cell debris chiefly from necrotic alveolar lining pneumocytes. Areas of dys- and
atelectasis are found out too. The deficiency of which substance plays a leading role in the
development of disease
A. vitamin K

B. fetal hemoglobin
C. immunoglobulins A and G
D. tyrosine
E. * surfactant
431. In autopsy of a 6-month-old infant it was found out: small cysts in pancreatic glands, signs of chronic
bronchitis with atelectasis, bronchiectasis, fatty changes and cholestasis in liver, coprostasis in
intestine. Described changes are typical for…:
A. Hemolytic disease of newborn
B. Pneumonia in newborn
C. Respiratory distress syndrome of newborn
D. Edimatous hemorrhagic syndrome
E. * Mucoviscidosis
432. In microscopic examination of a 2-day-old pre-term-newborn it was found out: diffuse edema and
numerous hemorrhages in the lungs, pulmonary capillaries overfull by blood. What is the probable
diagnosis?
A. Hemolytic disease of newborn
B. Birth injury
C. Pneumonia in newborn
D. Respiratory distress syndrome of newborn
E. * Edematous-hemorrhagic syndrome
433. In autopsy of still-born it was found out: teratoma in ovary, nevus on skin, rupture of tentorium
cerebelli, hypoplasia of thymus, polydactilia. What is the cause of death?
A. Hypoxia
B. Anoxia
C. Birth injury
D. Pneumonia
E. * Rupture of cerebellar palatinum
434. In autopsy of still-born it was found out: teratoma in ovary, nevus on skin, rupture of tentorium
cerebelli, hypoplasia of thymus, polydactilia. What is the variant of perinatal pathology?
A. Hemolytic disease of newborn
B. Pneumonia in newborn
C. Respiratory distress syndrome of newborn
D. Edimatous hemorrhagic syndrome
E. * Birth injury
435. On autopsy of a 71-year-old male, who had worked as a stone grinder for 14 years and died from
cardiopulmonary insufficiency, his lungs were enlarged and dense, they revealed numerous miliary
and larger grey or grey-black nodules rounded, oval or irregular in shape and dense in consistency. A
microscopic examination revealed some peribronchial and perivascular vegetation of the connective
tissue, scleroid interalveolar septa, catarrhal-desquamative bronchitis, bronchoectases, a diffuse
emphysema. The nodules were represented by clusters of coniophages with collagen fibres among
them, some nodules had a fibrous structure. What disease is characterized by the above changes in
the lungs?
A. Anthracosis
B. Bronchiectatic disease
C. Chronic bronchitis
D. Bronchial asthma
E. * Silicosis

436. An autopsy of a 59-year-old male, who had worked in mine for 19 years, revealed the following
changes in the lungs: chronic deforming bronchitis, chronic bronchopneumonia, an expressed
emphysema, sclerotic changes in the vascular walls, irregular caverns with crumbling black walls and
some black contents. The lymph nodes were enlarged and black. Histologically, the lungs revealed
perivascular and peribronchial sclerosis, sclerosis of interalveolar septa and a large number of
macrophages whose cytoplasm contained some grey-black or slate pigment. Name the most probable
pathology.
A. Siderosis
B. Silicosis
C. Aluminosis
D. Asbestosis
E. * Anthracosis
437. An autopsy of a young male, who served aboard a nuclear submarine and during his life-time
revealed severe anaemia, leukopenia, thrombocytopenia and an expressed haemorrhagic syndrome,
revealed the following changes: panmyelophthisis, decomposition of lymphocytes in the lymph
nodes, spleen and lymphatic apparatus of the gastrointestinal tract, as well as haemorrhages in the
mucosal membranes of the stomach, intestines and adrenal glands. What disease developed in this
case?
A. Acute hypoplastic anaemia
B. Vibration disease
C. Acute leukaemia
D. Decompression sickness
E. * Acute radiation sickness
438. An autopsy of a 56-year-old male, who had worked as a driller of boreholes during his life-time,
revealed dry gangrene of the toes and feet, an atrophy of the forearm, deltoid and rhomboid muscles.
Microscopically, the muscles were characterized by an atrophy, the distal epiphyses of the radial and
ulnar bones had foci of osteoporosis and sclerosis, the carpal bones had numerous cysts, marks of
pathological fractures and deformities; in the arteries there was a sharp narrowing of their lumens up
to the absolute obliteration. What disease developed in this case?
A. Decompression sickness
B. Deforming arthrosis
C. Myopathy
D. Obliterating endarteritis
E. * Vibration disease
439. An autopsy of a 43-year-old male, who had worked as a diver during his life-time, revealed in the
long bones of his lower extremities some foci of osteoporosis surrounded by an area of sclerosis, as
well as foci of aseptic necrosis of the bone tissue and osteomyelitis. The joints were characterized by
deformity, an atrophy of the cartilage, phenomena of arthritis. What disease developed in this man?
A. Obliterating endarteritis
B. Deforming arthrosis
C. Myopathy
D. Vibration disease
E. * Decompression sickness
440. An autopsy of a female, who had worked as a radiologist for 20 years and died from double
pneumonia, revealed numerous haemorrhages of various remoteness on the skin, mucous and serous
membranes, general haemosiderosis. The marrow of the sternum was yellow, the liver and kidneys
were yellow-clay and flaccid, the myocardium was flaccid, from the side of the endocardium it had
yellow-white lines. Microscopically, the marrow was characterized by panmyelophthisis, there was
fatty degeneration of the parenchymatous organs and purulent pneumonia in the lungs. What disease
developed in this case?

A. Hypoplastic anaemia
B. Aplastic anaemia
C. Acute radiation sickness
D. AIDS
E. * Chronic radiation sickness
Назва наукового напрямку (модуля): Семестр: 6
Патоморфологія
Опис:
Крок 1
Перелік питань:
1. 48 hours after tuberculine test (Mantoux test) a child had a papule 10 mm in diameter on the spot of
tuberculine injection. What hypersensitivity mechani-sm underlies these changes?

A. * Cellular cytotoxicity

B. Anaphylaxy

C. Antibody-dependent cytotoxicity

D. Immunocomplex cytotoxicity

E. Granulomatosis

2. A 17 year old boy fell seriously ill, the body temperature rose up to 38, 5oC, there appeared cough,
rhinitis, lacrimati-on, nasal discharges. What inflammation is it?

A. * Catarrhal

B. Serous

C. Fibrinous

D. Purulent

E. Hemorrhagic

3. A patient with skin mycosis has di-sorder of cellular immunity. The most typical characteristic of it is
reduction of the following index:

A. * T-lymphocytes

B. Immunoglobulin G

C. Immunoglobulin E

D. B-lymphocytes

E. Plasmocytes

4. 14 days after quinsy a 15-year-old chi-ld presented with morning facial swelling, high blood
pressure, "meat slops"urine. Immunohistological study of a renal biopsy sample revealed deposition
of immune complexes on the basement membranes of the capillaries and in the glomerular
mesangium. What disease developed in the patient?

A. * Acute glomerulonephritis
B. Acute interstitial nephritis

C. Lipoid nephrosis

D. Acute pyelonephritis

E. Necrotizing nephrosis

5. 2 days after labour a woman developed shock along with DIC syndrome that caused her death.
Autopsy revealed purulent endomyometritis, regi-onal purulent lymphangitis, lymphadeni-tis and
purulent thrombophlebitis. There were also dystrophic alterations and interstitial inflammation of
parenchymal organs. What is the most likely diagnosis?

A. * Septicemia

B. Syphilis

C. Tuberculosis of genital organs

D. Chorioadenoma destruens

E. Hydatid mole

6. 2 days after labour a woman developed shock along with DIC syndrome that caused her death.
Autopsy revealed purulent endomyometritis, regional purulent lymphangitis, lymphadenitis and
purulent thrombophlebitis. There were also dystrophic alterations and interstitial inflammation of
parenchymal organs. What is the most likely diagnosis?

A. * Septicemia

B. Syphilis

C. Tuberculosis of genital organs

D. Chorioadenoma destruens

E. Hydatid mole

7. 2 hours after a skeletal extension was performed to a 27 year old patient with multiple traumas
(closed injury of chest, closed fracture of right thigh) his condition abruptly became worse and the
patient died from acute cardiopulmonary decompensation. Histological examination of pulmonary
and cerebral vessels stained with Sudan III revealed orange drops occluding the vessel lumen. What
complication of polytrauma was developed?

A. * Fat embolism

B. Thromboembolism

C. Gaseous embolism

D. Microbal embolism
E. Air embolism

8. 40-year-old patient with the progressing staphylococcal purulent peri-odontitis developed purulent
inflammati-on of bone marrow spaces of the alveolar process, and then of the body of mandi-ble.
Microscopy revealed thinning of bone trabeculae, foci of necrosis, bone sequesters surrounded by the
connecti-ve tissue capsule. What is the most likely diagnosis?

A. * Chronic osteomyelitis

B. Acute osteomyelitis

C. Parodontome

D. Chronic fibrous periostitis

E. Purulent abscess

9. 6 months after delivery a woman had uterine bleeding. Gynecological exami-nation revealed in the
uterine cavity a dark-red tissue with multiple cavities that resembled of "sponge". Microscopic
examination of the tumour revealed some atypic light epithelial Langhans cells and giant cells of
cyncytiotrophoblast in blood lacunas. What tumour is it?

A. * Chorioepithelioma

B. Squamous cell nonkeratinous carci-noma

C. Adenocarcinoma

D. Fibromyoma

E. Vesicular mole

10. A 10 year old child had the mantoux tuberculin test administered. 48 hours later a papule up to 8 mm
in diameter appeared on the site of the injection. What type of hypersensitivity reaction developed
after the tuberculin injection?

A. * Type IV hypersensitivity reaction

B. Arthus phenomenon

C. Seroreaction

D. Atopic reaction

E. Type II hypersensitivity reaction

11. A 10-year-old child has painful swallowing, neck edema, temperature rise up to 39, 0oC, the whole
body is covered with bright-red petechial rash. Back of the throat and tonsils are hyperemic, the
tongue is crimsoncolored. Tonsillar surface is covered with isolated grayish-colored necrosis nidi.
What disease is it?

A. * Scarlet fever
B. Meningococcal nasopharyngitis

C. Diphtheria

D. Influenza

E. Measles

12. A 10-year-old child was found to have a congenital hypoplasia of the left kidney. Ultrasound
examination revealed that the right kidney was markedly enlarged and had regular shape. No
functional disorders were revealed. Specify the process that developed in the right kidney:

A. * Vicarious hypertrophy

B. Working hypertrophy

C. Hypertrophic growth

D. Pseudohypertrophy

E. Metaplasia

13. A 12-year-old child developed nephritic syndrome (proteinuria, hematuria, cylindruria) 2 weeks after
a case of tonsillitis, which is a sign of affected glomerular basement membrane in the kidneys. What
mechanism is the most likely to cause the basement membrane damage?

A. * Immune complex

B. Granulomatous

C. Antibody-mediated

D. Reaginic

E. Cytotoxic

14. A 12-year-old patient has white non-pigmented spots on the skin. The spots appeared after the patient
became 10 years old, and they constantly grow. This spots appeared due to the lack of the following
skin cells:

A. * Melanocytes

B. Adipocytes

C. Fibrocytes

D. Plasmocytes

E. Labrocytes
15. A 16-year-old female patient has fainted after quickly changing her body position from horizontal to
vertical one. Which process from the ones listed below has caused the loss of consciousness in the
first place?

A. * Decreasing venous return

B. Increasing venous return

C. Increasing central venous pressure

D. Decreasing oncotic pressure of blood plasma

E. Increasing arterial pressure

16. A 17 year old boy fell seriously ill, the body temperature rose up to 38,5oC there appeared cough,
rhinitis, lacrimation, nasal discharges. What inflammation is it?

A. * Catarrhal

B. Purulent

C. Fibrinous

D. Hemorrhagic

E. Serous

17. A 19-year-old female patient has had low haemoglobin rate of 90-95 g/l si-nce childhood. Blood
count results obtai-ned after hospitalisation are as follows: erythrocytes - 3, 2 · 1012/l, Hb- 85 g/l,
colour index - 0,78; leukocytes - 5, 6 · 109/l, platelets - 210 · 109/l. Smear examinati-on revealed
anisocytosis, poikilocytosis and target cells. Reticulocyte rate is 6%. Iron therapy was ineffective.
What blood pathology corresponds with the described clinical presentations?

A. * Thalassemia

B. Enzymopathy

C. Membranopathy

D. Sickle-cell anemia

E. Favism

18. A 2 year old child had acute respiratory viral infection and died from cardiopulmonary
decompensati-on. Autopsy revealed that his right lung was hyperemic; in the 2nd, 6th and 10th
segments and on the incision there were airless yellowish foci of irregular form, from several mm up
to 1 cm large. Microscopical examination revealed exudate consisting mainly of neutrophils in the
given areas of pulmonary tissue in the alveoles,bronchioles and bronchial tubes. What is the most
probable diagnosis?

A. * Focal pneumonia
B. Interstitial pneumonia

C. Croupous pneumonia

D. Acute bronchitis

E. Pulmonary abscess

19. A 20 year old patient complains of morbid thirst and huperdiuresis (up to 10 l daily). Glucose
concentration in blood is normal but it is absent in urine. The pati-ent has been diagnosed with
diabetes insi-pidus. What hormonal drug is the most appropriate for management of this di-sorder?

A. * Vasopressin

B. Cortisol

C. Thyroxin

D. Oxytocin

E. Insulin

20. A 20 year old patient died from intoxication 8 days after artificial illegal abortion performed in her
14-15th week of pregnancy. Autopsy of the corpse revealed yellowish colour of eye sclera and of
skin, necrotic suppurative endometritis, multiple pulmonary abscesses, spleen hyperplasia with a big
number of neutrophils in its sinuses. What complication after abortion was developed?

A. * Septicopyemia

B. Chroniosepsis

C. Hemorrhagic shock

D. Septicemia

E. Viral hepatitis type A

21. A 22 y.o. woman has enlarged lymph nodes. Histologically: a lymph node contains lymphocytes,
histiocytes, reti-cular cells, small and big Hodgkin’s cells, multinucleated Sternberg cells, isolated
foci of caseous necrosis. What disease are these changes typical for?

A. * Lymphogranulomatosis

B. Lymphosarcoma

C. Chronic leukosis

D. Acute leukosis

E. Lung cancer metastasis


22. A 22 year old patient from the West Ukraine complains of laboured nasal breathing. Morphological
exami-nation of biopsy material of nasal mucous membrane revealed lymphoid, epitheli-oid, plasma
cells as well as Mikulicz’s cells. What is the most probable diagnosis?

A. * Rhinoscleroma

B. Glanders

C. Tuberculosis

D. Leprosy

E. Syphilis

23. A 22-year-old patient was admitted to the hospital with complaints of heavy nasal breathing. During
the examination of her nasal cavity the doctors found thi-ckened mucous membrane, a lot of mucus
and nodular infiltrates without erosions in the nose.The nasal rhinoscleroma was diagnosed. The
biopsy was taken. What typical morphological changes may be found?

A. * Granulomas with Mikulicz’s cells

B. Granulomas with Virchow’s cells

C. Granulomas with Langhan’s cells

D. Granulomas with foreign body cells

E. Interstitial inflammation

24. A 23 year old man has perforation of hard palate. In the area of this perforati-on there was a compact
well-defined formation. Microscopic examination of the resected formation revealed a large focus of
caseous necrosis surrounded by granulation tissue with endovasculitis, cellular infiltration composed
of lymphocytes, epithelioid cells (mainly plasmocytes). What is the most probable diagnosis?

A. * Syphilis

B. Tuberculosis

C. Scleroma

D. Sarcoma

E. Leprosy

25. A 25-year-old man has spent a long ti-me in the sun under high air humidity. As a result of it his
body temperature rose up to 39oC. What pathological process is it?

A. * Hyperthermia

B. Infectious fever

C. Hypothermia
D. Noninfectious fever

E. Burn disease

26. A 28 year old patient had high arterial pressure, hematuria and facial edemat A. In spite of treatment
renal insufficiency was progressing. 6 months later the patient died from uremi A. Microscopic
examination of his kidneys and their glomerules revealed proliferation of capsule nephrothelium and
of podocytes with "demilune" formation, sclerosis and hyalinosis of glomerules. What disease
corresponds with the described picture?

A. * Subacute glomerulonephritis

B. Acute pyelonephritis

C. Chronic glomerulonephritis

D. Nephrotic syndrome

E. Acute glomerulonephritis

27. A 28 year old woman has been di-agnosed with extrauterine pregnancy complicated by tha fallopian
tube rupture. The blood is most likely to penetrate the following peritoneal space:

A. * Rectouterine

B. Vesicouterine

C. Right mesenteric sinus

D. Left mesenteric sinus

E. Intersigmoid sinus

28. A 30 year old man had been suffering from acute respiratory disease and died from cardiopulmonary
decompensation. Autopsy revealed fibrinous-haemorrhagic inflammation in the mucous membrane
of larynx and trachea, destructive panbronchitis, enlarged lungs that look black due to the multiple
abcesses, haemorrhages, necrosis. What is the most probable postmortem diagnosis?

A. * Influenza

B. Parainfluenza

C. Respiratory syncytial infection

D. Measles

E. Adenoviral infection

29. A 30 year old woman has applied a lipstick with a fluorescent substance for a long tim E. Then she
got a limited erythema and slight peeling on her lip border, later there appeared trANSWERsal striae
and cracks. Special methods of microscopic examination of the affected area helped to reveal
sensibilized lymphocytes and macrophages in the connective tissue; cytolysis. What type of
immunological hypersensitivity was developed?

A. * IV type (cellular cytotoxicity)

B. Granulomatosis

C. II type (antibody cytotoxicity)

D. I type (reaginic)

E. III type (immune complex cytotoxicity)

30. A 30-year-old patient has undergone keratoplasty in the transplantation center, cornea has been taken
fron a donor, who died in a road accident. What kind of transplantation was performed?

A. * Allotransplantation

B. Autotransplantation

C. Xenotransplantation

D. Explantation

E. Heterotransplantation

31. A 33 year old man died from uraemia. Autopsy revealed enlarged kidneys wei-ghing 500,0 each and
consisting of multi-ple cavities 0,5-2 cm in diameter. The cavi-ties were full of light-yellow
transparent liquid. Renal pelvis and ureters had no pecularities. What renal disease caused uraemia?

A. * Bilateral polycystic renal disease

B. Chronic pyelonephritis

C. Renal tumour

D. Renal tuberculosis

E. Rapidly progressing glomerulonephritis

32. A 35-year-old female patient has undergone biopsy of the breast nodules. Histological examination
has revealed enhanced proliferation of the small duct and acini epithelial cells, accompanied by the
formation of glandular structures of various shapes and sizes, which were located in the fi-brous
stroma. What is the most likely diagnosis?

A. * Fibroadenoma

B. Adenocarcinoma

C. Cystic breast

D. Invasive ductal carcinoma


E. Mastitis

33. A 37-year-old female patient complains of headache, vertigo, troubled sleep, numbness of limbs. For
the last 6 years she has been working at the gas-discharge lamp-producing factory in the
lead-processing shop. Blood test findings: low hemoglobin and RBC level, serum iron concentrati-on
exceeds the norm by several times. Specify the type of anemia:

A. * Iron refractory anemia

B. Iron-deficiency anemia

C. Minkowsky-Shauffard disease

D. Hypoplastic anemia

E. Metaplastic anemia

34. A 37-year-old male patient developed pseudoarthrosis after a closed fracture of the femur. Specify
the type of regeneration in the patient:

A. * Pathological hypo-regeneration

B. Pathological hyper-regeneration

C. Reparative

D. Physiological

E. -

35. A 37-year-old man, who was worki- ng in a caisson, after being lifted to the surface suddenly
developed signs of acute cerebral circulation disturbance and loss of consciousness. Several days
later he died. On autopsy in the left cerebral hemisphere there was detected a gray soft irregular focus
5?6?3,5 cm in size. What process had occurred in the brain?

A. * Ischemic stroke

B. Hemorrhagic infarction

C. Abscess

D. Cyst

E. Tumor

36. A 38 year old man died all of a sudden. Autopsy revealed myocardial infarction in the posterior wall
of the left ventri-cle. What are the most likely alterations in myocardiocyte structure that can be
revealed microscopically in the infarcti-on focus?

A. * Karyolysis

B. Adipose degeneration
C. Carbohydrate degeneration

D. Calcification

E. Protein degeneration

37. A 38 year old patient with full-blown jaundice, small cutaneous hemorrhages, general weakness and
loss of appetite underwent puncture biopsy of liver. Histological examination revealed disseminated
dystrophy, hepatocyte necrosis, Councilman's bodies. Lobule periphery has signs of significant
infiltration by lymphocytes, there are also individual multinuclear hepatocytes. What is the most
probable diagnosis?

A. * Acute viral hepatitis

B. Chronic hepatitis

C. Toxic degeneration of liver

D. Acute alcoholic hepatitis

E. Miliary hepatic cirrhosis

38. A 38-year-old male patient has been ill with systemic lupus erythematosus for three years. He was
diagnosed with diffuse renal affection accompanied by massi-ve edemata and expressive proteinuria.
What is the most likely cause of protei-nuria development?

A. * Autoimmune renal affection

B. Aseptic renal affection

C. Ischemic renal affection

D. Urinary bladder inflammation

E. Urinary tracts inflammation

39. A 38-year-old man died in the attempt of lifting weight. He had collaptoid state. Autopsy revealed an
extensive aneurism rupture of thoracic aorta. He suffered from visceral syphilis during his lifeti-me.
What pathological process caused weakness of aortic wall, its dilatation and rupture?

A. * Vanishing of elastic fibers

B. Vanishing of collagen fibers

C. Muscle layer atrophy

D. Intima changes by shagreen leather type

E. Vascularization

40. A 39 y.o. woman went through an operation in course of which surgeons removed her uterine tube
that was enlarged and a part of an ovary with a big cyst. Histological examination of a tube wall
revealed decidual cells, chorion vi-lli. What was the most probable diagnosis made after examination
of the uterine tube?

A. * Tubal pregnancy

B. Placental polyp

C. Choriocarcinoma

D. Papyraceous fetus

E. Lithopedion

41. A 39 year old man who had been operated for the stomach ulcer died 7 days after the surgery.
Autopsy revealed that peritoneal leaves were dull, plephoric, covered with massive yellow-greenish
fi-lms, the peritoneal cavity contained for about 300 ml of thick yellow-greenish liquid. What
pathologic process was revealed in the peritoneal cavity?

A. * Fibrinous suppurative peritonitis

B. Serous peritonitis

C. Fibrinous serous peritonitis

D. Peritoneal commissures

E. Fibrinous haemorrhagic peritonitis

42. A 39-year-old woman has madescence in the region of mammilla, a small ulcer with inflammatory
hyperemia and cutaneous edema. Histologic exami-nation of tissue sampling from this area revealed
in the malpighian layer of thi-ckened epidermis atypical cells with li-ght and optically empty
cytoplasm, with no intracellular bridges. Such cells were also found in the orifice of big mammal
gland ducts. What is the most probable diagnosis?

A. * Paget’s disease

B. Intraductal cancer

C. Basal cell carcinoma

D. Epidermoid cancer

E. Melanocarcinoma

43. A 3-year-old child has conti-nuous fever, lymph nodes are enlarged, the amount of lymphocytes in
blood is significantly increased. Enzyme-linked immunosorbent assay (ELISA) revealed antigen of
Epstein-Barr virus. What diagnosis can be made based on the information given above?

A. * Infectious mononucleosis

B. Burkitt’s lymphoma
C. Herpetic lymphadenopathy

D. Generalized infection caused by herpes-zoster

E. Cytomegalovirus infection

44. A 3-year-old child with meni-ngeal symptoms died. Postmortem macroscopy of the pia matter
revealed miliary nodules which were mi-croscopically represented by a focus of caseous necrosis
with masses of epi-thelioid and lymphoid cells with some crescent-shaped large cells inbetween
having peripheral nuclei. Specify the type of meningitis in the child:

A. * Tuberculous

B. Syphilitic

C. Brucellar

D. Grippal

E. Meningococcal

45. A 3-year-old child with meningeal symptoms died. Postmortem macroscopy of the pia matter
revealed miliary nodules which were microscopically represented by a focus of caseous necrosis with
masses of epithelioid and lymphoid cells with large cells containing crescent-shaped peripheral
nuclei situated between them. Specify the type of meningitis in the child:

A. * Tuberculous

B. Syphilitic

C. Brucellar

D. Grippal

E. Meningococcal

46. A 3-year-old girl with mental retardati- on has been diagnosed with sphingomyelin lipidosis
(Niemann-Pick disease). In this condition synthesis of the following substance is disrupted:

A. * Sphingomyelinase

B. Glycosyltransferase

C. Sphingosine

D. Ceramides

E. Gangliosides

47. A 4 year old child complained of pain during deglutition, indisposition. Objectively: palatine arches
and tonsils are moderately edematic and hyperemic, there are greyish-white films up to 1 mm thick
closely adhering to the subjacent ti-ssues. What pathological process are these changes typical for?
A. * Inflammation

B. Dystrophy

C. Necrosis

D. Metaplasia

E. Organization

48. A 40-year-old female patient has undergone thyroidectomy. Histological study of thyroid gland
found the folli-cles to be of different size and contain foamy colloid, follicle epithelium is hi-gh and
forms papillae, there is focal lymphocytic infiltration in stroma. Di-agnose the thyroid gland disease:

A. * Basedow’s disease

B. Hashimoto’s thyroiditis

C. Riedel’s thyroiditis

D. De Quervain’s disease

E. Nodular goiter

49. A 40-year-old man developed skin redness and swelling in the neck area, where eventually a small
abscess appeared. On section the focus is dense and yellow- green colored. In the purulent masses
there are white granules. Histologically there are fungal druses, plasma and xanthome cells, and
macrophages detected. Specify the most correct etiological name of this pathological process:

A. * Actinomycosis

B. Furuncle

C. Carbuncle

D. Syphilis

E. Leprosy

50. A 40-year-old patient with the progressing staphylococcal purulent periodontitis developed purulent
inflammation of bone marrow spaces of the alveolar process, and then of the body of mandible.
Microscopy revealed thinning of bone trabeculae, foci of necrosis, bone sequesters surrounded by the
connective tissue capsule. What is the most likely di-agnosis?

A. * Chronic osteomyelitis

B. Acute osteomyelitis

C. Parodontome

D. Chronic fibrous periostitis


E. Purulent abscess

51. A 40-year-old prisoner died of tuberculosis in the corrective labor camp. Autopsy of the body
revealed deformation and diminishing of both lung apices; in the both upper lobes there are multiple
cavi- ties with dense walls 2-3 mm thick; in the lower lung lobes there are disseminated foci of
caseous necrosis varying from 5 mm to 2 cm in diameter. Diagnose the type of tuberculosis:

A. * Secondary fibro-cavitary tuberculosis

B. Secondary fibrous-focal tuberculosis

C. Hematogenous macrofocal pulmonary tuberculosis

D. Primary tuberculosis, primary affect development

E. Secondary cirrhotic tuberculosis

52. A 40-year-old woman has had a feeli-ng of abdominal discomfort for the past 8 months. On pelvic
examination, there is the right adnexal mass. Abdominal CT scan demonstrates a 7 cm cystic mass
involving the right ovary with small areas of calcification. The uterus is normal in size. The right
fallopian tube and ovary have been removed surgically. Grossly, the mass on sectioning is filled with
abundant hair and sebum. Microscopi-cally, the mass has glandular spaces li-ned by columnar
epithelium, squamous epithelium with hair follicles, cartilage, and dense connective tissue. What
type of tumour is it?

A. * Teratoma

B. Squamous cell carcinoma of ovary

C. Melanoma

D. Sarcoma of ovary

E. Metastase of cervical carcinoma

53. A 40-year-old woman has undergone thyroidectomy. Histological study of thyroid gland found the
follicles to be of different size and contain foamy colloid, follicle epi- thelium is high and forms
papillae, there is focal lymphocytic infiltration in the stroma. Diagnose the thyroid gland disease:

A. * Basedow’s disease

B. Hashimoto’s thyroiditis

C. Riedel’s thyroiditis

D. De Quervain’s disease

E. Nodular goiter

54. A 40-year-old woman was diagnosed with glomerulonephritis based on her clinical symptoms and
the results of urine analysis. Anamnesis states chronic tonsillitis. What microorganisms are the most
likely cause for the kidney damage in this case?
A. * Streptococci

B. Staphylococci

C. Escherichia

D. Mycoplasma

E. Meningococci

55. A 41-year-old male patient has a history of recurrent attacks of heartbeats (paroxysms), profuse
sweating, headaches. Examination revealed hypertension, hyperglycemia, increased basal metabolic
rate, and tachycardia. These clinical presentati-ons are typical for the following adrenal pathology:

A. * Hyperfunction of the medulla

B. Hypofunction of the medulla

C. Hyperfunction of the adrenal cortex

D. Hypofunction of the adrenal cortex

E. Primary aldosteronism

56. A 41-year-old man has a hi- story of recurrent attacks of heartbeats (paroxysms), profuse sweating,
headaches. Examination revealed hypertension, hyperglycemia, increased basal metabolic rate, and
tachycardia. These clinical presentations are typical of the following adrenal pathology:

A. * Hyperfunction of the medulla

B. Hypofunction of the medulla

C. Hyperfunction of the adrenal cortex

D. Hypofunction of the adrenal cortex

E. Primary aldosteronism

57. A 42 year old patient complains of pain in the epigastral area, vomiti-ng; vomit masses have the
colour of "coffee-grounds", the patient has also melena. Anamnesis records gastric ulcer. Blood
formula: erythrocytes - 2, 8 · 1012/l, leukocytes - 8 · 109/l, Hb- 90 g/l. What complication is it?

A. * Haemorrhage

B. Penetration

C. Perforation

D. Canceration

E. Pyloric stenosis
58. A 43-year-old patient suffers from acute pancreatitis with disrupted common bile duct patency. What
condition can develop in this case?

A. * Mechanical jaundice

B. Hemolytic jaundice

C. Hepatocellular jaundice

D. Hepatic coma

E. Portal hypertension

59. A 44 year old woman complains of general weakness, heart pain, significant increase of body weight.
Objectively: moon face, hirsutism, AP is 165/100 mm Hg, height - 164 cm, weight - 103 kg; the fat
is mostly accumulated on her neck, thoracic girdle, belly. What is the main pathogenetic mechanism
of obesity?

A. * Increased production of glucocorticoids

B. Reduced production of thyroid hormones

C. Increased insulin production

D. Reduced glucagon production

E. Increased mineralocorticoid production

60. A 45 y.o. patient consulted a doctor about plaque-shaped formation on his neck. Histological
examination of biopsy skin material revealed tumourous cells of round and oval form with thin ring
of basophilic cytoplasma that resemble of cells of basal epidermal layer. What tumour is it?

A. * Basalioma

B. Epidermal cancer

C. Hydradenoma

D. Trichoepithelioma

E. Syringoadenoma

61. A 45 year old male died from di-sseminated tuberculosis. On autopsy the symptoms of tuberculosis
were confirmed by both microscopical and histological analyses. All the affected organs had
epithelioid cell granulomas with caseous necrosis in the centre. What kind of hypersensitivity
reaction underlies the process of granuloma development?

A. * Delayed

B. Antibody-dependent cytotoxicity

C. Complement-dependent cytotoxicity
D. Anaphylactic

E. Immune complex

62. A 45 year old man consulted a doctor about a plaque-like formation on his neck. Histological
examination of a skin bioptate revealed clusters of round and oval tumour cells with a narrow border
of basophilic cytoplasm resembling of cells of basal epidermal layer. What tumour is it?

A. * Basal cell carcinoma

B. Syringoadenoma

C. Trichoepithelioma

D. Epidermal cancer

E. Hydroadenoma

63. A 45-year-old woman has breast cancer. Her left arm has symptoms of lymphatic system
insufficiency - limb edema, lymph node enlargement. What form of lymphatic circulation
insufficiency is it?

A. * Mechanic insufficiency

B. Dynamic insufficiency

C. Resorption insufficiency

D. Combined insufficiency

E. -

64. A 46 year old patient who had been suffering from tuberculosis for 6 years di-ed from massive
pulmonary haemorrhage. Autopsy revealed different-sixed foci of sclerosis and caseous necrosis in
lungs, in the upper part of the right lung there was a cavity 5 cm in diameter with dense grey walls,
the cavity contained liquid blood and blood clots. What type of tuberculosis is it?

A. * Fibrocavernous

B. Acute cavernous

C. Infiltrative

D. Fibrous focal

E. Acute focal

65. A 46 year-old man complains of di-fficult nose breathing. Mikulich cells, storage of epithelioid cells,
plasmocytes, lymphocytes, hyaline balls are discovered in the biopsy material of the nose
thi-ckening. What is the most likely di-agnosis?
A. * Scleroma

B. Virus rhinitis

C. Allergic rhinitis

D. Rhinovirus infection

E. Meningococcal nasopharyngitis

66. A 46-year-old man complains of di-fficulties with nasal breathing. Mikulicz’s cells, accumulation of
epithelioid cells, plasmocytes, lymphocytes, hyaline balls were discovered in the biopsy material of
the thickened nasal mucosa. What is the most likely diagnosis?

A. * Scleroma

B. Virus rhinitis

C. Allergic rhinitis

D. Rhinovirus infection

E. Meningococcal nasopharyngitis

67. A 46-year-old patient suffering from the diffuse toxic goiter underwent resecti-on of the thyroid
gland. After the surgery the patient presents with appetite loss, dyspepsia, increased neuromuscular
excitement. The body weight remained unchanged. Body temperature is normal. Which of the
following has caused such a condition in this patient?

A. * Reduced production of parathormone

B. Increased production of thyroxin

C. Increased production of calcitonin

D. Increased production of thyroliberin

E. Reduced production of thyroxin

68. A 48 y.o. patient was admitted to the hospital with complaints about weakness, irritability, sleep
disturbance. Objectively: skin and scleras are yellow. In blood: conjugated bilirubin, cholalemia.
Feces are acholic. Urine is of dark colour (bi-lirubin). What jaundice is it?

A. * Mechanic

B. Hemolytic

C. Parenchymatous

D. Gilbert’s syndrome

E. Crigler-Najjar syndrome
69. A 49 year old woman spent a lot of time standing. As a result of it she got leg edema. What is the
most likely cause of the edema?

A. * Increase in hydrostatic pressure of blood in veins

B. Decrease in hydrostatic pressure of blood in veins

C. Decrease in hydrostatic pressure of blood in arteries

D. Increase in oncotic pressure of blood plasma

E. Increase in systemic arterial pressure

70. A 49-year-old patient with croupous pneumonia died from pneumococcal septicemia. Autopsy
revealed up to 700 ml of turbid greenish-yellow foul-smelling liquid in the left pleural cavity. The
pleural leaflets were dull and plethoric. What form of pleural inflammation is it?

A. * Empyema

B. Chronic abscess

C. Acute abscess

D. Phlegmon

E. Fibrinous inflammation

71. A 50 year old patient has been taki-ng treatment thrice for the last 6 months because of fractures
caused by domestic accidents. Microscopical examination of bony tissue revealed foci of lacunar
resolution, giant-cell granulomas in the tumour-like formations, cysts. Bony tissue was substituted by
fibrous connective ti-ssue. Examination revealed also adenoma of parathyroid gland and
hypercalcemia. What is the most probable diagnosis?

A. * Parathyroid osteodystrophy

B. Myelomatosis

C. Osteomyelitis

D. Osteopetrosis

E. Paget’s disease

72. A 50 year old patient suffers from essential hypertension. After a physical stress he experienced
muscle weakness, breathlessness, cyanosis of lips, skin and face. Respiration was accompanied by
distinctly heard bubbling rales. What mechanism underlies the development of this syndrome?

A. * Acute left-ventricular failure

B. Chronic right-ventricular failure

C. Chronic left-ventricular failure


D. Collapse

E. Cardiac tamponade

73. A 50 year old patient underwent resection of tumour of large intestine wall. Microscopically it
presents itself as fascicles of divergent collagen fibers of different thickness and form and some
monomorphous fusiform cells that are irregularly distributed among the fibers. Cellular atypia is not
evident. What tumour is it?

A. * Hard fibroma

B. Fibromyoma

C. Soft fibroma

D. Desmoma

E. Fibrosarcoma

74. A 50-year-old man has felt vague abdominal discomfort within past 4 months. Physical examination
revealed no lymphadenopathy, and no abdomi-nal masses or organomegaly at palpation. Bowel
sounds are heard. An abdominal CT scan shows a 20 cm retroperitoneal soft tissue mass obscuring
the left psoas muscle. A stool specimen tested for occult blood is negative. Which of the following
neoplasms is this man most likely to have?

A. * Lipoma

B. Melanoma

C. Hamartoma

D. Adenocarcinoma

E. Lymphoma

75. A 53-year-old male patient complains of acute pain in the right hypochondrium. Objective
examinati-on revealed scleral icterus. Laboratory tests revealed increased ALT activity, and
stercobilin was not detected in the stool. What disease is characterized by these symptoms?

A. * Cholelithiasis

B. Hemolytic jaundice

C. Hepatitis

D. Chronic colitis

E. Chronic gastritis

76. A 53-year-old man suffering from di-abetes mellitus has developed a painful conical induration,
bluish-red with yellow center, on the skin of his neck. Such changes are characteristic of:
A. * Furuncle

B. Abscess

C. Carbuncle

D. Phlegmon

E. Empyema

77. A 54-year-old female was brought to the casualty department after a car acci-dent. A traumatologist
diagnosed her with multiple fractures of the lower extremiti-es. What kind of embolism is most likely
to develop in this case?

A. * Adipose

B. Tissue

C. Thromboembolism

D. Gaseous

E. Air

78. A 54-year-old female was brought to the casualty department after a car accident. A traumatologist
diagnosed her with multiple fractures of the lower extremities. What kind of embolism is most likely
to develop in this case?

A. * Fat

B. Tissue

C. Thromboembolism

D. Gaseous

E. Air

79. A 54-year-old woman was brought to a casualty department after a car accident. A traumatologist
diagnosed her with multiple fractures of the lower extremities. What kind of embolism is most likely
to develop in this case?

A. * Adipose

B. Tissue

C. Thromboembolism

D. Gaseous

E. Air
80. A 54-year-old woman was brought to the emergency department after a car acci- dent. A
traumatologist diagnosed her with multiple fractures of the lower extremities. What kind of
embolism is the most likely to develop in this case?

A. * Adipose

B. Tissue

C. Thromboembolism

D. Gaseous

E. Air

81. A 55-year-old male patient was hospi-talised to a surgical clinic for suspected septicemia. What
material should be taken for analysis?

A. * Blood, sugar broth

B. Liquor, serum agar

C. Urine, beef-extract broth

D. Pus, yolk saline agar

E. Lymph node punctate, cysteine agar

82. A 55-year-old patient with a characteristic rash, fever, dizziness has been admitted to a hospital. He
has been provisionally diagnosed wi-th typhus. No similar cases have been reported. In his youth (15
years old) the patient suffered typhus in a boardi-ng school. What disease is it?

A. * Brill’s disease

B. Typhoid fever

C. Measles

D. Rubella

E. Cholera

83. A 56 y.o. patient has been suffering from thyreotoxicosis for a long time. What type of hypoxia can
be developed?

A. * Tissue

B. Hemic

C. Circulatory

D. Respiratory
E. Mixed

84. A 56 year old patient suffering from cardiac insufficiency has edema of feet and shins, edematous
skin is pale and cold. What is the leding mechanism of edema pathogenesis?

A. * Rise of hydrostatic pressure in venules

B. Drop of oncotic pessure in capillaries

C. Increase of capillary permeability

D. Disorder of lymph outflow

E. Positive water balance

85. A 59-year-old man has symptoms of parenchymatous jaundice and portal hypertension. Histological
examinati-on of the puncture of the liver bi- optate has revealed an affected beam-lobule structure,
part of hepatocytes has signs of fat dystrophy, port-portal connective tissue septa with formation of
pseudo-lobules, with periportal lympho-macrophage infiltrations. What is the most probable
diagnosis?

A. * Liver cirrhosis

B. Alcohol hepatitis

C. Chronic hepatosis

D. Viral hepatitis

E. Toxic dystrophy

86. A 5-year-old child who often fells ill with respiratory diseases has eczematous appearances after
consumption of some food products, tendency to prolonged course of inflammatory processes. What
kind of diathesis can be suspected in this case?

A. * Exudative-catharral

B. Hemmorhagic

C. Arthritism

D. Lymphohypoplastic

E. Asthenic

87. A 60-year-old patient with a long hi- story of atherosclerosis and a previous myocardial infarction
developed an attack of retrosternal pain. 3 days later the patient was hospitalized and then died of
progressive cardiovascular insufficiency. During autopsy a white fibrous depressed area about 3 cm
in diameter with clear margins was found within the area of posterior wall of the left ventricle and
interventricular septum. The dissector considered these changes to be:

A. * Focal cardiosclerosis
B. Myocardial ischemia

C. Myocardial infarction

D. Myocarditis

E. Myocardial degeneration

88. A 60-year-old patient with a long history of atherosclerosis and a previ-ous myocardial infarction
developed an attack of retrosternal pain. 3 days later the patient was hospitalized and then di-ed of
progressive cardiovascular insuffici-ency. At autopsy a white fibrous depressed area about 3 cm in
diameter with clear boundaries was found in the posterior wall of the left ventricle and
interventri-cular septum. The dissector evaluated these changes as:

A. * Focal cardiosclerosis

B. Myocardial ischemia

C. Myocardial infarction

D. Myocarditis

E. Myocardial degeneration

89. A 62 year old woman complai-ns of frequent pain attacks in the area of her chest and backbone, rib
fractures. Her doctor suspected myeloma (plasmocytoma). What of the following laboratory
characteristics will be of the greatest diagnostic importance?

A. * Paraproteinemia

B. Hyperalbuminemia

C. Proteinuria

D. Hypoglobulinemia

E. Hypoproteinemia

90. A 63 year old male patient who had been suffering from chronic di-ffuse obstructive disease,
pulmonary emphysema, for 15 years died from cardi-ac insufficiency. Autopsy revealed nutmeg liver
cirrhosis, cyanotic induration of ki-dneys and spleen, ascites, edemata of lower limbs. These changes
of internal organs are typical for the following di-sease:

A. * Chronic right-ventricular insufficiency

B. Acute right-ventricular insufficiency

C. Chronic left-ventricular insufficiency

D. Acute left-ventricular insufficiency

E. General cardiac insufficiency


91. A 63-year-old man, who has been suffering from chronic fibrous-cavernous pulmonary tuberculosis
for 24 years, has been delivered to a nephrology department with uremia. Intravital diagnostic test for
amyloid in the kidneys was positive. What amyloidosis is it in this case?

A. * Secondary systemic

B. Primary systemic

C. Localized (focal)

D. Hereditary (genetic)

E. Senile

92. A 65-year-old male suddenly lost the vision in one eye due to the reti-nal detachment. The patient
underwent enucleation. Histological examination of the removed eye retina and choroid revealed
clusters of atypical cells wi-th marked polymorphism of cells and nuclei, with a moderate number of
mi-toses including the pathological ones. The cell cytoplasm and intercellular medi-um contained
brown pigment giving a positive DOPA reaction. Perls’ reaction was negative. What is the most
likely di-agnosis?

A. * Melanoma

B. Pigmented mole

C. Hemorrhage

D. Cysticercosis

E. Wilson’s disease

93. A 67-year-old patient with clini-cal diagnosis of chronic bronchitis, pneumosclerosis, and
cardiopulmonary decompensation has the biopsy material taken from the suspicious area in his right
bronchus mucosa. Cellular and tissue atypi-sm along with pearly bodies can be histologi-cally
detected. What pathologic process is characterized by the described histological changes?

A. * Squamous cell carcinoma of bronchus with keratinization

B. Polypoid chronic bronchitis

C. Bronchiectasis

D. Acute bronchitis

E. Squamous cell metaplasia of bronchial mucosa

94. A 70-year-old male patient died from acute coronary insufficiency. He had knee joint swelling,
gonycampsis and gonalgia during his lifetime. Pathomorphological examination of the deformed
joi-nts and synovial membranes revealed membrane hyperaemia with multiple perivascular
inflammatory infltrations made by lymphocytes, plasmocytes and macrophagocytes. There was an
accumulation of organized fibrin covering some areas of synovium membrane and looking like rice
grains in the articular li-quid. What is the most likely diagnosis?

A. * Atrophic arthritis

B. Periarteritis nodosa

C. Ankylosing spondylitis

D. Tuberculous arthritis

E. Deforming arthrosis

95. A 71 year old man had been presenti-ng with diarrhea for 10 days. The feces had admixtures of blood
and mucus. He was delivered to a hospital in grave condi-tion and died 2 days later. Autopsy of the
body revealed the following: diphtheri-tic colitis with multiple irregularly-shaped ulcers of different
depth in both sigmoid colon and rectus. Bacteriological analysis revealed Shigella. What was the
main di-sease?

A. * Dysentery

B. Typhoid fever

C. Salmonellosis

D. Nonspecific ulcerous colitis

E. Yersiniosis

96. A 7-year-old boy died of acute posthemorrhagic anemia caused by profuse hemorrhage in the
gastrointestinal tract. Postmortem study revealed the followi- ng: macroscopically there were acutely
enlarged various groups of the lymph nodes, thymomegaly, hepatosplenomegaly, and bright red bone
marrow; microscopically there was hypercellular bone marrow wi- th monomorphic infiltrations
composed of blasts and diffuse-focal tumor infiltrations in the liver, spleen, lymph nodes, brain
substance and tunics. Make the diagnosis:

A. * Acute lymphoblastic leukemia

B. Acute myeloblastic leukemia

C. Acute undifferentiated leukemia

D. Acute monoblastic leukemia

E. Acute plasmablastic leukemia

97. A 7-year-old boy got ill with di-phtheria. On the third day he di-ed of asphyxiation. At autopsy the
mucosa of the larynx, trachea and bronchi had thickened, edematous, lustreless appearance and was
covered with gray films which could be easily removed. Specify the type of laryngeal inflammation:

A. * Croupous
B. Diphtheritic

C. Purulent

D. Catarrhal

E. Intermediate

98. A 7-year-old child has acute onset of disease: temperature rise up to 38oC , rhinitis, cough,
lacrimation, and large-spot rash on the skin. Pharyngeal mucosa is edematous, hyperemic, with
whitish spots in the buccal area. What kind of inflammation causes the changes in the buccal
mucosa?

A. * Catarrhal inflammation

B. Suppurative inflammation

C. Fibrinous inflammation

D. Hemorrhagic inflammation

E. Serous inflammation

99. A 9 m.o. child has delayed dentition, it is also out of order. Upper jaw confi-guration is horizontal
("high"palate); mi croscopically - irregular mineralization of tooth enamel, wrinkled enamel prisms,
some of them are vacuolized. Predentin zone is extended; there are solitary denti-cles. What disease
is it?

A. * Early rickets

B. Late rickets

C. Osteomalacia

D. Gout

E. Hypervitaminosis D

100. A 9-year-old boy has acute onset of di-sease: sore throat, body temperature rise up to 39, 5oC ; on the
second day diffuse skin rash was detected all over his skin exept for nasolabial triangle. On
examination of oral cavity: crimson tongue, "flaming pharynx", necrotic tonsillitis. What diagnosis is
the most likely?

A. * Scarlet fever

B. Measles

C. Diphtheria

D. Influenza

E. Meningococcemia
101. A child has abnormal formation of tooth enamel and dentin as a result of low concentration of
calcium ions in blood. Such abnormalities might be caused by deficiency of the following hormone:

A. * Parathormone

B. Thyrocalcitonin

C. Thyroxin

D. Somatotropic hormone

E. Triiodothyronine

102. A child is 10 years old. The followi-ng presentations have developed: sharp pain during swallowing,
swollen neck, body temperature rise up to 39, 0oC , bright-red fi-nely papular rash all over the body.
Pharynx and tonsils are sharply hyperemic ("flaming pharynx"), "crimson tongue". On the tonsi-ls
surface there are isolated greyish necrosis focuses. What disease it might be?

A. * Scarlet fever

B. Meningococcal nasopharyngitis

C. Diphtheria

D. Influenza

E. Measles

103. A child with point mutation presents with absence of glucose 6-phosphatase, hypoglycemia, and
hepatomegaly. What pathology are these signs characteristic of?

A. * Von Gierke’s disease (Glycogen storage disease type I)

B. Cori’s disease (Glycogen storage disease type III)

C. Addison’s disease (Primary adrenal insufficiency)

D. Parkinson’s disease

E. McArdle’s disease (Glycogen storage disease type V)

104. A child’s blood presents high content of galactose, glucose concentration is low. There are such
presentations as cataract, mental deficiency, adipose degeneration of liver. What disease is it?

A. * Galactosemia

B. Diabetes mellitus

C. Lactosemia

D. Steroid diabetes
E. Fructosemia

105. A coprological survey revealed light-colored feces containing drops of neutral fat. The most likely
reason for this condi-tion is the disorder of:

A. * Bile inflow into the bowel

B. Gastric juice acidity

C. Pancreatic juice secretion

D. Intestinal juice secretion

E. Intestinal absorption

106. A disaster fighter at a nuclear power plant developed hemorrhagic syndrome on the background of
acute radiation di-sease. What is the most important factor of syndrome pathogenesis?

A. * Thrombocytopenia

B. Vascular wall damage

C. Increased activity of fibrinolysis factors

D. Increased activity of anticoagulative system factors

E. Decreased activity of coagulative factors

107. A diseased child has a high fever, sore throat, swelling of submandibular lymph nodes. Objectively:
pharyngeal mucosa is edematous, moderately hyperemic, the tonsils are enlarged, covered with
grayish membrane tightly adhering to the tissues above. Attempts to remove the membrane produce
the bleeding defects. What di-sease are these presentations typical for?

A. * Diphtheria

B. Catarrhal tonsillitis

C. Scarlet fever

D. Meningococcal disease

E. Measles

108. A female patient complains of vision impairment. On examination she was found to have obesity,
fasting hyperglycemia. What complication of di-abetes can cause vision impairment?

A. * Microangiopathy

B. Macroangiopathy

C. Atherosclerosis

D. Neuropathy
E. Glomerulopathy

109. A female patient suffering from bronchial asthma had got a viral infection that provoked status
asthmatics with fatal outcome. Histological examination of lungs revealed spasm and edema of
bronchioles, apparent infiltration of their walls with lymphocytes, eosinophils and other leukocytes;
labrocyte degranulation. What mechanism of hypersensitivity underlies the described alterations?

A. * Reagin reaction

B. Inflammatory

C. Autoimmune

D. Immune complex

E. Immune cytolysis

110. A forensic medical expert exami-nes the body of a 58 y.o. man who had been consuming large
amounts of alcochol for a long time and died at home. Microscopicaly: the right lung is dense and
enlarged, its incision revealed that the tissue is greyish and homogenous, pleura is covered with
greyish layers. Mi-croscopically - alveolar cavities contain fibrin, hemolyzed erythrocytes. Make a
diagnosis:

A. * Croupous pneumonia

B. Focal pneumonia

C. Interstitial pneumonia

D. Primary pulmonary tuberculosis

E. Caseous pneumonia

111. A girl has been diagnosed with adrenogenital syndrome (pseudohermaphroditism). This pathology is
caused by hypersecretion of the following adrenal hormone:

A. * Androgens

B. Estrogens

C. Mineralocorticoids

D. Glucocorticoids

E. Catecholamines

112. A highly injured person has gradually died. Please choose the indicator of bi-ological death:

A. * Autolysis and decay in the cells

B. Disarray of chemical processes


C. Loss of consciousness

D. Absence of palpitation and breathing

E. Absence of movements

113. A histologic specimen shows an organ’s parenchyma which is presented by lymphoid tissue making
some lymph nodes. The nodes are located diffusi-vely and contain a central artery. What anatomic
formation might have such morphological structure?

A. * Spleen

B. Red bone marrow

C. Thymus

D. Tonsil

E. Lymph node

114. A histological specimen presents an artery. One of the membranes of its wall has flat cells lying on
the basal membrane. What type of cells is it?

A. * Endothelium

B. Mesothelium

C. Smooth myocytes

D. Fibroblasts

E. Macrophages

115. A male patient is 28 years old. Histological study of a cervical lymph node revealed a change of its
pattern due to the proliferation of epithelioid, lymphoid cells and macrophages having nuclei in form
of a horseshoe. In the center of some cell clusters there were non-structured light-pink areas with
fragments of nuclei. What disease are these changes typical for?

A. * Tuberculosis

B. Hodgkin’s disease

C. Actinomycosis

D. Tumor metastasis

E. Syphilis

116. A man died 8 days after the beginni-ng of the disease. He was diagnosed with dysentery. At the
autopsy it was found out a thickened wall of the sigma and rectum, fibrinous membrane on the
surface of mucous membrane. Histologically: there is a deep necrosis of mucous membrane with
infiltration of necrotic masses with fi-brin. What kind of colitis does correspond to the changes?

A. * Diphtheritic

B. Catarrhal

C. Ulcerative

D. Chronic

E. Gangrenous

117. A man had worked in a coal mine for over 20 years. After his death autopsy revealed that his lungs
were dense, grayish-black and had large areas of neogenic connective tissue containing a lot of
microphages with black pigment in the cytoplasm. What is the most likely diagnosis?

A. * Anthracosis

B. Anthracosilicosis

C. Silicoanthracosis

D. Talcosis

E. Siderosis

118. A man has suffered multiple bone fractures of his lower extremities during a traffic accident. During
transportation to a hospital his condition was further aggravated: blood pressure decreased, there
were signs of pulmonary artery embolism. What kind of embolism is the most likely in the given
case?

A. * Fat embolism

B. Air embolism

C. Gas embolism

D. Tissue embolism

E. Thromboembolism

119. A man is 28 years old. Histological investigation of the cervical lymph node revealed a change of its
pattern due to proliferation of epithelioid, lymphoid cells and macrophages with horseshoe-shaped
nuclei. In the center of some cell clusters there were non-structured light-pink areas with fragments
of nuclei. What disease are these changes typical of?

A. * Tuberculosis

B. Hodgkin’s disease

C. Actinomycosis
D. Tumor metastasis

E. Syphilis

120. A man with a wound of his limb that had been suppurating for a long time di-ed from intoxication.
Autopsy revealed extreme emaciation, dehydration, brown atrophy of liver, myocardium, spleen and
cross-striated muscles as well as renal amyloidosis. What diagnosis corresponds with the described
presentations?

A. * Chroniosepsis

B. Septicopyemia

C. Septicemia

D. Chernogubov’s syndrome

E. Brucellosis

121. A microscopic examination of the enlarged neck gland of a 14-year-old girl revealed destruction of
the tissue structure of the node, absence of the lymph follicles, sclerotic and necrosis parts. Cell
constitution of the node is polymorphous, lymphocites, eosinophi-les, atypical cells of the large size
wi-th multiple-lobule nuclei (Beresovsky-Shternberg cells) and onenucleus large size cells are
observed. What is the most likely diagnosis?

A. * Lymphogranulomatous

B. Acute lympholeucosis

C. Chronic lympholeucosis

D. Berkitt’s lymphoma

E. Fungous mycosis

122. A middle-aged man went to a foreign country because he had been offered a job there. However he
had been unemployed for quite a long time. What endocrine glands were exhausted most of all in this
man?

A. * Adrenal glands

B. Parathyroid glands

C. Seminal glands

D. Substernal gland

E. Thyroid gland

123. A mother consulted a doctor about her 5-year-old child who develops erythemas, vesicular rash and
skin itch under the influence of sun. Laboratory studies revealed decreased iron concentration in the
blood serum, increased uroporphyrinogen I excretion with the urine. What is the most likely inherited
pathology in this child?

A. * Erythropoietic porphyria

B. Methemoglobinemia

C. Hepatic porphyria

D. Coproporphyria

E. Intermittent porphyria

124. A pathology-histology laboratory received a vermiform appendix up to 2,0 cm thick. Its serous
membrane was pale, thick and covered with yellowish-green blooms. The wall was fattened, of
grayish-red color. The appendix lumen was dilated and folded with yellowish-green substance.
Histological examination revealed that the appendix wall was infiltrated with neutrophils. Specify the
appendix disease:

A. * Acute phlegmonous appendicitis

B. Acute gangrenous appendicitis

C. Acute superficial appendicitis

D. Acute simple appendicitis

E. Chronic appendicitis

125. A patient complains of hydruria (7 liters per day) and polydipsia. Examinati-on reveals no disorders
of carbohydrate metabolism. These abnormalities might be caused by the dysfunction of the
following endocrine gland:

A. * Neurohypophysis

B. Adenohypophysis

C. Islets of Langerhans (pancreatic islets)

D. Adrenal cortex

E. Adrenal medulla

126. A patient consulted a doctor with complaints of dyspnea occurring after physical exertion. Physical
examination revealed anemia, paraprotein was detected among gamma globulins. What value should
be determined in the patient’s urine to confirm the diagnosis of myeloma?

A. * Bence Jones protein

B. Bilirubin

C. Hemoglobin
D. Ceruloplasmin

E. Antitrypsin

127. A patient died 3 days after the operation because of perforated colon with manifestations of diffuse
purulent peritonitis. The autopsy revealed: colon mucos membrane was thickened and covered with a
fibrin film, isolated ulcers penetrated at different depth. The histology result: mucous membrane
necrosis, leukocytes infiltration with hemorrhages focuses. What disease complication caused the
patient’s death?

A. * Dysentery

B. Typhoid

C. Nonspecific ulcerative colitis

D. Crohn’s disease

E. Amebiasis

128. A patient died from acute cardiac insufficiency, among clinical presentations there was
gastrointestinal haemorrhage. Examination of mucous membrane of sromach revealed some defects
reachi-ng myenteron; their edges and bottom were mostly even and loose, some of them contained
dark-red blood. What pathological process was revealed?

A. * Acute ulcers

B. Chronic ulcers

C. Erosions

D. Thrombosis

E. Inflammation

129. A patient died from acute cardiac insufficiency. The histological examinati-on of his heart revealed
the necrotized section in myocardium of the left ventri-cle, which was separated from undamaged
tissue by the zone of hyperimic vessels, small hemorrhages and leukocytic infi-ltration. What is the
most likely diagnosis?

A. * Myocardial infarction

B. Myocardial ischemic dystrophy

C. Focal exudate myocarditis

D. Diffuse exudate myocarditis

E. Productive myocarditis

130. A patient died from cardiopulmonary decompensation. Histological examination revealed diffused
pulmonary lesion together with interstitial edema, infiltration of tissue by limphocytes, macrophages,
plasmocytes; pulmonary fibrosis, panacinar emphysem A. What disease corresponds with the
described picture?

A. * Fibrosing alveolitis

B. Pulmonary atelectasis

C. Bronchial asthma

D. Bronchopneumonia

E. Chronic bronchitis

131. A patient died from cardiopulmonary decompensation. Histological examinati-on revealed diffused
pulmonary affection along with interstitial edema, infiltration of tissue by limphocytes, macrophages,
plasmocytes; pulmonary fibrosis, panaci-nar emphysema. What is the most likely diagnosis?

A. * Fibrosing alveolitis

B. Chronic bronchitis

C. Bronchopneumonia

D. Pulmonary atelectasis

E. Bronchial asthma

132. A patient died from progressive heart failure. Autopsy revealed that the heart was enlarged in
diameter, flabby. The muscle section exhibited irregular blood supply. Histological study of
myocardium revealed hyperemia, the stroma was found to have lymphohistiocytic infiltrates wi-th
degeneration of cardiomyocytes. The revealed morphological changes are indi-cative of:

A. * Non-purulent interstitial myocarditis

B. Venous plethora

C. Cardiomyoliposis

D. Cardiosclerosis

E. Myocardial infarction

133. A patient died of cancerous cachexia with primary localization of cancer in the stomach. Autopsy
revealed acutely enlarged liver with uneven surface and numerous protruding nodes; the nodes had
clear margins in the section, rounded shape, gray-pink color, varying density, someti- mes contained
necrotic foci. Histologically: there are atypical cells in the nodes. What pathologic process occurred
in the liver?

A. * Cancer metastases

B. Abscesses
C. Regeneratory nodes

D. Infarction

E. Hepatic cancer

134. A patient died under conditions of cardiovascular insufficiency. Autopsy results: postinfarction
cardiosclerosis, myocardium hypertrophy and dilatati-on of its cavities, especially of its right
ventricle. Liver is enlarged, its surface is smooth, incision revealed that it was plethoric, with
dark-red specks against the background of brownish tissue. Hi-stologically: plethora of central parts
of lobules; peritheral parts around portal tracts contain hepatocytes in a state of adipose degeneration.
How are these li-ver changes called?

A. * Nutmeg liver

B. Pseudonutmeg liver

C. Amyloidosis

D. Liver cirrhosis

E. Liver steatosis

135. A patient has a cluster of matted together dense lymph nodes on his neck. Histological examination
of a removed lymph node revealed proliferation of reticular cells, presense of Reed-Sternberg cells.
What disease is meant?

A. * Lymphogranulomatosis

B. Myeloblastic leukosis

C. Lymphocytic leukosis

D. Lymphoblastic leukosis

E. Myelocytic leukosis

136. A patient has been admitted to the contagious isolation ward with signs of jaundice caused by
hepatitis virus. Which of the symptoms given below is strictly specific for hepatocellular jaundice?

A. * Increase of ALT, AST level

B. Hyperbilirubinemia

C. Bilirubinuria

D. Cholemia

E. Urobilinuria
137. A patient has been diagnosed with acute glomerulonephritis that developed after he had had
streptococcal infecti-on. It is most likely that the affection of basal glomerular membrane is caused
by an allergic reaction of the following type:

A. * Immune complex

B. Anaphylactic

C. Cytotoxic

D. Delayed

E. Stimulating

138. A patient has hoarseness of voice. Duri- ng laryngoscopy a gray-white larynx tumor with papillary
surface has been detected. Microscopic investigation has shown the following: growth of connective
tissue covered with multilayer, strongly keratinized pavement epithelium, no cellular atypia. What is
the most likely diagnosis?

A. * Papilloma

B. Fibroma

C. Polyp

D. Angioma

E. Angiofibroma

139. A patient has undergone an amputati-on of lower extremity. Some time later painful nodules
appeared in a stump. Amputatious neuromas were found out at the microscopic examination. To what
pathological processes do those formati-ons relate?

A. * Regeneration

B. Dystrophy

C. Inflammation

D. Hyperemia

E. Metaplasia

140. A patient hospitalized due to mercury intoxication presents with the following processes in the
kidneys: focal necrotic changes of tubules of major renal regions, edema, leukocyte infiltration and
hemorrhages in the interstitial tissue, venous congestion. What condition developed in the patient?

A. * Acute necrotic nephrosis

B. Acute glomerulonephritis

C. Chronic renal failure


D. Acute pyelonephritis

E. Chronic pyelonephritis

141. A patient ill with diabetes mellitus felt acute pain in his right foot. Objecti-vely: foot thumb is black,
foot tissues are edematous, there are foci of epidermis desquamation, stinking discharges. What
clinicopathological form of necrosis is it?

A. * Moist gangrene

B. Bedsore

C. Sequestrum

D. Dry gangrene

E. Infarction

142. A patient ill with thrombophlebitis of his lower limbs had chest pain, blood spi-tting, progressing
respiratory insufficiency that led to his death. Autopsy diagnosed multiple lung infarctions. What is
the most probable cause of their development?

A. * Thromboembolism of pulmonary artery branches

B. Thrombosis of pulmonary artery branches

C. Thrombosis of bronchial arteries

D. Thromboembolism of bronchial arteries

E. Thrombosis of pulmonary veins

143. A patient ill with tuberculosis died from progressing cardiopulmonary decompensation. Autopsy in
the area of the right lung apex revealed a cavity 5 cm in diameter communicating with lumen of a
segmental bronchus. On the inside cavity walls are covered with caseous masses with epithelioid and
Langhans cells beneath them. What morphological form of tuberculosis is it?

A. * Acute cavernous tuberculosis

B. Tuberculoma

C. Caseous pneumonia

D. Infiltrative tuberculosis

E. Acute focal tuberculosis

144. A patient in three weeks after acute myocardial infarction has pain in the heart and joints and
pneumonia. What is the main mechanism of development of post-infarction Dressler’s syndrome?

A. * Autoimmune inflammation
B. Ischemia of myocardium

C. Resorption of enzymes from necrotized area of myocardium

D. Secondary infection

E. Vessels thrombosis

145. A patient presents with icteriti-ousness of skin, scleras and mucous membranes. Blood plasma the
total bili-rubin is increased, stercobilin is increased in feces, urobilin is increased in urine. What type
of jaundice is it?

A. * Haemolytic

B. Gilbert’s disease

C. Parenchymatous

D. Obturational

E. Cholestatic

146. A patient suffering from chronic myeloleukemia has got the followi-ng symptoms of anemia:
decreased number of erythrocytes and low haemoglobin concentration, oxyphilic and
polychromatophilic normocytes, mi-crocytes. What is the leading pathogenetic mechanism of anemia
development?

A. * Substitution of haemoblast

B. Intravascular hemolysis of erythrocytes

C. Deficiency of vitamin B12

D. Reduced synthesis of erythropoietin

E. Chronic haemorrhage

147. A patient suffering from trombophlebitis of deep veins suddenly died. The autopsy has shown freely
lying red friable masses with dim crimped surface in the trunk and bifurcation of the pulmonary
artery. What pathologic process was revealed by the morbid anatomist?

A. * Tromboembolism

B. Thrombosis

C. Tissue embolism

D. Embolism with foreign body

E. Fat embolism

148. A patient suffers from high fever, apnoea, pain in the thorax on the right. Pleurocentesis yielded 700
ml of yellow- green viscous liquid. Make the diagnosis:
A. * Pleural empyema

B. Bronchial pneumonia

C. Serous pleurisy

D. Hemorrhagic pleurisy

E. Pleural carcinomatosis

149. A patient underwent biopsy of the soft palate arches for a suspected tumor (macroscopy revealed an
ulcer with a dense floor). Study of the biopsy materi-al revealed mucosal necrosis with infi-ltration of
lymphocytes, epithelioid cells, plasma cells, single neutrophils in the submucosa. There were also
apparent si-gns of endovasculitis and perivasculitis. The described changes are typical for:

A. * Primary syphilis

B. Aphthous stomatitis

C. Ulcerative stomatitis

D. Necrotizing ulcerative Vincent stomatitis

E. Pharyngeal diphtheria

150. A patient underwent surgical removal of a cavitary liver lesion 2 cm in diameter. It was revealed that
the cavity wall was formed by dense fibrous connective tissue; the cavi-ty contained muddy, thick,
yellowish-greenish fluid with an unpleasant odor. Microscopically, the fluid consi-sted mainly of
polymorphonuclear leukocytes. What pathological process are these morphological changes typi-cal
for?

A. * Chronic abscess

B. Acute abscess

C. Phlegmon

D. Empyema

E. -

151. A patient was admitted to the hospi-tal with an asphyxia attack provoked by a spasm of smooth
muscles of the respiratory tracts. This attack was mai-nly caused by alterations in the following parts
of the airways:

A. * Small bronchi

B. Median bronchi

C. Large bronchi
D. Terminal bronchioles

E. Respiratory part

152. A patient was admitted to the surgical department with suspected inflammation of Meckel's
diverticulum. What part of bowels should be examined in order to discover the diverticulum in
course of an operation?

A. * Ileum

B. Caecum

C. Jejunum

D. Colon ascendens

E. Duodenum

153. A patient was ill with burn disease that was complicated by DIC syndrome. What stage of DIC
syndrome can be suspected if it is known that the patient’s blood coagulates in less than 3 minutes?

A. * Hypercoagulation

B. Transition phase

C. Hypocoagulation

D. Fibrinolysis

E. Terminal

154. A patient who abuses smoking has chronic bronchitis. Biopsy of his primary bronchus revealed
multilayer pavement epithelium. What pathological process was revealed in the bronchus?

A. * Metaplasia

B. Physiological regeneration

C. Reparative regeneration

D. Hyperplasia

E. Dysplasia

155. A patient who has been abusing tobacco smoking for a long time has got cough accompanied by
excretion of vi-scous mucus; weakness after minor physi-cal stress, pale skin. The patient has also
lost 12,0 kg of body weight. Endoscopic examination of biosy material his illness was diagnosed as
squamous cell carci-noma. Name a pathological process that preceded formation of the tumour:

A. * Metaplasia

B. Hypoplasia
C. Hyperplasia

D. Necrosis

E. Sclerosis

156. A patient with a history of chronic glomerulonephritis presents with azotemia, oliguria, hypo- and
isosthenuria, proteinuria. What is the leading factor in the pathogenesis of these symptoms
development under chronic renal failure?

A. * Mass decrease of active nephrons

B. Intensification of glomerular filtration

C. Tubular hyposecretion

D. Disturbed permeability of glomerular membranes

E. Intensification of sodium reabsorption

157. A patient with android-type obesity had been suffering from arterial hypertension, hyperglycemia,
glycosuria for a long time and died from the cerebral hemorrhage. Pathologic examination revealed
pituitary basophil adenoma, adrenal cortex hyperplasia. What is the most likely diagnosis?

A. * Itsenko-Cushing’s syndrome

B. Diabetes mellitus

C. Acromegalia

D. Pituitary nanism

E. Adiposogenital dystrophy

158. A patient with chronic heart failure presents with increased blood viscosity. Capillaroscopy detected
damage to the vessel walls of the microcirculation system. What disorder is possible in the given
case?

A. * Blood ”sludge” phenomenon

B. Thrombosis

C. Embolism

D. Arterial hyperemia

E. Venous hyperemia

159. A patient with femoral neck fracture, who for a long time had to remain in bed in a forced (supine)
position, has developed dark-brown lesions along the backbone; soft tissues are swollen, in the areas
of maceration there is a foul-smelling liquid. Name the clinicopathologic type of necrosis:
A. * Bed-sore

B. Infarction

C. Sequestrum

D. Coagulation necrosis

E. Dry gangrene

160. A patient with femoral neck fracture, who for a long time had to remain in bed in a forced (supine)
position, has developed dark-brown lesions along the backbone; soft tissues are swollen, in the areas
of maceration there is a foul-smelling liquid. Name the clinicopathologic type of necrosis:

A. * Bedsore

B. Infarction

C. Sequestrum

D. Coagulation necrosis

E. Dry gangrene

161. A patient with high-titer antinuclear antibodies died from progressing renal impairment. Autopsy
revealed mesangi-oproliferative glomerulonephritis and abacterial polypous endocarditis. There was
periarterial bulbar sclerosis in spleen and productive proliferative vasculitis in skin. What is the most
likely diagnosis?

A. * Systemic lupus erythematosus

B. Nephrotic syndrome

C. Rheumatism

D. Dermatomyositis

E. Periarteritis nodosa

162. A patient with marked pneumofi-brosis that developed after infiltrati-ng pulmonary tuberculosis has
been di-agnosed with respiratory failure. What is its pathogenetic type?

A. * Restrictive

B. Obstructive

C. Dysregulatory

D. Reflex

E. Apneistic
163. A patient with obliterating atherosclerosis underwent sympathectomy of femoral artery in the region
of femoral trigone. What type of arterial hyperemia was induced by the operation?

A. * Neuroparalytic

B. Reactive

C. Metabolic

D. Neurotonic

E. Functional

164. A patient with obliterating endarteri-tis underwent ganglionic sympathectomy. What type of arterial
hyperaemia should have developed as a result of the surgery?

A. * Neuroparalytic

B. Neurotonic

C. Metabolic

D. Functional

E. Reactive

165. A patient with severe course of respiratory viral infection presented with clinical signs of progressing
heart failure that led to his death in the 2nd week of disease. Autopsy revealed that the heart cavities
were much dilated, the heart was flabby. Histological examination of the myocardium revealed
microvascular plethora and diffuse stroma infiltration with lymphocytes and histiocytes. What is the
most likely diagnosis?

A. * Myocarditis

B. Stenocardia

C. Acute coronary insufficiency

D. Myocardial infarction

E. Cardiomyopathy

166. A patient with signs of osteoporosis and urolithiasis has been admitted to an endocrinology
department. Blood test revealed hypercalcemia and hypophosphatemia. These changes are associated
with abnormal synthesis of the following hormone:

A. * Parathyroid hormone

B. Calcitonin

C. Cortisol
D. Aldosterone

E. Calcitriol

167. A patient with thrombophlebitis of lower extremities had got chest pai-ns, blood spitting, growing
respiratory failure that caused his death. Autopsy revealed multiple pulmonary infarctions. What is
the most probable reason of their development?

A. * Pulmonary artery embolism

B. Pulmonary artery thrombosis

C. Bronchial artery thrombosis

D. Bronchial artery embolism

E. Pulmonary venous thrombosis

168. A patient with tuberculosis di-ed from progressing cardiopulmonary decompensation. Autopsy in the
region of the right lung apex revealed a cavity 5 cm in diameter communicating with lumen of a
segmental bronchus. On the insi-de cavity walls are covered with caseous masses with epithelioid
and Langhans cells beneath them. What morphological form of tuberculosis is it?

A. * Acute cavernous tuberculosis

B. Tuberculoma

C. Caseous pneumonia

D. Infiltrative tuberculosis

E. Acute focal tuberculosis

169. A patient, having suffered a thermal burn, developed painful boils filled with turbid liquid in the skin.
What morphological type of inflammation has developed in the patient?

A. * Serous

B. Proliferative

C. Croupous

D. Granulomatous

E. Diphtheritic

170. A section of the left lung was found to have an area of dense red tissue. The area was cone-shaped,
stood out distinctly from the healthy tissue, with its base directed to the pleura. The dissected tissue
was granular, dark-red. What is the most likely diagnosis?

A. * Haemorrhagic infarction

B. Lung abscess
C. Lung gangrene

D. Primary tuberculous affection

E. Croupous pneumonia

171. A specimen shows an organ covered wi- th connective tissue capsule with trabeculae radiating
inward the organ. The organ’s cortex contains lymph nodules; there are medullary cords made of
lymphoid cells. What organ is under study?

A. * Lymph node

B. Thymus

C. Spleen

D. Red bone marrow

E. Tonsils

172. A specimen shows organ parenchyma to consist of lymphoid tissue that forms lymph nodules; the
nodules are located diffusely and have a central artery. What anatomical structure has such
morphological characteristics?

A. * Spleen

B. Tonsil

C. Lymph node

D. Thymus

E. Red bone marrow

173. A stillborn child was found to have thickened skin resembling of the tortoise shell, underdeveloped
auricles. Histological examination of skin revealed hyperkeratosis, atrophy of the granular epidermis
layer; inflammatory changes were not present. What is the most likely diagnosis?

A. * Ichthyosis

B. Leukoplakia

C. Xerodermia

D. Erythroplakia

E. Dermatomyositis

174. A surgeon has detected inflammation of the Meckel’s diverticulum in a patient. During surgical
invasion it can be located in the:
A. * Ileum

B. Jejunum

C. Colon

D. Duodenum

E. Sigmoid colon

175. A tooth extraction in a patient with chronic persistent hepatitis was complicated with prolonged
hemorrhage. What is the reason for the haemorrhagic syndrome?

A. * Decrease in thrombin production

B. Increase in thromboplastin production

C. Decrease in fibrin production

D. Increase in fibrinogen synthesis

E. Fibrinolysis intensification

176. A viral infection has damaged cells that form walls of bile capillaries. This sti-mulated conditions for
inflow of bile into the blood of sinusoidal capillaries. What cells are damaged?

A. * Hepatocytes

B. Kupffer’s cells

C. Ito cells

D. Pit-cells

E. Endotheliocytes

177. A woman complains of visual impairment. Examination revealed obesity in the patient and her
fasting plasma glucose level is hyperglycemic. What diabetes complication can cause visual
impairment/blindness?

A. * Microangiopathy

B. Macroangiopathy

C. Atherosclerosis

D. Neuropathy

E. Glomerulopathy

178. A woman delivered a dead child with multiple developmental defects. What protozoan disease might
have caused the intrauterine death?
A. * Toxoplasmosis

B. Leishmaniasis

C. Malaria

D. Amebiasis

E. Lambliasis

179. A woman gave birth to a stillborn baby with numerous malformations. What protozoan disease could
cause intrauterine death?

A. * Toxoplasmosis

B. Leishmaniasis

C. Malaria

D. Amebiasis

E. Lambliasis

180. A woman suffering from dysfuncti-onal metrorrhagia was made a diagnostic abortion. Histologically
in the scrape there were a lot of small stamped glandulars covered with multirowed epi-thelium. The
lumens of some glandulars were cystically extended. Choose the vari-ant of general pathologic
process in the endometrium.

A. * Glandular-cystic hyperplasia of endometrium

B. Atrophy of endometrium

C. Metaplasia of endometrium

D. Neoplasm of endometrium

E. Hypertrophic growth

181. A worker of a cattle farm fell acutely ill and then died from the progressing intoxication. Autopsy
revealed enlarged, hyposthenic spleen of dark-cherry colour when dissected; excessive pulp scraping.
At the base and fornix of brain pia maters are edematous, soaked with blood, dark-red ("scarlet hat").
Microscopic exami-nation revealed serous haemorrhagic inflammation of brain tissues and tuni-cs
along with destruction of small vessel walls. What is the most likely diagnosis?

A. * Anthrax

B. Tularemia

C. Brucellosis

D. Plaque
E. Cholera

182. A worker of an agricultural enterprise had been suffering from an acute disease with aggravating
intoxication signs, which resulted in his death. On autopsy: the spleen is enlarged, flaccid, dark
cherry-red in the section, yields excessive pulp scrape. Soft meninges of fornix and base of the brain
are edematous and saturated with blood (”cardinal’s cap”). Microscopically: serous- hemorrhagic
inflammation of meninges and cerebral tissues. Make the diagnosis:

A. * Anthrax

B. Tularemia

C. Plague

D. Cholera

E. Brucellosis

183. A worker of an agricultural enterprise had been suffering from an acute disease with aggravating
intoxication signs, whi- ch resulted in his death. On autopsy: the spleen is enlarged, flaccid, dark
cherry-red on section, yields excessive pulp scrape. Soft meninges of the fornix and base of the brain
are edematous and saturated with blood (”cardinal’s cap”). Microscopically: serous- hemorrhagic
inflammation of meninges and cerebral tissues. Make the diagnosis:

A. * Anthrax

B. Tularemia

C. Plague

D. Cholera

E. Brucellosis

184. A young man consulted a doctor about disturbed urination. Examinati-on of his external genitals
revealed that urethra is split on top and urine runs out of this opening. What anomaly of external
genitals development is the case?

A. * Epispadia

B. Phimosis

C. Hermaphroditism

D. Paraphimosis

E. Hypospadia

185. A young man has a painlessl formati-on without marked borders in the soft ti-ssues of his thigh. On
the tissue bioptate the formation looks like flesh of fish and consists of immature fibroblast-like cells
with multiple mitosis growing through the muscles. What is the most likely di-agnosis?
A. * Fibrosarcoma

B. Myosarcoma

C. Fibroma

D. Cancer

E. Myoma

186. A young man has an unpainfull formation without marked borders in the soft tissues of his thigh. On
the tissue bi-optate the formation lookes like flesh of fish and consists of immature fibroblast-like
cells with multiple mitosis growing through the muscles. What is the most li-kely diagnosis?

A. * Fibrosarcoma

B. Myosarcoma

C. Fibroma

D. Cancer

E. Myoma

187. Acute renal impairment caused death of a bleeding patient. Autopsy revealed enlarged kidneys with a
broad pale pink cortical layer expressively demarcated from dark red renal pyrami-ds. Macroscopic
examination revealed lack of epithelial nuclei of convoluted tubules, tubulorrhexis, phlebostasis. The
cell nuclei of choroid glomus and straight tubules were present. What pathology is it?

A. * Necronephrosis

B. Infarction

C. Glomerulonephritis

D. Pyelonephritis

E. Nephrosis

188. After a road accident a victim has tachycardia, arterial blood pressure 130/90 mm Hg, tachypnoe, the
skin is pale and dry, excitation of central nervous system is observed. What shock stage is the patient
most likely in?

A. * Erectile

B. Terminal

C. Torpid

D. Preshock (compensation stage)

E. Agony
189. After a tourniquet application a patient was found to have petechial haemorrhages. The reason for it
is the dysfunction of the following cells:

A. * Platelets

B. Eosinophils

C. Monocytes

D. Lymphocytes

E. Neutrophils

190. After an immunoassay a chi-ld was diagnosed with immunodefici-ency of humoral immunity. What
is the reason for the primary immunodeficiency development in the child?

A. * Hereditary abnormality of immune system

B. Embryonal development abnormalities

C. Pathometabolism in mother’s organism

D. Immune responsiveness and resistance disorders

E. Toxic damage of B-lymphocytes

191. After mushroom poisoning the pati- ent developed signs of acute hepatic fai- lure leading to his
death. On autopsy the liver is diminished, flaccid; the capsule is wrinkled; the tissue is ochre-yellow
on section. Microscopically: fatty degeneration of hepatocytes, necrotic central segments of the
hepatic lobes. These changes are characteristic of:

A. * Massive progressive necrosis

B. Fatty hepatosis

C. Acute exudative hepatitis

D. Acute productive hepatitis

E. Hepatolenticular degeneration

192. After sensitization a test animal recei- ved subcutaneously a dose of antigen. At the site of injection a
fibrinous inflammati- on developed with alteration of vessel walls, basal substance, and fibrous
structures of connective tissue. The inflammation took form of mucoid and fibrinoid degeneration,
fibrinoid necrosis. What immune response occurred in the test animal?

A. * Immediate hypersensitivity

B. Delayed hypersensitivity

C. Transplantation immune reaction


D. Normergic reaction

E. Granulomatosis

193. After the prior sensibilization an experimental animal was given a subcutaneous injection of an
antigen. The place of injection exhibited a fibrinous inflammation with alteration of the vessel walls,
basal substance and fibrous structures of the connective tissue in form of mucoid and fibrinoid
swelling and necrosis. What immunological reaction is it?

A. * Immediate hypersensitivity

B. Delayed-type hypersensitivity

C. Reaction of transplantation immunity

D. Normergic reaction

E. Granulomatosis

194. Amniocentesis revealed two sex chromatin bodies (Barr bodies) in each cell of the sample. What
disease is this character typical for?

A. * Trisomy X

B. Klinefelter syndrome

C. Turner’s syndrome

D. Down’s syndrome

E. Patau syndrome

195. An 18-year-old patient has enlarged inguinal lymphnodes, they are painless thickened on palpation.
In the area of genital mucous membrane there is a small-sized ulcer with thickened edges and
"laquer"bottom of greyish colour. What is the most probable diagnosis?

A. * Syphilis

B. Tuberculosis

C. Lepra

D. Trophic ulcer

E. Gonorrhea

196. An 8-year-old child was admitted to the infectious department with fever (up to 38oC) and punctuate
bright-red skin rash. The child was diagnosed as having scarlet fever. Objectively: mucous membrane
of pharynx is apparently hyperemic and edematic, the tonsils are enlarged and have dull
yellowish-grey foci with some black areas. What inflammation is the reason for the pharynx
alterations?
A. * Purulent necrotic

B. Fibrinous

C. Haemorrhagic

D. Serous

E. Catarrhal

197. An aged man had raise of arterial pressure under a stress. It was caused by activation of:

A. * Sympathoadrenal system

B. Parasympathetic nucleus of vagus

C. Functions of thyroid gland

D. Functions of adrenal cortex

E. Hypophysis function

198. An athlete (long-distance runner) during a contest developed a case of acute cardiac insufficiency.
This pathology resulted from:

A. * Cardiac volume overload

B. Disrupted coronary circulation

C. Direct damage to myocardium

D. Pericardium pathology

E. Cardiac pressure overload

199. An autopsy revealed large (1-2 cm) brownish-red, easy crumbling formations covering ulcerative
defects on the external surface of the aortic valve. What is the most likely diagnosis?

A. * Polypus-ulcerative endocarditis

B. Recurrent warty endocarditis

C. Acute warty endocarditis

D. Fibroplastic endocarditis

E. Diffusive endocarditis

200. An autopsy revealed: soft arachnoid membrane of the upper parts of cerebral hemisphere is plethoric,
of yellowish-green color, soaked with purulent and fibrose exudate, it lookes like a cap. What disease
is characterised by these symtoms?

A. * Meningococcal meningitis
B. Tuberculous meningitis

C. Influenza meningitis

D. Meningitis at anthrax

E. Meningitis at typhus

201. An HIV-positive patient’s cause of death is acute pulmonary insufficiency resulting from pneumonia.
Pathohistological investigation of lungs has revealed intersti-tial pneumonia, alveolocyte
desquamati-on and methamorphoses: alveolocyte enlargement, large intranuclear inclusi-ons
surrounded by lightly-coloured areas. Transformed cells resemble owl’s eye. Name the causative
agent of pneumonia:

A. * Cytomegalovirus

B. Pneumococcus

C. Influenza virus

D. Candida fungi

E. Toxoplasma

202. An intraoperational biopsy of mammal gland has revealed the si-gns of atypical tissue with disorder
of parenchyma stroma proportion with domination of the last, gland structures of different size and
shape, lined with single-layer proliferative epithelium. What is the most probable diagnosis?

A. * Fibroadenoma

B. Papilloma

C. Noninfiltrative cancer

D. Infiltrative cancer

E. Mastitis

203. Analysis of a punction biopsy materi-al of liver revealed hepatocyte dystrophy with necroses as well
as sclerosis with di-sorder of beam and lobulous structure, with formation of pseudolobules and
regenerative nodes. What is the most probable diagnosis:

A. * Liver cirrhosis

B. Chronic hepatosis

C. Chronic hepatitis

D. Progressive massive liver necrosis

E. Acute hepatitis
204. As a result of careless handling of an iron, a 34-year-old female patient has got acute pain, redness,
swelling of her right index finger. A few mi-nutes later, there appeared a blister fi-lled with a
transparent liquid of straw-yellow color. The described changes are a manifestation of the following
pathological process:

A. * Exudative inflammation

B. Traumatic edema

C. Alternative inflammation

D. Proliferative inflammation

E. Vacuolar degeneration

205. As a result of past encephalitis, a male patient has developed an increase in cerebrospinal fluid
pressure in the right lateral ventricle. What can be the cause of this condition?

A. * Closure of right interventricular foramen

B. Closure of left interventricular foramen

C. Atresia of tubus medullaris

D. Atresia of sylvian aqueduct

E. Atresia of the fourth ventricle foramina

206. At autopsy the occipital lobe of brain was found to have a cavity 2,5x1,5 cm large filled with a
transparent liquid. The cavity had smooth brownish walls. What process had developed in the brain?

A. * Cyst on the site of a hemorrhage

B. Softening of the cerebrocortical grey matter

C. Brain abscess

D. Paracephalia

E. Cyst on the site of the softening of the cerebrocortical grey matter

207. At the post-mortem examination the stomach of a patient with renal failure was found to have a
yellow-brown coating on the thickened mucosa. The coating was firmly adhering to its surface and
had si-gnificant thickness. Microscopy revealed congestion and necrosis of mucosal and submucosal
layers, fibrin presence. What is the most likely diagnosis?

A. * Fibrinous gastritis

B. Croupous gastritis

C. Gastric abscess
D. Esogastritis

E. Corrosive gastritis

208. At the post-mortem examination the stomach of a patient with renal failure was found to have a
yellow-brown coating on the thickened mucosa. The coating was firmly adhering to its surface and
had significant thickness. Microscopy revealed congestion and necrosis of mucosal and submucosal
layers, fibrin presence. What is the most likely diagnosis?

A. * Fibrinous gastritis

B. Croupous gastritis

C. Gastric abscess

D. Esogastritis

E. Corrosive gastritis

209. At the post-mortem examination the stomach of a patient with renal failure was found to have a
yellow-brown coating on the thickened mucosa. The coating was fi- rmly adhering to its surface and
had si- gnificant thickness. Microscopy revealed congestion and necrosis of mucosal and submucosal
layers, fibrin presence. What is the most likely diagnosis?

A. * Diphtheritic gastritis

B. Croupous gastritis

C. Phlegmonous gastritis

D. Catarrhal gastritis

E. Corrosive gastritis

210. Autopsy has revealed shrunken kidneys weighing 50 mg, with fine-grained surface and uniformly thi
nned substance. Microscopic investi-gation has shown the thickening of arteriole walls due to
accumulation of homogeneous anhistic pink-coloured masses in them. Glomerules were undersized,
sclerotic, with atrophi-ed tubules. What disease are these changes characteristic of?

A. * Essential hypertension

B. Pyelonephritis with kidney shrinkage

C. Renal amyloidosis

D. Acute glomerulonephritis

E. Membranous nephropathy

211. Autopsy of a 1,5-year-old child revealed hemorrhagic skin rash, moderate hyperemia and edema of
nasopharyngeal mucous membrane, small hemorrhages in the mucous membranes and internal
organs; dramatic dystrophic alterations in liver and myocardium; acute necrotic nephrosis; massive
hemorrhages in the adrenal glands. What disease are these alterations the most typical for?

A. * Meningococcal infection

B. Scarlet fever

C. Diphtheria

D. Measles

E. Epidemic typhus

212. Autopsy of a 12-year-old girl revealed: multiple cutaneous hemmorhages (mostly into the skin of
buttocks, lower extremities), serous and mucous memrane hemmorhages, cerebral hemmorhages.
Adrenal glands show focal necrosis and massive hemmorhages; ki-dneys show necrotic nephrosis,
suppurati-ve arthritis, iridocyclitis, vasculitis. What is the most probable diagnosis?

A. * Meningococcemia

B. Epidemic typhus

C. Periarteritis nodosa

D. Systemic lupus erythematosus

E. Radiation sickness

213. Autopsy of a 17 year old girl who died from pulmonary failure revealed a small area of caseous
necrosis in the inferior lobe of the right lung, and occurences of caseous necrosis in the
bronchopulmonary, bronchial and bi-furcational lymph nodes. What is the most probable postmortem
diagnosis?

A. * Primary tuberculosis

B. Hematogenous progression of primary tuberculosis

C. Hematogenous tuberculosis with predominant lung affection

D. Tuberculoma

E. Caseous pneumonia under secondary tuberculosis

214. Autopsy of a 28-year-old patient, who had been suffering from rheumatism and died of heart failure,
revealed pancarditis. Histological investigation of myocardium of the left ventricle posterior wall and
interventricular septum detected perivascular cellular focal infiltrates composed of macrophages and
creating palisade structures surrounding areas of fibrinoid necrosis. Determine the type of
myocarditis:

A. * Granulomatous

B. Diffuse interstitial productive


C. Diffuse interstitial exudative

D. Focal interstitial exudative

E. -

215. Autopsy of a 40-year-old woman, who died of cerebral hemorrhage during hypertensic crisis,
revealed: upper- body obesity, hypertrichosis, hirsutism, stretchmarks on the skin of thighs and
abdomen. Pituitary basophil adenoma is detected in the anterior lobe. What di- agnosis is the most
likely?

A. * Cushing’s disease

B. Essential hypertension

C. Alimentary obesity

D. Simmonds’ disease

E. Hypothalamic obesity

216. Autopsy of a 46-year-old man revealed multiple brown-and-green layers and hemmorhages on the
mucous membrane of rectum and sigmoid colon; slime and some blood in colon lumen;
hi-stologically - fibrinous colitis. In course of bacteriological analysis of colon contents S.Sonne
were found. What is the most probable diagnosis?

A. * Dysentery

B. Cholera

C. Salmonellosis

D. Yersiniosis

E. Crohn’s disease

217. Autopsy of a 48 y.o. man revealed a round formation 5 cm in diameter with clear-cut outlines in the
region of the 1st segment of his right lung. This formation was encircled with a thin layer of
connecti-ve tissue full of white brittle masses. Make a diagnosis of the secondary tuberculosis form:

A. * Tuberculoma

B. Caseous pneumonia

C. Acute cavernous tuberculosis

D. Acute focal tuberculosis

E. Fibrous cavernous tuberculosis

218. Autopsy of a 49-year-old woman who died from chronic renal insufficiency, revealed: kidneys were
dense, reduced, multicoloured, with hemorrhagic areas. Microscopic examination revealed some
hematoxylin bodies in the nuclei of the renal tubule epithelium; "wire-loop" thickening of the
glomerular capillary basement membrane; here and there in the capillaries some hyaline thrombi and
foci of fibrinoid necrosis were present. What is the most likely diagnosis?

A. * Systemic lupus erythematosus

B. Rheumatism

C. Arteriosclerotic pneumosclerosis

D. Amyloidosis

E. Atherosclerotic nephrosclerosis

219. Autopsy of a 5 year old child revealed in the area of vermis of cerebellum a soft greyish-pink node 2
cm in diameter with areas of haemorrhage. Histologi-cally this tumour consisted of atypical
monomorphous small roundish cells wi-th big polymorphous nuclei. What tumour is it?

A. * Medulloblastoma

B. Meningioma

C. Glioblastoma

D. Astrocytoma

E. Oligodendroglioma

220. Autopsy of a 50-year-old male who had tuberculosis revealed a dense gray-white nidus in form of a
nodule 2 cm in diameter in the subpleural portion of the upper right lobe. The pleura in this region
was thickened, in the pleural cavity there was a small amount of serous hemorrhagic fluid.
Histological study of the region revealed some glandular structures wi-th signs of cellular atypia and
abnormal mitoses, which were found within the fibrous connective tissue. What other pathology had
developed in the lungs?

A. * Adenocarcinoma

B. Squamous cell carcinoma

C. Adenoma

D. Fibrosarcoma

E. Fibroma

221. Autopsy of a 50-year-old man revealed the following changes: his right lung was moderately
compact in all parts, the dissected tissue was found to be airless, consolidated, dryish. Visceral pleura
had greyish-brown layers of fibrin. What is the most likely diagnosis?

A. * Croupous pneumonia

B. Tuberculosis
C. Bronchopneumonia

D. Interstitial pneumonia

E. Pneumofibrosis

222. Autopsy of a 56 y.o. man revealed in the right temporal part of brain a big focus of softened grey
matter that was semi-liquid and light grey. Arteries of cerebral tela contain multiple whitish-yellow
thi-ckenings of intima that abruptly narrow the lumen. What is your diagnosis?

A. * Ischemic stroke

B. Brain abscess

C. Hemorrhage

D. Hemorrhagic infarction

E. Brain edema

223. Autopsy of a 58 y.o. man revealed that bicuspid valve was deformed, thickened and unclosed.
Microscopically: foci of collagen fibrilla are eosinophilic, react positively to fibrin. The most
probably it is:

A. * Fibrinoid swelling

B. Fibrinous inflammation

C. Mucoid swelling

D. Hyalinosis

E. Amyloidosis

224. Autopsy of a 58 year old man revealed that bicuspid valve was deformed, thi-ckened and unclosed.
Microscopically: foci of collagen fibrilla are eosinophi-lic, react positively to fibrin. The most
probably it is:

A. * Fibrinoid swelling

B. Fibrinous inflammation

C. Mucoid swelling

D. Hyalinosis

E. Amyloidosis

225. Autopsy of a 5-year-old child revealed in the area of the vermis of cerebellum a soft grayish-pink
node 2 cm in diameter with blurred margins and areas of haemorrhage. Histologically this tumour
consisted of atypical monomorphous small round cells with large polymorphous nuclei. What tumour
is it?

A. * Medulloblastoma

B. Meningioma

C. Glioblastoma

D. Astrocytoma

E. Oligodendroglioma

226. Autopsy of a 62-year-old woman revealed a dense well-circumscribed node of 6 cm in diameter in


the cranial cavity. The node was attached to the dura mater and histologically consisted of clusters
and micro-concentric structures of endotheli-al cells, psammoma bodies. What kind of tumor was
found at autopsy?

A. * Meningioma

B. Glioblastoma

C. Medulloblastoma

D. Melanoma

E. Cancer metastasis

227. Autopsy of a 73-year-old man who had been suffering from the coronary heart disease along with
cardiac insufficiency for a long time revealed: nutmeg liver, brown induration of lungs, cyanotic
induration of kidneys and spleen. What kind of circulation disorder was the cause of such effects?

A. * General chronic venous congestion

B. Arterial hyperemia

C. General acute venous congestion

D. Acute anemia

E. Chronic anemia

228. Autopsy of a 75 year old patient who had been suffering from dissemi-nated atherosclerosis and died
under chronic cardiac failure revealed constricti-on and deformation of coronary arteries, tuberous
intima whose section appeared to be white and petrosal. Specify the stage of atherosclerosis
morphogenesis:

A. * Atherocalcinosis

B. Lipoidosis

C. Liposclerosis
D. Bilipid

E. Atheromatosis

229. Autopsy of a 78-year-old patient revealed that retroperitoneal tissue was soaked with blood, the
abdominal aorta had a sacciform protrusion including a defect with irregular edges. The wall of the
aorta was here and there of stone-like density. This is the complication of the following disease:

A. * Atherosclerosis

B. Essential hypertension

C. Systemic vasculitis

D. Visceral syphilis

E. Marfan syndrome

230. Autopsy of a 9-year-old child shows numerous irregular defects of varying depth with uneven
margins and gray-white films tightly attached to the underlying tissue on the rectal mucosa of the
body. What disease can be suspected?

A. * Dysentery

B. Salmonellosis

C. Cholera

D. Typhoid fever

E. Amebiasis

231. Autopsy of a body revealed bone marrow hyperplasia of tubular and flat bones (pyoid marrow),
splenomegaly (6 kg) and hepatomegaly (5 kg), enlargement of all lymph node groups. What disease
are the identified changes typical of?

A. * Chronic myelogenous leukemia

B. Chronic lymphocytic leukemia

C. Multiple myeloma

D. Polycythemia vera

E. Hodgkin’s disease

232. Autopsy of a man ill with severe hypothyroidism revealed that connective tissue, organ stroma,
adipose and cartilaginous tissues were swollen, semitransparent, mucus-lik E. Microscopic
examination of tissues revealed stellate cells having processes with mucus between them. What type
of dystrophy is it?

A. * Stromal-vascular carbohydrate
B. Parenchymatous adipose

C. Parenchymatous proteinaceous

D. Stromal-vascular adipose

E. Stromal-vascular proteinaceous

233. Autopsy of a man who died from burn disease revealed brain edema, li-ver enlargement as well as
enlargement of kidneys with wide light-grey cortical layer and plethoric medullary area.
Mi-croscopic examination revealed necrosis of tubules of main segments along with destruction of
basal membranes, intersti-cium edema with leukocytic infiltrati-on and haemorrhages. What is the
most probable postmortem diagnosis?

A. * Necrotic nephrosis

B. Tubulointerstitial nephritis

C. Pyelonephritis

D. Gouty kidney

E. Myeloma kidney

234. Autopsy of a man who died from chronic cardiacvascular collapse revealed "tiger heart". Sidewards
of endocardium a yellowish-white banding can be seen; myocardium is dull, dark-yellow. What
process caused this pathology?

A. * Fatty parenchymatous degeneration

B. Carbohydrate degeneration

C. Hyaline degeneration

D. Fatty vascular-stromal degeneration

E. Amyloidosis

235. Autopsy of a man who died from ethylene glycol poisoning revealed that his kidneys are a little bit
enlarged, edematic; their capsule can be easily removed. Cortical substance is broad and light-grey.
Medullary substance is dark-red. What pathology had this man?

A. * Necrotic nephrosis

B. Acute pyelonephritis

C. Acute glomerulonephritis

D. Acute tubular-interstitial nephritis

E. Lipoid nephrosis
236. Autopsy of a man who died from influenza revealed that his heart was sli-ghtly enlarged, pastous,
myocardium was dull and had specks. Microscopical exami-nation of myocardium revealed signs of
parenchymatous adipose and hydropic dystrophy; stroma was edematic with poor macrophagal and
lymphocytic infiltrati-on, vessels were plethoric; perivascular analysis revealed petechial
hemorrhages. What type of myocarditis was developed in this case?

A. * Serous diffuse

B. Interstitial proliferative

C. Serous focal

D. Purulent

E. Granulomatous

237. Autopsy of a man who died from influenza revealed that the heart was slightly enlarged and pastose.
The surface of the incision of myocardium appeared to be pale, with specks. Microscopic
examination revealed signs of parenchymatous adipose and hydropic degeneration, edematic stroma
with scant lymphocytic and macrophage infiltration; plethoric vessels; perivascular petechial
hemorrhages. What type of myocarditis is it?

A. * Serous diffuse

B. Interstitial proliferative

C. Serous focal

D. Purulent

E. Granulomatous

238. Autopsy of a man who had been working as a miner for many years and died from cardiopulmonary
decompensation revealed that his lungs were airless, sclerosed, their apexex had emphysematous
changes, the lung surface was greyish-black, the incised lung tissue was coal-black. What disease
caused death?

A. * Anthracosis

B. Silicosis

C. Aluminosis

D. Asbestosis

E. Talcosis

239. Autopsy of a man who had tuberculosis revealed a 3x2 cm large cavi-ty in the superior lobe of the
right lung. The cavity was interconnected with a bronchus, its wall was dense and consi-sted of three
layers: the internal layer was pyogenic, the middle layer was made by tuberculous granulation tissue
and the external one was made by connective ti-ssue. What is the most likely diagnosis?
A. * Fibrous cavernous tuberculosis

B. Fibrous focal tuberculosis

C. Tuberculoma

D. Acute focal tuberculosis

E. Acute cavernous tuberculosis

240. Autopsy of a man with a malignant stomach tumour who had died from cancer intoxication revealed
in the inferior and posterior lung fields some dense, grayish-red irregular foci protruding above the
section surface. Microscopic examination revealed exudates containing a large amount of neutrophils
in the lumen and walls of small bronchi and alveoli. Such pulmonary alterations indicate the
following disease:

A. * Acute purulent bronchopneumonia

B. Acute bronchitis

C. Croupous pneumonia

D. Intermittent pneumonia

E. Acute serous bronchopneumonia

241. Autopsy of a man with tuberculosis revealed a 3x2 cm large cavity in the superior lobe of the right
lung. The cavity was interconnected with a bronchus, its wall was dense and consisted of three
layers: the internal layer was pyogenic, the middle layer was made of tuberculous granulation tissue
and the external one was made of connective tissue. What is the most likely diagnosis?

A. * Fibrous cavernous tuberculosis

B. Fibrous focal tuberculosis

C. Tuberculoma

D. Acute focal tuberculosis

E. Acute cavernous tuberculosis

242. Autopsy of a man, who had been suffering from the multiple bronchiectasis for 5 years and died from
chronic renal insufficiency, revealed that kidneys were dense and enlarged, with thickened cortical
layer of white color with greasy luster. What renal disease might be suspected?

A. * Secondary amyloidosis

B. Glomerulonephritis

C. Chronic pyelonephritis

D. Necrotic nephrosis
E. -

243. Autopsy of a man, who served on a nuclear submarine, revealed the followi- ng pathologies: bone
marrow atrophy (panmyelophthisis), anemia, leukopenia, thrombocytopenia, lymphocytes disi-
ntegration in the lymph nodes, spleen, gastrointestinal lymphatic system, and hemorrhages into the
adrenal glands. What disease had developed in this case?

A. * Acute radiation sickness

B. Decompression sickness

C. Acute leukemia

D. Acute anemia

E. Vibration disease

244. Autopsy of a Middle-Eastern woman, who had been suffering from wasting fever for a long time,
revealed enlarged blackened liver and spleen. Bone marrow was hyperplastic and black-colored as
well. Cerebral cortex was smoky grey. What disease is it characteristic of?

A. * Malaria

B. AIDS

C. Epidemic typhus

D. Sepsis

E. Hepatitis

245. Autopsy of a patient who suffered from croupous pneumonia and died from pneumococcal sepsis
revealed 900 ml of turbid greenish-yellow liquid in the ri-ght pleural cavity. Pleural leaves are dull,
plephoric. Name the clinicopathological form of inflammation in the pleural cavity:

A. * Empyema

B. Fibrinous inflammation

C. Phlegmon

D. Chronic abscess

E. Acute abscess

246. Autopsy of a woman with cerebral atherosclerosis revealed in the left cerebral hemisphere a certain
focus that is presented by flabby, anhistic, greyish and yellowish tissue with indistinct edges. What
pathological process is the case?

A. * Ischemic stroke

B. Multifocal tumor growth with cystic degeneration


C. Multiple foci of fresh and old cerebral hemorrhage

D. Focal encephalitis

E. Senile encephalopathy

247. Autopsy of an 8-month-old boy, who died of severe pneumonia complicated with sepsis, revealed
absence of thymus. Lymph nodes have no lymphoid follicles and corti- cal substance. In the spleen
the follicles are decreased in size and have no light centers. What is the cause of such changes?

A. * Thymus agenesis

B. Thymus aplasia

C. Thymus atrophy

D. Thymus hypoplasia

E. Accidental thymic involution

248. Autopsy of the 58-year-old man hads revealed that mitral valve is deformed, thi-ckened, does not
totally close. Microscopi-cally: centers of collagen fibers are eosi-nophilic, have positive fibrin
reaction. The most probable diagnosis is:

A. * Fibrinoid swelling

B. Fibrinoid inflammation

C. Mucoid swelling

D. Hyalinosis

E. Amyloidosis

249. Autopsy of the dead patient who died from pulmonary edema revealed a large yellow-grey nidus in
the myocardium, and a fresh thrombus in the coronary artery. What is the most likely diagnosis?

A. * Myocardial infarction

B. Cardiosclerosis

C. Myocarditis

D. Amyloidosis

E. Cardiomyopathy

250. Autopsy of the patient revealed bone marrow hyperplasia of tubular and flat bones (pyoid marrow),
splenomegaly (6 kg) and hepatomegaly (5 kg), enlargement of all lymph node groups. What disease
are the identified changes typical for?
A. * Chronic myelogenous leukemia

B. Chronic lymphocytic leukemia

C. Multiple myeloma

D. Polycythemia vera

E. Hodgkin’s disease

251. Autopsy revealed that right lung is enlarged, solid, there are fibrin layers on the pleura. Lung tissue is
light grey color on incision with muddy liqued exudates. What lung disease are these symptoms
typical for?

A. * Croupous pneumonia

B. Bronchopneumonia

C. Interstitial pneumonia

D. Pulmonary gangrene

E. Fibrosing alveolitis

252. Blood analysis of a patient wi-th jaundice reveals conjugated bilirubi-nemia, increased concentration
of bile aci-ds. There is no stercobilinogen in urine. What type of jaundice is it?

A. * Obstructive jaundice

B. Hepatocellular jaundice

C. Parenchymatous jaundice

D. Hemolytic jaundice

E. Cythemolytic jaundice

253. Blood count of an athlete is as follows: erythrocytes - 5, 5 · 1012/l, Hb-g/l, leukocytes - 7 · 109/l,
neutrophils - 64%, basophils - 0,5%, eosinophils - 0,5%, monocytes - 8%, lymphocytes - 27%. First
of all, such results indicate the stimulation of:

A. * Erythropoiesis

B. Leukopoiesis

C. Lymphopoiesis

D. Granulocytopoiesis

E. Immunogenesis
254. Brain autopsy revealed an edema, hyperemia, and small hemorrhages in the medulla oblongata.
Microscopically chromatolysis, hydropia and nerve cell necrosis are observed; within the cytoplasm
of hippocampal nerve cells there are eosi- nophilic structures (Negri bodies) detected. What
diagnosis corresponds with the descri- bed morphological signs?

A. * Rabies

B. Meningococcal meningitis

C. Encephalitis

D. Encephalomyelitis

E. Brucellosis

255. Chronic inflammation and transformati-on of the one-layer ciliated epitheli-um into multiple-layers
flat epithelium was revealed in the thickened mucous membrane of the bronchus bioptate of the
patient with smoke abuse. Which of the processes is the most likely?

A. * Metaplasia

B. Hyperplasia of the epithelium

C. Squamous cancer

D. Leucoplacia

E. Epithelium hypertrophy

256. Colonoscopy of a patient ill wi-th dysentery revealed that mucous membrane of his large intestine is
hyperemic, edematic, its surface was covered with grey-and-green coats. Name the morphological
form of dysenteric collitis:

A. * Fibrinous

B. Catarrhal

C. Ulcerous

D. Purulent

E. Necrotic

257. Development of both immune and allergic reactions is based upon the same mechanisms of immune
system response to an antigen. What is the main difference between immune and allergic reactions?

A. * Development of tissue lesion

B. Amount of released antigen

C. Antigen structure

D. Routes by which antigens are delivered into the body


E. Hereditary predisposition

258. Diabetic nephropathy with uremia has developed in a patient with pancreatic diabetes. The velocity
of glomerular filtration is 9 ml/min. What mechanism of a decrease in glomerular filtration velocity
and chronic renal fai-lure development is most likely in the case of this patient?

A. * Reduction of active nephron mass

B. Decrease in systemic arterial pressure

C. Obstruction of nephron tubules with hyaline casts

D. Tissue acidosis

E. Arteriolar spasm

259. Diagnostic scraping was performed to the woman with dysfunctional uteri-ne bleeding. Multiple
convoluted glands, ganglially dilated lumens of some glands were revealed histologically in the
scrape. Name the type of general pathological process in endometry:

A. * Hyperplasia glandulocystica

B. Atrophy

C. Metaplasia

D. Displasia

E. Hypertrophic excrescence

260. During autopsy of a 9-month-old girl’s body, who died due to severe pneumonia complicated with
sepsis, lack of thymus is observed. In the lymph nodes the lymphoid follicles and cortical substance
are absent; follicles of spleen are reduced in size with no light zones and plasma cells. What is the
cause of such structural changes?

A. * Thymus agenesis

B. Accidental involution of thymus

C. Thymus hypoplasia

D. Thymus atrophy

E. Thymus aplasia

261. During autopsy of a man, who died of acute transmural cardiac infarction, the following has been
detected on the pericardium surface: fibrous whitish-brown deposit connecting parietal and visceral
pericardial layers. What kind of inflammation occurred in the pericardium?

A. * Croupous
B. Diphtheritic

C. Serous

D. Suppurative

E. Granulomatous

262. During autopsy of a man, who had been suffering from mitral stenosis, the lungs are revealed to be
dense and brown-colored. What pathologic process had occurred in the lungs?

A. * Hemosiderosis

B. Hemochromatosis

C. Jaundice

D. Hemomelanosis

E. Lipofuscinosis

263. During autopsy the following has been revealed: the meninges of the upper cerebral hemispheres are
extremely plethoric, of yellow-green color and are soaked with purulent effluent. What kind of
meni-ngitis is characterised by such clinical presentations?

A. * Meningococcal meningitis

B. Tuberculous meningitis

C. Grippal meningitis

D. Anthrax-induced

E. Epidemic typhus-induced

264. During blood transfusion a patient has developed intravascular erythrocyte hemolysis. What kind of
hypersensitivity does the patient have?

A. * II type (antibody-dependent)

B. I type (anaphylactic)

C. III type (immune complex)

D. IV type (cellular cytotoxicity)

E. IV type (granulomatosis)

265. During hystological analysis of the lymph node situated in the posterior neck triangle of an
18-year-old patient a morphologist detected a cluster of cells including the following: isolated
multinucleate Reed-Sternberg cells, large and small Hodgkin’s cells and numerous lymphocytes,
isolated plasma cells, eosi-nophils. What disease has developed in the patient?
A. * Lymphogranulomatosis

B. Nodular lymphoma

C. Burkitt’s lymphoma

D. Lymphocytic lymphoma

E. Chronic lymphocytic leukemia

266. During influenza epidemic 40% of pupils who didn’t go in for sports were affected by the disease,
and among the pupils who regularly did physical exercises this index was only 20%. What
adaptati-ve mechanisms determined such a low si-ckness rate of pupils participating in the sports?

A. * Cross adaptation

B. Specific adaptation

C. Physiological adaptation

D. Biochemical adaptation

E. Genetic adaptation

267. During intravenous saline transfusion a patient’s condition deteriorated drasti-cally, and the patient
died from asphyxiation. Autopsy revealed acute venous congestion of internal organs with sharp
right heart dilatation. When the right ventricle was punctured underwater, the bubbles escaped. What
pathological process occurred in the patient?

A. * Air embolism

B. Gaseous embolism

C. Adipose embolism

D. Tissue embolism

E. Thromboembolism

268. During pathomorphological renal investigation of a patient, who for a long time had been suffering
from osteomyelitis and died of progressing renal failure, the following was revealed: deposits of
homogeneous eosinophilic masses in glomerularmesangium, arterial and arteriolar walls, and stroma,
which colored red when stained with Congo red. What pathological process is this?

A. * Amyloidosis

B. Mucoid swelling

C. Calcinosis

D. Carbohydrate degeneration
E. Hyalinosis

269. During postembryonal haemopoiesis in the red bone marrow the cells of one of the cellular differons
demonstrate a gradual decrease in cytoplasmic basophi-lia as well as an increase in oxyphilia, the
nucleus is being forced out. Such morphological changes are typical for the following haemopoiesis
type:

A. * Erythropoiesis

B. Lymphopoiesis

C. Neutrophil cytopoiesis

D. Eosinophil cytopoiesis

E. Basophil cytopoiesis

270. During surgical manipulations a pati-ent has been given novocaine injection for anesthesia. 10
minutes later the patient developed paleness, dyspnea, hypotensi-on. What type of allergic reaction is
it?

A. * Anaphylactic immune reaction

B. Cellulotoxic immune reaction

C. Aggregate immune reaction

D. Stimulating immune reaction

E. Cell-mediated immune reaction

271. During the intravenous transfusi-on of the saline the patient’s conditi-on deteriorated dramatically,
and the patient died from asphyxiation. Autopsy revealed acute venous congestion of internal organs
with the dramatic ri-ght heart dilatation. When the right ventricle was punctured underwater, the
bubbles escaped. What pathological process occurred in the patient?

A. * Air embolism

B. Gaseous embolism

C. Adipose embolism

D. Tissue embolism

E. Thromboembolism

272. ECG of a 44-year-old patient shows signs of hypertrophy of both ventricles and the right atrium. The
patient was di-agnosed with the tricuspid valve insuffici-ency. What pathogenetic variant of cardiac
dysfunction is usually observed in case of such insufficiency?

A. * Heart overload by volume


B. Heart overload by resistance

C. Primary myocardial insufficiency

D. Coronary insufficiency

E. Cardiac tamponade

273. Examination of a 2-year-old child revealed physical developmental lag, the child often has
pneumonias. The child was diagnosed with nonclosure of ductus arteriosus. Haemodynamics
disorder was caused by the intercommunication of the following vessels:

A. * Aorta and pulmonary trunk

B. Pulmonary trunk and pulmonary veins

C. Superior cava and aorta

D. Superior cava and pulmonary trunk

E. Aorta and pulmonary veins

274. Examination of a 42 year old patient revealed a tumour of adenohypophysis. Objectively: the
patient’s weight is 117 kg, he has moon-like hyperemic face, red-blue striae of skin distension on his
belly.Osteoporosis and muscle dystrophy are present. AP is 210/140 mm Hg. What is the most
probable diagnosis?

A. * Cushing’s disease

B. Cushing’s syndrome

C. Conn’s disease

D. Diabetes mellitus

E. Essential hypertension

275. Examination of a 55 year old woman revealed under the skin of submandibular area a movable
slowly growing pasty formation with distinct borders 1,0x0,7 cm larg. Histological examination
revealed lipocytes that form segments of diffrent forms and sizes separated from each other by thin
layers of connective tissue with vessels. What is the most probable diagnosis?

A. * Lipoma

B. Liposarcoma

C. Fibrosarcoma

D. Angioma

E. Fibroma
276. Examination of a 55 year old woman revealed under the skin of submandi-bular area a movable
slowly growing pasty formation with distinct borders 1,0x0,7 cm large. Histological examination
revealed lipocytes that form segments of diffrent forms and sizes separated from each other by thin
layers of connective tissue with vessels. What is the most probable di-agnosis?

A. * Lipoma

B. Fibroma

C. Angioma

D. Liposarcoma

E. Fibrosarcoma

277. Examination of a 66 year old patient revealed a lytic tumour in the locus of pathological rib fracture.
Hi-stologically this tumour consists of atypi-cal plasmoblasts. Further examinati-on revealed
osteoporosis in the bones of vertebral column and pelvis. These changes are typical for:

A. * Myelomatosis

B. Tuberculous osteomyelitis

C. Ewing’s osteosarcoma

D. Neuroblastoma

E. Metastatic lung cancer

278. Examination of a bronchial tissue sample revealed atrophy of mucous membrane, cystic degeneration
of glands, focal metaplastic changes of lining prismatic epithelial cells into multilayer squamous
cells; increase in goblet cell number; in some parts of bronchial wall and especially in the mucous
membrane there was marked cellular inflammatory infiltration and growth of granulation tissue
bulging into the bronchial lumen in form of a polyp. What is the most likely diagnosis?

A. * Chronic bronchitis

B. Lobar pneumonia

C. Acute bronchitis

D. Bronchopneumonia

E. Interstitial pneumonia

279. Examination of a newborn boy’s genitals revealed a cleft of urethra that opens on the inferior surface
of his penis. What developmental anomaly is it?

A. * Hypospadia

B. Hermaphroditism

C. Epispadia
D. Monorchism

E. Cryptorchism

280. Examination of a patient revealed a dense, movable skin tumour that is standing out distinctly from
the surrounding tissues. Its section is found to be white and composed of fibrous tissue. Microscopic
examination revealed interlacing collagen fibers and few cells. What tumour is it?

A. * Fibroma

B. Myoma

C. Histiocytoma

D. Dermatofibroma

E. Desmoid

281. Examination of a patient revealed overgrowth of facial bones and soft tissues, tongue enlargement,
wide interdental spaces in the enlarged dental arch. What changes of the hormonal secretion are the
most likely?

A. * Hypersecretion of the somatotropic hormone

B. Hyposecretion of the somatotropic hormone

C. Hypersecretion of insulin

D. Hyposecretion of thyroxin

E. Hyposecretion of insulin

282. Examination of a pregnant woman having Rh-negative blood revealed hi-gh level of antierythrocytic
antibodies. For its reduction she was implanted wi-th her husband’s Rh-positive skin graft. The graft
was rejected in two weeks. Its microscopic examination revealed ci-rculatory disturbance, edema and
cellular infiltration with lymphocytes, neutrophils and macrophages predominance. What is the most
likely pathology?

A. * Graft immunity

B. Immediate hypersensitivity

C. Delayed-type hypersensitivity

D. Granulomatous inflammation

E. Interstitial inflammation

283. Examination of a young woman revealed a node-like, soft and elastic homogenous tumour of
pinkish-white colour along the acoustic nerv E. The tumour contains cell bundles with oval nuclei.
Cellular fibrous bundles form rhythmic structures made up by parallel rows of regularly oriented
cells arranged in form of a palisade with cell-free homogenous zone (Verocay bodies) between them.
What tumour is it?

A. * Neurinoma

B. Neuroblastoma

C. Ganglioneurinoma

D. Ganglioneuroblastoma

E. Malignant neurinoma

284. Examination of a young woman revealed a tumour up to 3 cm in diameter in form of a knot localized
along the acoustic nerve. The tumour is homogenous, soft and elastic, of pink-and-white colour.
Microscopically the tumour contains clusters of cells with oval nuclei. Fibrous cell clusters form
regular structures made up by parallel rows of regularly oriented cells arranged in form of a palisade.
Zones between the rows of cells are acellular and homogenous (Verocai bodies). What tumour is it?

A. * Neurinoma

B. Malignant neurinoma

C. Ganglioneuroma

D. Neuroblastoma

E. Ganglioneuroblastoma

285. Examination of an ovary specimen stained by hematoxylin-eosine revealed a follicle in which


follicular epithelium consisted of 1-2 layers of cubic cells. There was also a bright red membrane
around the ovocyte. What follicle is it?

A. * Primary

B. Primordial

C. Secondary

D. Mature

E. Atretic

286. Examination of coronary arteries revealed atherosclerotic calcific plaques that close vessel lumen by
1/3. The muscle has multiple whitish layers of connecti-ve tissue. What process was revealed in
myocardium?

A. * Diffuse cardiosclerosis

B. Tiger heart

C. Postinfarction cardiosclerosis
D. Myocarditis

E. Myocardium infarction

287. Examination of initial molecular structure revealed substitution of the glutamic acid by valine. What
inherited pathology is it typical for?

A. * Sickle-cell anemia

B. Thalassemia

C. Minkowsky-Shauffard disease

D. Favism

E. Hemoglobinosis

288. Examination of patients with peri-odontitis revealed the interdependence between the rate of
affection of peri-odontal tissues and the amount of lysozymes in saliva and gingival liquid. These
results can be obtained during studying the following protection system of an organism:

A. * Non-specific resistance

B. Humoral immunity

C. Cellular immunity

D. Autoresponsiveness

E. Tolerance

289. Examination of the anterior abdomi-nal wall of a pregnant woman revealed a tumour-like formation
that arose on the spot of a tumour that was removed two years ago. The neoplasm was well-defined,
dense, 2?1 cm large. Histological examination revealed that the tumour was composed of
differentiated connective ti-ssue with prevailing collagen fibres. What tumour might be suspected?

A. * Desmoid

B. Lipoma

C. Fibrosarcoma

D. Hibernoma

E. Leiomyoma

290. Examination of the anterior abdominal wall of a pregnant woman revealed a tumour-like formation
that arose on the spot of a tumour that was removed two years ago. The neoplasm was well-defined,
dense, 2o1 cm large. Histological examination revealed that the tumour was composed of
differentiated connective tissue with prevailing collagen fibers. What tumour might be suspected?

A. * Desmoid
B. Lipoma

C. Fibrosarcoma

D. Hibernoma

E. Leiomyoma

291. Examination of the coronary arteries revealed atherosclerotic plaques with calci- nosis that close the
arterial opening by 1/3. In the muscle there are numerous whiti- sh layers of connective tissue. Name
the process detected in the myocardium:

A. * Diffuse cardiosclerosis

B. Tiger heart

C. Postinfarction cardiosclerosis

D. Myocarditis

E. Myocardial infarction

292. Examination of the removed stomach revealed a deep roundish defect with regular edges at the lesser
curvature of the antrum. The defect reached the muscular tunic and was 1,5 cm in diameter. Within
the defect floor there was a translucent dense area resembling of a hyaline carti-lage. What process
had developed in the floor of the stomach defect?

A. * Local hyalinosis

B. Amyloidosis

C. Mucoid swelling

D. Fibrinoid alterations

E. General hyalinosis

293. Extensive thromboembolic infarcti-on of the left cerebral hemispheres, large septic spleen,
immunocomplex glomerulonephritis, ulcers on the edges of the aortic valves, covered with polypous
thrombus with colonies of staphylococcus were revealed on autopsy of the young man who died in
coma. What disease caused cerebral thromboemboly?

A. * Septic bacterial endocarditis

B. Septicemia

C. Acute rheumatic valvulitis

D. Septicopyemia

E. Rheumatic thromboendocarditis
294. For a long time a 49-year-old woman had suffered from glomerulonephritis which caused death. The
autopsy revealed that the size of her kidneys was 7 ? 3 ? 2,5 cm, weight 65,0 g, they were dense and
small-grained. Microscopically: fibrinogenous inflammation of serous and mucous membranes,
dystrophic changes of parenchymatous organs, brain edema. What complication can cause such
changes of serous membranes and inner organs?

A. * Uraemia

B. Anemia

C. Sepsis

D. DIC-syndrome

E. Thrombopenia

295. For the purpose of myocardium infarction treatment a patient was injected with embryonal stem cells
derived from this very patient by means of therapeutic cloning . What transplantation type is it?

A. * Autotransplantation

B. Isotransplantation

C. Heterotransplantation

D. Allotransplantation

E. Xenotransplantation

296. Granulomas containing lymphocytes and macrophages were detected during analysis of skin biopsy
material. Among macrophages there are large cells with fat inclusions, which contain
microorganisms in spheric packages (Virchow’s cells). The following disease is based on the
described type of hypersensitivity:

A. * Leprosy

B. Syphilis

C. Tuberculosis

D. Rhinoscleroma

E. Epidemic typhus

297. Gynecological examination of the uterine cervix in a 30-year-old woman revealed some bright-red
lustrous spots that easily bleed when touched. Biopsy showed that a part of the uterine cervix was
covered with cylindrical epithelium with papillary outgrowths; in the depth of tissue the growth of
glands was present. What pathology of the uterine cervix was revealed?

A. * Pseudoerosion

B. True erosion
C. Endocervicitis

D. Glandular hyperplasia

E. Leukoplakia

298. Histologic analysis of uterus mucous membrane revealed twisting glands, serrated and spinned, they
were extended by stroma growth with proliferation of its cells. Formulate a diagnosis:

A. * Glandular hyperplasia of endometrium

B. Acute endometritis

C. Leiomyoma

D. Cystic mole

E. Placental polyp

299. Histologic preparation stained wi- th orcein demonstrates from 40 to 60 fenestrated elastic
membranes within the middle coat of the vessel. Name this vessel:

A. * Elastic artery

B. Muscular artery

C. Mixed type artery

D. Muscular vein

E. Nonmuscular vein

300. Histological examination of a 40 y.o. man’s thymus revealed decreased share of parenchymatous
gland elements, increased share of adipose and loose connective tissue, its enrichment with thymus
bodies. The organ’s mass was unchanged. What phenomenon is it?

A. * Age involution

B. Accidental involution

C. Hypotrophy

D. Dystrophy

E. Atrophy

301. Histological examination of a 40 year old man’s thymus revealed reduced share of parenchymatous
elements, increased share of adipose and loose connective tissue, its enrichment with thymus bodies.
The organ’s mass was unchanged. What is this phenomenon called?

A. * Age involution
B. Accidental involution

C. Hypotrophy

D. Dystrophy

E. Atrophy

302. Histological examination of a skin tissue sampling revealed granulomas consisting of macrophagal
nodules with lymphocytes and plasmatic cells. There are also some big macrophages with fatty
vacuoles containing causative agents of a disease packed up in form of spheres (Virchow's cells).
Granulation tissue is well vascularize D. What disease is this granuloma typical for?

A. * Lepra

B. Syphilis

C. Rhinoscleroma

D. Tuberculosis

E. Glanders

303. Histological examination of bi-opsy samples taken from the thickened edges of a gastric ulcer
revealed small clusters of small, markedly atypical hyperchromatic epithelial cells that were localized
in the overdeveloped stroma. Specify the tumor:

A. * Scirrhous undifferentiated carci-noma

B. Medullary carcinoma

C. Adenocarcinoma

D. Undifferentiated sarcoma

E. Adenoma

304. Histological examination of the bi-opsy material obtained from the lower third of the esophagus of a
57-year-old male with the symptoms of continuous reflux revealed the change of the strati-fied
squamous epithelium to the single-layer columnar glandular epithelium wi-th signs of mucus
production. Specify the pathological process in the mucous membrane:

A. * Metaplasia

B. Hyperplasia

C. Hypertrophy

D. Organization

E. Regeneration
305. Histological examination of the removed skin neoplasm revealed clusters and cords of atypical cells
of stratified squamous epithelium, growing into the underlying tissue. What diagnosis can be
assumed?

A. * Non-keratinizing squamous cell carci-noma

B. Keratinizing squamous cell carcinoma

C. Carcinoma in situ

D. Papilloma

E. Adenoma

306. Histological investigation of the uterine scrape of the 45-year-old woman with di- sturbed ovarian
menstrual cycle revealed increased number of endometrial glands, some of which are serrated, while
others are dilated and cyst-like. Make the diagnosis:

A. * Endometrial cystic glandular hyperplasia

B. Placental polyp

C. Atypical endometrial hyperplasia

D. Glandular endometrial polyp

E. Endometrial adenocarcinoma

307. Histological specimen of the ovary shows large hollow structures. Primary oocyte within these
structures is surrounded with transparent membrane and radiati- ng crown and is situated in the
cumulus oophorus, the wall is made of follicular cell layer and theca. What ovarian structure can be
characterized by these morphological features?

A. * Mature (tertiary) follicle

B. Primordial follicle

C. Primary follicle

D. Corpus luteum

E. Corpus atreticum

308. In a body of a 37-year-old woman, who died with signs of pulmonary edema, there was detected
acute deformation of the aortic valve: it is shortened, thickened, ulcerated, has areas of stone-like
density. On its external surface there are large, up to 2 cm in diameter, thrombotic plaques. Left
ventricle wall is 2,2 cm thick. Cardiac muscle is dull, matt, and flaccid. What type of endocarditis
corresponds with described alterations of the aortic valve?

A. * Ulcerative polypoid endocarditis

B. Diffuse endocarditis
C. Acute verrucous endocarditis

D. Recurrent verrucous endocarditis

E. Fibroplastic endocarditis

309. In course of an experiment a big number of stem cells of red bone marrow was in some way
destructed. Regenerati-on of which cell populations in the loose connective tissue will be inhibited?

A. * Of macrophags

B. Of fibroblasts

C. Of pigment cells

D. Of lipocytes

E. Of pericytes

310. In course of severe respiratory viral infection there appeared clinical signs of progressing cardiac
insufficiency that caused death of a patient in the 2nd week of disease. Autopsy revealed that the
heart was sluggish, with significant cavity dilatation. Histological examination of myocardium
revealed plethora of microvessels and diffuse infiltration of stroma by lymphocytes and histiocytes.
What disease corresponds with the described picture?

A. * Myocarditis

B. Acute coronary insufficiency

C. Myocardium infarction

D. Stenocardia

E. Cardiomyopathy

311. In response to a change in body position from horizontal to vertical blood circulation system
develops reflex pressor reaction. Which of the following is its compulsory component?

A. * Systemic constriction of the venous vessels

B. Systemic dilatation of the arterial resi-stive vessels

C. Decrease in the circulating blood volume

D. Increase in the heart rate

E. Weakening of the pumbing ability of heart

312. Inflammatory processes cause synthesis of protein of acute phase in an organism. What substances
stumulate their synthesis?
A. * Interleukin-1

B. Immunoglobulins

C. Interferons

D. Biogenic amins

E. Angiotensin

313. Medical examination of a 20-year-old woman revealed a dense incapsulated node 1 cm in diameter
that was palpated in the mammary gland. The postoperati-ve biopsy revealed connective tissue
overgrowth around the mammary ducts and glandular components of different di-ameter that didn’t
make lobules and bore no signs of cellular abnormality. What di-agnosis will be made?

A. * Fibroadenoma

B. Fibroma

C. Metastatic cancer

D. Adenoma

E. Fibrocarcinoma

314. Microscopic examination of the enlarged neck gland of a 14 year old girl revealed destruction of the
ti-ssue structure of the node, absence of the lymph follicles, sclerotic areas and necrosis foci, cell
constitution of the node is polymorphous, lymphocites, eosinophi-les, big atypical cells with
multilobular nuclei (Beresovsky-Sternberg cells) and mononuclear cells of the large size are present.
What is the most likely diagnosis?

A. * Lymphogranulomatosis

B. Acute lympholeucosis

C. Chronic lympholeucosis

D. Berkitt’s lymphoma

E. Fungoid mycosis

315. Microscopical examination of a removed appendix revealed an edema, di-ffuse neutrophilic


infiltration of appendix wall along with necrosis and defect of mucous membrane with affection of its
muscle plate. What appendicitis form was developed?

A. * Ulcerophlegmonous

B. Phlegmonous

C. Gangrenous

D. Superficial
E. Apostematous

316. Microscopical examination of an enlarged cervical lymph node revealed blurring of its structure,
absence of lymphoid follicles; all the microscopic fi-elds showed cells with roundish nuclei and thin
limbus of basophil cytoplasm. It is known from the clinical data that other groups of lymph nodes are
also enlarged as well as spleen and liver. What disease might be suspected?

A. * Lymphoid leukosis

B. Lymphogranulomatosis

C. Lymphosarcoma

D. Myeloid leukosis

E. Multiple myeloma

317. Microscopical renal examination of a 36 y.o. woman who died from renal insufficiency revealed in
the glomerules proliferation of capsule nephrothelium as well as of podocytes and phagocytes
accompanied by formation of "crescents", capillary loop necrosis, fibrinous thrombs in their lumens;
sclerosis and hyalinosis of glomerules, atrophy of tubules and fi-brosis of renal stroma. What is the
most probable diagnosis?

A. * Subacute glomerulonephritis

B. Acute glomerulonephritis

C. Chronic glomerulonephritis

D. Focal segmentary sclerosis

E. Membranous nephropathy

318. Microscopically examination of an enlarged cervical lymph node revealed blurring of its structure,
absence of lymphoid follicles; all the microscopic fields showed cells with roundish nuclei and thin
limbus of basophil cytoplasm. It is known from the clinical data that other groups of lymph nodes are
also enlarged as well as spleen and liver. What disease might be suspected?

A. * Lymphoid leucosis

B. Lymphogranulomatosis

C. Lymphosarcoma

D. Myeloid leucosis

E. Multiple myeloma

319. Microscopy of the bronchial wall revealed atrophy of the mucosa, metaplastic change from columnar
to squamous epithelium, an increase in the number of goblet cells, diffuse infiltration of the bronchial
wall with lymphoplasmacytic elements with a large number of neutrophilic granulocytes, pronounced
sclerosis. Spesi-fy the morphological form of bronchitis:
A. * Chronic purulent bronchitis

B. Acute bronchitis

C. Polypoid chronic bronchitis

D. Acute purulent bronchitis

E. Chronic bronchitis

320. Microscopy of the coronary artery of a dead 53-year-old patient revealed lumi-nal occlusion due to a
fibrous plaque wi-th some lipids. The most likely form of atherosclerosis in this case is:

A. * Liposclerosis

B. Lipidosis

C. Prelipid stage

D. Atheromatosis

E. Ulceration

321. Mucous membrane of the right palatine tonsil has a painless ulcer wi-th smooth lacquer fundus and
regular cartilagenous edges. Microscopically: inflammatory infiltration that consists of lymphocytes,
plasmocytes, a small number of neutrophils and epithelioid cells; endovasculitis and perivasculitis.
What disease is it?

A. * Syphilis

B. Actinomycosis

C. Tuberculosis

D. Pharyngeal diphtheria

E. Ulcerous necrotic Vincent’s angina

322. Necrosis focus was observed in the area of hyperemia and skin edema in a few hours after burn.
What mechani- sm strengthens destructive effects in the inflammation area?

A. * Secondary alteration

B. Primary alteration

C. Emigration of lymphocytes

D. Diapedesis of erythrocytes

E. Proliferation of fibroblasts
323. On autopsy a 35-year-old man the focus of carnification 5 cm in diametre enclosed in a thin capsule
was revealed in the second segment of the right lung . The focus consists of a tough dry friable tissue
with a dim surface. For what disease are these morphological changes typical?

A. * Tuberculoma

B. Lung cancer

C. Chondroma

D. Tumorous form of silicosis

E. Postinflammatory pneumosclerosis

324. On autopsy it was revealed: pia mater of the upper parts of cerebral hemisphere is plethoric, of
yellowish-green color, soaked with purulent and fibrose exudate, looks like a cap. What disease is it
typical for?

A. * Meningococcal meningitis

B. Tuberculous meningitis

C. Grippal meningitis

D. Meningitis connected with anthrax

E. Meningitis connected with typhus

325. On autopsy of a 40-year-old woman, who had been suffering from rheumatoid arthritis, her liver is
found to be dense and enlarged. On dissection its tissue is red-brown colored, with enlarged follicles
resembling semi-transparent grayish-white granules. What is the most likely pathologi- cal process?

A. * Sago spleen

B. Sugar-coated spleen

C. Lardaceous spleen

D. Splenic hyalinosis

E. Porphyry spleen

326. On autopsy of the 58-year-old man it was revealed: mitral valve is deformed, thickened, not totally
closed. Microscopi-cally: foci of collagen fibers are eosinophi-lic, have positive fibrin reaction. The
most likely it is:

A. * Fibrinoid swelling

B. Fibrinoid inflammation

C. Mucoid swelling

D. Hyalinosis
E. Amyloidosis

327. On autopsy the dissector determined that the lungs are enlarged, pale, soft, do not deflate, crunch
when cut. Microscopically there are dilated alveolar ducts, alveolar septa are thin, and signs of
intracapillary sclerosis are observed. What pulmonary di- sorder are these presentations characteristic
of?

A. * Emphysema

B. Pneumosclerosis

C. Pneumothorax

D. Atelectasis

E. Pneumonia

328. On bronchoscopy there is a polypoid growth 1,0 cm in diameter with ulcer in its center in the upper
lobe of the right lung. Histological investigation revealed a tumor composed of lymphocyte-like cells
with hyperchromic nuclei, the cells form layers and bands. What is the most likely tumor type?

A. * Undifferentiated small cell carcinoma

B. Undifferentiated large cell carcinoma

C. Squamous cell carcinoma

D. Adenocarcinoma

E. Glandular squamous cell carcinoma

329. On examination of a newborn boy’s external genitalia a fissure in the urethra opening on the inferior
surface of his penis is detected. What maldevelopment is it?

A. * Hypospadias

B. Hermaphroditism

C. Epispadia

D. Monorchism

E. Cryptorchidism

330. On histological examination of uterine mucosa the following is detected: sinuous glands, serratiform
and corkscrew-shaped elongated growths of stroma with cell proliferation. Make the diagnosis:

A. * Glandular endometrial hyperplasia

B. Acute endometritis
C. Leiomyoma

D. Vesicular mole

E. Placental polyp

331. On the 24th day since the onset of disease, a male patient diagnosed with typhoid fever and
undergoing treatment in an infectious diseases hospital has suddenly developed cli-nical
presentations of acute abdomen leading to the death of the patient. During autopsy peritonitis has
been revealed, with numerous ulcers coveri-ng the colon mucosa and reaching as deep as muscular
and, in places, serous tunic. The ulcers have smooth edges and even floor. The intestinal wall is
perforated. What stage of typhoid fever has the lethal complication ari-sen at?

A. * Clean ulcer

B. Medullary swelling

C. Necrosis

D. Dirty ulcer

E. Regeneration

332. Parents of 5-year-old child report him o have frequent colds that develop into pneumonias, presence
of purulent rashes on the skin. Laboratory tests have revealed the following: absence of
immunoglobulins of any type, and naked cells are absent from the lymph nodes punctate. What kind
of immune disorder is it?

A. * X-linked hypogammaglobulinemia (Bruton type agammaglobulinemia)

B. Autosomal recessive agammaglobulinaemia (Swiss type)

C. Hypoplastic anemia

D. Agranulocytosis

E. Louis-Barr syndrome

333. Parents of a 5-year-old boy report him to have frequent colds that develop into pneumonias, presence
of purulent rashes on the skin. Laboratory tests have revealed the following: absence of
immunoglobulins of any type, and naked cells are absent from the lymph nodes punctate. What kind
of immune disorder is it?

A. * X-linked hypogammaglobulinemia (Bruton type agammaglobulinemia)

B. Autosomal recessive agammaglobuli-naemia (Swiss type)

C. Hypoplastic anemia

D. Agranulocytosis

E. Louis-Barr syndrome
334. Parodontitis is treated with calcium preparations and a hormone that sti-mulates tooth mineralization
and inhibits tissue resorption. What hormone is it?

A. * Calcitonin

B. Parathormone

C. Adrenalin

D. Aldosterone

E. Thyroxine

335. Poisoning caused by mercury (II) chloride (corrosive sublimate) occurred in the result of safety rules
violation. In 2 days the patient’s diurnal diuresis became 620 ml. The patient developed headache,
vomiting, convulsions, dyspnea; moist crackles are observed in the lungs. Name this pathology:

A. * Acute renal failure

B. Chronic renal failure

C. Uremic coma

D. Glomerulonephritis

E. Pyelonephritis

336. Pulmonary examination of a pati-ent who has worked as a stone grinder for 9 years revealed small
dense roundi-sh nodules consisting of connective ti-ssue. The nodules were found to have peripheral
macrophages. Such pulmonary alterations are indicative of the following disease:

A. * Silicosis

B. Acute pneumonia

C. Multiple bronchiectasis

D. Chronic bronchitis

E. Bronchial asthma

337. Section shows significant enlargement of the patient’s right kidney. There is a nephrolith at the place
of incision. Renal pelvic lumen is distended with accumulating urine. Renal parenchyma is acutely
thinned out. What is the most correct diagnosis?

A. * Hydronephrosis

B. Pyelectasis

C. Hydroureteronephrosis

D. Renal cyst
E. Nephroblastoma

338. Several minutes after a dentist admini-stered novocaine for local anaesthesia of a patient’s tooth, the
following symptoms sharply developed in the patient: fatigue,skin itching. Objectively the following
can be observed: skin hyperemia, tachycardia, BP dropped down to 70/40 mm Hg. What kind of
allergic reaction is this pathology?

A. * Anaphylactic

B. Cytotoxic

C. Stimulating

D. Cell-mediated immune reaction

E. Immune complex

339. The liver puncture biopsy of a patient with hepatocellular insufficiency revealed hydropic and
ballooning degenerationof hepatocytes, necrosis of certain cells, presence of Kaunsilmen’s bodies.
Portal and lobular stroma were infiltrated mostly with lymphocytes and macrophages as well as with
a small number of polymorphonuclear lymphocytes. What is the most likely diagnosis?

A. * Acute viral hepatitis

B. Chronic persistent hepatitis

C. Chronic aggressive hepatitis

D. Autoimmune hepatitis

E. Alcoholic hepatitis

340. The patient with acute miocardi-al infarction was given intravenously different solutions during 8
hours with medical dropper 1500 ml and oxygen intranasally. He died because of pulmonary edema.
What caused the pulmonary edema?

A. * Volume overload of the left ventricular

B. Decreased oncotic pressure due to hemodilution

C. Allergic reaction

D. Neurogenic reaction

E. Inhalation of the oxygen

341. The patient with complaints of permanent thirst applied to the doctor. Hyperglycemia, polyuria and
increased concentration of 17-ketosteroids in the urine were revealed. What disease is the most
likely?

A. * Steroid diabetes
B. Insulin-dependent diabetes mellitus

C. Myxoedema

D. Type I glycogenosis

E. Addison’s disease

342. The pulmonalis embolism has suddenly developed in a 40 year-old patient with opened fracture of
the hip. Choose the possible kind of embolism.

A. * Fat

B. Thrombus-embolus

C. Air

D. Tissue

E. Foreign body

343. The pulmonalis embolism has suddenly developed in a 40-year-old pati-ent with opened fracture of
the hip. Choose the possible kind of embolism

A. * Fat

B. Thrombus-embolus

C. Air

D. Tissue

E. Foreign body

344. The upper lobe of the right lung is enlarged, grey and airless, the inscision surface is dripping with
turbid liquid, the pleura has many fibrinogenous films; microscopical examination of alveoles
revealed exudate containing neutrophils, desquamated alveolocytes and fibrin fibers. The bronchus
wall is intact. What is the most probable diagnosis?

A. * Croupous pneumonia

B. Interstitial pneumonia

C. Pulmonary abscess

D. Focal pneumonia

E. Influenzal pneumonia

345. Tissue sampling of a 37-year-old male patient with chronic renal disease has revealed the following:
sclerosis, lymphocytic and plasmocytic infiltrati-on of renal pelvis and calices walls, dystrophy and
atrophy of tubules. Remaining tubules are enlarged and stretched with colloid masses, epitheli-um is
flattened out ("scutiform"or "shield-shaped"kidney). What is the most likely diagnosis?

A. * Chronic pyelonephritis

B. Tubular interstitial nephritis

C. Acute pyelonephritis

D. Glomerulonephritis

E. Nephrosclerosis

346. Violation of safety rules resulted in calomel intoxication. Two days later the daily diuresis was 620
ml. A pati-ent experienced headache, vomiting, convulsions, dyspnea, moist rales in lungs. What
pathology is it?

A. * Acute renal insufficiency

B. Chronic renal insufficiency

C. Uraemic coma

D. Glomerulonephritis

E. Pyelonephritis

347. While examining a patient an otolaryngologist noticed hyperaemia and significantly edematous
tonsils with a grayish bloom upon them. Microscopical examination of this bloom revealed some
gram-positive bacilli placed at an angle with each other. What disease might be suspected?

A. * Diphtheria

B. Angina

C. Scarlet fever

D. Meningococcal nasopharyngitis

E. Epidemic parotitis

348. While playing volleyball a sportsman jumped and then landed across the external edge of his foot.
This caused acute pain in the talocrural articulation, active movements became limited, passive
movements remained unlimited but painful. In the region of the external ankle a swelling appeared,
the skin turned red and became warmer to the touch. What type of peripheral circulation disorder has
developed in this case?

A. * Arterial hyperaemia

B. Stasis

C. Embolism
D. Venous hyperaemia

E. Thrombosis

349. While playing volleyball a sportsman made a jump and landed on the outside edge of his foot. He felt
acute pain in the talocrural joint, active movements are limited, passive movements are unlimited but
painful. A bit later there appeared a swelling in the area of external ankle, the skin became red and
warm. What type of peripheral circulation disturbance is the case?

A. * Arterial hyperemia

B. Venous hyperemia

C. Stasis

D. Embolism

E. Thrombosis

350. Work in a mine is known to cause inhalation of large amounts of coal dust. Inhaled coal dust can be
detected in the following pulmonary cells:

A. * Alveolar macrophages

B. Respiratory epithelial cells

C. Secretory epithelial cells

D. Capillary endothelial cells

E. Pericapillary cells

351. Autopsy of a woman revealed the following morphologic changes: stenosis of the atrioventricular
opening, mitral insufficiency. Histologically there are focal cardiosclerosis and ”blooming” Aschoff
nodules in the myocardium. What is the most likely diagnosis?

A. * Rheumatism

B. Scleroderma

C. Dermatomyositis

D. Polyarteritis nodosa

E. Systemic lupus erythematosus

352. Histological investigation of the uterine scrape of the 45-year-old woman with disturbed ovarian
menstrual cycle revealed increased number of endometrial glands, some of which are serrated, while
others are dilated and cyst-like. Make the diagnosis:

A. * Endometrial cystic glandular hyperplasia


B. Placental polyp

C. Atypical endometrial hyperplasia

D. Glandular endometrial polyp

E. Endometrial adenocarcinoma

353. A 45-year-old man diagnosed with hepatic cirrhosis and ascites underwent drainage of 5 liters of
fluid from his abdominal cavity, which resulted in development of syncopal state due to insufficient
blood supply to the brain. What circulatory disorder occurred in the abdominal cavity in this case?

A. * Arterial hyperemia

B. Ischemia

C. Venous hyperemia

D. Thrombosis

E. Embolism

354. 4. During autopsy of a 34-year-old man, who died of chronic kidney failure due to renal
amyloidosis, in the lungs (mainly in the lower lobes) the pathologist detected multiple bronchial
dilations filled with purulent masses accumulated in the bronchial lumen. Surface of the lungs section
has fine-meshed pattern and resembles a honeycomb. Histologically there is a chronic inflammation
detected in the bronchial wall, muscle fibers are replaced with connective tissue. These changes in
the lungs can be defined as:

A. * Bronchiectases

B. Bronchopneumonia

C. Chronic bronchitis

D. Chronic pneumonia

E. Lung abscesses

355. 5. Autopsy revealed a large wedge-shaped patch of a dense dark red tissue with clear margins in the
upper lobe of the right lung. Histological examination detected there necrosis of the alveolar walls;
the alveolar lumen is tightly packed with erythrocytes. What process occurred in the lungs?

A. * Hemorrhagic infarction

B. Carneous degeneration

C. Gangrene

D. Hemorrhage

E. Atelectasis
356. 6. A patient, who had burn, developed painful boils filled with turbid liquid on the skin. What
morphological type of inflammation has developed in the patient?

A. * Serous

B. Proliferative

C. Croupous

D. Granulomatous

E. Diphtheritic

357. Coronary artery thrombosis resulted in development of myocardial infarction. What mechanisms of
cell damage are leading in this disease?

A. * Calcium

B. Lipid

C. Acidotic

D. Electroosmotic

E. Protein

358. A 65-year-old woman, who had been suffering from deep vein thrombophlebitis of the lower leg,
suddenly died when awaiting her appointment with the doctor. Autopsy revealed loose friable red
masses with corrugated dull surface in the main pulmonary artery and its bifurcation. What
pathologic process was discovered by the pathologist in the pulmonary artery?

A. * Thromboembolism

B. Thrombosis

C. Tissue embolism

D. Foreign body embolism

E. Fat embolism

359. Examination of the coronary arteries revealed atherosclerotic plaques with calcinosis that narrow the
arterial opening by 1/3. In the muscle there are numerous whitish layers of connective tissue. Name
the process detected in the myocardium:

A. * Diffuse cardiosclerosis

B. Tiger heart

C. Postinfarction cardiosclerosis

D. Myocarditis
E. Myocardial infarction

360. Microscopy of the myocardium of a girl, who died of diphtheria due to heart failure, shows fatty
degeneration, multiple foci of cardiomyocytes necrosis, and small foci of cellular infiltrates in the
interstitium. What type of myocarditis is it?

A. * Alterative

B. Diffuse exudative

C. Focal exudative

D. Interstitial

E. Granulomatous

361. A 40-year-old man developed skin redness and swelling in the neck area, where eventually a small
abscess appeared. On section the focus is dense and yellow- green colored. In the purulent masses
there are white granules. Histologically there are fungal druses, plasma and xanthome cells, and
macrophages detected. Specify the most correct etiological name of this pathological process:

A. * Actinomycosis

B. Furuncle

C. Carbuncle

D. Syphilis

E. Leprosy

362. Renal examination shows the kidney to be swollen and plethoric, with easily removable capsule.
Renal pelvis and calyces are dilated and filled with turbid urine; their mucosa is dull and has
hemorrhagic foci. On section the renal tissue is variegated, there are yellow-gray areas surrounded
with plethoric and hemorrhagic zone. What disease can be characterized by such results of
macroscopic examination?

A. * Acute pyelonephritis

B. Acute glomerulonephritis

C. Renal amyloidosis

D. Nephrolithiasis

E. Polycystic kidney

363. Autopsy of the body of a man, who for a long time had been working at the factory with high content
of silicon dioxide in the air, revealed enlarged dense lungs with numerous round and oval sclerotic
foci. The foci were miliary and larger in size and colored gray or gray-black. What is the most likely
diagnosis?
A. * Nodular silicosis

B. Diffuse sclerotic silicosis

C. Anthracosilicosis

D. Silicoanthracosis

E. Asbestosis

364. Autopsy of the body of a 50-year- old man, who had been suffering from tuberculosis and died of
cardiopulmonary decompensation, shows lobar affection of the lungs: the upper lobe of the right lung
is enlarged, dense, yellow on section, crumbling, with pleural fibrin deposition. What type of
secondary tuberculosis can be characterized by this pathology?

A. * Caseous pneumonia

B. Fibrous focal tuberculosis

C. Infiltrative tuberculosis

D. Tuberculoma

E. Acute focal tuberculosis

365. A 34-year-old man died in a comatose state. According to his family after a business trip to an
African country he developed periodical jaundice attacks. Autopsy shows the following: dense
enlarged spleen with slateblack pulp; enlarged plethoric liver, gray-black on section; cerebral gray
matter is brown-gray; cerebral white matter contains numerous small hemorrhages. What infectious
disease can be suspected?

A. * Malaria

B. Meningococcemia

C. Prion infection

D. Generalized herpetic infection

E. Generalized cryptococcosis

366. Acute renal impairment caused death of a patient with hemorrhage. Autopsy revealed enlarged
kidneys with broad pale pink cortical layer expressively demarcated from dark red renal pyramids.
Macroscopic examination revealed lack of epithelial nuclei of the convoluted tubules, tubulorrhexis,
phlebostasis. The cell nuclei of the choroid glomus and straight tubules were present. What
pathology is it?

A. * Necronephrosis

B. Infarction

C. Glomerulonephritis
D. Pyelonephritis

E. Nephrosis

367. Autopsy of the body revealed waxy degeneration of the rectus abdominis muscles. In the terminal
segment of the small intestine there are ulcers 3-5 cm in diameter. The ulcer walls are covered in a
crumbling grayish-white substance. The ulcer edges are markedly raised above the mucosa. Widal
test is positive. Make the diagnosis:

A. * Typhoid fever

B. Nonspecific ulcerative colitis

C. Relapsing fever

D. Crohn’s disease

E. Dysentery

368. A 30-year-old man has undergone surgical removal of a thyroid tumor. Histologically the tumor is
made up of papillary structures varying in size that emerge from the inner surface of dilated cystic
follicles and are covered with atypical epithelium. What is the most likely diagnosis?

A. * Papillary carcinoma

B. Colloid nodular goiter

C. Macrofollicular adenoma

D. Follicular carcinoma

E. Medullar carcinoma

369. Histological analysis of a removed skin tumor shows clusters and bands composed of atypical
stratified squamous epithelium cells that penetrate into the underlying tissue. What preliminary
diagnosis can be made?

A. * Non-keratinizing squamous cell carcinoma

B. Solid cancer

C. Carcinoma in situ

D. Papilloma

E. Adenoma

370. Autopsy of a patient, who died of bilateral bronchopneumonia, shows in the left lung lower lobe a
cavity 5 cm in diameter, filled with yellowish-white liquid. What complication of the patient’s
pneumonia had developed?
A. * Abscess

B. Gangrene

C. Granuloma

D. Sequestrum

E. Tuberculoma

371. A patient presents with enlarged cervical lymph nodes. Other lymph nodes and internal organs are
without changes. Peripheral blood test results are normal. Histological examination of biopsy
material taken from the cervical lymph node shows smoothed-out pattern, absent follicles,
homogeneous cell composition represented by lymphoblasts. The cells penetrate into the lymph node
capsule. What diagnosis can be made?

A. * Lymphoblastic leukemia

B. Myeloblastic leukemia

C. Erythroblastic leukemia

D. Sezary disease

E. Burkitt lymphoma

372. A young woman presents with a tumor along the auditory nerve. The tumor is node-shaped, 3 cm in
diameter, soft and elastic, pink-white colored, and has homogeneous structure. Microscopically the
tumor contains bundles of cells with oval nuclei. These cellular fibrous bundles form regular
structures made up of parallel rows of regularly oriented cells arranged in the form of a palisade with
acellular homogeneous area in between (Verocay bodies). Name this type of tumor:

A. * Neurinoma

B. Malignant neurinoma

C. Ganglioneuroma

D. Neuroblastoma

E. Ganglioneuroblastoma

373. When examining a biopsy material obtained from the thyroid gland, the pathologist discovered
lymphocyte infiltration of the thyroid tissues and destruction of the parenchymal elements. Diffuse
lymphocyte infiltration with lymphoid follicles was detected in the stroma. What is the most likely
diagnosis?

A. * Hashimoto’s thyroiditis (chronic lymphocytic thyroiditis)

B. Papillary thyroid cancer

C. Undifferentiated thyroid carcinoma


D. Solid adenoma of the thyroid

E. Graves’ disease (toxic diffuse goiter)

374. A victim of a traffic accident was received by the intensive care unit. The patient is in a grave
condition that can be characterized as a severe pathologic process that leads to exhaustion of vital
functions and puts the patient into the marginal state between life and death due to critical reduction
of capillary circulation in the affected organs. The patient is in the state of:

A. * Shock

B. Collapse

C. Coma

D. Agony

E. Preagony

375. Autopsy of the body of an elderly man revealed yellow spots and streaks in the aortic intima and
white-yellow protruding plaques in the area of aortic bifurcation. Microscopy (with hematoxylin and
eosin staining) shows round cavities in the thickened aortic intima. The cavities color orange when
stained with Sudan III and are surrounded with overgrown connective tissue. What process
developed in the aortic intima?

A. * Metabolic disorder of cholesterol and cholesterol ethers

B. Local hyalinosis

C. Metabolic disorder of neutral fat

D. Systemic hyalinosis

E. Secondary amyloidosis

376. Autopsy of a 50-year-old woman, who had been suffering from systemic connective tissue disease,
reveals small dense kidneys with lumpy surface. Microscopy of the renal glomeruli detects there foci
of fibrinoid necrosis, hyaline thrombi, hematoxylin bodies, and socalled ” wire loop thickening ” of
the basement membranes. What disease can be characterized by such changes in the kidneys?

A. * Systemic lupus erythematosus

B. Rheumatism

C. Scleroderma

D. Rheumatoid arthritis

E. Dermatomyositis
377. A 63-year-old man suffers from esophageal carcinoma, presents with metastases into the mediastinal
lymph nodes and cancerous cachexia. What pathogenetic stage of neoplastic process is observed in
the patient?

A. * Progression

B. Promotion

C. Transformation

D. Initiation

E. -

378. A 45-year-old woman presents with breast cancer. Metastases can spread in this case to the following
regional lymph nodes:

A. * Axillary, parasternal

B. Abdominal, cervical

C. Cervical, parasternal

D. Parasternal, mediastinal

E. Aortic, mediastinal

379. A 63-year-old man, who has been suffering from chronic diffuse obstructive pulmonary emphysema
for 15 years, died of progressive heart failure. Autopsy shows nutmeg liver cirrhosis, cyanotic
induration of kidneys and spleen, ascites, and edemas of the lower limbs. What type of heart failure
can be characterized by such changes in the internal organs?

A. * Chronic heart failure

B. Chronic atrial failure

C. Acute right ventricular failure

D. Acute global heart failure

E. Acute left ventricular failure

380. A patient has gradually developed a skin plaque on his face. In the center of this plaque there are
necrotic patch and an ulcer. Histopathological analysis of the biopsy material reveals proliferation of
atypical epithelial cells with large number of pathologic mitoses. What is the most likely diagnosis?

A. Fibroma

B. Sarcoma

C. Papilloma
D. * Skin cancer

E. Trophic ulcer

381. Autopsy revealed a large wedge-shaped patch of a dense dark red tissue with clear margins in the
upper lobe of the right lung. Histological examination detected there necrosis of the alveolar walls;
the alveolar lumen is tightly packed with erythrocytes. What process occurred in the lungs?

A. Hemorrhage

B. * Hemorrhagic infarction

C. Carneous degeneration

D. Gangrene

E. Atelectasis

382. Autopsy of a 30-year-old man shows enlarged spleen (weight - 900.0 g), enlarged liver (weight -
4000.0 g), and enlarged lymph nodes. Bone marrow of the femoral shaft is bright red and moist.
Microscopically in the liver there are thick infiltrates located mostly along the portal tract. The
infiltrates consist of juvenile hemopoietic cells with round nuclei and thin layer of cytoplasm. What
disease can be suspected?

A. * Acute lymphoblastic leukemia

B. Chronic myeloid leukemia

C. Acute myeloblastic leukemia

D. Chronic lymphocytic leukemia

E. Generalized lymphogranulomatosis

383. Histological analysis of a biopsy skin sample obtained from a 24-year-old patient detects caseous
necrosis surrounded with cellular infiltrate consisting of lymphocytes, among which there are single
giant cells; proliferation of connective tissue and endovasculites are observed. Characterize this
pathologic process:

A. Catarrhal inflammation

B. Abscess

C. * Proliferative granulomatous inflammation

D. Proliferative interstitial inflammation

E. Ichorous inflammation

384. A 27-year-old woman has undergone a sector resection of mammary gland tissue. Macroscopy
detects a dense white node, 4 cm in diameter, with clear margins in the excised tissue. Immediate
histological analysis shows the tumor to consist of a large amount of fibrous stroma with stromal
proliferation around the small canaliculi. Canalicular epithelium overlays the basement membrane
and retains its polarity. Make the diagnosis:

A. Cancer

B. Adenocarcinoma

C. Dyshormonal disorders

D. Sarcoma

E. * Pericanalicular fibroadenoma

385. An inflammation can be characterized by hemocapillary dilation in the affected area, decreased blood
circulation, and increased vessel wall permeability. What cells play the key role in this process?

A. Fibroblasts

B. Macrophages

C. Plasma cells

D. Eosinophils

E. * Tissue basophils

386. In the course of an urgent surgery, the vermiform appendix of the patient was excised. The appendix
was acutely distended and gray-black throughout its whole length. In the distal segment a defect of
the appendix wall was detected, through which a foul-smelling gray-brown substance was being
discharged from the appendix lumen. Histological analysis shows necrotization of the appendix wall
with hemorrhagic foci; lumen of the mesenteric artery is filled with a trombus. What type of
appendicitis is it?

A. * Acute gangrenous

B. Acute phlegmonous

C. Acute simple

D. Chronic

E. Acute superficial

387. A 34-year-old man died in a comatose state. According to his family after a business trip to an
African country he developed periodical jaundice attacks. Autopsy shows the following: dense
enlarged spleen with slate-black pulp; enlarged plethoric liver, gray-black on section; cerebral gray
matter is brown-gray; cerebral white matter contains numerous small hemorrhages. What infectious
disease can be suspected?

A. Meningococcemia

B. Prion infection
C. Generalized cryptococcosis

D. Generalized herpetic infection

E. * Malaria

388. A 39-year-old man underwent a surgary for peptic ulcer disease of the stomach.He died 7 days after
the surgery. On autopsy the peritoneal layers are plethoric, dull, and covered with massive
yellow-green membranous deposits. The peritoneal cavity contains approximately 300 mL of thick
yellow-green fluid. What pathologic process was detected in the peritoneal cavity?

A. * Fibrinopurulent peritonitis

B. Serofibrinous peritonitis

C. Peritoneal commissures

D. Serous peritonitis

E. Fibrinohemorrhagic peritonitis

389. Regional lymph nodes surrounding an infected wound are enlarged. Histological examination shows
increased number of macrophages, lymphocytes, and lymphatic follicles in the cortical layer of the
lymph nodes, as well as a large amount of plasma cells. What process in the lymph nodes is indicated
by these histological changes?

A. Transplant rejection

B. * Antigen stimulation

C. Congenital deficiency of lymphoid tissue

D. Neoplastic aberration

E. Acquired deficiency of lymphoid tissue

390. A patient is diagnosed with severe B12- deficiency anemia resulting in disturbed hematopoiesis and
appearance of atypical erythrocytes in the blood. The patient has a history of total gastric resection.
This diagnosis can be confirmed if the following cells are present in the peripheral blood:

A. * Megalocytes

B. Microcytes

C. Elliptocytes

D. Anulocytes

E. Normocytes

391. Autopsy of a man, who served on a nuclear submarine, revealed the following pathologies: bone
marrow atrophy (panmyelophthisis), anemia, leukopenia, thrombocytopenia, lymphocytic
disintegration in the lymph nodes, spleen, gastrointestinal lymphatic system, and hemorrhages into
the adrenal glands. What disease had developed in this case?

A. Decompression sickness

B. * Acute radiation sickness

C. Vibration disease

D. Acute leukemia

E. Acute anemia

392. Autopsy of a 58-year-old man, who for a long time has been drinking alcohol in large amounts and
died at home, is being conducted. Macroscopically the right lung is dense and enlarged, its tissue is
gray and homogeneous on section, its pleura is covered with grayish membranous deposits.
Microscopically the alveolar cavities contain fibrin threads, neutrophils, and hemolysed erythrocytes.
Make the diagnosis:

A. Primary pulmonary tuberculosis

B. Interstitial pneumonia

C. * Croupous pneumonias

D. Caseous pneumonia

E. Focal pneumonia

393. Autopsy of a 49-year-old woman who died of chronic kidney failure shows small dense striated
kidneys with areas of hemorrhages. Microscopically nuclei of epithelial channels contain
hematoxylin bodies; glomerular capillaries resemble wire loops, have thickened basement
membranes, and in places contain hyaline thrombi and foci of fibrinoid necrosis. What is the most
likely diagnosis?

A. Rheumatism

B. * Systemic lupus erythematosus

C. Amyloidosis

D. Arteriolosclerotic nephrosclerosis

E. Atherosclerotic nephrosclerosis

394. Some diseases of large-intestine lead to me changes in the qnantitative ratio between mucosal
epithelial cells. What cell types are normally predominant in the cryptal epithelium of the large
intestine?

A. * Goblet cells

B. Ciliated columnar epithelial cells


C. Poorly differentiated cells

D. Endocrine cells

E. Cells with acidophilic granules

395. Autopsy of a 3-year-old child shows a tumor in the cerebellum. The tumor has no clear margin
separating it from the surrounding tissues. Histologically it is made of small atypical cells with
hyperchromic nuclei. This tumor is most likely a:

A. * Medulloblastoma

B. Glioblastoma

C. Cancer metastasis

D. Medullary sarcoma

E. Sarcoma metastasis

396. Autopsy of a man, who had been suffering from mitral stenosis, reveals dense brown lungs. What
pathologic process had occurred in the lungs?

A. Hemochromatosis

B. Lipofuscinosis

C. * Hemosiderosis

D. Hemomelanosis

E. Jaundice

397. Autopsy of a 60-year-old woman, who for a long time had been suffering from essential
hypertension, shows significantly diminished kidneys (weight of both kidneys is 80 g) with finely
granular surface. Uniform renal cortical thinning can be observed on section. Name the described
changes in the kidneys:

A. * Primary contracted kidney

B. Diabetic nephrosclerosis

C. Pyelonephritic contracted kidney

D. Secondary contracted kidney

E. Amyloid contracted kidney

398. A 60-year old man with a history of hypertension, diabetes and hyperlipidemia had a sudden onset of
right-sided weakness. By the time the ambulance arrived, he had difficulty speaking. Unfortunately,
the patient died within the next 2 hours and an autopsy was performed immediately. The gross
examination of the cerebral left hemisphere showed brain swelling, widened gyri and poorly
demarcated gray-white junction. Which of the following is the most likely cause of this patient’s
death?

A. * Ischemic stroke

B. Tumor

C. Intracerebral hemorrhage

D. Cyst

E. Abscess

399. A 65-year-old woman presents to the emergency department because of shortness of breath and chest
pain that started a few hours ago. She did not have a fever, expectoration, or any accompanying
symptoms. She has a history of right leg deep vein thrombosis that occurred 5 years ago. Some time
later, she dies of severe respiratory distress. A pulmonary autopsy specimen reveals red loose mass
that is lodged in the bifurcation of the pulmonary trunk with extensions into both the left and right
main pulmonary arteries. Which of the following is the most likely diagnosis?

A. Pneumothorax

B. Myocardial infarction

C. * Thromboembolism

D. Pneumonia

E. -

400. A 40-year-old woman dies of intracerebral hemorrhage after the hypertensive emergency. During an
autopsy, the pathologist reveals severe obesity, excess of body hair and wide purplish stria on the
abdomen. Microscopic examination of pituitary gland reveals hyperplastic acini populated by a
homogenous cluster of deeply basophilic cells. Which of the following was the most likely
underlying disease?

A. Sheehan’s syndrome

B. Arterial hypertension

C. -

D. Hyperthyroidism

E. * Cushing disease
1. An experimental animal received a subcutaneous dose of an antigen preceded by sensitization. At the
place of the injection, some fibrinous inflammation developed with an alteration of the vascular
walls, the main substance and fibrous structures of the connective tissue in the form of a mucoid and
fibrinoid swelling, a fibrinoid necrosis. Which of the diagnoses listed below was the most probable?
A. Delayed hypersensitivity
B. Transplantation immunoreaction
C. Normergy
D. Granulomatosis
E. * Immediate hypersensitivity
2. A histological examination of the lungs if a male, who suffered for many years from atopic bronchial
asthma and died of asphyxia, revealed much mucus with an admixture of eosinophils in the lumens of
the bronchioles and small bronchi, sclerosis of interalveolar septa, dilation of alveolar lumens. Which
of the mechanisms in the development of a hypersensitivity reaction took place when a fit of
asphyxia developed?
A. Cytotoxic reaction
B. Immunocomplex reaction
C. Cytolysis owing to lymphocytes
D. Granulomatosis
E. * Reaginic reaction
3. A study of the thymus of a 5-year-old child, who died from acute destructive staphylococcal
pneumonia, revealed a decrease in the weight of the gland down to 3.0 g. On histological
examination, a smaller size of the lobules of the gland with a collapse of the stroma, an inversion of
the layers, and cyst-like Hassal’s bodies were found out. Which of the diagnoses listed below was the
most probable?
A. Thymomegaly
B. Hypoplasia of the thymus
C. Dysplasia of the thymus
D. Agenesia of the thymus
E. * Accidental reaction
4. An examination of a pregnant woman with a rhesus-negative group of blood revealed a high level of
antierythrocyte antibodies; in order to decrease it, a skin flap of her rhesus-positive husband was
grafted to her. Two weeks later the flap was rejected; its microscopic examination revealed
disturbances of circulation, an oedema, a cellular infiltration mostly by lymphocytes, neutrophils and
macrophages. Which of the pathological processes listed below was the most probable?
A. Immediate hypersensitivity
B. Delayed hypersensitivity
C. Granulomatous inflammation
D. Interstitial inflammation
E. * Transplantation immunity
5. An experimental animal received a subcutaneous dose of an antigen preceded by sensitization. At the
place of the injection, some fibrinous inflammation developed with an alteration of the vascular
walls, the main substance and fibrous structures of the connective tissue in the form of a mucoid and
fibrinoid swelling, a fibrinoid necrosis. Which of the diagnoses listed below was the most probable?
A. Delayed hypersensitivity
B. Transplantation immunoreaction

C. Normergy
D. Granulomatosis
E. * Immediate hypersensitivity
6. A histological examination of the lungs if a male, who suffered for many years from atopic bronchial
asthma and died of asphyxia, revealed much mucus with an admixture of eosinophils in the lumens of
the bronchioles and small bronchi, sclerosis of interalveolar septa, dilation of alveolar lumens. Which
of the mechanisms in the development of a hypersensitivity reaction took place when a fit of
asphyxia developed?
A. Cytotoxic reaction
B. Immunocomplex reaction
C. Cytolysis owing to lymphocytes
D. Granulomatosis
E. * Reaginic reaction
7. A study of the thymus of a 5-year-old child, who died from acute destructive staphylococcal
pneumonia, revealed a decrease in the weight of the gland down to 3.0 g. On histological
examination, a smaller size of the lobules of the gland with a collapse of the stroma, an inversion of
the layers, and cyst-like Hassal’s bodies were found out. Which of the diagnoses listed below was the
most probable?
A. Thymomegaly
B. Hypoplasia of the thymus
C. Dysplasia of the thymus
D. Agenesia of the thymus
E. * Accidental reaction
8. An examination of a pregnant woman with a rhesus-negative group of blood revealed a high level of
antierythrocyte antibodies; in order to decrease it, a skin flap of her rhesus-positive husband was
grafted to her. Two weeks later the flap was rejected; its microscopic examination revealed
disturbances of circulation, an oedema, a cellular infiltration mostly by lymphocytes, neutrophils and
macrophages. Which of the pathological processes listed below was the most probable?
A. Immediate hypersensitivity
B. Delayed hypersensitivity
C. Granulomatous inflammation
D. Interstitial inflammation
E. * Transplantation immunity
9. On supersonic examination of a 48-year-old male patient, a hepatic neoplasm was diagnosed and a
puncture biopsy was made. Microscopically, the tumour consisted of atypical hepatocytes which
formed trabeculae, acini or tubules. The tumour stroma was poor and had thin-walled blood vessels.
Which of the kinds of tumours listed below was the most probable?
A. Hepatocellular adenoma
B. Metastasis of adenocarcinoma
C. Cholangiocellular carcinoma
D. Solid carcinoma
E. * Hepatocellular carcinoma
10. An encapsulated tumour, 2 cm in diameter, surgically removed from an amputation stump of a lower
extremity, microscopically consists of spindle cells of the monomorphous kind with rod-shaped
nuclei which form "fence-like" structures together with fibres. Which of the tumours listed below is
the most probable?
A. Neurofibroma
B. Malignant neurilemmoma
C. Soft fibroma

D. Fibrosarcoma
E. * Benign neurilemmoma
11. A microscopic examination of a biopsy from a deformed mucous membrane of a lobar bronchus of a
45-year-old male, who smoked for many years, revealed a carcinoma consisting of atypical epithelial
cells with hyperchromatic nuclei and numerous pathological mitoses. The growth of the tumour did
not spread to the basal membrane of the epithelium. Name the histological form of carcinoma.
A. Squamous cell carcinoma
B. Adenocarcinoma
C. Solid carcinoma
D. Small-cell carcinoma
E. * Carcinoma in situ
12. A 45-year-old male underwent surgical removal of a tumour, 4 x 3 cm in size, from the lateral
ventricle of his brain; the tumour surface had small papillae, and it was connected with a vascular
plexus. Microscopically, the tumour consisted of villus-like vegetations covered with epithelial cells
of the cubical and columnar shape and the monomorphous kind. Which of the tumours listed below
was the most probable?
A. Ependymoma
B. Ependymoblastoma
C. Choriocarcinoma
D. Glioblastoma
E. * Choriopapilloma
13. An encapsulated tumour, 2 cm in diameter, surgically removed from an amputation stump of a lower
extremity, microscopically consists of spindle cells of the monomorphous kind with rod-shaped
nuclei which form "fence-like" structures together with fibres. Which of the tumours listed below is
the most probable?
A. Neurofibroma
B. Malignant neurilemmoma
C. Soft fibroma
D. Fibrosarcoma
E. * Benign neurilemmoma
14. A microscopic examination of a biopsy from a deformed mucous membrane of a lobar bronchus of a
45-year-old male, who smoked for many years, revealed a carcinoma consisting of atypical epithelial
cells with hyperchromatic nuclei and numerous pathological mitoses. The growth of the tumour did
not spread to the basal membrane of the epithelium. Name the histological form of carcinoma.
A. Squamous cell carcinoma
B. Adenocarcinoma
C. Solid carcinoma
D. Small-cell carcinoma
E. * Carcinoma in situ
15. A microscopic examination of a gastrobiopsy from a tumour of the pyloroduodenal portion of the
stomach revealed layers of atypical epithelial cells with a large number of mitoses; the tumour
architectonics is characterized by prevalence of the parenchyma over the stroma. Which of the
histological forms of carcinoma listed below was the most probable?
A. Adenocarcinoma
B. Solid carcinoma
C. Mucinous carcinoma
D. Small-cell carcinoma
E. * Medullary carcinoma
16. On supersonic examination of a 48-year-old male patient, a hepatic neoplasm was diagnosed and a
puncture biopsy was made. Microscopically, the tumour consisted of atypical hepatocytes which
formed trabeculae, acini or tubules. The tumour stroma was poor and had thin-walled blood vessels.
Which of the kinds of tumours listed below was the most probable?
A. Hepatocellular adenoma
B. Metastasis of adenocarcinoma
C. Cholangiocellular carcinoma
D. Solid carcinoma
E. * Hepatocellular carcinoma
17. A tumour was found in the locus of a pathological fracture of a rib in a male patient. The case history
contained information about persistent proteinuria with presence of abnormal proteins of Bence-
Jones type, as well as presence of osteolytic foci in the bones of the spine, skull and pelvis.
Histologically, the tumour cells were represented by plasmablasts and plasmacytes. What is your
diagnosis?
A. Primary macroglobulinaemia
B. Heavy-chain disease
C. Osteosarcoma
D. Fibrosarcoma
E. * Multiple myeloma
18. An autopsy of a male, who died from chronic renal insufficiency, revealed numerous nodes with soft
elastic consistency in the ribs, bones of the vault of the skull and the breastbone. The osseous
substance was decalcified according to the nodes. The kidneys were enlarged, light grey, dense, their
section had some greasy lustre. What is your diagnosis?
A. Primary amyloid nephropathy
B. Parathyroid osteodystrophy
C. Osteoma
D. Osteosarcoma
E. * Multiple myeloma
19. Some tumour, which was mobile and clearly delimited from the surrounding tissues, was revealed in
the skin. On section, the tumour tissue was white and fibrous. Microscopically, the tumour consisted
of chaotically interlaced collagenous fibres and a small number of connective tissue cells. Name the
tumour.
A. Soft fibroma
B. Histiocytoma
C. Dermatofibroma
D. Desmoid
E. * Hard fibroma
20. An enlarged dense tuberous prostate has been sent for a histological examination. On section, there
were tumour nodes, 1-2 cm in diameter, surrounded by connective-tissue layers. Microscopically,
against a background of fibrosis there were glandular complexes with atypical epithelial cells,
hyperchromatic nuclei and pathological mitoses. Which of the tumours listed below was the most
probable?
A. Solid carcinoma
B. Adenoma
C. Fibroma
D. Fibrosarcoma
E. * Adenocarcinoma
21. A thick node without any clear borders, about 10 cm in diameter, is contoured on the outer surface of
a thigh. Microscopically, the tumour consists of immature fibroblast-like cells with pathological
mitoses and collagenous fibres. The tumour cells grow among the muscular fibres. Indicate the
diagnosis which was the most probable one of those listed below.
A. Malignant histiocytoma
B. Hard fibroma
C. Soft fibroma
D. Dermatofibroma
E. * Fibrosarcoma
22. A newborn baby has some red-blue flattened tumor, 5 x 4 x 0.3 cm in size, in a capsule on the skin of
its face. Microscopically, the tumour consists of large thin-walled vascular cavities which have an
endothelial lining and are filled with blood. Name the tumour.
A. Venous haemangioma
B. Capillary haemangioma
C. Hemangiopericytoma
D. Lymphangioma
E. * Cavernous haemangioma
23. A histological examination of a thyroid gland revealed small cysts, which were lined with atypical
epithelium and filled with papillae, the latter originating from the walls of the cysts and growing into
their capsules. Name the tumour.
A. Papillary adenoma
B. Follicular carcinoma
C. Solid carcinoma
D. Carcinoma simplex
E. * Papillary carcinoma
24. A histological examination of some spherical neoplasm located under the surface of the skin,
revealed papilliform vegetations of the epithelium with phenomena of acanthosis and
hyperkeratinization. The tumour stroma consisted of a large amount of the connective tissue and
vessels. What tumour took place?
A. Keratoacanthoma
B. Carcinoma in situ
C. Keratinizing squamous cell carcinoma
D. Nonkeratinizing squamous cell carcinoma
E. * Papilloma
25. A bronchoscopy of the mucous membrane of the main bronchus revealed some tumour. A
microscopic examination of the tumour biopsy showed that it consisted of lymphocyte-like cells with
hyperchromatic nuclei growing in the form of layers or bands and involving the submucous layer.
The tumour had many pathological mitoses. Which of the histological forms of carcinoma listed
below was the most probable?
A. Squamous cell carcinoma
B. Adenocarcinoma
C. Adenoacanthoma
D. Scirrhous carcinoma
E. * Small-cell carcinoma
26. Examination of a patient revealed a dense, movable skin tumour that is standing out distinctly from
the surrounding tissues. Its section is found to be white and composed of fibrous tissue. Microscopic
examination revealed interlacing collagen fibers and few cells. What tumor is it?
A. Myoma
B. Histiocytoma
C. Dermatofibroma
D. Desmoid
E. * Fibroma
27. A 50-year-old man has felt vague abdominal discomfort within past 4 months. Physical examination
revealed no lymphadenopathy, and no abdominal masses or organomegaly at palpation. Bowel
sounds are heard. An abdominal CT scan shows a 20 cm retroperitoneal soft tissue mass obscuring
the left psoas muscle. A stool specimen tested for occult blood is negative. Which of the following
neoplasms is this man most likely to have?
A. Myoma
B. Histiocytoma
C. Dermatofibroma
D. Desmoid
E. * Fibroma
28. A 40-year-old woman has had a feeling of abdominal discomfort for the past 8 months. On pelvic
examination, there is the right adnexal mass. Abdominal CT scan demonstrates a 7 cm cystic mass
involving the right ovary with small areas of calcification. The uterus is normal in size. The right
fallopian tube and ovary have been removed surgically. Grossly, the mass on sectioning is filled with
abundant hair and sebum. Microscopically, the mass has glandular spaces lined by columnar
epithelium, squamous epithelium with hair follicles, cartilage, and dense connective tissue. What
type of tumour is it?
A. Squamous cell carcinoma of ovary
B. Melanoma
C. Sarcoma of ovary
D. Metastase of cervical carcinoma
E. * Teratoma
29. An autopsy of a 76-year-old male, who smoked for a long period of time, lived sedentary life and had
redundant weight, revealed in the intima of the aorta some grey-yellow spots and stripes, fibrous
plaques, calcified areas with haemorrhages and calcinosis. What disease do these changes indicate?
A. Nonspecific aortoarteritis
B. Hypertensive disease
C. Systemic lupus erythematosus
D. Visceral syphilis
E. * Atherosclerosis
30. An autopsy of a 27-year-old male, who died suddenly, revealed in the intima of the abdominal aorta
some yellow foci in the form of spots and stripes, which did not rise above the surface of the intima
but after staining with sudan III became orange. What stage in the morphogenesis of atherosclerosis
was revealed?
A. Atherocalcinosis
B. Liposclerosis
C. Atheroma
D. Prelipid
E. * Lipoidosis
31. A microscopic examination of the wall of an aorta revealed a focal infiltration of the intima by lipids
and proteins. The lipids impregnated the intima and accumulated in the muscle cells and
macrophages. Determine the stage of atherosclerosis.
A. Prelipid
B. Liposclerosis
C. Atheromatosis
D. Atherocalcinosis

E. * Lipoidosis
32. A 75-year-old male was hospitalized complaining of a sharp pain in the abdominal cavity, weakness,
filiform pulse. During an operation it was found that the paraaortic fat was imbibed with blood. The
abdominal aorta had a sac-like protrusion, its wall was thinned and had an area of rupture. What
disease caused the complication?
A. Coronary disease
B. Hypertensive disease
C. Cardiomyopathy
D. Systemic vasculitis
E. * Atherosclerosis
33. A 59-year-old patient receiving chemotherapy with the anthracycline Adriamycin develops severe
heart failure. Sections from an endocardial biopsy specimen reveal vacuolization of the endoplasmic
reticulum of the myocytes. Adriamycin therapy most frequently causes what type of
cardiomyopathy?
A. Restrictive cardiomyopathy
B. Hyperplastic cardiomyopathy
C. Hypertrophic cardiomyopathy
D. Obliterative cardiomyopathy
E. * Dilated cardiomyopathy
34. A 3-month-old girl is being evaluated for feeding difficulty and failure to thrive. Physical
examination finds pallor, peripheral cyanosis, tachypnea, and fine expiratory wheezing. Chest x-ray
shows cardiac enlargement. She is admitted to the hospital, quickly develops severe cardiac failure,
and dies 3 days after admission. At the time of autopsy the endocardium is found to have a “cream
cheese” gross appearance. Histologic sections from this area reveal thickening of the endocardium
due to a proliferation of fibrous and elastic tissue. Which of the following is the most likely
diagnosis?
A. Dilated cardiomyopathy
B. Hypertrophic cardiomyopathy
C. Infective endocarditis
D. Libman-Sachs endocarditis
E. * Restrictive cardiomyopathy
35. Shortly before death a patient got an electrocardiographically based diagnosis of acute myocardial
infarction. Autopsy revealed that the myocardial cavity contained 200 ml of liquid blood and 400 g of
clots; the posterior wall of the left ventricle had a perforation up to 2 cm long. What complication of
myocardial infraction is it?
A. Haemorrhagic pericarditis
B. Exudative pericarditis
C. Idiopathic myocarditis
D. Stone heart
E. * Myocardial rupture with cardiac tamponade
36. Dystrophic changes in the heart muscle are accompanied by an enlargement of the heart cavity, a
decrease in the strength of the heart, an increase in the volume of blood that remains during systole in
the cavity of the heart, overflow of the veins. For what heart condition is this characteristic?
A. Tamponade of heart
B. Tonic dilatation
C. Emergency stage of hyperfunction and hypertrophy
D. cardiosclerosis
E. * Myogenic dilatation

37. An autopsy of a male patient, who died from heart failure, revealed an enlarged heart weighing 550
g, fibrinous pericarditis, as well as contracted dense kidneys weighing 50 g each and having a fine-
grained surface. Microscopically, the kidneys were characterized by an expressed hyalinosis of
arteioles and glomeruli. Name the basic disease.
A. Atherosclerosis
B. Rheumatism
C. Pericarditis
D. Cardiomyopathy
E. * Hypertensive disease
38. A 67-year-old male patient was suffering from hypertensive disease during 20 years. He died from
chronic renal insufficiency. What appearance did his kidneys have on autopsy?
A. Large, motley
B. Large, red
C. Large, white, dense
D. Small, dense, macrotuberous
E. * Small, dense, with a fine-grained surface
39. Against a background of hypertensive crisis, a male patient with hypertensive disease developed
acute renal insufficiency which caused his death. What morphological changes in the renal arteioles
were the most probable?
A. Stenosing atherosclerosis
B. B. Hyperelastosis
C. Hyalinosis
D. Sclerosis
E. * Fibrinoid necrosis
40. A 56-year-old male patient with elevated blood pressure (250/120 mm Hg) died from an impairment
of his cerebral circulation. An autopsy of the brain revealed a red focus in the thalamus, 2.5 cm in
diameter, which sank on section. Microscopically, there was fibrinoid necrosis of the vascular walls
and impregnation of the necrotized brain tissue with blood. Which of the diagnoses listed below was
the most probable?
A. Cerebral haematoma
B. Anaemic infarct of brain
C. Mixed infarct
D. Atheromatosis
E. * Haemorrhagic infarct of brain
41. A 2-month-old girl is being examined for a routine checkup. She was born at term, and there were no
problems or complications during the pregnancy. The baby appeared normal at birth and has been
asymptomatic. Physical examination at this time finds a soft systolic murmur with a systolic thrill.
No cyanosis is present, and her peripheral pulses are thought to be within normal limits. An ECG
reveals slight left ventricular hypertrophy. Which of the following is the most likely diagnosis?
A. Coarctation of the aorta
B. Patent ductus arteriosus
C. Persistent truncus arteriosus
D. Tetralogy of Fallot
E. * Ventricular septal defect
42. In a clinical study of tetralogy of Fallot, patients are examined before surgery to determine predictors
observed on echocardiography that correlate with the severity of the disease and the need for more
careful monitoring. A subset of patient is found to have more severe congestive heart failure, poor
exercise tolerance, and decreased arterial oxygen saturation levels. Which of the following is most
likely to predict a worse clinical presentation for these patients?

A. Size of the left ventricle


B. Size of the ventricular septal defect
C. Diameter of the tricuspid valve
D. Presence of an atrial septal defect
E. * Degree of pulmonary stenosis
43. A 7-year-old child has thick, painless nodules 1-2 mm in size on the extensor surfaces of the elbow
and knee joints. In the biopsy of the nodules - large foci of fibrinoid necrosis of the connective tissue
with lymphocytes and macrophages on the periphery. Specify the disease in which such nodules are
observed?
A. Systemic lupus erythematosus
B. Rheumatoid arthritis
C. Scleroderma
D. Periarteritis nodosa
E. * Rheumatism
44. In a patient who died of heart failure, during a pathological examination, it was discovered: the mitral
valve leaflets are deformed, thickened, joined at the edges; in the connective tissue of the
myocardium there are diffusely located nodules, which consist of areas of fibrinoid necrosis, around
which are clusters of macrophages resembling giant multinucleated cells. The foci are surrounded by
lymphocytes and single plasma cells. Which of the following granulomas takes place in this patient?
A. Leprous
B. Tuberculous
C. Actinomycotic
D. Syphilitic
E. * Rheumatic
45. The disease that is characterized by the systolic murmur without signs of mitral valve failure is called
as:
A. Rheumatic heart disease
B. Congenitally bicuspid aortic valve
C. Mitral annular calcification
D. Degenerative calcific aortic valve stenosis
E. * Mitral valve prolapse
46. Etiologically and pathogenetically, rheumatic fever and rheumatic heart disease are characterized by
all of the following, except:
A. Initial attack of illness some weeks after streptococcal infection
B. Elevated serum titers of antibodies to streptolysin and hyaluronidase
C. Sterile tissue lesions not resulting from direct bacterial invasion
D. Recurrent acute illness following the streptococcal infection
E. * Decreased serum protein levels
47. Heart lesions in acute rheumatic fever are called as which of the following:
A. Tuberculoma
B. Mitral stenosis
C. Foreign bodies
D. Gumma
E. * Aschoff bodies
48. The Anitschkow cells are characterized by all of the following, except:
A. Abundant amorphophilic cytoplasm
B. Central round-to-ovoid nucleus
C. Central disposed chromatin (caterpillar cells)

D. Monocytic cytogenesis
E. * B-lymphocytic cytogenesis
49. Child 3 month old, which has been not vaccinated, has died because of suppurative peritonitis. The
perforative ulcer of small intestine has been found in autopsy. The mesenteric lymph nodule have
been dense, their cut surface had caseous mass. Diagnosis of tuberculosis has been determined. Call
the rout of spreading of tuberculosis.
A. Child has primary tuberculosis
B. The primary tuberculose complex is found in autopsy
C. Tuberculosis is transmitted with food
D. The exudative and necrotic reactions are predominate
E. * Disease appears due to transplacental passway
50. During the histological examination of the biopsy the pathologist has found out granulomas within
the livers. They contain mainly T-lymphocytes and epithelioid cells, and solitary giant Langhan’s
cells. In the center of granulomas there was a small area of caseous necrosis. What pathologic process
is characterized by those changes?
A. Coagulative necrosis.
B. Liquefactive necrosis.
C. Neoplasm.
D. Exudative inflammation.
E. * Proliferative inflammation.
51. Autopsy of man aged 60 demonstrated numerous whitish millet-grain-sized nodes in the lungs and
liver. Microscopy demonstrated granulomas with foci of necrosis in the center and epithelioid,
lymphoid, plasmatic cells on the periphery as well as macrophages and numerous Pirogov-Langhans
cells in the infiltrations. Which of the listed granulomas is most probable?
A. Phagocytic
B. Epithelioidcellular
C. Macrophage
D. Foreign-body granuloma
E. * Giant-cell
52. During the microscopic examination of bioptic fragment of the skin the granulomas containing
epithelioid cells, surrounded with T-lymphocytes were found out. Between the epithelioid cells the
solitary giant polynuclear Langhan’s cells located. There were areas of caseous necrosis in the center
of some granulomas. Blood vessels were absent. What disease do such changes characterize?
A. Syphilis
B. Leprosy
C. Rhinoscleroma
D. Hodgkin’s disease
E. * Tuberculosis
53. The disease in a male hunter began with an elevation of his body temperature up to 37-38(C,
increased reflex excitability, a disturbance of sleep and hydrophobia. Later these signs were
accompanied by spasms of the muscles of the larynx and pharynx, as well as those of respiration. The
patient’s death was caused by arrest of respiration. On autopsy, an oedema and plethora of the brain,
as well as small haemorrhages in the region of the myelencephalon were found. On histological
examination of the brain, its stem part, walls of the 3rd ventricle and hippocampus revealed necrosis
of the nerve cells which were surrounded (as well as small vessels) by nodules consisting of clusters
of microglial and lymphoid cells. The cytoplasm of the nerve cells of the hippocampus contained
some rounded eosinophilic inclusions (Babes-Negri bodies). What disease is characterized by the
picture described?
A. Typhoid fever
B. Epidemic typhus

C. Poliomyelitis
D. Tick-borne encephalitis
E. * Rabies
54. The disease in a 67-year-old woman acutely began with an expressed oedema and tenderness of the
skin and soft tissues of the neck. A phlegmon of neck and mediastinitis were diagnosed. The patient
died under the increasing phenomena of intoxication. On autopsy, the left tonsil was slightly enlarged
and dense; on section, it was yellowish-greenish and had a lot of small cavities which imparted a
honeycomb structure to it. The soft tissues of the neck and the fat of the anterior mediastinum had
signs of purulent melting. Microscopically, the tissue had a lot of small abscesses, their centres
having intensive basophilic formations, which consisted of short rod-like elements connected with
their one end to the common centre. What is your diagnosis?
A. Giardiasis
B. Leishmaniasis
C. Brucellosis
D. Amoebiasis
E. * Actinomycosis
55. A male patient, who came from the Central Asia, had persistent diarrhoeae, a loss of body weight and
signs of intoxication against whose background he died. An autopsy revealed numerous hepatic
abscesses, the caecum was characterized by dingy green areas of necrosis of its mucous membrane,
these areas slightly rose above its surface and penetrated into the muscular layer. The ulcers resulting
from the necrosis were characterized by undermined edges which hung over their bottom. The
inflammatory reaction in the intestinal wall was poorly expressed. What was the most probable
disease in that case?
A. Salmonellosis
B. Cholera
C. Typhoid fever
D. Bacterial dysentery
E. * Amoebiasis
56. An autopsy of a 45-year-old male, who had had a fever with signs of intoxication during his life-time,
revealed an enlarged dense spleen (500 g); on section, its pulp had numerous grey-white and white-
yellow miliary necroses of follicles, and there were infarct-like foci of necrosis under the capsule. A
histological examination revealed hyperplasia of the follicles with breakdown of leukocytes and
accumulation of neutrophils, and numerous thrombi in the vessels. Which of the diagnoses listed
below was the most probable?
A. Plague
B. Haematogenous general miliary tuberculosis
C. Tularaemia
D. Typhoid fever
E. * Relapsing fever
57. The examination of the child with measles showed the non-clear border edematous fluctuated areas
of red-black color in the soft tissues of the cheeks and perineum. What complication did develop in
the child?
A. Dry gangrene
B. Gas gangrene
C. Bedsore
D. Trophic ulcer
E. * Wet gangrene (noma)

58. A 8 year-old child was ill acutely with clinical signs of vomiting, headache and severe intoxication.
After two days of the disease he has died. In autopsy the pathologist has found out: meninges
thickened, yellowish color on basal surface, edema and hyperemia. Meningococcus was detected
from liquor fluid. Diagnose this disease.
A. Scarlet fever
B. Pertussis
C. Diphtheria
D. Measles
E. * Meningococcal infection
59. A 5 year-old girl has died because of asphyxia owing to true croup. In the autopsy it was established;
mucosa of larynx, trachea and bronchi dwarfed, edematous, dull, coated by grayish fibrinous plaques,
which were easily removed. Described morphological changes are characteristic for…:
A. Flu
B. Measles
C. Pertussis
D. Scarlet fever
E. * Diphtheria
60. A 6 year-old child, was ill acutely with signs of intoxication. In 2 day the patient has died. In autopsy
the pathologist has found out: meninges of brain with edema, hyperemia, yellow-grey exudate.
Tissue of brain was edematous. Microscopic investigation: there were neutrophils, hyperemia,
hemorrhages and edema in meninges. Described changes are most typical for:
A. Flu
B. Pertussis
C. Diphtheria
D. Measles
E. * Meningococcal meningitis
61. Patient has suffered from cholera. Clinical dates are dehydratating, cyanosis and convulsions. In the
result of massive infusion therapy the exicosis has been diminished, but anuria has been remained.
Patient has dead because of uremia. What morphological features in kidney have been found out?
A. Choleric typhoid is developed
B. Development of uremia is connected with acute glomerulonephritis
C. Fibrinous colitis is found in autopsy
D. Exicosis is due to action of virus exotoxin
E. * Necrotic nephrosis with cortical necrosis takes place in the kidneys
62. An autopsy of a 45-year-old male, who suffered from numerous pathological fractures during his
lifetime, revealed changes in his long tubular bones: the bones of the thigh and shin were bent, in
some places they resembled spirals, their surface was tuberous, a section revealed an obliterated
medullary channel and a change in the compact structure of the cortical layer by the spongy type.
Microscopically, there was a mosaic type of the bone structures: against a background of a disordered
thin-fibrous or lamellar structure of the bone fragments there were numerous cavities of sinusal
resorption combined with signs of new formation of the osseous tissue. The arteries, which supplied
the bone tissue, were dilated and convoluted. Name a diagnosis.
A. Osteopetrosis
B. Parathyroid osteodystrophy
C. Fibrous dysplasia
D. Chronic osteomyelitis
E. * Deforming osteodystrophy

63. A 20-year-old girl developed complaints about an expressed fatiguability of her ocular, masticatory,
speech and deglutitive groups of muscles, when the normal contraction of the muscles after great
activity absolutely discontinued, but after some rest the functioning of the muscles was restored
again. Some time later the pathological process involved the muscles of the extremities and
intercostal ones. An inadequate ventilation of the lungs resulted in development of the secondary
lobular pneumonia which caused the patient’s death. An autopsy revealed an atrophy of the striated
muscles, their dystrophy with focal clusters of the lymphocytes in the interstice. An enlarged thymus
was characterized by follicular hyperplasia. What was the most probable diagnosis?
A. Pseudohypertrophic muscular dystrophy
B. Amyotrophic lateral sclerosis
C. Werdnig-Hoffman spinal amyotrophy
D. Erb’s muscular dystrophy
E. * Myasthenia
64. An autopsy of a male, who died from uraemia, revealed deformity of the spinal column with a sharp
limitation of mobility. The articular cartilages of small joints of the spinal column were destroyed,
there were some expressed signs of a prolonged chronic inflammation in the articular tissues, the
cavities of the joints were filled with the connective tissue, but in some places with the osseous one
together with formation of ankyloses. The aorta, heart and lungs revealed a chronic inflammation and
focal sclerosis. The kidneys were characterized by amyloidosis. What diagnosis was the most
probable in this case?
A. Rheumatoid arthritis
B. Paget’s disease (deforming osteosis)
C. Parathyroid osteodystrophy
D. Osteopetrosis (marble bone disease)
E. * Bekhterev’s disease
65. On autopsy of a male, who died from uraemia, it was found that the pancreas was reduced in size, his
contracted kidneys had a fine-grained surface, the liver was enlarged, yellow and flaccid.
Microscopically, the pancreatic tissue revealed an atrophy of the parenchyma, including islets of
Langerhans, the atrophied parenchyma was substituted for hyperplastic connective and fatty tissues.
The kidneys were characterized by sclerosis and hyalinosis of the glomeruli, as well as by a
glycogenic infiltration of the tubules; there was a fatty degeneration in the liver and a fibrinous
inflammation in the mucous coats of the trachea, bronchi and stomach. What disease did the died
person suffer from?
A. Chronic glomerulonephritis
B. Hypertensive disease
C. Chronic indurative pancreatitis
D. Steatosis
E. * Diabetes mellitus
66. An autopsy of a male, who died from chronic renal insufficiency, revealed atherosclerosis of the
aorta and large arteries, small and dense kidneys with a fine-grained surface, an enlarged yellow-
brown and flaccid liver, the pancreas was reduced in size. Microscopically, there was
atherocalcinosis of the aorta and arteries, an atrophy of the parenchyma, sclerosis and lipomatosis of
the pancreas; the kidneys were characterized by hyalinosis of the mesangium and glomeruli, a
glycogenic infiltration of the epithelium of the tubules, with large-drop adiposis in the hepatocytes.
What pathological process took place in the kidneys?
A. Arterial nephrosclerosis
B. Chronic pancreatitis
C. Chronic glomerulonephritis
D. Steatosis
E. * Diabetic nephrosclerosis

67. A 52-year-old male died from renal insufficiency. On microscopic examination of his organs, the
pancreas revealed lipomatosis and sclerosis with an atrophy of islets of Langerhans, the kidneys had
hyalinosis of the mesangium and glomeruli (Kimmelstiel-Wilson syndrome) and a glycogenic
infiltration of the epithelium of the tubules, the liver was characterized by fatty degeneration. Which
of the diagnoses listed below was the most probable?
A. Arterial nephrosclerosis
B. Amyloid shrunk kidneys
C. Chronic glomerulonephritis
D. Goodpasture’s syndrome
E. * Diabetic glomerulosclerosis
68. A 53-year-old male patient, who suffered from peptic ulcer of the stomach for more than 25 years,
was admitted to a surgical department with complaints about frequent vomiting after taking food,
progressing loss of weight, severe thirst. At the hospital, the signs of oliguria and later anuria
developed. The patient died. An autopsy revealed a cicatricial stenosis of the pylorus and a sharp
enlargement of the stomach which practically reached the pelvic region. Which of the complication
of peptic ulcer listed below caused the patient’s death?
A. Peritonitis
B. Penetration of ulcer
C. Erosive haemorrhage
D. Malignancy
E. * Chlorhydropenic uraemia
69. A 47-year-old woman underwent radical mastectomy for a neoplasm. A histological examination of
the mammary gland revealed an eczematous lesion of the nipple and areola, a cancerous lesion of the
ducts of the gland and presence of large light cells in the epidermis of the nipple and areola. Make a
diagnosis.
A. Intralobular carcinoma in situ
B. Acneiform carcinoma
C. Papillary carcinoma
D. Fibrous carcinoma
E. * Paget's disease
70. A histological express examination of a tumour node of a mammary gland revealed some
encapsulated formation with proliferation of alveoli and intralobular ducts; the interstitial connective
tissue grew either around or inside the ducts. Which of the tumours took place?
A. Foliaceous tumour
B. Noninfiltrating intralobular carcinoma
C. Infiltrating intralobular carcinoma
D. Paget's disease
E. * Fibroadenoma
71. During an operation on a woman, her cyst-like changed ovary was removed; it was a thin-walled
cavity filled with some yellowish transparent fluid and having a smooth inner surface. Histologically,
the cavity wall was lined with the cubical epithelium. Name the kind of the tumour.
A. Mucinous cystadenoma
B. Serous cystadenocarcinoma
C. Pseudomucinous cystocarcinoma
D. Granulosa cell tumour
E. * Serous cystadenoma

72. A histological examination of a biopsy from a uterine cervix revealed that its tissue was covered with
a wide layer of the stratified squamous epithelium having foci of proliferation of atypical cells with
pathological mitoses, but the basal membrane of the epithelium was not affected. What is your
diagnosis?
A. Nonkeratinizing squamous cell carcinoma
B. Keratinizing squamous cell carcinoma
C. Leukoplakia
D. Epithelial dysplasia
E. * Carcinoma in situ
73. An autopsy of a male, who died from chronic renal insufficiency, revealed atherosclerosis of the
aorta and large arteries, small and dense kidneys with a finegrained surface, an enlarged yellow-
brown and flaccid liver, the pancreas was reduced in size. Microscopically, there was
atherocalcinosis of the aorta and arteries, an atrophy of the parenchyma, sclerosis and lipomatosis of
the pancreas; the kidneys were characterized by hyalinosis of the mesangium and glomeruli, a
glycogenic infiltration of the epithelium of the tubules, with large-drop adiposis in the hepatocytes.
What pathological process took place in the kidneys?
A. Arterial nephrosclerosis
B. Chronic pancreatitis
C. Chronic glomerulonephritis
D. Steatosis
E. * Diabetic nephrosclerosis
74. On autopsy of a male, who died from uraemia, it was found that the pancreas was reduced in size, his
contracted kidneys had a fine-grained surface, the liver was enlarged, yellow and flaccid.
Microscopically, the pancreatic tissue revealed an atrophy of the parenchyma, including islets of
Langerhans, the atrophied parenchyma was substituted for hyperplastic connective and fatty tissues.
The kidneys were characterized by sclerosis and hyalinosis of the glomeruli, as well as by a
glycogenic infiltration of the tubules; there was a fatty degeneration in the liver and a fibrinous
inflammation in the mucous coats of the trachea, bronchi and stomach. What disease did the died
person suffer from?
A. Chronic indurative pancreatitis
B. Chronic glomerulonephritis
C. Hypertensive disease
D. Steatosis
E. * Diabetes mellitus
75. An autopsy of a male revealed a tumour in the anterior lobe of the hypophysis, enlarged adrenal
glands, a reduction of the gonads in size, a hypertrophy of the left cardiac ventricle, the pancreas was
reduced in size and thickened. Histologically, there was a pituitary basophil adenoma and a
hyperplasia of the cortical layer in the adrenal glands. The pancreas was characterized by a
moderately expressed atrophy of the parenchyma, including islets of Langerhans. What disease did
the patient suffer from?
A. Diabetes mellitus
B. Adiposogenital dystrophy
C. Cushing's syndrome
D. Simmonds disease
E. * Cushing's disease
76. For a histological examination, a lobe and a part of the isthmus of the thyroid gland were received.
The tissue of the gland was dense and tuberous, on section it was pale brown and had grey-whitish
foci. Microscopically, against a background of an atrophy of the follicles of the gland, there was
some diffuse lymphoplasmacytic infiltration of the stroma with formation of lymphoid follicles.
What pathological process were these changes typical for?

A. Toxic goiter
B. Thyroid adenoma
C. Colloid goiter
D. Sporadic goiter
E. * Allergic thyroiditis
77. A histological examination of a thyroid gland revealed follicles of various size and shape which were
lined with the columnar epithelium; the latter proliferated and formed papillae of various size. The
follicular lumens contained some liquid and vacuolized colloid. The stroma of the gland was
characterized by a lymphoplasmacytic infiltration, in some places with formation of lymphatic
follicles having light centres. Which of the diagnoses was the most probable?
A. Colloid goiter
B. Nodular goiter
C. Hashimoto's disease
D. Ligneous thyroiditis
E. * Toxic goiter
78. An autopsy of a 48-year-old male, who died from vascular collapse, revealed an increased
pigmentation of the skin, the adrenal glands were reduced in size, the brown-yellow liver was
enlarged. On histological examination, foci of necrosis with a tuberculous granulation tissue were
found in the adrenal glands. The liver was characterized by phenomena of fatty degeneration. Which
of the diagnoses was the most probable?
A. Steatosis
B. Primary aldosteronism
C. Cushing's syndrome
D. Lipofuscinosis
E. * Addison's disease
79. An autopsy of a 45-year-old male, who suffered from numerous pathological fractures during his
lifetime, revealed changes in his long tubular bones: the bones of the thigh and shin were bent, in
some places they resembled spirals, their surface was tuberous, a section revealed an obliterated
medullary channel and a change in the compact structure of the cortical layer by the spongy type.
Microscopically, there was a mosaic type of the bone structures: against a background of a disordered
thin-fibrous or lamellar structure of the bone fragments there were numerous cavities of sinusal
resorption combined with signs of new formation of the osseous tissue. The arteries, which supplied
the bone tissue, were dilated and convoluted. Name a diagnosis.
A. Osteopetrosis
B. Parathyroid osteodystrophy
C. Fibrous dysplasia
D. Chronic osteomyelitis
E. * Deforming osteodystrophy
80. An autopsy of a male, who suffered from right-sided pneumonia in the lower lobe during his life-
time and for a long period of time expectorated sputum of a purulent character, revealed some cavity
with dense edges that was located in the 9th-10th segments of the lung and was filled with yellowish
cream-like masses. There was some whitish path from the cavity to the root of the lung.
Microscopically, the cavity was separated from the intact pulmonary tissue with a membrane which
consisted of a fibrous connective tissue from the outside and a granulation one from inside. Which of
the diagnoses was the most probable?
A. Bronchiectatic disease
B. Pulmonary gangrene
C. Acute pulmonary abscess
D. Chronic pneumonia
E. * Chronic abscess

81. For a histological examination, a vermiform process (appendix) was sent. Its size is increased, the
serous membrane is dim, plethoric and covered with greyish films, the wall is thickened and some
pus is discharged from the lumen. Microscopically, a plethora of the vessels, an oedema of all the
layers and their diffuse infiltration by leukocytes are observed. Name the kind of inflammation in the
vermiform process.
A. Catarrhal
B. Putrid
C. Mixed
D. Fibrinous
E. * Phlegmonous
82. An examination of a 7-year-old child, who was referred to infectious department with complaints
about a sharp pain in his throat, difficult swallowing, an elevated body temperature up to 39cc, an
oedema of his neck, revealed that the tonsils were enlarged, their mucosa was plethoric and covered
with a large number of yellow-whitish films which were closely adjacent to the mucosa. an attempt to
remove a film results in a deep bleeding defect. what kind of inflammation takes place?
A. Suppurative
B. Serous
C. Croupous
D. Haemorrhagic
E. * Diphtheritic
83. A male was treated for purulent otitis. On the 9th day of his staying at an inpatient department he
died from a brain oedema. On autopsy, the temporal region of the left hemisphere revealed a cavity
with uneven rough inner edges which was filled with some yellowish-greenish thick dull fluid. The
outer wall of the cavity was represented with the cerebral tissue. What pathological process was it?
A. Colliquative necrosis
B. Phlegmon
C. Empyema
D. Chronic abscess
E. * Acute abscess
84. An autopsy revealed 0.5 I of some yellowish transparent fluid with small white crumble clots in the
right pleural cavity. The parietal and visceral pleurae were covered with a white crumble coat. What
kind of exudative inflammation was it?
A. Suppurative
B. Serous
C. Putrid
D. Catarrhal
E. * Croupous
85. A 6-year-old girl fell ill with diphtheria and three days later died of asphyxia resulting from
membranous croup. On autopsy, the mucous membranes of the larynx, trachea and bronchi were
thickened, oedematous and covered with greyish films which were easily separated. What kind of
inflammation did the morphological changes in the larynx indicate?
A. Serous
B. Haemorrhagic
C. Diphtheritic
D. Croupous
E. * Catarrhal
86. An autopsy of a 77-year-old male, who died from dysentery, revealed some grey-yellow films which
were closely connected with the underlying tissues in the colon and separated with formation of
ulcers. Name the kind of inflammation.

A. Serous
B. Catarrhal
C. Croupous
D. Suppurative
E. * Diphtheritic
87. A microscopic examination of the tissue dissected from some postoperative infiltrate revealed
granulomata with giant multinucleate cells around the suture material. What kind of granulomata did
they belong to?
A. Tuberculous
B. Rheumatic
C. Lepromatous
D. Mycotic
E. * Foreign-body
88. An examination of a renal biopsy revealed some mostly perivascular and periglomerular
lymphocytic, plasmacytic and macrophagal infiltration of the interstice against a background of its
sclerosis. Name the most probable kind of inflammation.
A. Productive diffuse
B. Granulomatous
C. Exudative diffuse
D. Exudative focal
E. * Productive focal
89. A microscopic examination of the myocardium in a male, who died from cardiac decompensation,
revealed sclerosis of the perivascular connective tissue and its diffuse infiltration by lymphocytes,
macrophages, plasmacytes and solitary neutrophils. Which of the listed kinds of inflammation was
the most probable?
A. Granulomatous productive
B. Alterative
C. Exudative diffuse
D. Exudative focal
E. * Interstitial productive
90. A little girl died of asphyxia resulting from membranous croup. A girl illed with diphtheria. On
autopsy, the mucous membranes of the larynx, trachea and bronchi were thickened, oedematous and
covered with greyish films which were easily separated. What type of inflammation
pathomorpholodist investigated?
A. Serous
B. Haemorrhagic
C. Diphtheritic
D. Croupous
E. * Catarrhal
91. A surgeon removed a tumour in the liver of a 47-year-old male patient. It was macroscopically
revealed that the wall of the cavity was formed by a dense fibrous connective tissue; the cavity
contained some yellow-greenish dull thick fluid which had an unpleasant odour and microscopically
consisted mainly of polymorphonuclear leukocytes. What pathological process did such
morphological changes correspond to?
A. Phlegmon
B. Acute abscess
C. Empyema
D. Colliquative necrosis
E. * Chronic abscess

92. A children died from asphyxia at the diphtheria. On autopsy were identified membranous plates in
space of respiratory ways, the mucous membranes of the larynx, trachea and bronchi were thickened,
oedematous and covered with greyish films which were easily separated. What kind of inflammation
did the morphological changes in the larynx indicate?
A. Serous
B. Haemorrhagic
C. Diphtheritic
D. * Croupous
E. Catarrhal
93. An autopsy of a 58-year-old male, who suffered from croupous pneumonia during his life-time and
died of cardiopulmonary insufficiency, revealed 900 ml of some yellow-greenish dull fluid in his
right pleural cavity. The pleural leaves were dull and plethoric. Name the clinical-morphological
form of the inflammation in the pleural cavity.
A. Dry pleurisy
B. Phlegmon
C. Chronic abscess
D. Acute abscess
E. * Empyema
94. An autopsy of a woman, who suffered from the right-sided purulent otitis during her life-time,
revealed a cavity 4 x 3 cm in size in the region of the right temporal lobe that contained some yellow-
green dull viscous fluid. The inner layer of the wall was represented with a yellowish crumble tissue,
the outer layer consisted of a whitish dense tissue. What process were the described changes in the
brain characteristic of?
A. Acute abscess
B. Empyema
C. Grey softening of the brain
D. Haemorrhage
E. * Chronic abscess
95. Microscopic analysis of tissue sampling from patient's skin reveals granulomas that consist of
epithelioid cells surrounded mostly by T-lymphocytes. Among epithelioid cells there are solitary
giant multinuclear cells of Pirogov-Langhans type. In the centre of some granulomas there are areas
of caseous necrosis. Blood vessels are absent. What disease are the described granulomas typical for?
A. * Tuberculosis
B. Syphilis
C. Leprosy
D. Rhinoscleroma
E. Glanders
96. A pathology-histology laboratory received a vermiform appendix up to 2,0 cm thick. Its serous
membrane was pale, thick and covered with yellowish-green films. The wall was flaccid, of grayish-
red colour. The appendix lumen was dilated and filled with yellowish-green substance. Histological
examination revealed that the appendix wall was infiltrated with neutrophils. Specify the appendix
disease:
A. * Acute phlegmonous appendicitis
B. Acute gangrenous appendicitis
C. Acute superficial appendicitis
D. Acute simple appendicitis
E. Chronic appendicitis

97. Autopsy of a 50-year-old man revealed the following changes: his right lung was moderately
compact in all parts, the dissected tissue was found to be airless, fine-grained, dryish. Visceral pleura
had greyish-brown layers of fibrin. What is the most likely diagnosis?
A. * Croupous pneumonia
B. Tuberculosis
C. Bronchopneumonia
D. Interstitial pneumonia
E. Pneumofibrosis
98. A patient with android-type obesity had been suffering from arterial hypertension, hyperglycemia,
glycosuria for a long time and died from the cerebral haemorrhage. Pathologic examination revealed
pituitary basophil adenoma, adrenal cortex hyperplasia. What is the most likely diagnosis?
A. * Itsenko-Cushing's syndrome
B. Diabetes mellitus
C. Acromegalia
D. Pituitary nanism
E. Adiposogenital dystrophy
99. Mucous membrane of the right palatine tonsil has a painless ulcer with smooth lacquer fundus and
accurate edges of cartlaginous consistency. Microscopically: inflammatory infiltrate that consists of
lymphocytes, plasmocytes, a small number of neutrophils and epithelioid cells; endovasculistis and
perivasculitis. What disease is in question?
A. * Syphilis
B. Actinomycosis
C. Tuberculosis
D. Pharyngeal diphtheria
E. Necrotic (Vincent's) tonsillitis
100. On the 5th day of illness a 12 year old child who was treated in the infectious department on account
of influenza felt severe headache, sickness, dizziness, got meningeal signs. The child died 24 hours
later from increasing brain edema. Dissection of cranial cavity revealed that pia maters of brain are
edematic, plethoric, saturated diffusively with bright red liquid. Convolutions and sulci of brain are
flattened. What influenza complication is in question?
A. * Hemorrhagic meningitis
B. Cerebral hemorrhage
C. Venous hyperemia of brain membranes
D. Suppurative leptomeningitis
E. Serous meningitis
1. During autopsy approximately 2,0 liters of pus have been found in the abdominal cavity of the body.
Peritoneum is dull and of grayish shade, serous tunic of intestines has grayish-colored coating that is
easily removable. Specify the most likely type of peritonitis in the patient:
A. Hemorrhagic peritonitis
B. Serous peritonitis
C. Tuberculous peritonitis
D. Necrosis
E. * Fibrinopurulent peritonitis
2. The most characteristic manifestation of malignant tumor is which of the following:
A. Cellular atypia and pleomorphism
B. Compression of surrounding tissue
C. Large size
D. Necrosis
E. * Metastases
3. The most important feature to distinguish the malignant tumor from a benign one is which of the
following:
A. Lack of encapsulation
B. High mitotic rate
C. Necrosis
D. Nuclear pleomorphism
E. * Metastases
4. All of the following neoplasms are malignant, except:
A. Adenocarcinoma
B. Melanoma
C. Seminoma
D. Chorionepithelioma
E. * Papillary cystadenoma
5. Cytokines secreted by tumors that induce angiogenesis and assist the tumor in establishing its blood
supply include all of the following, except:
A. Fibroblast growth factor
B. Transforming growth factor - a
C. Transforming growth factor - b
D. Platelet-derived growth factor
E. * Tumor necrosis factor
6. All of the following genes (known as " tumor suppressor genes ") provide the negative control over
cell proliferation, except:
A. The p53gene
B. The DCC ("Deleted in Colon Cancer") gene
C. The Rb gene
D. The WT-1 gene
E. * The bcl-2 gene
7. The factor assessed in the histological grading of a malignant tumor is which of the following:
A. The number of lymph node metastases

B. The size (diameter) of the primary tumor


C. The extent of invasion of the primary tumor into surrounding structures
D. The presence or absence of liver metastases
E. * The degree of cytological differentiation of the primary tumor
8. A malignant tumor is characterized by all of the following, except:
A. Increased abnormal tissue mass
B. Uncoordinated invasive growth
C. Relatively autonomous growth
D. Metastases
E. * Decreased abnormal tissue mass
9. The growth of neoplasms is critically dependent on which of the following:
A. Localization
B. Neutrophil immigration
C. Lymphatic rainage
D. Inflammatory reaction
E. * Tumor stroma
10. Dysplasia is characterized by all of the following, EXCEPT:
A. Abnormal organization of cells;
B. Loss in the uniformity of individual cells;
C. Loss in cell architectural organization;
D. Variation of cells in size and shape
E. * Replacement of one adult cell type by another adult cell type;
11. Hyperplasia is characterized by which of the following:
A. Increase in the size of cells;
B. Increase in the number of nuclei in cells;
C. Shrinkage in the size of cells;
D. Atypia of cells.
E. * Increase in the number of cells;
12. Hypertrophy as a process is characterized by which of the following:
A. Shrinkage in the size of cells and of the organ;
B. Increase in the number of cells;
C. Abnormal organization of cells;
D. Variation of cells in size and shape
E. * Increase in the size of cells and of the organ;
13. Hypertrophy as an adaptive response is charactirized by which of the following:
A. Pathologic hypertrophy of the breast duringlactation;
B. Pathologic hypertrophy of the uterus duringpregnancy;
C. Hypertrophy of the skeletal muscle cells in apatient with immobilized broken limb;
D. Hypertrophy of the endometrium due toovarian tumor.
E. * Hypertrophy of the skeletal muscle cells in a body-builder;
14. Hypoplasia is characterized by all of the following, EXCEPT
A. Incomplete development of an organ;
B. Decreased number of cells;
C. Underdevelopment of an organ;
D. Decreased function of an organ.
E. * Increased number of cells;

15. Metaplasia is characterized by which of the following:


A. Reversible increase in the size of cells;
B. Irreversible change in which one adult celltype isreplaced by another adult cell type;
C. Reversible abnormal organization of cells;
D. Reversible increase in the number of cells.
E. * Reversible change in which one adult celltype is replaced by another adult cell type;
16. One of the variants of physiologic atrophy is:
A. Atrophy of skeletal muscle by the immobi lized broken limb;
B. Kidney atrophy from pressure (hydronephrosis) ;
C. Atrophy of the endometrium by ovarian tumor;
D. Atrophy of the brain in atherosclerosis
E. * Atrophy of uterus after parturition;
17. Pressure-overloaded (concentric) cardiac hypertrophy is characterized by all of the following,
EXCEPT:
A. Hypertrophy of left ventricle;
B. Increased wall thickness;
C. Normal left cavity diameter;
D. Reduced left cavity diameter.
E. * Dilated left cavity diameter;
18. Simple endometrial hyperplasia is characterized by all of the following, EXCEPT:
A. Increase in the number and size of endometrial glands;
B. Complex endometrial glands;
C. Increase in gland-to-stroma ratio;
D. Dilated endometrial glands.
E. * Atypia of gland cells;
19. The causes of hypertrophy are all of the following, EXCEPT:
A. Mechanical triggers;
B. Trophic triggers;
C. Vasoactive agents.
D. Polypeptide growth factors;
E. * Nervous triggers;
20. The hydronephrosis is characterized by all of the following, EXCEPT:
A. Thinning of the renal parenchyma;
B. Dilatation of the renal pelvis;
C. Dilatation of the renal calyces;
D. Progressive atrophy of the kidney.
E. * Thickening of the renal parenchyma
21. The hyperplasia of hepatic cells that occurs after partial hepatoectomy is an example of:
A. Pathologic hyperplasia;
B. Hormonal hyperplasia;
C. Hormonal hypertrophy;
D. Compensatory hypertrophy.
E. * Compensatory hyperplasia;
22. The hypertrophic and dilated (eccentric) heart is characterized by all of the following, EXCEPT:
A. Decreased mass and diminished left wallthickness;
B. Increased mass and normal left wallthickness;
C. Normal mass and left wall thickness;
D. Normal mass and diminished left wallthickness;
E. * Increased mass and diminished left wallthickness.
23. The nomenclature of tumors is based on which of the following:
A. Stromal component
B. Localization
C. Inflammatory changes
D. Vascular component
E. * Parenchymal component histogenesis
24. Malignant tumors arising from mesenchymal tissue are referred to as:
A. Adenocarcinomas
B. Papillomas
C. Cystadenomas
D. Polyps
E. * Sarcomas
25. The important factors associated with the increasing incidence of tumors are au of the following,
except:
A. Age
B. Diet
C. Environment
D. Genetic makeup
E. * Acute inflammation
26. Morphologic atypia is characterized by all of the following, except:
A. Cell pleomorphism
B. Alteration of parenchyma/ stroma ratio
C. Atypical mitoses
D. Enlarged hyperchromatic nuclei
E. * Metaplasia
27. Anaplasia is characterized by all of the following, except:
A. Cellular and nuclear pleomorphism
B. Formation of tumor giant cells
C. Formation of atypical mitotic figures
D. Hyperchromatic nuclei
E. * Formation of Langhans giant cells
28. Benign tumor arising from smooth muscle cells is called:
A. Myoma
B. Chondroma
C. Rhabdomyoma
D. Fibroma
E. * Leiomyoma
29. Tumor cell invasion into the extracellular matrix can be characterized by all of the following, except:
A. Detachment of the tumor cells from each other
B. Attachment to matrix components
C. Degradation of extracellular matrix
D. Migration of tumor cells
E. * Intravasation
30. Character of growth of high differentiated tumors in relation to surrounding tissues:
A. Exophytic
B. Endophytic
C. Infiltrating
D. Invasive
E. * Expansive
31. Character of growth of malignant tumors in relation to surrounding tissues.
A. Exophytic
B. Expansive
C. Endophytic
D. Apposition
E. * Invasive
32. Secondary changes in tumors are all of the following, except:
A. Necrosis
B. Inflammation
C. Calcification
D. Hemorrhages
E. * Metastases
33. Desmoid is located more often in:
A. Legs
B. Hands
C. Skin of back
D. Langs
E. * Anterior abdominal wall
34. Rhabdomyoma is formed from:
A. Collagen fibers
B. Smooth muscles
C. Histiocytes
D. Fibroblasts
E. * Striated muscles
35. Leiomyoma is formed from:
A. Collagen fibers
B. Histiocytes
C. Myoblast
D. Fibroblasts
E. * Smooth muscles
36. Capillary Hemangioma more frequently occur in:
A. Mans
B. Young women
C. Old age people
D. Women in menopause
E. * Children
37. Osteosarcoma is a malignant tumour which arising from:
A. Chondrocytes
B. Lypoblasts
C. Histiocytes

D. Chondroblast
E. * Atypical cells of osteoblastic type with a lot of mitosises
38. Astroblastoma metastasing by:
A. Blood
B. Lymph
C. Contact|
D. Implantation
E. * Cerebrospinal fluid
39. Ependymoma is glioma connected with:
A. Mesencephalon
B. Cerebelli
C. Spinal cord
D. Medulla oblongate
E. * Ventricular ependyma
40. Name of high differentiated tumor of the peripheral nervous system:
A. Hybernoma
B. Leiomyoma
C. Fibroma
D. Glioma
E. * Schwannoma
41. Name differentiated tumor from fatty tissue:
A. Fibroma
B. Desmoid
C. Histiocytoma
D. Myxoma
E. * Lipoma
42. Thyroid gland with small cysts, lined with atypical epithelium and filled with papillae, the latter
originating from the walls of the cysts and growing into their capsules. Choose rihgt diagnosis:
A. Papillary adenoma
B. Follicular carcinoma
C. Solid carcinoma
D. Carcinoma simplex
E. * Papillary carcinoma
43. Uterine cervix tissue covered with a wide layer of the stratified squamous epithelium having
roliferation of atypical cells with pathological mitoses, but the basal membrane of the epithelium was
not affected. Choose rihgt diagnosis:
A. Nonkeratinizing squamous cell carcinoma
B. Keratinizing squamous cell carcinoma
C. Leukoplakia
D. Epithelial dysplasia
E. * Carcinoma in situ
44. Deformed mucouse membrane of a lobar bronchus consistinyof hyperchromatic nuclei and numerous
pathological mitoses, the growth of the tumour did not spread to the basal membrane of the
epithelium. Choose rihgt diagnosis:
A. Squamous cell carcinoma
B. Adenocarcinoma
C. Solid carcinoma

D. Small-cell carcinoma
E. * Carcinoma in situ
45. The epithelial tumors without specific localization develop from:
A. Mesotheliocytes
B. Fibroblastes
C. Epithelioidcells
D. Epitheliocytes
E. * Squamous epithelium
46. The quiet formed benign tumor with squamous epithelium is known as:
A. Sarcoma
B. Fibroma
C. Lipoma
D. Mioma
E. * Papilloma
47. Not formed malignant tumors are named:
A. Fibroma
B. Adenoma
C. Mioma
D. Papilloma
E. * Carcinoma
48. The quiet formed benign tumor with glandular cells is known as:
A. Fibroma
B. Sarcoma
C. Mioma
D. Papilloma
E. * Adenoma
49. The papilloma looks like:
A. A noddle with narrow flat
B. Cyst
C. Stria
D. Scar
E. * A noddle with papilly flat
50. Adenoma is the quiet formed tumor with:
A. Transitional epithelium
B. Squamous epithelium
C. Epymisium
D. Perimisium
E. * Glandular epithelium
51. All of the following neoplasms are malignant, EXCEPT:
A. Adenocarcinoma
B. Sarcoma
C. Seminoma
D. Chorionepithelioma
E. * Mioma
52. All of the following neoplasms are malignant, EXCEPT:
A. Adenocarcinoma

B. Sarcoma
C. Seminoma
D. Chorionepithelioma
E. * Fybroma
53. A tumor that tends to spread over the surfaces of organs or body cavities rather than metastastring via
blood vessels or lymphatics is which of the following:
A. Colon carcinoma
B. Thyroid carcinoma
C. Renal cell carcinoma
D. Hepatocellular carcinoma
E. * Mesothelioma
54. A sequence of metaplasia-dysplasia-neoplasia typically is known to occur in the oncogenesis of all of
the following cancers, EXCEPT:
A. Squamous cell carcinoma of the bladder
B. Squamous cell carcinoma of the lung
C. Adenocarcinoma of the esophagus
D. Squamous cell carcinoma of the endocervix
E. * Carcinoid tumor of the small bowel
55. Poor prognosis in breast cancer is associated with all of the following, EXCEPT:
A. Extensive angiogenesis and presence of aneuploidy
B. Absence of estrogen receptors
C. Presence of aneuploidy
D. Extensive angiogenesis
E. * Overexpression of the N-myc oncogene
56. The most common benign tumor of the female breast is which of the following:
A. Cystadenoma
B. Sarcoma
C. Fibroma
D. Adenocarcinoma
E. * Fibroadenoma
57. A black tumour, 2 cm in diameter, from the skin of his thigh, microscopically, the tumour consisted
of polymorphous cells, the cytoplasm of most of them includs some brown pigment (with a positive
reaction to DOPA), large number of pathological mitoses was registered. Choose rihgt diagnosis:
A. Carcinoma
B. Sarcoma
C. Carcinosarcoma
D. Nevus
E. * Melanoma
58. Massive exophytic carcinoma on the lesser curvature of the stomach with metastases to the ovaries.
What kind of metastatic spreading took place?
A. Haematogenous
B. Lymphogenous orthograde
C. Implantation
D. Perineural
E. * Lymphogenous retrograde
59. What type of metaplasia may occur in the respiratory tract in habitual cigarette smoker?
A. Epithelial metaplasia: squamous to columnar

B. Metaplasia to undifferentiated mesenchymal cells


C. Connective tissue metaplasia
D. Compensatory metaplasia
E. * Epithelial metaplasia columnar to squamous
60. Carcinoid syndrome development results from producing of which the following substances:
A. Melanin
B. Bilirubin
C. Ferritin
D. Hemosiderin
E. * Serotonin
61. The most common malignant epithelial tumor of the stomach is the following:
A. Gastric lymphoma
B. Gastric myoma
C. Gastric leiomyoma
D. Adenomatous polyp
E. * Gastric carcinoma
62. Invasive breast adenocarcinomas of the female breast include all subtypes, EXCEPT:
A. Scirrhous adenocarcinoma
B. Adenocarcinoma
C. Medullary carcinoma
D. Tubular carcinoma
E. * Carcinoma in situ
63. A main morphologic features of Graves disease include all of the following, EXCEPT:
A. Tall and crowded follicular epithelial cells;
B. Small papillae into the follicular lumen;
C. Lack of fibrovascular cores in papillae;
D. Pale colloid with scalloped margins;
E. * Large leukocytic aggregates in the stroma.
64. A most common cause of the glomeruli damage in systemic lupus erythematosus is which of the
following:
A. Local anaphylaxis;
B. Immunologic tolerance;
C. Genetic deficiency of themonocyte/macrophage system;
D. Genetic deficiency of the complement system.
E. * Immune complex injury;
65. At morphologically exam, delayed-type hypersensitivity is characterized by all of the following,
EXCEPT:
A. Accumulation of the mononuclear cells around venules;
B. Dermal edema;
C. Deposition of fibrin in the interstitium of derma;
D. Accumulation of the mononuclear cellsaround small vein;
E. * Deposition of hemosiderin in the derma.
66. Name the macrophage cytokines which are characterized by all of the following, EXCEPT:
A. Mediate natural immunity;
B. Regulate lymphocytes growth, activation anddifferentiation;
C. Activate inflammatory cells;
D. Affect leukocyte movements.

E. * Inhibit hematopoesis;
67. Organs most commonly and seriously damaged in systemic lupus erythematosus are all of the
following, EXCEPT:
A. Skin;
B. Heart and vessels
C. Joints;
D. Serosal surfaces;
E. * Eye;
68. The all of the following diseases result from Type III hypersensithity (immune complex disorders),
EXCEPT:
A. Glomerulonephritis;
B. Farmer's lung;
C. Systemic lupus erythematosus
D. Hemolitic anemia;
E. * Myocardial infarction;
69. The autoimmune diseases involving a single organ or cell type are all of the following, EXCEPT:
A. Hashimoto thyroiditis;
B. Goodpasture syndrome;
C. Insulin-dependent diabetes mellitus;
D. Myasthenia gravis.
E. * Myocardial infarction;
70. The causes of pathologic atrophy are all of the following, EXCEPT:
A. Loss of endocrine stimulation;
B. Loss of innervation;
C. Diminished blood supply;
D. Decreased workload;
E. * Increased workload
71. The causes of pathologic atrophy are all of the following, EXCEPT:
A. Aging;
B. Pressure;
C. Inadequate nutrition;
D. Denervation
E. * Intracellular fat accumulation;
72. The CD4+ T cells influence the function of all of the following immune system cells, EXCEPT:
A. B lymphocytes;
B. Natural killer cells;
C. Macrophages;
D. CD8+ T cells.
E. * Fibroblasts;
73. The cells affected by HIV are all of the following, EXCEPT:
A. Macrophages;
B. Dendritic cells;
C. Monocytes;
D. CD4+ T cells.
E. * CD8+ T cells;
74. The cells of immune system which have numerous fine dendritic cytoplasmic processus are called:
A. Fibroblasts;

B. Macrophages;
C. Leucocytes;
D. CD4+ T cells
E. * Dendritic cells;
75. The cells which compose the granuloma in type IV hypersensitivity reactions are all of the following,
EXCEPT:
A. Lymphocytes;
B. Macrophages;
C. Epithelioid cells;
D. Giant cells.
E. * Erythrocytes;
76. The complement activation gives rise for all of the following proinflammatory effects, EXCEPT:
A. Release of C3b;
B. Production of chemotactic factors;
C. Release of anaphylatoxins;
D. Formation of membrane attack complex.
E. * Formation of antigens
77. The Disorders of the immune system include all of the following, EXCEPT:
A. Hypersensitivity reactions;
B. Autoimmune diseases;
C. Immunologic deficiency syndromes;
D. Amyloidosis.
E. * Hyalinosis
78. Atrophy is characterized by which of the following:
A. Variation of cells in size and shape;
B. Increase in the size of cells;
C. Abnormal organisation of cells.
D. Increase in the number of cells;
E. * Shrinkage in the size of cells by loss of cell substance;
79. Conditions leading to endometrial hyperplasia include all of the following, EXCEPT:
A. Polycystic ovarian disease;
B. Functioning granulosa cell tumors of the ovary;
C. Prolonged administration of estrogenic substances;
D. Excessive ovarian cortical function;
E. * Prolonged administration of analgesic substances
80. The local factors that influence wound healing are all of the following, EXCEPT:
A. Wound infection;
B. Mechanical factors;
C. Foreign bodies;
D. Size, location and type of the wound
E. * Hormones (glucocorticoids);
81. The massive growth of the gravid uterus with large plump cells is an example of:
A. Pathologic hypertrophy;
B. Pathologic hyperplasia;
C. Dysplasia;
D. Metaplasia.

E. * Hormone induced physiologic hypertrophy


82. The most common clinical manifestation of endometrial hyperplasia is:
A. Menses stopping;
B. Pains;
C. Purulent discharges;
D. Mucous discharges
E. * Abnormal uterine bleeding;
83. What pigment can be found in the cytoplasm of heart and muscle cells in aging atrophy?
A. Melanin;
B. Hemosiderin;
C. Bilirubin;
D. Ferritin.
E. * Lipofuscin
84. Which pathologic process results from dysplasia?
A. Aplasia;
B. Hypoplasia;
C. Hyperpigmentation;
D. Calcification;
E. * Neoplasia
85. Which type of metaplasia may occur in the respiratory tract in habitual cigarette smoker?
A. Epithelial metaplasia: squamous to columnar;
B. Metaplasia to undifferentiated mesenchymalcells;
C. Connective tissue metaplasia;
D. None of these.
E. * Epithelial metaplasia: columnar to squamosus
86. Give the determination of gynecomastia
A. Non-cancerous cystous dysplasia of mammary gland
B. Non-cancerous hyperplasia of mammary gland
C. Malignant dysplasia of mammary gland
D. Non-cancerous sclerotic dysplasia of mammary gland
E. * Non-cancerous dysplasia of breast in men
F. Is compatible with a normal life span if treated
87. Increased blood viscosity (hyperviscosity syndrome) is major complication of all of the following
disorders, EXCEPT:
A. Polycythemia vera
B. Immunoglobulin A myeloma
C. * "Hairy-cell" leukemia
D. Waldenstrom's macroglobulinemia
E. Heavy-chain disease
88. Factor that provides for a definitive diagnosis of hemolytic anemia is which of the following:
A. Red blood cell antibodies
B. * Red blood cell destruction
C. Red blood cell enzyme deficiency
D. Bone marrow erythroid hyperplasia
E. Abnormal hemoglobin
89. Nonimmune hemolytic anemia occurs in patients with which of the following disease:

A. Systemic lupus erythematosus


B. * Malaria
C. Chronic lymphocytic leukemia
D. Hodgkin's disease
E. All named
90. The disorder that most commonly causes iron deficiency anemia is which of the following:
A. Liver cirrhosis
B. Cardiomyopathy
C. * Peptic ulcer
D. Pancreatitis
E. Hemorrhagic stroke
91. The similarity between thalassemia major and thalassemia minor is which of the following:
A. Severity of anemia
B. Incidence of infection
C. Life spans
D. * Characterised by elevated HbF level and reduced HbA2 level.
E. Homozygous form
92. Name causes when the volume of stored blood transfused exceeds their normal blood volume, it
results in dilutional thrombocytopenia and dilution of coagulation factors.
A. * Massive transfusion
B. Transmission of infection
C. Air embolism
D. Thrombophlebitis
E. Circulatory overload
93. An increase in the amount of erythrocytes is the highest in:
A. Patients with erythromia
B. After transfusion of erythrocytes
C. In patients with emphysema of the lungs
D. In patients with heart failure
E. * People who live in mountains
94. Of the following anemias hyperchromic is:
A. B-12 deficiency
B. Metaplasmatic
C. Hemolytic
D. Post-hemmorhagic anemia
E. * Iron-deficiency anemia
95. Megalowblastic hyperplasia of bone marrow arises because of hypovitaminosis of:
A. A
B. В-1
C. C
D. PP
E. * B-12
96. The most characteristic cells of blood in bone marrow during B-12 deficient anemia are:
A. Poikilocyte
B. Megalowblasts
C. Hypochromatic erythrocytes

D. Normoblasts
E. * Anyzocytes
97. Leucosis-systemic tumor diseases:
A. Lymphoid tissue
B. Mesenchymal tissue
C. Endothelial tissue
D. Epithelial tissue
E. * Hemopoetic tissue
98. Bone marrow is red juicy, sometimes a gray tint during:
A. Undifferentiated form of leukosis
B. Jewler’s form of leucosis
C. Lymphogranulamatosis
D. During a child’s form of leucosis
E. * Early form of leucosis
99. All of the following statements correctly describe hereditary spherocytosis, EXCEPT:
A. * Red blood cells have the membrane-associated protein spectrin
B. Mutation in the ankyrin gene is present in most cases
C. Anaplastic crisis
D. Splenectomy is invariably therapeutic
E. -
100. Features of megaloblastic anemias include all of the following, EXCEPT:
A. Hypersegmented neutrophils
B. * Lack of the membrane-associated protein spectrin
C. Increased intramedullary hemolysis
D. Increased extramedullary hemolysis
E. Pancytopenia
101. All of the following statements correctly describe aplastic anemia, EXCEPT:
A. Production of all hematopoietic bone marrow elements is reduced
B. Chemical drug exposure is the most common cause
C. Fanconi's anemia represents an inherited form of the disease
D. No underlying etiology is evident in 50% of cases
E. * Splenomegaly is a characteristic clinical finding
102. Osmotic fragility is characteristic of the erythrocytes in which of the following diseases:
A. Fanconi's anemia
B. Sickle cell anemia
C. Glucose-6-phosphate dehydrogenase deficiency
D. * Hereditary spherocytosis
E. Pernicious anemia
103. Myelophthisic anemia can occur in patients with any of the following conditions, EXCEPT:
A. Miliary tuberculosis
B. Carcinomatosis
C. Myelofibrosis
D. Multiple myeloma
E. * Uremia
104. During chronic lympho leucosis the liver is
A. Shrunken

B. Atrophic
C. Infiltrated by scar tissue
D. Cirrhotic
E. * Enlarged, gray-brown color
105. To the group of paraprotein leukosis belong disease of
A. Hodgkins
B. Waldenstrom’s
C. Langerhans’
D. Letterer-Siwe
E. * Mielomic
106. The well developed atheromatous plaque results from all of the following processes, except:
A. Endothelial injury with adhesion of monocytes and platelets
B. Migration of monocytes from the lumen and smooth muscle cells from the media into the intima
C. Smooth muscle cell proliferation, collagen deposition
D. Lipid absorption by macrophages and smooth muscle cells
E. * Action of membrane-bound tissue factor
107. The chronic endothelial injury in atherosclerosis may be caused by all of the following factors,
except:
A. Hyperlipidemia
B. Hypertension
C. Smoking
D. Immune reactions
E. * Genetic defects
108. The chronic endothelial injury in atherosclerosis includes all pathologic processes, except:
A. Increased endothelial permeability
B. Adhesion of blood monocytes to the endothelium
C. Adhesion of platelets to the intima
D. Insudation of lipoproteins into the vessel wall
E. * Regeneration of endothelial cells
109. The special stain used to identify "foam cells" in atherosclerotic plaque is which of the following:
A. PAS-reaction
B. Picrofuchsin
C. Hematoxylin and eosin
D. Congo red stain
E. * SudanIII
110. The major nonmodifiable risk factors for atherosclerosis are all of the following, except:
A. Increasing age
B. Genetic abnormalities
C. Male gender
D. Family history
E. * Increased water resorption
111. The potentially controllable risk factors for atherosclerosis are all of the following, except:
A. Hyperlipidemia
B. Hypertension
C. Diabetes mellitus
D. Cigarette smoking

E. * Genetic abnormalities
112. What vessels are struck by atherosclerosis?
A. Capillaries
B. Aorta
C. V. cava inferior
D. Veins
E. * Arteries
113. Aschoff giant cells are large mesenchymal cells and are believed to be derived from which of the
following:
A. Fibroblasts
B. Lymphocytes
C. Eosinophils
D. Erythrocytes
E. * Macrophages
114. Atherosclerosis commonly affects the arteries supplying all of the following organs, except:
A. Lower extremities
B. Heart
C. Brain
D. Kidney
E. * Upper extremities
115. Blood vessels affected in atherosclerosis are which of the following:
A. Aorta only
B. Large arteries and large veins
C. Large veins
D. Aorta and arterioli
E. * Aorta and large arteries
116. Cholesterol-laden macrophages and smooth muscle cells in atheromatous plaque are also referred to
as:
A. Giant cells
B. "Popcorn" -cells
C. Epithelioid cells
D. Vacuolated cells
E. * "Foam" cells
117. Material accumulated in macrophages and smooth muscle cells in atherosclerotic plaque is which of
the following:
A. Glycogen
B. Lipohyaline
C. Triglycerides
D. Lipoproteids
E. * Cholesterol esters
118. Type of uncomplicated macroscopic changes of intima arteries in atherosclerosis.
A. Postural thrombosis
B. Plaque detachment
C. Fibrous plaque with ulceration
D. Intramural hematoma
E. * Fat stains and stripes
119. Macroscopic characteristic of fibrous plaques in atherosclerosis

A. A Intimacy is similar to "shagreen skin"


B. Localized in the media
C. Do not rise above intimacy
D. Localized in adventitia
E. * Rise above intimacy
120. A type of calcinosis in atherosclerotic "plaques".
A. Metabolic
B. Primary
C. Metastatic
D. Secondary
E. * Dystrophic
121. Nature foam cells in atherosclerotic plaques
A. Granulocytes
B. Eosinophils
C. Lymphocytes
D. Plasmocytes
E. * Macrophages
122. Morphogenetic factors of parietal thrombosis at atherosclerosis.
A. Laminar flow of blood
B. Changing the rheological properties of the blood
C. Sclerosis
D. Deposition of calcium into a plaque
E. * Turbulent blood flow in the plaque area
123. Coronary artery supplying blood to the anterior part of the inter-ventricular septum is which of the
following:
A. Left main coronary artery
B. Left anterior descending artery
C. Left circumflex coronary artery
D. Proximal marginal coronary artery
E. * Right coronary artery
124. Coronary artery suppying blood to the posterior portion of the interventricular septum is which of the
following:
A. Left main coronary artery
B. Left anterior descending artery
C. Left circumflex coronary artery
D. Proximal marginal coronary artery
E. * Right coronary artery
125. Macroscopic characteristic of fatty spots and strips in atherosclerosis.
A. Rising over intimacy
B. Covered with ulcers
C. Localized in the media
D. Localized in adventitia
E. * Do not rise above intimacy
126. Changes in the aging heart are all of the following, except:
A. Epicardial coronary artery tortuosity
B. Decreased left ventricular cavity size
C. Decreased subepicardial fat

D. Brown atrophy of myocardium


E. * Increased number of inflammatory cells in myocardium
127. Blood vessels affected in systemic hypertension are which of the folowing:
A. Aorta and large arteries only
B. Aorta only
C. Large arteries and large veins
D. Venules
E. * Arterioli
128. The antihypertensive substances produced in the kidney are all of the following, except:
A. Prostaglandines
B. Urinary kallikrein-kinin system
C. Platelet-activating factor
D. Nitric oxide
E. * Thromboxane
129. What is coronary heart disease:
A. Myocardial disease due to exogenous intoxication
B. Myocardial disease due to endogenous intoxication
C. Myocardial disease due to exposure to infections
D. Myocardial disease due to metabolic disorders
E. * Myocardial disease due to absolute or relative failure of the coronary circulation
130. Specify the type of heart attack, depending on the timing of the development since the first signs of
ischemia:
A. Dystrophic
B. Necrotizing
C. Metabolic
D. Chronic
E. * Acute
131. What fatal complication may develop chronic thrombosis of the aneurysm of the heart:
A. Gangrene
B. Adrenal insufficiency
C. Chronic renal failure
D. Renal insufficiency
E. * Cerebral infarction
132. Name form of acute ischemic heart disease:
A. Metabolic myocardial injury
B. Fatty degeneration of the myocardium
C. Interstitial myocarditis
D. Productive myocarditis
E. * Acute ischemic dystrophy
133. Name stage of a myocardial infarction
A. Late
B. Metabolic
C. Compensatory
D. Early
E. * Necrotic
134. Name the morphological expression of chronic the ischemic heart disease:

A. Fatty degeneration of the myocardium


B. Heart obesity
C. Myocardial hypertrophy
D. Heart defect
E. * Diffuse small-focal cardiosclerosis
135. How deadly complications can occur in patients with left side chronic heart aneurism with parietal
thrombosis?
A. Pulmonary infarction
B. Anasarca
C. Fatty degeneration of the myocardium
D. Pulmonary embolism
E. * Spleen infarction
136. The favorable outcome of myocardial infarction:
A. Cyst rupture
B. Purulent fusion
C. Fusion of myocardium
D. A heart attack
E. * Scarring
137. Changes in muscle cells at the stage of acute ischemia
A. Lysis of nuclei
B. Coagulation cytoplasm
C. Lysis of the cytoplasm
D. Increased amount of glycogen
E. * Disappearance of glycogen
138. The most common cause of death in patients with chronic ischemic heart disease:
A. Hemopericardium
B. Cardiogenic shock
C. Asystole
D. Acute heart failure
E. * Chronic heart failure
139. The type of myocardial infarction, depending on the developmental date since the first signs of
ischemia.
A. Chronic
B. Hemorrhagic, ischemic
C. Acute
D. Subacute
E. * Recurrent
140. Specify the type of heart attack, depending on the developmental date since the first symptoms of
ischemia.
A. Focal
B. Necrotic
C. Ischemic
D. Transmural
E. * Primary
141. Macroscopic changes in the heart when acute myocardial infarction.
A. In a myocardium a necrosis of a triangular shape
B. In the myocardium, many small sclerosis centers

C. In the myocardium, the center of a sclerosis of irregular shape


D. The heart has a tiger's look
E. * In a myocardium a necrosis center of irregular shape
142. The morphological changes in the kidney during malignant form of hypertension.
A. Acute nephritis
B. Tubulointerstitial nephrosis
C. Cysts
D. Purulent inflammation
E. * Fibrinoid necrosis of arterioles and capillary loops of the kidney’s glomeruli
143. The cause of death in malignant nephrosclerosis
A. Acute heart failure
B. Chronic heart failure
C. Acute liver failure
D. Chronic renal failure
E. * Acute renal failure
144. Morphology changes in renal hypertension benign course.
A. Heart attack
B. Regeneration glomerular hypertrophy
C. Glomeruli hypertrophy
D. Fibrinoid necrosis of arterioles and glomeruli
E. * Glomeruli sclerosis
145. Morphology preclinical stages of hypertension.
A. Compensatory hypertrophy of the left ventricle
B. Spasm of arterioles
C. Hypertrophy of the muscle layer of arteries
D. Hypertrophy elastic structures arterioles
E. * All the above named
146. Macroscopic characteristics of renal hypertension benign course.
A. Greasy kidney
B. The kidneys are enlarged in size
C. Red
D. Pale
E. * The surface of fine-grained
147. Clinical expression kidney form hypertension.
A. Acute heart failure
B. Chronic heart failure
C. Acute renal failure
D. * Chronic renal failure
E. Amyloidosis
148. Changes in large vessels in hypertension in gross examination.
A. Arteriosclerosis
B. Liposclerosis
C. Calcification
D. Sclerosis
E. * Fatty spots and stripes
149. The clinical syndrome of malignant hypertension includes all ofthe following symptoms, except:

A. Papilledema
B. Retinal hemorrhages
C. Diastolic pressure >120 mmHg
D. Renal failure
E. * Systolic pressure >120 mmHg
150. The humoral constriction factors that lead to the peripheral resistence in hypertension are all of the
following, except:
A. Angiotensin II
B. Catecholamines
C. Leukotriens
D. Thromboxanes
E. * Prostaglandines
151. All of the following factors may contribute to vasoconstriction, except:
A. Increased sensitivity of vascular smooth muscles to constricting agent
B. Behavioral factors
C. Neurogenic factors
D. Increased release of vasoconstric-tive agents
E. * Acute coagulation factor
152. The malignant hypertension can be identified microscopically by all of the following features,
except:
A. Onion-skin concentric laminated thickening of the arteriolar walls
B. Progressive narrowing of the arteriolar lumina
C. Perivascular fibrosis
D. Fibrinoid necrosis of the arteriolar walls
E. * Cholesterol-laden "foam cells" in the arteriolar walls
153. On gross examination, the kidneys in systemic chronic hypertension are characterized by all of the
following features, except:
A. Reduced in size and weight
B. Fine granularity on cortical surface
C. Grain leather-like cortical surface
D. Narrowed cortical surface on section
E. * Hemorrhages on cortical surface
154. The pathologic process in arterioles in malignant hypertension is referred to as:
A. Endothrombovasculitis
B. Thromboangitis obliterans
C. Necrotizing atherosclerosis
D. Productive arteriolitis
E. * Necrotizing arteriolitis
155. Nonbacterial thrombotic endocarditis is characterized by all of the following, except:
A. Mitral and/or aortic valve involvement
B. Systemic emboli development
C. Small (1 to 5 mm), sterile vegetations
D. Vegetations along the line of valve closure
E. * Tricuspid valve involvement
156. Microscopically, most cases of acute myocarditis are characterized by all of th following, except:
A. Nonspecific inflammatory infiltrate
B. Variable amount of interstitial edema

C. Variable degree of myofiber degeneration


D. Variable degree of myofiber necrosis
E. * Marked fibrosis of interstitium
157. The correct order of histologic phases in the development of Aschoff bodies is which of the
following:
A. Early — pathognomonic — intermediate
B. Granulomatous — late — exudative
C. Acute — chronic — healed
D. Nonspecific — specific
E. * Exudative - proliferative – fibrous
158. The valvular effects of chronic rheumatic heart disease are all of the following, except:
A. Thickened and deformed valve leaflets
B. Valvular calcification
C. Thickened and shortened chordae tendineae
D. "Fish-mouth" appearance of mitral valve
E. * Valvular abscess formation
159. Left-sided heart failure is most often caused by all of the following, except:
A. Ischemic heart disease
B. Nonischemic heart disease
C. Hypertension
D. Aortic and mitral valvular diseases
E. * Brown atrophy of myocardium
160. The nonrheumatic degenerative calcific aortic stenosis is characterized by all of the following,
except:
A. Heaped-up calcified masses within aortic cusps
B. Primary involvement of the valvular fibrosa
C. Distorted cuspal architecture
D. Not involved free cuspal edges
E. * Lipid deposition and cellular proliferation
161. The mitral annular calcification is characterized by all of the following, except:
A. Absence of inflammatory changes
B. Irregular, stony hard nodule appearance
C. Possible thromboembolic complications
D. Infective endocarditis
E. * Severely impaired valvular function
162. The Aschoff bodies consist of all of the following, except:
A. Lymphocytes (primarily T cells)
B. Anitschkow cells
C. Plasma cells
D. Fibrinoid degeneration
E. * "Foam cells"
163. Infective endocarditis is characterized by all of the following, except:
A. Microbic invasion into the heart valves
B. Destructive vegetation formation
C. Mural thrombosis
D. Destruction of the underlying cardiac tissues
E. * Amyloid mass deposition

164. Systemic embolism in infective endocarditis may cause infarcts in all of the following organs, except:

A. Brain
B. Kidneys
C. Heart
D. Spleen
E. * Lungs
165. Microscopycally the vegetations of subacute infective endocarditis are characterized by all of the
following, except:
A. Fibrosis
B. Microbic body calcifications
C. Chronic inflammatory infiltrate
D. Granulation tissue at vegetation's bases
E. * Granuloma formation
166. Microscopically, chronic rheumatic heart disease is characterized by all of the following, except:
A. Diffuse fibrosis of leaflets
B. Neovascularization of leaflets
C. Aschoff bodies in perivascular connective tissue
D. Destruction of original leaflet architecture
E. * Acute purulent inflammation
167. Fibrosis bridging across the valvular commissures and calcification in chronic rheumatic heart
disease cause which of the following:
A. Granulomatosis
B. "Tigered effect"
C. Fibrofatty atheromas
D. Starry sky appearance
E. * Fish mouth or buttonhole stenosis
168. The morphological and clinical effects of pure (isolated) right-sided heart failure are all of following,
except:
A. Anasarca
B. Congestive hepatomegaly
C. Ascites
D. Pleural and pericardial effusions
E. * Cerebral hematoma
169. Etiologically and pathogenetically, rheumatic fever and rheumatic heart disease are characterized by
all of the following, except:
A. Initial attack of illness some weeks after streptococcal infection
B. Elevated serum titers of antibodies to streptolysin and hyaluronidase
C. Sterile tissue lesions not resulting from direct bacterial invasion
D. Recurrent acute illness following the streptococcal infection
E. * Decreased serum protein levels
170. Heart lesions in acute rheumatic fever are called as which of the following:
A. Tuberculoma
B. Foreign bodies
C. Gumma
D. Mitral stenosis
E. * Aschoff bodies

171. Nonembolic complications of infective endocarditis include all of the following, except;
A. Valvular insufficiency or stenosis with cardiac failure
B. Myocardial ring abscess
C. Suppurative pericarditis
D. Endocarditis of artificial valves
E. * Acute myocardial infarction
172. Disturbance of connective tissue in rheumatic diseases includes:
A. cell reaction (granulematosis) and sclerosis
B. hyaline droplets dystrophy
C. amyloidosis
D. hydropic dystrophy
E. * Hyalinosis
173. In rheumatism sensitizing factor antibodies:
A. B-hemolytic streptococcus, type B
B. B-hemolytic streptococcus type AB
C. B-hemolytic staphylococcus type D
D. B-hemolytic staphylococcus type C
E. * B-hemolytic streptococcus typeA
174. What antibodies are sensitizing factor in rheumatic conditions:
A. B-hemolytic streptococcus, type B
B. B-hemolytic streptococcus type AB
C. B-hemolytic streptococcus type D
D. B-hemolytic staphylococcus type C
E. * B-hemolytic staphylococcus type A
175. B-hemolytic streptococci type A has affinity with antigens antibodies against
A. liver
B. kidney
C. tonsils
D. connective tissue of mediastinum
E. * connective tissue of heart
176. Kinship antibodies against B-hemolytic streptococci type A antigens from connective tissue of the
heart explains the predominant lesion in patients with rheumatism:
A. liver
B. only epikard
C. only endocard
D. more pericardiocentesis
E. * heart
177. In rheumatoid arthritis in patients with damage mainly to the connective tissue:
A. heart
B. kidney
C. renal capsule
D. liver
E. * capsule of the joints
178. Connective tissue structures are damaged in rheumatoid arthritis:
A. heart
B. kidney

C. renal capsule
D. liver
E. * capsule of the joints
179. In the pathogenesis of rheumatoid arthritis are important:
A. none ANSWER among following is correct
B. immune complexes, where there are different classes of immunoglobulins (Ig N, Ig Q, Ig B)
C. complexes, where there are different classes of immunoglobulins (Ig Y, Ig H, Ig L)
D. immune complexes, where the antigen is a globulin of different classes (Ig M, Ig G, Ig A)
E. * immune complexes, where the antibodies are different classes of immunoglobulins (Ig M, Ig G, Ig A)

180. In systemic lupus erythematosus is violated:


A. exchange of DNA and antigens to components formed the nucleus and cytoplasm
B. exchange of RNA and formation of antibodies to components of the nucleus and cytoplasm
C. exchange of RNA and formation of antibodies
D. exchange of DNA and antigens to components formed the nucleus and cytoplasm
E. * exchange of DNA and formation of antibodies to components of the nucleus and cytoplasm
181. What disease does not belong to a group of rheumatic diseases:
A. Periarthritis nodosa
B. Rheumatic fever
C. Dermatomyositis
D. Scleroderma
E. * Hypertension
182. Bekhterev disease - a chronic rheumatic disease with the damage:
A. Articular apparatus
B. Large joints
C. Small and large joints
D. Heart
E. * The small joints of the spine
183. What disease refers to a group of rheumatic:
A. Atherosclerosis
B. Obesity
C. Myocardial infarction
D. Amyloidosis
E. * SLE
184. What type of necrosis can be found in tuberculous granuloma?
A. Coagulation necrosis
B. Liquefactive necrosis
C. Enzymatic fat necrosis
D. Fibrinoid necrosis
E. * Caseous necrosis
185. Mycobacterium tuberculosis is characterized by all of the following features, EXCEPT:
A. Red colored in acid-fast staining
B. Aerobic
C. Non-spore-forming
D. Nonmotile
E. * Pili-forming

186. Named the least frequent location for miliary extrapulmonary seeding:
A. Bone marrow
B. Kidneys
C. Liver
D. Spleen
E. * Uterus
187. Miliary tuberculosis is associated with which of the following pathologic conditions:
A. Reinfection
B. Localized caseation in the lungs
C. Localized caseation in the lymph node
D. Primary infection
E. * Hematogenous tuberculosis
188. Secondary tuberculosis is characterized by all of the following features, EXCEPT:
A. Caseous necrosis and cavities in the lung
B. Progression directly into the disseminated disease
C. Reinfection with Mycobacteria
D. Reactivation of dormant disease
E. * Primary focus in the lung
189. Caseous focus in tuberculosis may progress into a cavity in which of the following pathologic
conditions:
A. Reactivation of dormant disease
B. Reinfection
C. Caseous pneumonia
D. Lobar exudate consolidation
E. * Erosion into the bronchiole (drainage)
190. Growth and multiplication of the tubercle bacilli in cavernous fibrocaseous tuberculosis are favored
by which of the following pathologic conditions:
A. Lymphatic drainage
B. Progressive hypoxia
C. Increased perfusion
D. Sludging of blood in alveolar capillaries
E. * Increased oxygen tension
191. The cavity in cavernous fibrocaseous tuberculosis is characterized by all of the following features,
EXCEPT:
A. Localized in the apex of the lung
B. Lined by yellow-grey caseous material
C. Walled by fibrous tissue
D. Drained by bronchus
E. * Filled with suppurative exudate
192. Complications of tuberculosis osteomyelitis include all of the following, EXCEPT:
A. Tuberculosis arthritis
B. Sinus tract formation
C. Cold abscess formation
D. Amyloidosis
E. * Caseation in the lung
193. The intestine may be affected in which of the following forms of tuberculosis:
A. Secondary tuberculosis

B. Cavernous fibrocaseous tuberculosis


C. Dormant tuberculosis
D. Miliary tuberculosis
E. * Primary tuberculosis complex
194. Healed lesions in primary tuberculosis include all of the following, EXCEPT:
A. Fibrous incapsulation
B. Focal pleural adhesions
C. Fibrocalcific scar
D. Foci of ossification
E. * Caseous pneumonia
195. Tuberculoma is macroscopically characterized by all of the following features, EXCEPT:
A. Intraparenchymal single mass
B. Greyish-white appearance
C. Well-circumscribed mass
D. Several centimeters in diameter
E. * Several millimeters in diameter
196. Tuberculous salpingitis can be found in which of the following forms of tuberculosis:
A. Secondary tuberculosis
B. Cavernous fibrocaseous tuberculosis
C. Dormant lesions
D. Primary tuberculosis
E. * Miliary tuberculosis
197. The cells playing the most important role in chronic tuberculous inflammation are which of the
following:
A. Leukocytes
B. Eosinophils
C. Erythrocytes
D. Plasma cells
E. * Macrophages
198. The miliary lung tuberculosis is characterized by which type of inflammation:
A. Serous
B. Fibrinous
C. Suppurative
D. Hemorrhagic
E. * Granulomatous
199. Complications of primary tuberculosis:
A. Heart disease
B. Acute pyelonephritis
C. Scleroderma
D. Pneumosclerosis
E. * Meningitis
200. Influenza virus of type A infects all of the following, EXCEPT:
A. Horses
B. Humans
C. Pigs
D. Birds

E. * Dogs
201. The incubation period of the flu continues:
A. 7-10 days
B. 10-20 days
C. 30 days
D. 12-24 years
E. * 2-4 days
202. The electron micrograph of epithelial cells infected by influenza virus is characterized by all of the
following, EXCEPT:
A. Presense of influenza virus particles
B. Hyperchromatic and enlarged nuclei
C. Dilation of cysterns of endoplasmic reticulum
D. Large vacuoles
E. * Displasia of epithelial cells
203. Influenza tracheitis is characterized by all of the following, EXCEPT:
A. Hyperemic and swollen mucosa
B. Narrowing of trachea lumen
C. Pinpoint hemorrhages
D. Grey-yellow mucosa covering films within trachea
E. * Purulent exudate
204. Morphology of influenza pneumonia is characterized by all of the following features, EXCEPT:
A. Diffuse neutrophil infiltration of the bronchiolar walls
B. Dense fibrin-rich neutrophil exudate into adjacent alveoli
C. Foci of necrosis and hemorrhages in lung tissue
D. Foci of necrosis and ulceration in bronchiolar mucosa
E. * Granuloma formation in bronchiolar walls
205. Figuratively the influensa pneumonia is often referred to as:
A. Hemosiderosis of the lung
B. Silicosis
C. Lung infarct
D. Farmer's lung
E. * Multicolored lung
206. Morphologically, measles pneumonia is characterized by all of the following, EXCEPT:
A. Diffuse neutrophil infiltration of the bronchiolar walls
B. Destruction of some fragments of bronchiolar walls and squamous cell metaplasia
C. Exudate with giant cells in the adjacent alveoli
D. Interstitial lung inflammation
E. * Large areas of hemorrhage in lung tissue
207. A histological specimen shows terminal secretory parts of glands made by conic cells with basophilic
cytoplasm and a roundish nucleus in the centre. Specify the type of terminal secretory parts by the
type of secretion:
A. Seromucous
B. Sebaceous
C. Combined
D. Mucous
E. * Serous

208. A 35-year-old patient complains of pain in the upper jaw, bleeding, a slight loosening of teeth. He
has been diagnosed with periodontitis. What is a typical pathological process in this case?
A. Bleeding
B. Caries
C. Pain
D. Redness
E. * Inflammation
209. B-cells secreting IgA in gastrointestinal infections are localized in which of the following parts of the
gastrointestinal tract:
A. Muscle layer
B. Submucosa
C. Serosa
D. Regional lymphatic nodes
E. * Mucosa-associated lymphoid tissue (MALT)
210. Haw many stages of changes of Peyer's patches does the pathogeny of enteric fever include?
A. 2
B. 3
C. 4
D. Hasn't stage development
E. * 5
211. Haw long does everyone of the stage of enteric fever continue mainly?
A. 3 days
B. 1 month
C. 2 weeks
D. 3 weeks
E. * 1 week
212. Haw many days does the incubative period of enteric fever continue mainly?
A. to 2 years
B. 2-3 weeks
C. 1 month
D. 1.5-2 months
E. * 10-14 days
213. Where do the main changes situated at the typhoid fever?
A. in retroperitoneal space
B. in submucous layer
C. in adventitia membrane
D. in serosal membrane (peritoneum)
E. * in mucous membrane
214. Which stage of changes between enumerated at the dysentery is the third?
A. fibrinous colitis
B. catarrhal colitis
C. no indicated here
D. healing of the wound
E. * ulcer formation
215. The preventive measures against gastrointestinal infections are all of the following, EXCEPT:
A. Disposal of waste and vermin
B. Cooking food

C. Cleaning drinking water


D. Hand washing
E. * Killing the parasites which are the transmitters of these infections
216. Normal defenses against ingested gut pathogens include all of the following, EXCEPT:
A. Lytic pancreatic enzymes and bile detergents
B. Secreted IgA antibodies
C. Acid gastric juice
D. Viscous mucous layer covering the gut
E. * Activated C3 complement
217. The term "dysentery" refers to diarrhea associated with all of the following symptoms, EXCEPT:
A. Abdominal cramping
B. Tenesmus
C. Stools containing blood
D. Stools containing pus and mucus
E. * "Rice-water" stools
218. Shigellae cause all of the following pathologic features, EXCEPT:
A. Hemorrhagic colitis
B. Hemolytic-uremic syndrome
C. Damage of endothelial cells in the colon microvasculature
D. Stools containing blood, pus and mucus
E. * Acute tubular necrosis
219. The pathogenic mechanisms of Vibrio cholera infection are all of the following, EXCEPT:
A. Adherence of enterotoxigenic organisms to the enterocyte
B. Changes in the underlying cell cytoplasm
C. Effacement of the apical enterocyte membrane
D. Destruction of the microvillus brush border
E. * Ability to synthesize prostaglandin
220. Measles virus is transmitted by which of the following:
A. Milk
B. Food
C. Respiratory droplets
D. Feces
E. * Blood
221. Measles virus multiplies inside all of the following cells, EXCEPT:
A. Upper respiratory epithelial cells
B. lymphocytes
C. Macrophages
D. T lymphocytes
E. * Hepatocytes
222. "Black measles" is characterized by which of the following:
A. Multiple nevi
B. Hypermelanosis
C. Icterus
D. Hyperchromatosis
E. * Hemorrhages
223. The measles rash results from which of the following:

A. Nonimmune inflammation
B. Delayed — type hypersensitivity
C. Systemic immune complex reaction
D. Antibody-mediated cellular dysfunction
E. * T-cell - mediated immunity
224. Ulcerated mucosal lesions in the oral cavity near the opening of Stensen ducts are referred to as:
A. Vegetations
B. Luschka spots
C. Burkitt spots
D. Paget spots
E. * Koplik spots
225. The term "Koplik spots" referes to which of the following:
A. Ulcerated gastric mucosal lesions
B. Foci of granulomatous inflammation of the oral cavity
C. Foci of purulent inflammation in the skin
D. Ulcerated esophageal mucosal lesions
E. * Ulcerated mucosal lesions in the oral cavity
226. Pathological feature that can be found in the heart in diphtheria is which of the following:
A. Fibrinous pericarditis
B. Heart aneurysm
C. Bacterial endocarditis
D. Myocardial infarction
E. * Toxic myocarditis
227. Nervous system pathology in diphtheria includes which of the following:
A. Intracerebral hemorrhage
B. Ischemic necrosis
C. Hydrocephalus
D. Encephalitis
E. * Polyneuritis
228. The most common cause of death in patients with diphtheria is which of the following:
A. Intracerebral hemorrhage
B. Chronic heart failure
C. Chronic lung failure
D. Acute tubular necrosis
E. * Acute heart failure
229. The disease causing predominant derangement of the upper airways is which of the following:
A. Bacterial pneumonia
B. Tuberculosis
C. Meningococcal nasopharyngitis
D. Cholera
E. * Diphtheria
230. Diphtheria damage of the larynx is characterized by which of the following types of inflammation:
A. Serous
B. Catarrhal
C. Hemorrhagic
D. Granulomatous

E. * Fibrinous
231. Consequences of diphtheria include all of the following, EXCEPT:
A. Hyperplasia of the spleen
B. Fatty myocardial changes
C. Focal necroses of the parenchyma of the organs
D. Polyneuritis
E. * Systemic hemosiderosis
232. Exotoxin in diphtheria damages all of the following organs, EXCEPT:
A. Thyroid gland
B. Nervous system
C. Adrenals
D. Kidneys
E. * Heart
233. Pathological feature in the skin found in meningococcemia is which of the following:
A. Erythematous rash
B. Koplic spots
C. Furuncle
D. Impetigo
E. * Hemorrhagic rash
234. Diphtheria is characterized by the derangement of all of the following organs, EXCEPT:
A. Nasopharynx
B. Oropharynx
C. Larynx
D. Trachea
E. * Esophagus
235. Microscopic features in diphtheria include all of the following, EXCEPT:
A. Neutrophilic infiltration
B. Vascular congestion
C. Interstitial edema
D. Fibrin exudation
E. * Hyaline deposition
236. Meningococcal meningitis is most commonly characterized by which of the following types of
inflammation:
A. Serous
B. Catarrhal
C. Fibrinous
D. Granulomatous
E. * Purulent
237. The stain used to identify the causative organism in pyogenic meningitis is which of the following:
A. Hematoxylin and eosin
B. PAS-reaction
C. Weigert's
D. Congo red
E. * Gram
238. Complications of pyogenic meningitis include all of the following, EXCEPT:
A. Leptomeningeal fibrosis

B. Hydrocephalus
C. Adhesive arachnoiditis
D. Focal encephalitis
E. * Intracerebral hemorrhage
239. The complications of scarlet fever include all of the following, EXCEPT:
A. Poststreptococcal glomeru-lonephritis
B. Retropharyngeal abscess
C. Purulent mastoiditis
D. Phlegmon of the neck
E. * Acute poststreptococcal hepatitis
240. All of the following pathologic processes can be found in the second period of scarlet fever,
EXCEPT:
A. Vasculitis
B. Acute glomerulonephritis
C. Arthritis
D. Endocarditis
E. * Amyloidosis
241. Renal pathology that can be found in patients with scarlet fever is which of the following:
A. Lipoid nephrosis
B. Pyelonephritis
C. Amyloidosis
D. Glomerulosclerosis
E. * Glomerulonephritis
242. The causes of death in meningococcemia include all of the following, EXCEPT:
A. Acute adrenal insufficiency
B. Bacterial shock
C. Acute tubular necrosis
D. Acute heart failure
E. * Cerebral cachexia
243. Acute adrenal insufficiency syndrome is also referred to as:
A. Budd-Chiari syndrome
B. Kimmelstill-Wilson syndrome
C. Hamman-Rich syndrome
D. Zollinger-Ellison syndrome
E. * Waterhouse - Friderichsen syndrome
244. The causative agent of scarlet fever is:
A. Greening streptococcus
B. Diplococcus
C. Staphylococcus aureus
D. Intestinal sticks
E. * Streptococcus group A
245. All of the following clinical features are likely to be found in nephrotic syndrome, EXCEPT:
A. Proteinuria
B. Hypoalbuminemia
C. Hyperlipidemia
D. Edema

E. * Hematuria
246. All of the following statements correctly describe renal artery stenosis, EXCEPT:
A. It is an uncommon form of hypertension
B. It is the most common curable form of hypertension
C. It is usually caused by atherosclerotic plaque
D. It produces high renin levels in the venous blood of the ischemic kidney
E. * It is treated by hemodialysis
247. All of the following statements correctly describe analgesic abuse nephropathy, EXCEPT:
A. It is characterized by tubulo-interstitial component
B. It is often caused by phenacetin
C. It causes inability to concentrate urine
D. It often improves with drug withdrawal
E. * It predisposes to the development of renal cell carcinoma
248. All of the following statements regarding Goodpasture's syndrome are true, EXCEPT:
A. Patients present with hemoptysis and hematuria
B. Death occurs due to uremia and pulmonary hemorrhage
C. Electron microscopy shows the absence of electron-dense deposits
D. Immunofluorescence reveals linear deposits of IgG in the glomeruli
E. * Immunofluorescence reveals granular deposits of IgG in the glomeruli
249. All of the following statements regarding postinfectious glomerulonephritis are true, EXCEPT:
A. The disease follows streptococcus infection
B. Electron microscopy shows large subendothelial immune-type deposits
C. The histologic picture is that of diffuse proliferative glomerulonephritis
D. The clinical picture is characteristic of acute nephritis
E. * Most affected children develop chronic renal failure
250. An immunofluorescence-stained kidney specimen from a patient with poststreptococcal
glomerulonephritis is likely to show which of the following:
A. Linear deposits of Ig G
B. Granular deposits of IgA
C. Linear deposits of streptococcal antigen
D. Granular deposits of streptococcal antigen
E. * Granular deposits of Ig G
251. At extracapillar productive glomerulonephritis half moons are formed by
A. Red corpuscles
B. Neutrophilic leucocytes
C. Red corpuscles and neutrophilic leucocytes
D. Exfoliated endothelium and fibrin
E. * Exfoliated renal epithelium and fibrin
252. Benign nephrosclerosis is characterized by all of the following, EXCEPT:
A. Narrowing of the lumen of the arterioles and small arteries
B. Thickening and hyalinization of the vessels' walls
C. Foci of tubular atrophy
D. Deposition of collagen within the Bowman space
E. * Deposition of amyloid within the Bowman space
253. Changes in a colon at uremia are
A. Granulomatous inflammation

B. Stricture|
C. Polyposis|
D. Ulcerous colitis
E. * Fibrinous [diphtheritic] colitis
254. Diabetes mellitus is associated with all of the following renal disorders, EXCEPT:
A. Diffuse glomerulosclerosis
B. Nodular glomerulosclerosis
C. Benigh nephrosclerosis
D. Acute pyelonephritis
E. * Urate nephropathy
255. Ultrastructural changes in children primary nephrotie syndrome involve which of the following
glomerular elements:
A. Endothelium
B. Mesangium
C. Bloodvessels
D. Basement membrane
E. * Podocytes
256. Uremia is associated with all of the following abnormalities, EXCEPT:
A. Peripheral neuropathy
B. Gastritis
C. Pericarditis
D. Diffuse alveolar damage
E. * Polycythemia
257. What enumerate illness from the listed below relates to the large mottled kidney
A. Amiloidosis of kidney
B. Chronic pyelonephritis
C. Nephrolithiasis
D. Chronic glomerulonephritis
E. * Subacute glomerulonephritis
258. What disease can be complicated by amyloidosis of kidneys:
A. Atherosclerosis
B. Heart ischemic disease
C. Croupous pneumonia
D. Hypertension
E. * Fibrous cavernous tuberculosis of lungs
259. Most forms of chronic renal failure produce increased serum levels of all of the following substances,
EXCEPT:
A. Renin
B. Aldosterone
C. Phosphate
D. Parathormone
E. * Calcium
260. Mesangial cells can be characterized by all of the following properties, EXCEPT:
A. Ingestion of macromolecules
B. Connection with Lacis cells
C. Ability to contract
D. Production of basement membrane proteins

E. * Production of renin
261. In immunologicahy mediated glomerulonephritis all of the following cells contribute to the
glomerular injury, EXCEPT:
A. Macrophages
B. Platelets
C. Neutrophils
D. Mesangial cells
E. * Mast cells
262. Systemic lupus erythematosus gives rise to all of the following patterns of glomerular injury,
EXCEPT:
A. Focal proliferative glomerulonephritis
B. Diffuse membranous glomerulonephritis
C. Diffuse proliferative glomerulonephritis
D. Mesangial proliferative glomerulonephritis
E. * Lipoid nephrosis
263. Renal diseases producing systemic hypertension include all of the following, EXCEPT:
A. Acute glomerulonephritis
B. Chronic glomerulonephritis
C. Chronic pyelonephritis
D. Renal vasculitis
E. * Renal amyloidosis
264. Histologic features of malignant nephrosclerosis include all of the following, EXCEPT:
A. Fibrinoid necrosis of arterioles
B. Medial thickening of arterioles
C. Renal artery thrombosis
D. Focal renal parenchymal infarction
E. * Fibromuscular dysplasia of the renal artery
265. Which statement correctly characterizes membranous glomerulopathy?
A. It is the most common cause of nephrotie syndrome in children
B. Patients usually present with acute renal failure
C. It is characterized by diffuse proliferative glomerulonephritis
D. It is characterized by mesangial interposition phenomenon
E. * Electron microscopy demonstrates numerous subepithelial immunetype deposits
266. Subendothelial granular electron-dense deposits can be found in which of the following diseases:
A. Rapidly progressive glomerulonephritis
B. Side cell nephropathy
C. Membranous glomerulonephritis
D. Gouty nephropathy
E. * Systemic lupus erythematosus
267. Hydronephrosis is characterized by all of the following, EXCEPT:
A. Thinning of the renal parenchyma
B. Dilatation of the renal pelvis
C. Dilatation of the renal calyces
D. Progressive atrophy of the kidney
E. * Kidney infarct
268. Hydronephrosis is caused by all of the following, EXCEPT:
A. Large uterine leiomyoma
B. Renal calculi
C. Benign prostatic hypertrophy
D. Papillary transitional cell carcinoma of the ureter
E. * Chronic renal vein thrombosis
269. In nonobstructive chronic pyelonephritis the most common way for bacteria to gain entrance into the
kidney is which of the following:
A. Arterial bloodstream
B. The lymphatics
C. Venous bloodstream
D. Aberrant arteriovenous shunts
E. * Vesicoureteral reflux
270. All of the following statements correctly describe chronic pyelonephritis, EXCEPT:
A. It causes asymmetrically scarred kidneys
B. It is associated with vesicoureteral reflux in most cases
C. It may produce thyroidization of tubules
D. It is an important cause of secondary nephrosclerosis
E. * It spares the calyces and pelvis
271. All of the following conditions predispose to urolithiasis, EXCEPT:
A. Hyperparathyroidism
B. Gout
C. Proteus pyelonephritis
D. Enteric hyperoxaluria
E. * Sickle cell nephropathy
272. The factor least likely to cause acute pyelonephritis is which of the following:
A. Pregnancy
B. Nephrolithiasis
C. Catheterization of the bladder
D. Prostatic hypertrophy
E. * Septicemia
273. What pathologic condition of the kidneys is caused by mercury poisoning?
A. Renal papillary necrosis
B. Crescentic glomerulonephritis
C. Acute interstitial nephritis
D. Renal cell carcinoma
E. * Acute tubular necrosis
274. What develops in case of prolong obstruction of ureters by stone in a kidney
A. Heart attack
B. Gangrene
C. Glomerulonephritis
D. Amiloidosis
E. * Hydronephrosis
275. The tubular epithelial cells in acute tubular necrosis are characterized by all of the following
pathologic features, EXCEPT:
A. Karyolysis
B. Plasmolysis
C. Plasmorrhexis
D. Plasmocoagulation

E. * Tubulorrhexis
276. Hematuria is a characteristic clinical feature of all of the following diseases, EXCEPT:
A. Glomerulonephritis
B. Nephrolithiasis
C. Renal cell carcinoma
D. Bladder papilloma
E. * Malakoplakia
277. In the urinary tract obstruction all pathologic processes can be found, EXCEPT:
A. Dilatation of the pelvis and calyces;
B. Glomerular and tubular atrophy.
C. Interstitial fibrosis;
D. Interstitial inflammation;
E. * Acute tubular necrosis;
278. Microscopic examination revealed- necrosis of renal convoluted tubule epithelium, tubuloreksys,
stromal edema, ischemia cortical layer and the plethora of medulla. Name this pathological process.
A. Glomerulonephritis
B. Pyelonephritis
C. Policystosis
D. Chronic lung disease
E. * Necrotizing nephrosis
279. An autopsy of a male revealed a prostatic adenoma and large kidneys with sharply enlarged pelves
and calyces filled with some transparent fluid. Name the process in the kidneys.
A. Glomerulonephritis
B. Polycystic kidneys
C. Tuberculosis
D. Pyelonephritis
E. * Hydronephrosis
280. Name the disease of kidneys, which can be attributed to the group of tubulopathies:
A. Amyloidosis of kidneys
B. Chronic kidney insufficiency
C. Hydronephrosis
D. Pyelonephritis
E. * Acute kidney insufficiency
281. An autopsy has revealed that kidneys are enlarged, surface is large-granulary because of multiple
cavities with smooth wall, which are filled with clear fluid. What kidney disease did the patient have?

A. Infarction
B. Glomerulonephritis
C. Pyelonephritis
D. Necrotic nephrosis
E. * Polycystic kidney
282. In the urinary tract obstruction all pathologic processes can be found. EXCEPT:
A. Dilatation of the pelvis and calyces
B. * Ischemic tubular necrosis
C. Interstitial inflammation
D. Interstitial fibrosis
E. Glomerular and tubular atrophy

283. Indicate the localization of metastases of the uterus cancer?


A. Ovaries
B. Intestine
C. Lungs
D. Peritoneum
E. * Lymph nodes of the pelvis
284. What form of mastopathy transforms into cancer?
A. Tubular
B. Cyst
C. Fibrotic
D. Nonproliferative
E. * Proliferative
285. The most common tumor that occurs in the uterus
A. Rhabdomyoma
B. Abrikosov's tumor
C. Rabdomiosarcoma
D. lipoma
E. * Leiomyoma
286. The most common tumor that occurs in the breast
A. Rhabdomyoma
B. Leiomyoma
C. Liposarcoma
D. lipoma
E. * Fibromyoma
287. Advanced cervical carcinoma can extend by direct continuity to all of the following, EXCEPT:
A. Urinary bladder
B. Ureters
C. Rectum
D. Peritoneum
E. * Colon transversum
288. All of the following morphologic features characterize the ovary adenocarcinoma cells, EXCEPT:
A. Enlarged nucleoli
B. Atypical mitoses
C. Variation in size and shape
D. Hyperchromatic nuclei
E. * Nucleus inclusions
289. All statements concerning chorion-carcinoma are true, EXCEPT:
A. The tumor has a dimorphic pattern
B. Hemorrhages and necrosis are present
C. The tumor is malignant
D. The tumor is composed of cytotrophoblast and syncytiotrophoblast cells
E. * The tumor produces chorionic villi
290. All statements concerning leiomyomas are true, EXCEPT:
A. They regress or calcify after castration or menopause
B. They may undergo rapid increase in size during pregnancy
C. Their cause is unknown

D. They are found in 25% of reproductive women


E. * They do not respond to estrogens
291. An important factor in cervical oncogenesis is which of the following:
A. Herpes simplex
B. Herpes zoster
C. Human immunodeficiency virus
D. Respiratory syncytial virus
E. * HPV-infection
292. Chorioncarcinoma is commonly associated with which of the following:
A. Systemic hypertension
B. Obesity
C. Oral contraceptive steroid use
D. Diabetes mellitus
E. * Pregnancy
293. Conditions leading to endometrial hyperplasia include all of the following, EXCEPT:
A. Polycystic ovarian disease
B. Functioning granulosa cell tumors of the ovary
C. Excessive cortical function (cortical stroma hyperplasia)
D. Estrogen replacement therapy
E. * Endometriosis
294. Criteria to differentiate between benign and malignant tumors are all of the following, EXCEPT:
A. Maturity
B. Rate and character of growth
C. Metastases
D. Local invasion
E. * Edema
295. In a biopsy of сervix of a 26-year-old woman the diagnosis following was established: pseudo-
erosion. What microscopical changes has the pathologist revealed?
A. Cell-atypia of an epithelium of an mucosal epithelium
B. Keratinization of an epithelium
C. “Carcinomatous pearls”
D. Local inflammation and necrosis in mucosa
E. * Local changes of a stratified squamous epithelium on single-layer prismatic one
296. A 30-year-old woman had ectopic tubal pregnancy which finished with a location of a fetus in the
tubal cavity with bleeding. Call this pathology of pregnancy
A. Complete tubal abortion
B. Spontaneous abortion
C. Induced abortion
D. Criminal abortion
E. * Incomplete tubal abortion
297. The pathologist has found out in histological examination of a remote uterus a lot of glandular
formations with single cysts in myometrium, endometrium had usual structure. Described changes
are characteristic for …
A. Glandular hyperplasia of endometrium.
B. Adenocarcinoma of uterus.
C. Adenomatosis of a mucosa of a uterus
D. Leyomyosarcoma of uterus.

E. * Endometriosis
298. Distant hematogenic metastases of invasive cervical carcinoma occur in all of the following,
EXCEPT:
A. Liver
B. Lungs
C. Bone marrow
D. Kidney
E. * Lymph nodes
299. Endometrial hyperplasia is associated with which of the following:
A. High estrogenic stimulation with normal progestational activity
B. Normal estrogenic stimulation with increased progestational activity
C. High estrogenic stimulation and progestational activity
D. Normal estrogenic stimulation and progestational activity
E. * High estrogenic stimulation with diminished progestational activity
300. First hematogenous metastases of the endometrial carcinoma can be found in which of the following
organs:
A. Bones
B. Regional lymph nodes
C. Liver
D. Central nervous system
E. * Lungs
301. First lymphogenous metastases of the papillary mucinous cystadnocarcinoma of the ovary can be
found in which of the following organs:
A. Aortic lymph nodes
B. Liver
C. Lungs
D. Bones
E. * Pelvic lymph nodes
302. First metastases of the chorioncarcinoma can be found in which of the following organs:
A. Liver
B. Kidney
C. Bones
D. Brain
E. * Lung
303. Higher frequency of carcinoma of the endometrium is commonly associated with all of the following,
EXCEPT:
A. Obesity
B. Diabetes mellitus
C. Hypertension
D. Infertility (anovulatory cycles)
E. * Young age
304. The most common benign tumor of the ovary is which of the following:
A. Adenocarcinoma
B. Cystadenocarcinoma
C. Papilloma
D. Fibroadenoma
E. * Cystadenoma

305. The tumor composed of clusters of cuboid cytotrophoblast cells separated by streaming masses of
syncytiotrophoblast giant cells with marked atypia is referred to as:
A. Adenoma
B. Adenocarcinoma
C. Cystadenoma
D. Fibroadenoma
E. * Chorioncarcinoma
306. What pathological process the hypertrophy of prostatic gland is related to?
A. Chronic prostatitis
B. Acute prostatitis
C. Hyperfunction of sexual glands
D. Disorder of the urine outflow
E. * Hypofunction of sexual glands
307. Benign tumor arising from osteoblasts is called:
A. Chondroma
B. Adenoma
C. Fibroma
D. Papilloma
E. * Osteoma
308. All of the following genes (known as " tumor suppressor genes ") provide the negative control over
cell proliferation, except:
A. The p53gene
B. The DCC ("Deleted in Colon Cancer") gene
C. The Rb gene
D. The WT-1 gene
E. * The bcl-2 gene
309. Benign tumor arising from fibroblastic cells is called:
A. Chondroma
B. Adenoma
C. Osteoma
D. Papilloma
E. * Fibroma
310. Benign tumor arising from cartilaginous tissue is called:
A. Adenoma
B. Osteoma
C. Fibroma
D. Papilloma
E. * Chondroma
311. Dysplasia is characterized by all of the following, except:
A. Loss of cell uniformity
B. Hyperchromatic enlarged nuclei
C. Appearance of mitotic figures
D. Loss of cell architectural orientation
E. * Formation of tumor giant cells
312. Benign tumors composed of either cavernous spaces or serpentine capillary-like channels containing
blood or lymph are all the following, except:
A. Capillary lymphangiomas

B. Cavernous hemangiomas
C. Cavernous lymphangiomas
D. Capillary hemangiomas
E. * Capillary telangiectases
313. Type of Hemangioma:
A. Microcirculatory
B. Fibrous
C. Arterial
D. Granular-cell
E. * Venous
314. Differentiated synoviom arises from:
A. Capillries
B. Venous capillary
C. Arteriolas
D. Bone tissues
E. * Synovial elements of tendons
315. Schwannoma is formed:
A. Mallory’s bodies
B. Giant cells
C. Paccionian bodies
D. Melanin-producing cells
E. * Spinder-like cells
316. In the initial part of the upper lobe bronchus of the right lung some polyp-like formation, 1.0 cm in
diameter, with a superficial ulcer, consists of lymphocyte-like cells with hyperchromatic nuclei; the
cells grew in layers and bands. Choose rihgt diagnosis:
A. Undifferentiated large-cell carcinoma
B. Squamous cell carcinoma
C. Adenocarcinoma
D. Glandular squamous cell carcinoma
E. * Undifferentiated small-cell carcinoma
317. Аll of the following morphologic features characterize the adenocarcinoma cells, EXCEPT:
A. Variation in size and shape
B. Hyperchromatic nuclei
C. Enlarged nucleoli
D. Atypical mitoses
E. * Hypochromatic nuclei
318. The most significant histogenetic sign of well-differentiated squamosus carcinoma is which of the
following:
A. Necrotic foci
B. Areas of inflammation
C. Hemorrhages
D. Abscesses
E. * Keratinization
319. A loss of CD4+ T cells resulting from HIV infection leads to the decrease of all of the following,
EXCEPT:
A. Response to soluble antigens;
B. Specific cytotoxicity;

C. Killing of tumor cells;


D. Lymphokine secretion;
E. * Spontaneous secretion of IL-1.
320. A most common cause of endogenous hyperthyroidism is which of the following:
A. Hashimoto thyroiditis;
B. Subacute (granulomatous) thyroiditis;
C. Sporadic goiter.
D. Tuberculosis;
E. * Graves disease;
321. A point out the mechanism which is the basis for classification of the hypersensitivity diseases.
A. Byochemical;
B. Non-immunologic;
C. Chemical;
D. Physiologic.
E. * Immunologic;
322. A replacement of the normal secretory columnar epithelium by the nonfunctioning stratified
squamous epithelium may occur in all organs of the following, EXCEPT:
A. Bile ducts of the liver;
B. Excretory ducts of the salivary glands;
C. Excretory ducts of the pancreas;
D. Respiratory epithelium of the bronchi;
E. * Tubular epithelium of the kidney.
323. A type I hypersensitivity (anaphylactk type) is characterized by all of the following, EXCEPT:
A. Occurs in humans previously sensitized to theantigen;
B. Develops rapidly (within minutes);
C. Mediates by IgE antibodies in humans;
D. Mast cell or basophil degranulation.
E. * Develops slowly (within days);
324. Classical example of delayed type hypersensitivity is which of the following:
A. Arthus reaction;
B. Local anaphylaxis;
C. Complement-dependent reaction;
D. Systemic anaphylaxis
E. * Tuberculin reaction;
325. Predominant cells aggregating in the form of follicles in the spleen are which of the following:
A. CD4+T cells;
B. CD8+ T cells;
C. Macrophages;
D. Natural killer cells.
E. * B lymphocytes;
326. Secondary mediators released during Type I hypersensitivity (anaphylactic type) are all of the
following, EXCEPT:
A. Leukotriens;
B. Cytokines;
C. Prostaglandin D2.
D. Platelet-activating factor;
E. * Biogenic amines;

327. The all of the following cells are involved in immune response, EXCEPT:
A. Macrophages;
B. Natural killer cells;
C. T lymphocytes;
D. B lymphocytes.
E. * Eeosinophils;
328. The antinuclear antibodies in systemic lupus erythematosus are directed against all of the following,
EXCEPT:
A. DNA;
B. Histones;
C. Nonhiston proteins bound to RNA;
D. Nucleolar antigens.
E. * Nuclear membrane;
329. The cells which play an important role both in the induction and in the effector phase of immune
response are which of the following:
A. CD4+T cells;
B. CD8+ T cells;
C. B lymphocytes;
D. Natural killer cells.
E. * Macrophages;
330. The electron micrograph of a renal glomerular capillary loop from a patient with systemic lupus
erythematosus reveals which of the following:
A. Dense deposits in epithelial cells;
B. Dense deposits in capillary lumen;
C. Laminated deposits in subendothelial location;
D. Laminated deposits in epithelial cells.
E. * Dense deposits in subendothelial location;
331. Cardiac hypertrophy is characterized by all pathologic changes, EXCEPT:
A. Increased mass and size of the heart;
B. Increased protein synthesis;
C. Interstitial cardiac fibrosis;
D. Inadequate vasculature.
E. * Increased number of myocardial cells;
332. The proliferation of the glandular epithelium of a female during pregnancy is an example of:
A. Compensatory hyperplasia;
B. Pathologic hyperplasia;
C. Compensatory hypertrophy;
D. Hormonal hypertrophy.
E. * Hormonal hyperplasia;
333. Which type of wound healing can be termed as "keloid"?
A. Atrophic scar;
B. Scar with dysplasia;
C. Hyperplastic scar;
D. Hypotrophic scar;
E. * Hypertrophic scar
334. Polycythemia vera is a proliferative disorder of stem cells that:
A. Has an X-linked recessive mode of transmission

B. Is associated with high levels of erythropoietin


C. Produces abnormalities in the red cell series only
D. * Is rapidly fatal if untreated
E. Is compatible with a normal life span if treated
335. A deficiency of the red cell membrane component spectrin causes which of the following diseases:
A. Pernicious anemia
B. * Hereditary spherocytosis
C. Sickle cell anemia
D. Thalassemia major 5.
E. Iron - deficiency anemia
336. Acute leucosis is characterized by prolipheration of:
A. Developed healthy cells
B. Undifferentiated or patially differentiated blast cells
C. Erythrocytes
D. Leucocytes
E. * Endothelium
337. Acute leucosis is more commonly seen in
A. children
B. women
C. older people
D. young people
E. * Men
338. Iron-deficiency anemia is commonly associated with all of the following factors, EXCEPT:
A. Colon cancer
B. * Polycythemia
C. Gastrectomy
D. Normal menses
E. Pregnancy
339. Patients with polycythemia vera show all of the following characteristic features, EXCEPT:
A. High platelet count (thrombocytosis)
B. High hematocrit
C. * High erythropoietin level
D. High mean age at presentation
E. Higher than average chance of stroke or myocardial infarction
340. All of the following are stages of the atherosclerotic plaque development, except:
A. Fibrous plaque
B. Fibrofatty plaque
C. Atheromatous plaque
D. Fibrolipid plaque
E. * Fibrinous plaque
341. All of the following components can be found in the necrotic center of atheromatous plaque, except:
A. Cell debris
B. Cholesterol crystals
C. "Foam" cells
D. Calcium crystals
E. * Glycogen granules

342. Type of complicated macroscopic changes of intima arteries in atherosclerosis.


A. Fatty spots
B. Hyalineized plaques
C. Fibrosis plaques
D. Fatty stripes
E. * Intramural hematoma
343. Macroscopic characteristic of fatty spots and strips in atherosclerosis.
A. Rising over intimacy
B. Covered with ulcers
C. Localized in the media
D. Localized in adventitia
E. * Do not rise above intimacy
344. Material deposited in arteriolar walls in systemic hypertension is which of the following:
A. Amyloid
B. Glycogen
C. Cholesterol or cholesterol esters
D. Lipoprotein
E. * Hyaline
345. Specify the cause of death in patients with chronic ischemic heart disease:
A. Coma
B. Cardiogenic shock
C. Acute vascular insufficiency
D. Chronic obstructive pulmonary disease
E. * Chronic heart failure
346. Choose the reason of myocardial infarction:
A. Tumor compression of the coronary veins
B. Fatty degeneration of the myocardium
C. An inflammation of the coronary arteries
D. An inflammation of the coronary veins
E. * Coronary artery thrombosis
347. Name the morphological expression of chronic ischemic heart disease:
A. Myocardial infarction
B. Fatty degeneration of the myocardium
C. Heart defect
D. Obesity heart
E. * Chronic cardiac aneurysm
348. The cause of death in acute coronary heart disease:
A. Asphyxia
B. Chronic heart failure
C. Coma
D. Acute pulmonary insufficiency
E. * Cardiogenic shock
349. Name the morphological expression of chronic the ischemic heart disease:
A. Fatty degeneration of the myocardium
B. Heart obesity
C. Myocardial hypertrophy

D. Heart defect
E. * Diffuse small-focal cardiosclerosis
350. Name the morphological expression of acute ischemic heart disease:
A. Small-focal cardiosclerosis
B. Chronic aneurysm
C. Myocardial infarction
D. Heart defect
E. * Myocardial infarction
351. What complication can develop in patients with acute heart aneurysm:
A. A heart attack
B. Myocardial infarction
C. Cardiac hypertrophy
D. Cardiosclerosis
E. * Rupture of the ventricle
352. What are the stages of myocardial infarction?
A. Dystrophic
B. Metabolic
C. All of the above
D. Ischemic
E. * Necrotic
353. Name renal changes in hypertension.
A. Acute nephritis
B. Secondary wrinkled kidney
C. Atherosclerotic nephrosclerosis
D. Atrophy of the kidneys
E. * Primary wrinkled kidney
354. Name synonyms hypertension.
A. Symptomatic hypertension
B. Primary hypertension
C. Secondary hypertension '
D. Neurogenic hypertension
E. * Idiopathic hypertension
355. What are the pathological conditions can lead to symptomatic hypertension?
A. The disease of the CNS
B. Vascular disease
C. Kidney disease
D. Endocrine diseases
E. * All of the above
356. The leading factor in the pathogenesis of hypertension.
A. Hepatic factor
B. Kidney
C. Metabolic
D. Reception lipoprotein
E. * Hereditary
357. Symptoms of cardiac dysfunction include all of the following, except:
A. Failure of the pump itself

B. Obstruction of blood flow


C. Regurgitant blood flow
D. Disorders of cardiac conduction
E. * Disorders of blood coagulation
358. The renin-angiotensin system consists of all of the following components, except:
A. Renin
B. Angiotensin I
C. Plasma angiotensinogen
D. Angiotensin II
E. * Tissue angiotensinogen
359. The typical pathologic proces in arterioles and small arteries in systemic hypertension is referred to
as:
A. Hyaline atrophy
B. Hyaline hyperplasia
C. Fibroelastic hyalinosis
D. Hyaline atherosclerosis
E. * Hyaline arteriolosclerosis
360. The gene defects in enzymes involved in aldosterone metabolism in systemic hypertension lead to all
of the following effects, except:
A. Adaptive increase in aldosterone secretion
B. Increased salt resorption
C. Increased water resorption
D. Plasma volume expansion
E. * Increased lipoprotein resorption
361. Resulting from left-sided heart failure are all of these pathologic changes, except:
A. Brown induration of the lung
B. Hypoxic encephalopathy
C. Pulmonary edema
D. Prerenal azotemia
E. * Secondary amyloidosis
362. Secondary hypertension may be caused by all of the following diseases, except:
A. Acute glomerulonephritis
B. Gushing's syndrome
C. Pheochromocytoma
D. Coarctation of aorta
E. * Lobar pneumonia
363. Acute rheumatic carditis is characterized by all of the following, except:
A. Unremarkable gross appearance of the heart
B. The Aschoff bodies in myocardium
C. Diffuse nonspecific myocarditis
D. The Aschoff bodies in joints
E. * "Tigered effect" gross appearance
364. Non embolic complications of infective endocarditis include all of the following, except;
A. Valvular insufficiency or stenosis with cardiac failure
B. Myocardial ring abscess
C. Suppurative pericarditis
D. Endocarditis of artificial valves

E. * Acute myocardial infarction


365. The cardinal anatomic changes of the mitral (or tricuspid) valve in chronic rheumatic heart disease
are all of the following, except:
A. Fusion of the tendinous cords
B. Leaflet thickening
C. Commisural fusion
D. Shortening and thickening of the tendinous cords
E. * Leaflet softening
366. The clinical consequences of valvular dysfunction depend on all of the following, except:
A. Valve involved
B. Rate and quality of compensatory mechanisms
C. Degree of impairment
D. Rate of dysfunction development
E. * Sex of the patient
367. Systemic emboli in infective endocarditis may occur because of all of the following, except:
A. Involvement of aortic and mitral valves
B. Severe valvular damage
C. Numerous amount of vegetations
D. Friable nature of vegetations
E. * Chronic heart insufficiency
368. The Anitschkow cells are characterized by all of the following, except:
A. Abundant amorphophilic cytoplasm
B. Central round-to-ovoid nucleus
C. Central disposed chromatin (caterpillar cells)
D. Monocytic cytogenesis
E. * B-lymphocytic cytogenesis
369. What explains the predominant lesion in patients with rheumatism affinity antibodies against B-
hemolytic streptococci type A antigens from connective tissue of the heart:
A. liver
B. only epikard
C. only endocard
D. more pericardiocentesis
E. * heart
370. The disease is related to a group of rheumatic:
A. Atherosclerosis
B. Tuberculosis
C. Amyloidosis
D. Arthrosis
E. * Scleroderma
371. What is rheumatic disease - (a group of chronic diseases characterized by systemic lesion):
A. Joints
B. The circulatory system
C. Liver
D. Kidney
E. * Connective tissue and blood vessels
372. The decisive factor for the development of scleroderma is a violation of the synthesis:
A. Reticular fibers

B. Fibrin
C. Fibrinogen
D. Keloid
E. * Collagen
373. What disease refers to a group of rheumatic:
A. Atherosclerosis
B. Obesity
C. Myocardial infarction
D. Amyloidosis
E. * Dermatomyositis
374. Typical tyberculous granuloma is characterized by all of the following, EXCEPT:
A. Area of central necrosis
B. Epithelioid cells
C. Langhans-type giant cells
D. Lymphocytes
E. * Plasma cells
375. The most common sites of skeletal tuberculosis involvement are all of the following, EXCEPT:
A. Hips
B. Thoracic vertebrae
C. Lumbar vertebrae
D. Knees
E. * Skull bones
376. Substances that prevent complete phagocytosis of Mycobacterium tuberculosis by macrophages and
induce delayed type hypersensitivity are all of the following, EXCEPT:
A. Cord factor
B. Activated complement
C. Lipoarabinomanan (LAM)
D. Heart-shock protein
E. * Interleukin
377. A calcified focus (fibrocalcific scar) forming in the lung parenchyma and in the hilar lymph node
after the primary tuberculosis infection is also referred to as:
A. Keloid
B. Granuloma
C. Ghon focus
D. Simon focus
E. * Aschoff-Pule focus
378. All of these cells participate in immune response in primary lung tuberculosis, EXCEPT:
A. Type I pneumocytes
B. Double negative T-cells
C. CD4+ helper T-cells
D. CD8+ suppressor T-cells
E. * Alveolar macrophages
379. Severe destruction of vertebrae in spine tuberculosis may result in all of the following pathologic
conditions, EXCEPT:
A. Permanent compression fractures
B. Scoliotic deformities
C. Neurologic deficits

D. Kyphotic deformities
E. * Drainage tract (sequestrum) forming
380. The liver may be affected in which of the following forms of tuberculosis:
A. Secondary tuberculosis
B. Cavernous fibrocaseous tuberculosis
C. Dormant tuberculosis
D. Primary tuberculosis complex
E. * Miliary tuberculosis
381. The granuloma in tuberculosis is composed predominantly of which of the following cells:
A. Fibroblasts
B. Eosinophils
C. Plasma cells
D. Neutrophils
E. * Epithelioid cells
382. The possible causes of chronicity of inflammation in tuberculosis are all of the following, EXCEPT:
A. Persistence of certain microorganisms
B. Prolonged exposure to toxic agents
C. Incomplete phagocytosis
D. Resistance of etiologic agent
E. * Complete phagocytosis
383. Impairment of bronchociliary function in influenza results in which of the following:
A. Fungal superinfection
B. Chlamydial superinfection
C. Protozoal superinfection
D. Mycoplasmal superinfection
E. * Bacterial superinfection
384. Peribronchial pneumonia at measles often results in which of the following complications:
A. Peripheral lung tumor
B. Central lung tumor
C. Hemosiderosis of the lung
D. Lung gangrene
E. * Bronchiectasis
385. The severe form of measles pneumonia is characterized by which of the following:
A. Foci of hemorrhagic inflammation
B. Catarrhal bronchitis
C. Granulomatous inflammation
D. Caseous necrosis
E. * Destructive panbronchitis
386. Which stage of changes of Peyer's patches is the second in typhoid between enumerated?
A. no indicated here
B. ulcer formation
C. healing
D. medullar swelling
E. * necrosis
387. Uterine cervix tissue covered with a wide layer of the stratified squamous epithelium having
roliferation of atypical cells with pathological mitoses, but the basal membrane of the epithelium was
not affected. Choose rihgt diagnosis:

A. Nonkeratinizing squamous cell carcinoma


B. Keratinizing squamous cell carcinoma
C. Leukoplakia
D. Epithelial dysplasia
E. * Carcinoma in situ
388. Deformed mucouse membrane of a lobar bronchus consistinyof hyperchromatic nuclei and numerous
pathological mitoses, the growth of the tumour did not spread to the basal membrane of the
epithelium. Choose rihgt diagnosis:
A. Squamous cell carcinoma
B. Adenocarcinoma
C. Solid carcinoma
D. Small-cell carcinoma
E. * Carcinoma in situ
389. The epithelial tumors without specific localization develop from:
A. Mesotheliocytes
B. Fibroblastes
C. Epithelioidcells
D. Epitheliocytes
E. * Squamous epithelium
390. The quiet formed benign tumor with squamous epithelium is known as:
A. Sarcoma
B. Fibroma
C. Lipoma
D. Mioma
E. * Papilloma
391. What are the pathological conditions can lead to symptomatic hypertension?
A. The disease of the CNS
B. Vascular disease
C. Kidney disease
D. Endocrine diseases
E. * All of the above
392. The leading factor in the pathogenesis of hypertension.
A. Hepatic factor
B. Kidney
C. Metabolic
D. Reception lipoprotein
E. * Hereditary
393. Symptoms of cardiac dysfunction include all of the following, except:
A. Failure of the pump itself
B. Obstruction of blood flow
C. Regurgitant blood flow
D. Disorders of cardiac conduction
E. * Disorders of blood coagulation
394. The renin-angiotensin system consists of all of the following components, except:
A. Renin
B. Angiotensin I
C. Plasma angiotensinogen

D. Angiotensin II
E. * Tissue angiotensinogen
395. The typical pathologic proces in arterioles and small arteries in systemic hypertension is referred to
as:
A. Hyaline atrophy
B. Hyaline hyperplasia
C. Fibroelastic hyalinosis
D. Hyaline atherosclerosis
E. * Hyaline arteriolosclerosis
396. The gene defects in enzymes involved in aldosterone metabolism in systemic hypertension lead to all
of the following effects, except:
A. Adaptive increase in aldosterone secretion
B. Increased salt resorption
C. Increased water resorption
D. Plasma volume expansion
E. * Increased lipoprotein resorption
397. Resulting from left-sided heart failure are all of these pathologic changes, except:
A. Brown induration of the lung
B. Hypoxic encephalopathy
C. Pulmonary edema
D. Prerenal azotemia
E. * Secondary amyloidosis
398. Secondary hypertension may be caused by all of the following diseases, except:
A. Acute glomerulonephritis
B. Gushing's syndrome
C. Pheochromocytoma
D. Coarctation of aorta
E. * Lobar pneumonia
399. Acute rheumatic carditis is characterized by all of the following, except:
A. Unremarkable gross appearance of the heart
B. The Aschoff bodies in myocardium
C. Diffuse nonspecific myocarditis
D. The Aschoff bodies in joints
E. * "Tigered effect" gross appearance
400. Non embolic complications of infective endocarditis include all of the following, except;
A. Valvular insufficiency or stenosis with cardiac failure
B. Myocardial ring abscess
C. Suppurative pericarditis
D. Endocarditis of artificial valves
E. * Acute myocardial infarction
401. The cardinal anatomic changes of the mitral (or tricuspid) valve in chronic rheumatic heart disease
are all of the following, except:
A. Fusion of the tendinous cords
B. Leaflet thickening
C. Commisural fusion
D. Shortening and thickening of the tendinous cords
E. * Leaflet softening

402. The clinical consequences of valvular dysfunction depend on all of the following, except:
A. Valve involved
B. Rate and quality of compensatory mechanisms
C. Degree of impairment
D. Rate of dysfunction development
E. * Sex of the patient
403. Systemic emboli in infective endocarditis may occur because of all of the following, except:
A. Involvement of aortic and mitral valves
B. Severe valvular damage
C. Numerous amount of vegetations
D. Friable nature of vegetations
E. * Chronic heart insufficiency
404. Diabetes mellitus is associated with all of the following renal disorders, EXCEPT:
A. Diffuse glomerulosclerosis
B. Nodular glomerulosclerosis
C. Benigh nephrosclerosis
D. Acute pyelonephritis
E. * Urate nephropathy
405. Ultrastructural changes in children primary nephrotie syndrome involve which of the following
glomerular elements:
A. Endothelium
B. Mesangium
C. Bloodvessels
D. Basement membrane
E. * Podocytes
406. Uremia is associated with all of the following abnormalities, EXCEPT:
A. Peripheral neuropathy
B. Gastritis
C. Pericarditis
D. Diffuse alveolar damage
E. * Polycythemia
407. What enumerate illness from the listed below relates to the large mottled kidney
A. Amiloidosis of kidney
B. Chronic pyelonephritis
C. Nephrolithiasis
D. Chronic glomerulonephritis
E. * Subacute glomerulonephritis
408. What disease can be complicated by amyloidosis of kidneys:
A. Atherosclerosis
B. Heart ischemic disease
C. Croupous pneumonia
D. Hypertension
E. * Fibrous cavernous tuberculosis of lungs
409. Most forms of chronic renal failure produce increased serum levels of all of the following substances,
EXCEPT:
A. Renin
B. Aldosterone

C. Phosphate
D. Parathormone
E. * Calcium
410. Mesangial cells can be characterized by all of the following properties, EXCEPT:
A. Ingestion of macromolecules
B. Connection with Lacis cells
C. Ability to contract
D. Production of basement membrane proteins
E. * Production of renin
411. In immunologicahy mediated glomerulonephritis all of the following cells contribute to the
glomerular injury, EXCEPT:
A. Macrophages
B. Platelets
C. Neutrophils
D. Mesangial cells
E. * Mast cells
412. Systemic lupus erythematosus gives rise to all of the following patterns of glomerular injury,
EXCEPT:
A. Focal proliferative glomerulonephritis
B. Diffuse membranous glomerulonephritis
C. Diffuse proliferative glomerulonephritis
D. Mesangial proliferative glomerulonephritis
E. * Lipoid nephrosis
413. Renal diseases producing systemic hypertension include all of the following, EXCEPT:
A. Acute glomerulonephritis
B. Chronic glomerulonephritis
C. Chronic pyelonephritis
D. Renal vasculitis
E. * Renal amyloidosis
414. Histologic features of malignant nephrosclerosis include all of the following, EXCEPT:
A. Fibrinoid necrosis of arterioles
B. Medial thickening of arterioles
C. Renal artery thrombosis
D. Focal renal parenchymal infarction
E. * Fibromuscular dysplasia of the renal artery
415. Which statement correctly characterizes membranous glomerulopathy?
A. It is the most common cause of nephrotie syndrome in children
B. Patients usually present with acute renal failure
C. It is characterized by diffuse proliferative glomerulonephritis
D. It is characterized by mesangial interposition phenomenon
E. * Electron microscopy demonstrates numerous subepithelial immunetype deposits
416. Subendothelial granular electron-dense deposits can be found in which of the following diseases:
A. Rapidly progressive glomerulonephritis
B. Side cell nephropathy
C. Membranous glomerulonephritis
D. Gouty nephropathy
E. * Systemic lupus erythematosus

417. Hydronephrosis is characterized by all of the following, EXCEPT:


A. Thinning of the renal parenchyma
B. Dilatation of the renal pelvis
C. Dilatation of the renal calyces
D. Progressive atrophy of the kidney
E. * Kidney infarct
418. Hydronephrosis is caused by all of the following, EXCEPT:
A. Large uterine leiomyoma
B. Renal calculi
C. Benign prostatic hypertrophy
D. Papillary transitional cell carcinoma of the ureter
E. * Chronic renal vein thrombosis
419. Bone marrow is red juicy, sometimes a gray tint during:
A. Undifferentiated form of leukosis
B. Jewler’s form of leucosis
C. Lymphogranulamatosis
D. During a child’s form of leucosis
E. * Early form of leucosis
420. All of the following statements correctly describe hereditary spherocytosis, EXCEPT:
A. * Red blood cells have the membrane-associated protein spectrin
B. Mutation in the ankyrin gene is present in most cases
C. Anaplastic crisis
D. Splenectomy is invariably therapeutic
E. -
421. Features of megaloblastic anemias include all of the following, EXCEPT:
A. Hypersegmented neutrophils
B. * Lack of the membrane-associated protein spectrin
C. Increased intramedullary hemolysis
D. Increased extramedullary hemolysis
E. Pancytopenia
422. All of the following statements correctly describe aplastic anemia, EXCEPT:
A. Production of all hematopoietic bone marrow elements is reduced
B. Chemical drug exposure is the most common cause
C. Fanconi's anemia represents an inherited form of the disease
D. No underlying etiology is evident in 50% of cases
E. * Splenomegaly is a characteristic clinical finding
423. Osmotic fragility is characteristic of the erythrocytes in which of the following diseases:
A. Fanconi's anemia
B. Sickle cell anemia
C. Glucose-6-phosphate dehydrogenase deficiency
D. * Hereditary spherocytosis
E. Pernicious anemia
424. Myelophthisic anemia can occur in patients with any of the following conditions, EXCEPT:
A. Miliary tuberculosis
B. Carcinomatosis
C. Myelofibrosis

D. Multiple myeloma
E. * Uremia
425. During chronic lympho leucosis the liver is
A. Shrunken
B. Atrophic
C. Infiltrated by scar tissue
D. Cirrhotic
E. * Enlarged, gray-brown color
426. To the group of paraprotein leukosis belong disease of
A. Hodgkins
B. Waldenstrom’s
C. Langerhans’
D. Letterer-Siwe
E. * Mielomic
427. Metaplasia is characterized by which of the following:
A. Reversible increase in the size of cells;
B. Irreversible change in which one adult celltype isreplaced by another adult cell type;
C. Reversible abnormal organization of cells;
D. Reversible increase in the number of cells.
E. * Reversible change in which one adult celltype is replaced by another adult cell type;
428. One of the variants of physiologic atrophy is:
A. Atrophy of skeletal muscle by the immobi lized broken limb;
B. Kidney atrophy from pressure (hydronephrosis) ;
C. Atrophy of the endometrium by ovarian tumor;
D. Atrophy of the brain in atherosclerosis
E. * Atrophy of uterus after parturition;
429. Pressure-overloaded (concentric) cardiac hypertrophy is characterized by all of the following,
EXCEPT:
A. Hypertrophy of left ventricle;
B. Increased wall thickness;
C. Normal left cavity diameter;
D. Reduced left cavity diameter.
E. * Dilated left cavity diameter;
430. Simple endometrial hyperplasia is characterized by all of the following, EXCEPT:
A. Increase in the number and size of endometrial glands;
B. Complex endometrial glands;
C. Increase in gland-to-stroma ratio;
D. Dilated endometrial glands.
E. * Atypia of gland cells;
431. The causes of hypertrophy are all of the following, EXCEPT:
A. Mechanical triggers;
B. Trophic triggers;
C. Vasoactive agents.
D. Polypeptide growth factors;
E. * Nervous triggers;
432. The hydronephrosis is characterized by all of the following, EXCEPT:

A. Thinning of the renal parenchyma;


B. Dilatation of the renal pelvis;
C. Dilatation of the renal calyces;
D. Progressive atrophy of the kidney.
E. * Thickening of the renal parenchyma
433. The hyperplasia of hepatic cells that occurs after partial hepatoectomy is an example of:
A. Pathologic hyperplasia;
B. Hormonal hyperplasia;
C. Hormonal hypertrophy;
D. Compensatory hypertrophy.
E. * Compensatory hyperplasia;
434. The hypertrophic and dilated (eccentric) heart is characterized by all of the following, EXCEPT:
A. Decreased mass and diminished left wallthickness;
B. Increased mass and normal left wallthickness;
C. Normal mass and left wall thickness;
D. Normal mass and diminished left wallthickness;
E. * Increased mass and diminished left wallthickness.
435. The local factors that influence wound healing are all of the following, EXCEPT:
A. Wound infection;
B. Mechanical factors;
C. Foreign bodies;
D. Size, location and type of the wound
E. * Hormones (glucocorticoids);
436. The massive growth of the gravid uterus with large plump cells is an example of:
A. Pathologic hypertrophy;
B. Pathologic hyperplasia;
C. Dysplasia;
D. Metaplasia.
E. * Hormone induced physiologic hypertrophy
437. The most common clinical manifestation of endometrial hyperplasia is:
A. Menses stopping;
B. Pains;
C. Purulent discharges;
D. Mucous discharges
E. * Abnormal uterine bleeding;
438. What pigment can be found in the cytoplasm of heart and muscle cells in aging atrophy?
A. Melanin;
B. Hemosiderin;
C. Bilirubin;
D. Ferritin.
E. * Lipofuscin
439. The cells of immune system which have numerous fine dendritic cytoplasmic processus are called:
A. Fibroblasts;
B. Macrophages;
C. Leucocytes;
D. CD4+ T cells

E. * Dendritic cells;
440. The cells which compose the granuloma in type IV hypersensitivity reactions are all of the following,
EXCEPT:
A. Lymphocytes;
B. Macrophages;
C. Epithelioid cells;
D. Giant cells.
E. * Erythrocytes;
1. A 12-year-old child developed nephritic syndrome (proteinuria, hematuria, cylindruria) 2 weeks after
a case of tonsillitis, which is a sign of affected glomerular basement membrane in the kidneys. What
mechanism is the most likely to cause the basement membrane damage?
A. Cytotoxic
B. Granulomatous
C. Antibody-mediated
D. Reaginic
E. * Immune complex
2. Several minutes after a dentist administered Novocain for local anesthesia of a patient’s tooth, the
following symptoms sharply developed in the patient: fatigue, skin itching. Objectively the following
can be observed: skin hyperemia, tachycardia, BP dropped down to 70/40 mm Hg. What kind of
allergic reaction is this pathology?
A. Immune complex
B. Cytotoxic
C. Stimulating
D. Cell-mediated immune reaction
E. * Anaphylactic
3. During blood transfusion a patient has developed intravascular erythrocyte hemolysis. What kind of
hypersensitivity does the patient have?
A. IV type (granulomatosis)
B. I type (anaphylactic)
C. III type (immune complex)
D. IV type (cellular cytotoxicity)
E. * II type (antibody-dependent)
4. During autopsy of a 9-month-old girl’s body, who died due to severe pneumonia complicated by
sepsis, there is a lack of a thymus. In the lymph nodes, lymphoid follicles and cortex are absent;
spleen follicles are reduced in size, without light zones and plasma cells. What is the reason for such
structural changes?
A. Thymus aplasia
B. Accidental involution of thymus
C. Thymus hypoplasia
D. Thymus atrophy
E. * Thymus agenesis
5. A 10-year-old child was found to have a congenital hypoplasia of the left kidney. Ultrasound
examination revealed that the right kidney was markedly enlarged and had regular shape. No
functional disorders were revealed. Specify the process that developed in the right kidney:
A. Metaplasia
B. Working hypertrophy
C. Hypertrophic growth
D. Pseudohypertrophy
E. * Vicarious hypertrophy

6. Histological examination of the biopsy material obtained from the lower third of the esophagus of a
57-year-old male with the symptoms of continuous reflux revealed the change of the stratified
squamous epithelium to the single layer columnar glandular epithelium with signs of mucus
production. Specify the pathological process in the mucous membrane:
A. Regeneration
B. Hyperplasia
C. Hypertrophy
D. Organization
E. * Metaplasia
7. Chronic inflammation and transformation of the one-layer ciliated epithelium into multiple-layers flat
epithelium was revealed in the thickened mucous membrane of the bronchus bioptate of the patient
with smoke abuse. Which of the processes is the most likely?
A. Epithelium hypertrophy
B. Hyperplasia of the epithelium
C. Squamous cancer
D. Leucoplacia
E. * Metaplasia
8. 10 years ago a patient underwent extraction of his right kidney on account of a tumour. After that the
volume of his left tumour grew by 50%. What process was developed in the kidney?
A. Hypertrophic enlargement
B. Neurohumoral hypertrophy
C. Pseudohypertrophy
D. Functional hypertrophy
E. * Vicarious hypertrophy
9. Examination of a 40-year-old man ill with stenosing (without metastases) esophageal carcinoma
revealed the folowing changes: atrophy of skeletal muscles and fatty tissue.His skin is sallow,
epidermis is attenuated, heart has grown smaller. Myocardium and liver are brown. What is the most
probable diagnosis?
A. Addison’s disease
B. Myasthenia
C. Cancerous cachexia
D. Brown atrophy
E. * Alimentary cachexia
10. In a male patient, a visual examination of the skin of his back revealed some spherical tumour, 2 cm
in diameter, which was thick in consistency and had clear borders with the surrounding tissues.
Microscopically, the tumour consisted of some chaotically interlaced bundles of collagenous fibres
and a small numbt: of connective tissue cells. Name the tumour.
A. Leiomyoma
B. Haemangioma
C. Melanoma
D. Lipoma
E. * Fibroma
11. A 65-year-old woman underwent removal of some tumour, 1.0 x 1.0 x 0.8 cm in size, localized under
the skin of her thigh. Macroscopically, the tumour had a connective-tissue capsule and was
represented on section with a yellowish lobate tissue. Microscopically, there were large cells, which
had the sudanophilic cytoplasm and formed lobules separated with connective-tissue layers. Name
this tumour.
A. Hibernoma
B. Liposarcoma

C. Fibroma
D. Desmoid
E. * Lipoma
12. A thick encapsulated node, 2.0 cm in diameter, was surgically removed from the mammary gland of
a female patient. On section, the tissue of the node was white-pink and fibrous. Microscopically, the
tumour consisted of glandular structures, which had no signs of cellular atypism and were
compressed with a connective tissue vegetating around. In the tumour, the stroma prevailed over the
glandular parenchyma. What is your diagnosis?
A. Adenoma
B. Nonproliferative mastopathy
C. Proliferative mastopathy
D. Adenocarcinoma
E. * Fibroadenoma
13. On examination of a 6-year-old child with a tumour on the femoral diaphysis, several metastatic foci
of another osseous localization were found. A histological examination of the primary tumour
revealed that it consisted of some round cells, which had scanty cytoplasm, were characterized by an
insignificant tendency to formation of pseudorosettes, and manifested themselves with solitary
mitoses. What is your diagnosis?
A. Plasmacytoma
B. Chondroma
C. Cancer
D. Fibrosarcoma
E. * Ewing's sarcoma
14. A 40-year-old male patient underwent removal of a tumour, 2 cm in diameter, which was localized in
the region of the cerebellopontine angle of the brain stem and tended to grow into the auditory
meatus. Histologically, the tumour consisted of spindle cells with rod-shaped nuclei; the tumour cells
and fibres formed rhythmic structures. Name the kind of the tumour.
A. Medulloblastoma
B. Meningioma
C. Oligodendroglioma
D. Astrocytoma
E. * Schwannoma
15. A 6-year-old boy underwent removal of a tumour localized along the median line of the cerebellum.
Histologically, the tumour consisted of the cells which had a poor crown of the cytoplasm, a
hyperchromatic nucleus, demonstrated a mitotic activity and tended to form "rosettes". What is your
diagnosis?
A. Astrocytoma
B. Oligodendroglioma
C. Multiform spongioblastoma
D. Bipolar spongioblastoma
E. * Medulloblastoma
16. A tumour removed from the white matter of the right hemisphere of the brain is some soft "motley"
node, 4 cm in diameter, without any clear borders with the substance of the brain. Microscopically,
the tumour consists of polymorphous cells with numerous pathological mitoses, and it also reveals
foci of necrosis and haemorrhages which occurred at different time. Name the tumor.
A. Oligodendroglioma
B. Oligodendroglioblastoma
C. Astrocytoma
D. Astroblastoma

E. * Glioblastoma
17. A histological express examination of a tumour node of a mammary gland revealed some
encapsulated formation with proliferation of alveoli and intralobular ducts; the interstitial connective
tissue grew either around or inside the ducts. Which of the tumours took place?
A. Foliaceous tumour
B. Noninfiltrating intralobular carcinoma
C. Infiltrating intralobular carcinoma
D. Paget's disease
E. * Fibroadenoma
18. During an operation on a woman, her cyst-like changed ovary was removed; it was a thin-walled
cavity filled with some yellowish transparent fluid and having a smooth inner surface. Histologically,
the cavity wall was lined with the cubical epithelium. Name the kind of the tumour.
A. Mucinous cystadenoma
B. Serous cystadenocarcinoma
C. Granulosa cell tumour
D. * Serous cystadenoma
E. Pseudomucinous cystocarcinoma
19. Autopsy of a man who died of intoxication revealed cachexia, muscular atrophy, wrinkled skin,
decreased mass of the inner organs, stenosing tumor of the stomach with metastases to the liver and
regional nodes. Which type of cachexia is most probable?
A. Alimentary
B. Hypophyseal
C. Cerebral
D. In chronic infectious disease
E. * Cancerous
20. In a 37-year-old female patient, an enlarged dense mammary gland was revealed, the nipple with the
areola of the mammary gland were oedematous, the skin had an appearance of an "intradermal bleb".
On microscopic examination, the gland tissues revealed layers of tumour cells with polymorphous
nuclei and a large number of pathological mitoses. The tumour stroma was poorly expressed. Make
the diagnosis.
A. Adenofibroma
B. Scirrhous carcinoma
C. Paget's disease
D. Adenoma
E. * Medullary carcinoma
21. On bronchoscopy, an exophytic tumour was found; it was localized in the bronchus and significantly
narrowed its lumen. Histologically, the tumour consisted of complexes of polymorphous epithelial
cells with hyperchromatic nuclei and pathological mitoses. Among the tumour cells there were
eosinophilic concentric structures. Make a diagnosis of the tumour.
A. Nonkeratinizing squamous cell carcinoma
B. Large-cell carcinoma
C. Small-cell carcinoma
D. Adenoacanthoma
E. * Keratinizing squamous cell carcinoma
22. For a histological examination, an eyeball was sent; some black tumour, 1 x 0.4 cm in size, was
revealed in its vascular membrane. Microscopically, the tumour consisted of large polymorphous
cells grouped in alveolar structures. The cytoplasm of the cells contained some brown pigment. What
is your diagnosis?
A. Neurilemmoma
B. Angiosarcoma
C. Neuroblastoma
D. Ganglioneuroblastoma
E. * Melanoma
23. A microscopic examination of a biopsy from a large intestine revealed some tumour made of the
columnar epithelium which formed atypical glandular structures of various shapes and size. The
epithelial cells were polymorphous and with hyperchromatic nuclei, there were pathological mitoses.
What is your diagnosis?
A. Basal cell carcinoma
B. Solid carcinoma
C. Mucinous carcinoma
D. Carcinoma simplex
E. * Adenocarcinoma
24. A male patient, who suffered from chronic bronchitis for a long period of time, revealed a pulmonary
tumour, which was closely connected with the bronchial wall and grew in the form of a polyp.
Microscopically, the tumour consisted of complexes of polymorphous epithelial cells with a large
number of mitoses. Among the tumour cells there were stratified concentric oxyphilic structures.
Name the histological type of the tumour.
A. Mucinous carcinoma
B. Nonkeratinizing squamous cell carcinoma
C. Adenocarcinoma
D. * Keratinizing squamous cell carcinoma
E. Solid carcinoma
25. A male underwent surgical removal of a black tumour, 2 cm in diamete, from the skin of his thigh.
Microscopically, the tumour consisted of polymorphous cells, the cytoplasm of most of them having
some brown pigment (with a positive reaction to DOPA). A large number of pathological mitoses
was registered. Which of the tumours listed below was the most probable?
A. Carcinoma
B. Sarcoma
C. Carcinosarcoma
D. Nevus
E. * Melanoma
26. On bronchoscopy in the initial part of the upper lobe bronchus of the right lung some polyp-like
formation, 1.0 cm in diameter, with a superficial ulcer was found. A histological examination
revealed a tumour consisting of lymphocyte-like cells with hyperchromatic nuclei; the cells grew in
layers and bands. Indicate the most probable tumour.
A. Undifferentiated large-cell carcinoma
B. Squamous cell carcinoma
C. Adenocarcinoma
D. Glandular squamous cell carcinoma
E. * Undifferentiated small-cell carcinoma
27. A histological examination of some spherical neoplasm located under the surface of the skin,
revealed papilliform vegetations of the epithelium with phenomena of acanthosis and
hyperkeratinization. The tumour stroma consisted of a large amount of the connective tissue and
vessels. What tumour took place?
A. Keratoacanthoma
B. Carcinoma in situ
C. Keratinizing squamous cell carcinoma
D. Nonkeratinizing squamous cell carcinoma

E. * Papilloma
28. A 26-year-old male patient underwent surgical removal of a tumour, 4 x 5 cm in size, which was
surrounded by a capsule and located in the white matter of his brain. Microscopically, the tumour
consisted of the stellate and glia cells having various size and located among the glial fibres. Name
the tumour.
A. Oligodendroglioma
B. Astroblastoma
C. Glioblastoma
D. Ependymoma
E. * Astrocytoma
29. On supersonic examination of a 48-year-old male patient, a hepatic neoplasm was diagnosed and a
puncture biopsy was made. Microscopically, the tumour consisted of atypical hepatocytes which
formed trabeculae, acini or tubules. The tumour stroma was poor and had thin-walled blood vessels.
Which of the kinds of tumours listed below was the most probable?
A. Hepatocellular adenoma
B. Metastasis of adenocarcinoma
C. Cholangiocellular carcinoma
D. Solid carcinoma
E. * Hepatocellular carcinoma
30. Autopsy of a woman with cerebral atherosclerosis revealed in the left cerebral hemisphere a certain
focus that is presented by flabby, greyish and yellowish tissue with indistinct edges. What
pathological process is the case?
A. Senile encephalopathy
B. Multifocal tumor growth with cystic degeneration
C. Multiple foci of fresh and old cerebral hemorrhage
D. Focal encephalitis
E. * Ischemic stroke
31. Autopsy of a 75-year-old man with a long history of atherosclerosis revealed a grey irregular-shaped
focus of loose consistency in the right parieto-temporal region of brain. What is the most likely cause
of this process?
A. Thrombosis of tomentum cerebri
B. Thrombosis of the right anterior cerebral artery
C. Thrombosis of the right posterior cerebral artery
D. Thrombosis of basilar artery
E. * Thrombosis of the right medial cerebral artery
32. Autopsy of a 75-year-old patient who had been suffering from disseminated atherosclerosis and died
under chronic cardiac failure revealed constriction and deformation of coronary arteries, tuberous
intima whose section appeared to be white and petrosal. Specify the stage of atherosclerosis
morphogenesis:
A. Atheromatosis
B. Lipoidosis
C. Liposclerosis
D. Bilipid
E. * Atherocalcinosis
33. Morphological examination of an amputated gangrenous extremity revealed that the lumen of
femoral artery was constricted due to stony, partly ulcerated plaques with obturating thrombi. What is
the most likely diagnosis?
A. Nodular periarthritis
B. Obliterating endartheriitis

C. Non-specific aortoartheriitis
D. Obliterating thromboangiitis
E. * Atherosclerosis
34. A male patient developed substernal pains at 8 a.m., and at 9 a.m. myocardial infarction was
diagnosed by ECG data at the admission department. Ten minutes later the patient died. What most
reliable sign of myocardial infarction will be found on histological examination?
A. Vacuole dystrophy of cardiomyocytes
B. Fat infiltration of cardiomyocytes
C. Reduced activity of dehydrogenases in fibroblasts
D. Relaxation of myofibrils in cardiomyocytes
E. * Disappearance of glycogen in cardiomyocytes
35. On autopsy of a 68-year-old male, who died from cardiac decompensation, the myocardium of the
anterior wall in left ventricle of his heart contained an irregular grey focus, 5 x 4 cm in size, with a
dense consistency, fibrous structure and clear borders. What pathological process in the myocardium
did the pathologist reveal?
A. Myocarditis
B. Microfocal cardiosclerosis
C. Infarction
D. Rheumatism
E. * Postinfarction cardiosclerosis
36. On autopsy of a 66-year-old male, who died from acute cardiac failure, an acute venous plethora of
the internal organs was found. The cardiac cavities were dilated; a myocardial section revealed some
dim yellowish focus, 3.5 x 4 cm in size, in the anterior wall of the left ventricle. The coronary arteries
had stenosing atheromatous plaques. Which of the diagnoses listed below was the most probable?

A. Microfocal cardiosclerosis
B. Macrofocal cardiosclerosis
C. Fatty degeneration of myocardium
D. Myocarditis
E. * Myocardial infarction
37. Three weeks following a myocardial infarction, a 54-year-old man presents with fever, productive
cough, and chest pain. The pain is worse with inspiration, better when he is sitting up, and not
relieved by nitroglycerin. Physical examination finds a friction rub along with increased jugular
venous pressure and pulsus paradoxus (excess blood pressure drop with inspiration). Which of the
following is the most likely explanation for these findings?
A. Caplan’s syndrome;
B. Dressler’s syndrome;
C. Ruptured papillary muscle;
D. Ventricular aneurysm.
E. * Ruptured ventricular wall;
38. In a patient with hypertension disease, in the context of a hypertensive crisis, acute renal failure, from
which he died, developed. What are the most likely morphological changes in kidney arterioles?

A. Constrictive atherosclerosis
B. Hyperelastosis
C. Sclerosis
D. Hyalinosis
E. * Fibrinoid necrosis

39. Autopsy has revealed shrunken kidneys weighing 50 mg, with finegrained surface and uniformly
thinned substance. Microscopic investigation has shown the thickening of arteriole walls due to
accumulation of homogeneous anhistic pink-colored masses in them. Glomerules were undersized,
sclerotic, with atrophied tubules. What disease are these changes characteristic of?
A. Membranous nephropathy
B. Pyelonephritis with kidney shrinkage
C. Renal amyloidosis
D. Acute glomerulonephritis
E. * Essential hypertension
40. A 60-year-old woman has reported incrising fatigue over the past year. An abdominal ultrasound
scan shows that her kidneys are symmetrically smaller than normal whith fine-grained surface. The
microscopic appearance of the kidneys showed sclerosis and glomeruli hyalinosis. These finding are
most likely to indicate which of the following underlying conditions?
A. Escherichia coli septicemia
B. Polyarteritis nodosa
C. Atherosclerosis
D. Takayasu arteritis
E. * Systemic hypertension
41. In man of 60 years old, who had hypertension for a long time and died of chronic renal failure at
autopsy was revealed: both kidneys are greatly reduced in size, the surface is fine-grained;
histologically - the majority of glomerulars is hialinised and sclerosed, some glomerulars are -
hyperplastic; areas of sclerosis are detected in stroma, arteriolosclerosis and atherosclerosis,
elastofibrosis of large branches of renal artery. Name identified changes of kidneys.
A. Arteriosclerotic kidney
B. Atherosclerotic nephrosclerosis
C. Chronic glomerulonephritis
D. Chronic pyelonephritis
E. * Arteriolosclerotic nephrosclerosis
42. An autopsy of a 9-year-old child, who suffered from rheumatism and died of heart failure, revealed
dilatation of cavities in the ventricles of his heart. Microscopically, the myocardial stroma was
characterized by a plethora, oedema, diffuse infiltrations of histiocytes, lymphocytes, neutrophils and
eosinophils. What diagnosis was the most probable one?
A. Focal interstitial exudative myocarditis
B. Granulomatous productive myocarditis
C. Interstitial productive myocarditis
D. Alterative myocarditis
E. * Diffuse interstitial exudative myocarditis
43. An autopsy of a female revealed morphological manifestations of chronic heart failure in the right
ventricle, stenosis of the left atrioventricular aperture, insufficiency of the mitral valve.
Histologically, a connective-tissue disorganization in the form of some mucoid and fibrinoid swelling
was found with presence of blooming Aschoff s bodies against a background of focal cardiosclerosis
in the myocardium. Which of the diagnoses listed below was the most probable?
A. Scleroderma
B. Dermatomyositis
C. Polyarteritis nodosa
D. Systemic lupus erythematosus
E. * Rheumatism

44. A histological examination of the cusps of the mitral valve in the heart of a female, who died from
cardiac decompensation, revealed a focal desquamation of endotheliocytes with superimposed
thrombotic masses in these areas. The connective tissue of the valve cusp had signs of
disorganization, areas of sclerosis and angiomatosis. Diagnose the kind of valvular endocarditis.
A. Diffuse
B. Acute vegetative
C. Loffler's
D. Polypous-ulcerous
E. * Recurrent vegetative
45. An autopsy of a 7-year-old child, who died from progressing heart failure, revealed 200 ml of some
semitransparent fluid in the pericardial cavity, the surface of the epicardium had greyish filamentous
superpositions. Name the kind of pericarditis.
A. Serous
B. Purulent
C. Fibroplastic
D. -
E. * Fibrinous
46. A 45-year-old female, who complained of progressing muscular weakness, underwent a biopsy of
soft tissues on her shin. A histological examination of the biopsy revealed some microfocal
petrification of the derma and skeletal muscles, a reduced amount of glycogen and transversal
striation in the muscular fibres, some fibres were necrotized, the stroma was infiltrated by
lymphocytes, macrophages and plasma cells. Make a diagnosis of the disease.
A. Systemic scleroderma
B. Systemic lupus erythematosus
C. Rheumatism
D. Polyarteritis nodosa
E. * Dermatomyositis
47. A 38-year-old female suffers from an expressed deformity of joints of her lingers und Iocs.
Histologically, the periarticular connective tissue reveals some mucoid swelling, foci of fibrinoid
necrosis, clusters of macrophages and areas of sclerosis, the synovial membrane has an oedema of
villi, as well as their mucoid and fibrinoid swelling, the synovial cavity contains "rice bodies". Make
a diagnosis of the disease.
A. Rheumatism
B. Bekhterev's disease
C. Infectious polyarthritis
D. Polyarteritis nodosa
E. * Rheumatoid arthritis
48. An examination of the female, who died from renal insufficiency, revealed that her skin in the
regions of the bridge of the nose and lateral surfaces of the face was brown-reddish and
desquamative. On autopsy, the heart was enlarged, the cusps of the aortic valve were thickened,
dense and had thrombotic superpositions. The aortic wall had elastolysis and small scars in the
middle coat. The kidneys were motley, enlarged, with foci of haemorrhages in their cortical layer.
Microscopically, there were haematoxylin bodies in the epithelial nuclei, the basal membranes of
capillaries of the glomeruli were thickened and gave an appearance of wire loops, somewhere the
capillaries contained hyaline thrombi and foci of fibrinoid necrosis. Make a diagnosis.
A. Rheumatism
B. Arteriolar nephrosclerosis
C. Nephropathy amyloidosis
D. Arterial nephrosclerosis
E. * Systemic lupus erythematosus

49. Microscopy of the kidneys from a man died of systemic lupus erythematosus revealed sclerosed
glomeruli, the lumens of the small arteries and arterioles are narrow, the median membrane is thin,
homogeneous, eosinophilic masses are present in the subendothelial space. Immunologically these
masses contain immune complexes and fibrin. Which substance is present in the subendothelial
space?
A. Fat-protein detritus
B. Simple hyaline
C. Lipohyalin
D. Amyloid
E. * Complex hyalin
50. On autopsy of a 16-year-old male juvenile with reduced nourishment, who was ill with primary
pulmonary tuberculosis and died from an accompanying pneumonia, it was found that the 2nd
segment of the right lung had a Ghon’s focus, the peribronchial and bifurcation lymph nodes were
enlarged, united in packs, grey-yellow, dense in consistency and crumbled. Microscopically, the 3rd
segment of the right lung contained a primary affect surrounded by fibrosed tuberculous granulomata
and a connective-tissue capsule; the lymph nodes were characterized by caseous necrosis. Which of
the forms of tuberculosis was the most probable?
A. Caseous pneumonia
B. Acute focal pulmonary tuberculosis
C. Chronic course of primary tuberculosis
D. Haematogenous acute miliary tuberculosis
E. * Lymphogenous progressing of tuberculosis
51. The 48-year-old patient, suffered from fibrous-cavernous tuberculosis, has complained of weakness,
reduction of daily secretion of urine, edema of the body and extremities and increasing of blood
pressure to 180/90. The increasing of protein, presence of hyaline and grain cylinders and
erythrocytes were found in urine. The patient has died in a month because of the insufficiency of
kidney. In autopsy the enlargement of the heart and "lardaceous" kidneys with weight more than 240
g were found. What is the complication of fibrous-cavernous tuberculosis?
A. Nephrotic syndrome
B. Glomerulonephritis
C. Pyelonephritis
D. Nephrosclerosis
E. * Amyloidosis
52. In autopsy of patient, which suffered by tuberculosis for a long time, the cavity by the sizes 3х2 cm
connected with a bronchus was found out in the upper lobe of right lung. The wall of the cavity dense
has three layers: internal - pyogenic, middle - layer of tubercular granulation tissue, external – layer
of connective tissue. What is the most probable diagnosis which this formation relative to?
A. Tuberculoma.
B. Acute focal tuberculosis.
C. * Fibrous-cavernous tuberculosis.
D. Acute cavernous tuberculosis
E. Fibrous-focal tuberculosis.
53. During autopsy of the patient, which suffered by the progressing form of a secondary tuberculosis,
enlarged dense left lung was found out. The tissue of the lung had yellow color, on the pleura there
were fibrinous coverings. Microscopic examination showed the predominance of necrotic changes.
What from the offered diagnosis is most probable?
A. Fibrous-cavernous tuberculosis.
B. Infiltrative tuberculosis.
C. Acute focal tuberculosis.
D. Fibrous-focal tuberculosis.

E. * Caseous pneumonia.
54. A male patient complained of a fever, a severe headache, dyspnoea, and tachycardia. He died under
the phenomena of heart failure. On visual examination of the body, roseolae and petechiae on the
skin of the thorax, bedsores of the shins and a gangrene of the toes were found. A histological
examination of the myelencephalon tissue revealed a hyperaemia, stases, perivascular cuffs of plasma
cells and foci of a proliferation of the microglia (Popov’s granulomata). What is your diagnosis?

A. Meningococcal meningoencephalitis
B. Anthracic meningoencephalitis
C. Typhoid fever
D. Epidemic typhus
E. * Rubella
55. A 34-year-old male injection addict died from progressing respiratory insufficiency. A
pathoanatomical examination revealed the following changes: enlarged peribronchial, bifurcation,
paratracheal and paraaortic lymph nodes; the pulmonary tissue was air-free, the surface of its section
discharged a large amount of some mucous-foamy viscous fluid. Microscopically, the lumens of
alveoli contained foamy eosinophilic masses and clusters of macrophages, among which on silvering
some small round grey-black corpuscles with a light area in the centre were identified. Make a
diagnosis of the disease.
A. Streptococcal pneumonia
B. Lung candidiasis
C. Pneumococcal pneumonia
D. Cytomegalovirus pneumonia
E. * Pneumocystic pneumonia
56. A serviceman, who returned from Chechnya, was admitted to a hospital in a bad state with
disturbances of consciousness, swallowing and respiration. The skin of his extremities, trunk and the
conjunctiva of his eyes had petechial eruptions. The death was caused by arrest of respiration. A
pathoanatomical examination of the capillaries and arterioles of the grey matter, myelencephalon,
pons and subcortical ganglia revealed destructive-proliferative endothrombovasculitis with
perivascular nodules consisting of the glial cells, lymphocytes and histiocytes. The skin and kidneys
revealed vasculitis, the heart had focal isolated myocarditis. What disease should be suspected?
A. Brucellosis
B. Poliomyelitis
C. Polyarteritis nodosa
D. Rheumatism
E. * Epidemic typhus
57. A geologist, who several months before had been on an expedition in the Central Asia, had
paroxysms of a fever, an icteric colouring of the skin, hypochromic anaemia and loss of body mass
during his life-time. On autopsy, a sharp enlargement of the spleen and liver, as well as hyperplasia
of the marrow were found. Microscopically, there was haemomelanosis of the liver, spleen and
marrow. What disease are such changes typical for?
A. Relapsing fever
B. Chronic septicaemia
C. Epidemic typhus
D. Amoebiasis
E. * Malaria

58. A 15-year-old female patient died at an infectious hospital on the 3rd day of some disease from
intoxication. On autopsy, a punctate rash on the skin, excluding the region of the nasolabial triangle,
was found. The fauces, tonsils and tongue were brightly hyperaemic (the (glowing fauces(, a
(strawberry tongue(). Microscopically, the internal organs revealed prevalence of dystrophic changes
and a sharp disturbance of circulation. Which of the diagnoses was the most probable?
A. Measles
B. Septic form of scarlet fever
C. Allergic period of scarlet fever
D. Diphtheria of respiratory tract
E. * Toxic form of scarlet fever
59. A disease in an 8-year-old boy began acutely with a catarrh of the upper respiratory tract and
conjunctivitis, on the 3rd day of the disease he developed some macrofocal papular eruption behind
the ears, and later on the face, neck and trunk. The patient died on the 10th day under the phenomena
of severe respiratory insufficiency. An autopsy revealed lobular pneumonia with foci of suppuration.
Microscopically, he had purulonecrotic panbronchitis and bronchiolitis, peribronchial interstitial
pneumonia with a focal infiltration of the interstice by lymphoid, plasma cells, macrophages and
giant multinucleate cells. Which of the diagnoses listed below was the most probable?
A. Influenza
B. Respiratory-syncytial infection
C. Adenovirus infection
D. Scarlet fever
E. * Measles
60. A 24-year-old female, who 2 months before had had diphtheria and suffered from paralysis of the
soft palate, suddenly died from heart failure. On autopsy, an eccentric hypertrophy of the heart and a
dim flaccid myocardium were found. A microscopic examination of the glossopharyngeal, phrenic
nerves and vagus revealed a lumpy breakdown of the myelin and a plethora of the epineural vessels.
The 3rd cervical sympathetic ganglion and the nodose one of the vagus were characterized by an
impaired circulation and phenomena of cytolysis. Which of the diagnoses was the most probable?
A. Exudative myocarditis
B. Alterative myocarditis
C. Isolated myocarditis
D. Toxic myocarditis
E. * Late paralysis of heart
61. Against a background of a satisfactory general state and a subfebrile body temperature, a visual
examination of a 6-year-old child revealed eruptions on the face, head and trunk in the form of red
spots and vesicles having transparent contents, some vesicles were covered with crusts. From the side
of the fauces and upper respiratory tract, expressed catarrhal changes were determined.
Microscopically, the areas of the eruptions were characterized by a plethora of the vessels, a balloon
dystrophy and giant multinucleate cells in the spinous layer of the epidermis. The oral mucosa had
small erosions, plethoric vessels and perivascular lymphohistiocytic infiltrates. Which of the
diagnoses was the most probable?
A. Scarlet fever
B. Measles
C. Smallpox
D. Rubella
E. * Varicella
62. 14 days after quinsy a 15-year-old child presented with morning facial swelling, high blood pressure,
"meat slops" urine. Immunohistological study of a renal biopsy sample revealed deposition of
immune complexes on the basement membranes of the capillaries and in the glomerular mesangium.
What disease developed in the patient?

A. Necronephrosis
B. Acute interstitial nephritis
C. Lipoid nephrosis
D. Acute pyelonephritis
E. * Acute glomerulonephritis
63. Autopsy of a 52-year-old woman with a long history of chronic glomerulonephritis revealed
significantly reduced in size, dense kidneys with a surface of fine granularity; fibrinous inflammation
of serous and mucous membranes; dystrophic changes in parenchymatous organs; cerebral edema.
The described changes of serous membranes and internal organs are caused by the following
complication:
A. Thrombocytopenia
B. Anaemia
C. Sepsis
D. * Uraemia
E. DIC syndrome
64. A 45-year-old patient with an 8-year history of tuberculosis died in a hospital of chronic renal failure.
At autopsy, the kidneys were enlarged, the cross-section surface looked greasy, histological study
revealed profuse deposits of structureless homogeneous eosinophilic masses exhibiting marked
metachromasia when stained with Congo red. What pathological process developed in the kidneys?
A. Nephrosclerosis
B. Hematogenous renal tuberculosis
C. Acute glomerulonephritis
D. Toxic nephritis against the background of antibiotic therapy
E. * Secondary amyloidosis
65. The man of 23 years old fell ill after acute hypothermia. There was high blood pressure, hematuria,
swelling of the face. The treatment was ineffective. After 6 months he died of uremia. At the section
of kidneys they are increased, flabby, cortical layer of broad, swollen, yellow-gray, dull, with a red
specs, well bordereded from crimson brain substance. Microscopically detected proliferation of
capsule glomerular epithelium, podocytes and macrophages with formation of a half-moons. Choose
a diagnosis.
A. Acute glomerulonephritis
B. Chronic glomerulonephritis
C. Acute pyelonephritis
D. Renal amyloidosis
E. * Subacute glomerulonephritis
66. The patient died at signs of azotaemic uremia. At the autopsy: enlarged kidney; swollen, red-blooded,
easily removable capsule. Cavities of kidney pelvises and cups are enhanced, filled with cloudy urine
pus, their membranes are dull, with focuses hemorrhage. At section kidney tissue is mottled, yellow-
gray areas are surrounded by a zone of hyperemia and hemorrhage. What disease does correspond to
the macroscopic view of the kidneys?
A. * Acute pyelonephritis
B. Acute glomerulonephritis
C. Renal amyloidosis
D. Chronic pyelonephritis
E. Chronic glomerulonephritis

67. 42 year old man who suffered severe typhoid fever, ARF (acute renal failure) developed, from which
he died. At autopsy the kidneys are increased in size, swollen, fibrous capsule is removed easily, at
section - cortical substance is pale gray, dark red pyramids. At histological examination in the
majority of tubules the lumen is narrowed, epithelial cells increased in size without nuclei;
glomerulars are collapsed; in stroma - edema, small leukocyte infiltration, minor hemorrhage.
Determine the renal pathology that caused the death of the patient.
A. * Necronephrosis
B. Acute pyelonephritis
C. Acute glomerulonephritis
D. Pionephros
E. Nephrotic syndrome
68. Patient of 49 years hospitalised because pain in the waist. At ultrasound examination in dramatically
enhanced renal pelvis and cups of right kidney stones were found. Nephrectomy was performed. At
morphological study: kidney is sharply increased, atrophied parenchyma, cups and pelvises are
distended, filled with clear yellowish liquid. Microscopically revealed diffuse sclerosis, tubular and
glomerular atrophy saved tubules are distended with cysts. What complications of renal stone disease
emerged in the patient?
A. * Hydronephrosis
B. Pionefrosis
C. Pyelonephritis
D. Glomerulonephritis
E. Chronic paranephritis
69. During the autopsy woman died of uremia, kidneys of different sizes were found with
macrotubercular surface, between the renal capsule and renal surface there are dense adhesions. At
microscopic examination the following focal renal changes were identified: Multiple encapsulated
abscesses, expressed limfohistiocytic interstitial infiltration and widening of the connective tissue.
Mucous membrane of kidneys with focuses of metaplasia of transitional epithelium into multilayerd.
Tubules are partially atrophied, partially distended and filled with colloid-like content Which of the
following diagnoses is the most likely?
A. * Chronic pyelonephritis
B. Chronic glomerulonephritis
C. Acute pyelonephritis
D. Acute glomerulonephritis
E. Tubuloyinterstitial nephritis
70. A scrape from the uterine cavity revealed in the blood some particles of a neoplasm which had an
organoid structure and consisted of the columnar epithelial cells which formed glandular structures;
the stroma of the neoplasm was significantly developed, the area of its (pedicle( had glomi of thick-
walled vessels. Which of the tumours was the most probable?
A. Glandular hyperplasia of endometrium
B. Uterine fibroadenoma
C. Endometriosis
D. Uterine adenocarcinoma
E. * Endometrial polyp
71. On gynaecological examination of a 36-year-old female, a red focus, which was 0.8 x 0.5 cm in size
and had uneven contours, was found in the mucous coat of the vaginal portion of the uterine cervix
on its border with the mouth of the womb. A microscopical examination revealed substitution of the
stratified squamous epithelium for the columnar one, as well as accumulation and new formation of
glands from the cambial elements of the columnar epithelium in the cervical channel were observed
under the integmentary epithelium. Which of the diagnoses listed below was the most probable?
A. Adenomatosis of uterine cervix

B. Healing endocervicosis
C. Simple endocervicosis
D. Dysplasia of epithelium of uterine cervix
E. * Proliferative endocervicosis
72. In a biopsy of сervix of a 26-year-old woman the diagnosis following was established: pseudo-
erosion. What microscopical changes has the pathologist revealed?
A. Cell-atypia of an epithelium of an mucosal epithelium
B. Keratinization of an epithelium
C. “Carcinomatous pearls”
D. Local inflammation and necrosis in mucosa
E. * Local changes of a stratified squamous epithelium on single-layer prismatic one
73. Diagnostic scraping was performed to the woman with dysfunctional uterine bleeding. Multiple
convoluted glands, ganglially dilated cavities of some glands were revealed histologically in the
scrape. Name the type of general pathological process.
A. Hypertrophic excrescence
B. Metaplasia
C. Displasia
D. Atrophy
E. * Glandular- cystic hyperplasia
74. A patient with gastric juice hypersecretion has been recommended to exclude from the diet rich
broths and vegetable infused water. A doctor recommended it, because these food products stimulate
production of the following hormone:
A. * Gastrin
B. Secretin
C. Cholecystokinin
D. Somatostatin
E. Neurotensin
75. A 49-year-old patient was found to have a disproportionate enlargement of hands, feet, nose, ears,
superciliary arches and cheek bones. Blood test revealed hyperglycemia, impaired glucose tolerance.
What is the most likely cause of this pathology development?
A. * Hypersecretion of growth hormone
B. Posterior pituitary hormone hypersecretion
C. Insulin hyposecretion
D. Vasopressin hyposecretion
E. Glucocorticoid hypersecretion
76. An autopsy of the body of an elderly man, who was suffering from acute intestinal disorder during
his last 2 weeks of life, has revealed the following change in the rectum and sigmoid colon: brown
and green film covering the mucosa is detected. The itestinal wall is thickened; the cavi- ty sharply
narrows down. Microscopy reveals mucosa necrosis of varying depth, necrotic tissue is pierced
through with fibrin threads, leucocytic infiltrati- on is observed. What diagnosis is the most probable?

A. * Fibrinous colitis
B. Catharrhal colitis
C. Ulcerative colitis
D. Follicular colitis
E. -
77. A 43-year-old woman complains of weight loss, hyperhidrosis, low-grade fever, increased irritability.
She has been found to have hyperfunction of the sympathetic-adrenal system and basal metabolism.
These disorders can be caused by hypersecretion of the followi- ng hormone:

A. * Thyroxine
B. Somatotropin
C. Corticotropin
D. Insulin
E. Aldosterone
78. Specify the type of jaundice, during which there is no direct bilirubin in blood, and urine
urobilinogen level is high:
A. * Suprahepatic
B. Hepatic
C. Subhepatic
D. Mechanical
E. -
79. A 35-year-old woman complai- ns of swollen neck. Subtotal thyreoidectomy is performed. On
histological examination of the removed part of the thyroid gland the following was detected: atrophy
of parenchyma, moderate sclerosis development, duffuse infiltration by lymphocytes and plasma
cells leading to formation of lymphatic follicles. What pathology has developed in the thyroid gland?

A. * Hashimoto’s thyroiditis
B. Follicular adenoma
C. Riedel’s thyroiditis
D. Papillary carcinoma of the thyroid gland
E. Diffuse toxic goiter
80. A 55-year-old man had been suffering from chronic glomerulonephritis. He died from chronic renal
failure. Macroscopical examination revealed on the surface of epicardium and pericardium some
greyish-white villous depositions. After their removal dilated and plethoric vessels were uncovered.
What process took place in the pericardium?
A. Organization
B. Haemorrhagic inflammation
C. Proliferative inflammation
D. Arterial hyperemia
E. * Fibrinous inflammation
81. A 39-year-old man who had been operated for the stomach ulcer died 7 days after the surgery.
Autopsy revealed that peritoneal leaves were dull, plephoric, covered with massive yellow-greenish
films, the peritoneal cavity contained for about 300 ml of thick yellow-greenish liquid. What
pathologic process was revealed in the peritoneal cavity?
A. Peritoneal commissures
B. Fibrinous serous peritonitis
C. Fibrinous haemorrhagic peritonitis
D. Serous peritonitis
E. * Fibrinous suppurative peritonitis
82. Preventive examination of a 55-year-old patient revealed type II diabetes mellitus. An
endocrinologist revealed an increase in body weight and liver enlargement. The man is non-smoker
and doesn't abuse alcohol but likes to have a good meal. Histological examination by means of
diagnostic liver puncture revealed that the hepatocytes were enlarged mostly on the lobule periphery,
their cytoplasm had transparent vacuoles showing positive reaction with sudan III. What liver
pathology was revealed?
A. Acute viral hepatitis
B. Chronic viral hepatitis
C. Alcohol hepatitis

D. Portal liver cirrhosis


E. * Fatty hepatosis
83. A female patient with a tumour of pancreas has developed mechanic jaundice resulting from
compression of a bile-excreting duct. Which duct is compressed?
A. Ductus hepaticus dexter
B. * Ductus choledochus
C. Ductus hepaticus sinister
D. Ductus cysticus
E. Ductus hepaticus communis
84. Examination of a 32-year-old patient revealed disproportional skeleton size, enlargement of
superciliary arches, nose, lips, tongue, jaw bones, feet. What gland's function was disturbed?
A. Epiphysis
B. Pancreas
C. Suprarenal
D. Thyroid
E. * Hypophysis
85. Autopsy of a 67-year-old man who died after presenting with hypoglycemic coma revealed some
areas of connective tissue growth and necrosis foci, atrophy of Langerhans islets in pancreas. What
disease might have induced such changes in pancreas?
A. Acute pancreatitis
B. * Diabetes mellitus
C. Cancer of the head of pancreas
D. Pancreas hypoplasia
E. Mucoviscidosis
86. A 55 year old man had been suffering from chronic glomerulonephritis. He died from chronic renal
failure. Macroscopical examination revealed on the surface of epicardium and pericardium some
greyish-white villous depositions. After their removal dilated and plethoric vessels were uncovered.
What process took place in the pericardium?
A. * Fibrinous inflammation
B. Organization
C. Proliferative inflammation
D. Haemorrhagic inflammation
E. Arterial hyperemia
87. Microscopic examination of periodontium revealed plethoric vessels, edema of connective tissue
along with infiltration by single neutrophils. What type of exudative inflammation in the
periodontium is it?
A. * Serous
B. Purulent
C. Putrid
D. Fibrinous
E. Catarrhal
88. A 60 year old patient complains of tongue burning, excessive salivation and glossalgia effects that
came 5 days after he started using a metal dental bridge. Objectively: mucous membrane of oral
cavity is edematic and hyperemic. What form of stomatitis is it?
A. * Catarrhal
B. Purulent
C. Ulcerous
D. Gangrenous

E. Fibrinous
89. A 23 year old man got perforation of hard palate. There was also a solid welldefined formation.
Postoperative microscopical examination of this formation revealed a large focus of caseous necrosis
surrounded by granulation tissue with endovasculitis and cellular infiltrate consisting mainly of
plasmocytes but also of lymphocytes and epithelioid cells. What is the most probable diagnosis?
A. * Syphilis
B. Tuberculosis
C. Scleroma
D. Sarcoma
E. Lepra
90. A 40 year old male patient died from cerebral edema. In anamnesis the face carbuncle was registered.
Autopsy revealed hyperemia and edema of cerebral tissue. White matter of the left hemisphere had
two cavities 6х5,5 and 5х4,5 cm large filled with yellowish-green cream-like fluid. Walls of the
cavities were built up by nerve tissue with irregular rands. What complication of carbuncle was it?
A. * Acute abscesses
B. Chronic abscesses
C. Empyema
D. Colliquative necroses
E. Cysts
91. Examination of puncture biopsy material of liver revealed dystrophy of hepatocytes, their necrosis
and sclerosis along with disturbance of beam and lobulous structure and formation of pseudolobules
of regeneration nodes. What is the most probable diagnosis?
A. Chronic hepatosis
B. Chronic hepatitis
C. Progressing massive liver necrosis
D. Acute hepatitis
E. * Liver cirrhosis
92. Autopsy of a 34 y.o. man who di- ed from rheumatism revealed that epi- cardium surface was villous
and covered with grey films that can be easily removed. After their removal the surface is edematic
and plethoric. What is the most probable diagnosis?
A. * Fibrinous pericarditis
B. Purulent pericarditis
C. Hemorrhagic pericarditis
D. Proliferative pericarditis
E. Catarrhal pericarditis
93. Opening of a patient’s abdominal cavity revealed for about 2,0 L of purulent fluid. Peritoneum is
dull, greyish, serous tunic of intestines has grayish layers that can be easily removed. It is most likely
to be:
A. * Fibrinopurulent peritonitis
B. Hemorrhagic peritonitis
C. Serous peritonitis
D. Tuberculous peritonitis
E. –
94. In course of gastric endoscopy the biopsy material of mucous membrane was taken. Its histological
examination revealed the following: mucous membrane is intact, thickened, edematic, hyperemic,
with small dropli- ke hemorrhages, coated with thick mucus. Name the form of acute gastritis:
A. * Catarrhal
B. Erosive
C. Fibrinous

D. Purulent
E. Necrotic
95. During the histologic lung analysis of a man who died from cardiac insufficiency the inflammation
focuses were revealed. Alveoles were full of light-pink fluid, here and there with pinkish fibers that
formed a close-meshed reticulum with a small number of lymphocytes. What type of exudate is
present in lungs?
A. * Serofibrinous
B. Hemorrhagic
C. Serous
D. Purulent
E. Fibrinous
96. Histological examination of thyroid gland of a man who died from cardiac insufficiency
accompanied by hypothyroi- dism revealed diffuse infiltration of the gland by lymphocytes and
plasmocytes with formation of lymphoid follicles, as well as atrophy of parenchyma and growth of
connective tissue. What is the most probable diagnosis?
A. * Autoimmune Hashimoto’s thyroiditis
B. Adenoma of thyroid gland
C. Purulent thyroiditis
D. Thyrotoxic goiter
E. –
97. An autopsy of a 7-year-old child, who died from progressing heart failure, revealed 200 ml of some
semitransparent fluid in the pericardial cavity, the surface of the epicardium had greyish filamentous
superpositions. Name the kind of pericarditis.
A. Serous
B. Purulent
C. Fibroplastic
D. * Fibrinous
E. -
98. A 40-year-old female patient underwent an operation of thyroidectomy. A histological examination
of the thyroid tissue revealed that its follicles differed in size, contained some foamy colloid, the
follicular epithelium was high and in some places it formed papillae. The stroma of the gland had
clusters of lymphocytes which formed follicles with light centres. Make a diagnosis of the disease of
the thyroid gland.
A. Hashimoto's disease
B. Ligneous thyroiditis
C. Acute nonsuppurative thyroiditis
D. Nodular goiter
E. * Toxic goiter
99. A histological examination of the thyroid gland revealed a significant infiltration of its tissue by
lymphocytes, formation of lymphoid follicles, an atrophy of parenchymatous elements and a
significant vegetation of the connective tissue. What disease is characterized by this picture?
A. Colloid goiter
B. Endemic goiter
C. Diffuse toxic goiter
D. Parenchymatous goiter
E. * Hashimoto's disease

100. An autopsy of a young female, who died from adrenal insufficiency, revealed diffuse hypermelanosis
of the skin, hyperplasia of the cells in islets of Langerhans in the pancreas, the adrenal glands were
sharply reduced in size and their thinned cortical substance had foci of necrosis, haemorrhages and
sclerosis. What is your diagnosis?
A. Waterhouse-Friderichsen syndrome
B. Primary aldosteronism
C. Cushing's disease
D. * Addison's disease
E. Pheochromocytoma
101. In a young male, an abundant quantity of the somatotropic hormone and enlargement of the nose,
lips, ears, loWER jaw, hands and feet were revealed. What is your diagnosis?
A. Pituitary dwarfismB.
B. shing's disease
C. Addison's disease
D. Adiposogenital dystrophy
E. * Acromegaly
102. In a male patient with an increased level of the parathormone, a histological examination in the area
of a pathological fracture of his femur revealed foci of a lacunar resolution of the osteoid beams and
new formation of a fibrous tissue. What is your diagnosis?
A. Multiple myeloma
B. Osteoblastoclastoma
C. Paget's disease
D. Osteopetrosis
E. * Parathyroid osteodystrophy
103. A male patient with phenomena of hypothyroidism died from heart failure. On histological
examination, his thyroid gland revealed a diffuse infiltration of the gland by lymphocytes and
plasmacytes, an atrophy of the parenchyma and a vegetation of the connective tissue. What disease
was it?
A. Acute nonsuppurative thyroiditis
B. Ligneous thyroiditis
C. Nodular goiter
D. Toxic goiter
E. * Hashimoto's disease
104. An autopsy of a 45-year-old female, who was suffering from arterial hypertension, diabetes mellitus
and ovarian dysfunction during past 15 years, revealed obesity by the upper type, a pituitary basophil
adenoma in the anterior lobe of the hypophysis, hyperplasia of the adrenal cortex. Which of the
diagnoses listed below was the most probable?
A. Cushing's syndrome
B. Hypertensive disease
C. Addison's disease
D. Adiposogenital dystrophy
E. * Cushing's disease
105. A 50-year-old female took medical advice complaining of excretion of a large amount of urine and
excessive thirst. On examination, her nourishment was reduced, the skin was dry, density of the urine
ranged from 1001 to 1010, data of an ultrasound examination and computed tomography of the brain
revealed a tumour in the posterior lobe of the hypophysis. Indicate the most probable disease.
A. Acromegaly
B. Babinsky-Frelich disease
C. Simmonds disease

D. Cushing's disease
E. * Diabetes insipidus
106. A 46-year-old male patient, who suffered from bulimia, polydipsia, polyuria with glucosuria and
albuminuria, died of renal insufficiency. On autopsy, the kidneys were reduced in size, dense and had
a fine-grained surface. The pancreas was reduced and partially substituted for a fatty tissue.
Microscopically, islets of Langerhans were fine, in some places they were substituted for a
connective tissue, solitary ones were hypertrophic. The kidneys reveal intracapillary
glomerulosclerosis. Which of the diagnoses was the most probable?
A. Subacute glomerulonephritis
B. Chronic indurative pancreatitis
C. Diabetes insipidus
D. Chronic glomerulonephritis
E. * Diabetes mellitus
107. A 36-year-old female patient underwent resection of the both lobes of her thyroid gland; each of
them was 5 x 6 cm in size, pink-yellow, moderately dense and had a tuberous surface. A microscopic
examination revealed follicles of various size, some of them were dilated like cysts and filled with
some colloid; the follicular walls were lined with the smoothed cuboidal epithelium; the stroma of the
gland was redundantly developed owing to the connective tissue, there were foci of calcinosis. Which
of the diseases listed below corresponded most to the changes found?
A. Parenchymatous goiter
B. Toxic goiter
C. Hashimoto's disease
D. Ligneous thyroiditis
E. * Colloid goiter
108. An autopsy of a 24-year-old female (from her case history it is known that a year before the woman
had given birth to a child) revealed a sharp decrease of the body weight down to 38 kg, the skin was
dry and thin, the weight of the internal organs was loWER ed. Also, there was a sharp decrease in the
weight of the adenohypophysis, the latter had cicatrices; there were foci of dystrophy, necrobiosis
and hyalinosis in the diencephalon. The ovaries, thyroid and adrenal glands had phenomena of
hypotrophy, the mucous membrane of the intestines was atrophied. Which of the diagnoses was the
most probable?
A. Nutritional dystrophy
B. Suprarenal cachexia
C. Cachexia associated with chronic amoebiasis
D. Cachexia associated with pellagra
E. * Cerebrohypophysial cachexia
109. An autopsy of a 45-year-old female patient, who suffered from obesity by the upper type, steroid
diabetes mellitus, arterial hypertension and secondary ovarian dysfunction, revealed hypertrichosis,
hirsutism, striae on the skin of the thighs and abdomen. The anterior lobe of the hypophysis
contained a white-pink encapsulated tumour, 2.5 cm in diameter (microscopically, it was a pituitary
basophil adenoma); the adrenal glands were characterized by bilateral hyperplasia of the fascicular
layer. Which of the diagnoses was the most probable?
A. Cushing's syndrome
B. Adiposogenital dystrophy
C. Simmonds disease
D. Pituitary dwarfism
E. * Cushing's disease

110. A 52-year-old male died from renal insufficiency. On microscopic examination of his organs, the
pancreas revealed lipomatosis and sclerosis with an atrophy of islets of Langerhans, the kidneys had
hyalinosis of the mesangium and glomeruli (Kimmelstiel-Wilson syndrome) and a glycogenic
infiltration of the epithelium of the tubules, the liver was characterized by fatty degeneration. Which
of the diagnoses listed below was the most probable?
A. Arterial nephrosclerosis
B. Amyloid wrinkled kidneys
C. Chronic glomerulonephritis
D. Goodpasture's syndrome
E. * Diabetic glomerulosclerosis
1. In the pathomorphological laboratory sent a connective tissue with a large arterial vessel, in the
lumen of which a grayish-colored dry brittle mass, densely connected with the vessel wall. At the
histological examination, an arterial vessel has a mass of obliterating nature that contains fibrin,
platelets, and a small number of red blood cells with an overwhelming number of leukocytes with the
phenomenon of initial organization. What kind of process is there in this case?
A. Connective tissue in the lumen of blood vessels
B. Thromb-embolus in the lumen of the vessel
C. Hyaline thrombus in the lumen of the vessel
D. A blood vessel in the lumen of blood vessels
E. * Cladding white blood clot in the lumen of the arterial vessel
2. A man 56 years old, for 20 years has not suffered from chronic ischemic illness. Suddenly there were
pains in the heart area, sharply increased cynosiness of the face, there was a "feeling of fear of
death." Despite the onset of treatment, death has occurred. At the autopsy, a large left ventricular
heart attack has been detected. A large amount of pinkish-reddish foamy fluid flows from the surface
of the lung. In the liver a peculiar figure resembling muscat has been found, blood from dark cherry
drains from the surface of the cut. Morphological changes in the lungs and liver are manifestations:
A. local arterial hyperemia
B. general acute arterial hypertension
C. Local acute venous hyperemia
D. local chronic venous hyperemia
E. * general acute venous hyperemia
3. A 68-year-old woman developed acute myocardial infarction as a result of stenotic atherosclerosis of
the coronary arteries. During the section it was revealed: in the lumen of the anterior interventricular
branch of the left coronary artery there are brittle gray-yellow masses reminiscent of the
atheromatous detritus and completely lining the lumen of the vessel, in the thickness of the anterior
and lateral walls of the left ventricle - transmural portion of irregular shape, gray-yellow color,
periphery surrounded by a hemorrhagic crown. Diagnose the cause of a heart attack.
A. Tissue embolism
B. Embryos by extraneous bodies
C. Fatty embolism
D. Retrograde embolism
E. * Thrombosis
4. The section revealed: multiple hemorrhagic pulmonary heart attacks, some pulmonary vessels of a
lilac-colored dense mass, varicose veins of the lower extremities, in which the presence of fragile
dark-brown colored masses. What kind of pathological process is being discussed?
A. Fatty pulmonary artery embolism.
B. Tissue embolism of the vessels of the pulmonary artery.
C. Congestive thrombosis of the vessels of the pulmonary artery.
D. Hemorrhagic bronchopneumonia.
E. * Trombembolia of the vessels of the pulmonary artery
5. At the autopsy, an enlarged liver of a colorful species, with a pattern of nutmeg in the section, was
discovered. In the lumen of the liver veins, the wall clots are found. What kind of disruption of blood
circulation in the liver?
A. General venous plethora
B. Anemia

C. Hemorrhage
D. Bleeding
E. * Local venous hyperemia
6. In a pilot who died as a result of depressurizing the cockpit, a large number of vesicles were detected
in the histological examination of the internal organs in the blood vessels, and in the liver it was fecal
dystrophy. In the brain and spinal cord - multiple small ischemic centers of gray softening. Indicate
the most likely cause of such changes.
A. Fatty embolism
B. Air embolism
C. Thromboembolism
D. Tissue embolism
E. * Gas embolism
7. A man 57 years old, for 20 years suffered from emphysema of the lungs. He died of heart failure. At
the intersection: cirrhosis, mucous membranes and cerebellar cysts. Expressed swelling of the dermis
and subcutaneous tissue, hydrothorax, hydropericardium, ascites. The liver has a "muscat" ("heart")
cirrhosis. The described changes are a manifestation:
A. local acute arterial hyperemia
B. general acute arterial hyperemia
C. general acute venous hyperemia
D. local chronic venous hyperemia
E. * general chronic venous hyperemia
8. Name the mechanism lying in the basis of the pathogenesis of organ-nonspecific autoimmune
diseases.
A. Disturbance of the physiological isolation of organs and tissues to which there is no physiological
tolerance
B. Appearance of new nonshared antigens in the organism
C. Atrophy of the lymphoid system
D. * Disturbance of the immune homeostasis in the lymphoid system
E. Cellular immunodeficiency
9. Name the mechanism lying in the basis of the pathogenesis of organ-specific autoimmune diseases.
A. * Disturbance of the physiological isolation of organs and tissues to which there is no physiological
tolerance
B. Primary disturbance in the immunocompetent system
C. Appearance of new nonshared antigens in the organism
D. Disturbance in the control of immune homeostasis
E. Atrophy of the lymphoid system
10. At the patient, 29 years old, with a split-fold fracture of the right thigh for the 3rd day after injury,
complaints of pain in the chest appeared to the right, difficult breathing. A day later, with the
background of progressive cardio-respiratory failure, death has come. At histological examination in
the blood vessels of the lungs and brain were found sudanophilic drops of orange color, which
completely blocked the lumen of the vessels of the microcirculation bed. What complication is the
death of a patient?
A. Gas embolism.
B. Medication embolism.
C. Microbial embolism.
D. Thromboembolism.
E. * Fat embolism.

11. A man 21 years old died in the phenomena of acute pulmonary edema. At autopsy revealed: skin is
pale with pronounced acrocyanosis. Blood in vessels is thick, dark cherry color. Microscopically in
the vessels of the microcirculatory channel, small thrombi and phenomena of "slag phenomenon" are
detected. Which of the following processes are covered?
A. general chronic venous plethora
B. general acute venous plethora
C. local chronic venous glorification
D. local acute venous hypertension
E. * Condensation of blood
12. The mountaineers on the sixth day of ascension to Everest during a control examination in peripheral
blood revealed that the amount of erythrocytes reaches 10 million in 1 ml of blood. A slight increase
in blood pressure, redness of the skin of the face and visible mucous membranes is noted. What
disorder of blood circulation arose in climbers?
A. DIC-syndrome
B. Blood Condensation
C. general venous plethora
D. local arterial hypertension
E. * general arterial hypertension
13. A woman 23 years old, for several years suffering from fibrous-cavernous pulmonary tuberculosis.
Suddenly there was a pallor of the skin and mucous membranes, dizziness, and fainted. A few
minutes later there was death. At autopsy paid attention to the sharp pallor of the skin, visible mucous
membranes, serous membranes, tissues of the internal organs. Cavities of the heart and large vessels
are empty, spleen is small, wrinkled. Point and spotty hemorrhage under the left ventricular
endocardium of the heart (Minakov spots) was detected. What is a circulatory disorder out of the
following, found in this case?
A. bleeding
B. hemorrhage
C. general chronic anemia
D. Blood congestion
E. * general acute anemia
14. A woman is 29 years old, a worker of a mercury plant, with a marked pallor and light jaundice of the
skin, mucous membranes, and the presence of diapedetic hemorrhages. Complaints about fatigue,
weakness, decreased ability to work, dizziness, unconscious condition. Significant dystrophic
changes were detected in liver biopsy, and the presence of hemosiderin in Kupffer cells. What
general disorder of blood circulation from the below mentioned is in this case?
A. general acute anemia
B. Blood Condensation
C. hemohromatosis
D. local chronic anemia
E. * general chronic anemia
15. At the autopsy of a corpse of a young man who died after a bite of a snake from multiple organ
failure, a slag phenomenon and numerous thrombi in the microvessels of the lungs, kidneys, liver,
adrenal glands, pituitary gland, brain, gastrointestinal tract, skin were revealed. Along with this there
were multiple hemorrhages in the lungs, the brain, in combination with dystrophy and necrosis of
organs and tissues. What kind of blood circulation disorder is in this case?
A. thrombosis
B. general venous plethora
C. Blood Congestion
D. general acute anemia
E. * DIC-syndrome
16. A man 45 years old died of liver failure. At the intersection revealed thrombosis of the liver veins
(Kiri disease), muscat cirrhosis of the liver, ascites. At the anterior abdominal wall, the vessels are
sharply enlarged, filled with dark cherry blossoms, with a characteristic pattern of "jellyfish heads".
The above changes are due to:
A. general venous hypertension
B. a stasis of blood
C. compressive venous hyperemia
D. collateral venous congestion
E. * obstructive venous hyperemia
17. A woman 23 years old was taken to a surgical department diagnosed with "ectopic pregnancy" and a
picture of general acute anemia. During surgery, the surgeon found elastic clots in the cavity of the
small pelvis and about 1000 ml of liquid blood. What type of disorder of blood circulation on the
mechanism of its formation revealed a surgeon?
A. haemorrhagia over rhexin
B. hemorrhagia through diapedesin
C. petechiae autchichems
D. suggilatio
E. * haemorrhagia through diabrosin
18. A 57-year-old man suffering from a trophic ulcer of the left shin has pain, redness, a feeling of
warmth, swelling in the affected shin. During a surgery in the lumen of the veins, a dry clot of blood,
which is easily crumbly, layered, with a rough surface, is detected. The described changes are a
manifestation:
A. phlebothrombosis
B. DIC-syndrome
C. slag phenomenon
D. Blood stasis
E. * thrombophlebitis
19. A 32-year-old woman suffering from subacute septic endocarditis suddenly lost her vision in the
right eye. During examination, an ophthalmologist showed a sharp increase in the lumen of the
central artery of the eye and the presence of a blood clot in it. The detected changes are a
manifestation:
A. phlebothrombosis
B. DIC-syndrome
C. slag phenomenon
D. Blood stasis.
E. * thromboembolism
20. A histological examination of the lungs if a male, who suffered for many years from atopic bronchial
asthma and died of asphyxia, revealed much mucus with an admixture of eosinophils in the lumens of
the bronchioles and small bronchi, sclerosis of interalveolar septa, dilation of alveolar lumens. Which
of the mechanisms in the development of a hypersensitivity reaction took place when a fit of
asphyxia developed?
A. Cytotoxic reaction
B. Immunocomplex reaction
C. Cytolysis owing to lymphocytes
D. Granulomatosis
E. * Reaginic reaction
21. A study of the thymus of a 5-year-old child, who died from acute destructive staphylococcal
pneumonia, revealed a decrease in the weight of the gland down to 3.0 g. On histological
examination, a smaller size of the lobules of the gland with a collapse of the stroma, an inversion of
the layers, and cyst-like Hassal’s bodies were found out. Which of the diagnoses listed below was the
most probable?
A. Thymomegaly
B. Hypoplasia of the thymus
C. Dysplasia of the thymus
D. Agenesia of the thymus
E. * Accidental reaction
22. An examination of a pregnant woman with a rhesus-negative group of blood revealed a high level of
antierythrocyte antibodies; in order to decrease it, a skin flap of her rhesus-positive husband was
grafted to her. Two weeks later the flap was rejected; its microscopic examination revealed
disturbances of circulation, an oedema, a cellular infiltration mostly by lymphocytes, neutrophils and
macrophages. Which of the pathological processes listed below was the most probable?
A. Immediate hypersensitivity
B. Delayed hypersensitivity
C. Granulomatous inflammation
D. Interstitial inflammation
E. * Transplantation immunity
23. An autopsy of a 43-year-old female, who suffered from attacks of expiratory dyspnoea during her
life-time and died from asphyxia, revealed some dense glass-like mucus in the lumens of the bronchi,
their walls were thick, the lungs had foci of an emphysema and atelectases. A histological
examination of the pulmonary tissue revealed some mucus with an admixture of eosinophils in the
lumens of small bronchi, sclerosis of the peribronchial connective tissue and interalveolar septa,
dilation of the lumens in the alveoli. What mechanism of hypersensitivity formed the basis for the
development of asphyxia?
A. Cytotoxic reaction
B. Immunocomplex reaction
C. Cytolysis owing to lymphocytes
D. Granulomatosis
E. * Reaginic reaction
24. An 8-year-old child was done an intracutaneous tuberculin (Mantoux) test with a diagnostic purpose.
Forty-eight hours following the injection of tuberculin, a dense hyperaemic papule, 20 cm in
diameter, with necrosis in its centre formed. Name the mechanism of hypersensitivity which lay in
the basis of the above changes.
A. Anaphylaxis
B. Antibody-dependent cytotoxicity
C. Immunocomplex cytotoxicity
D. Granulomatosis
E. * Cellular cytotoxicity
25. A 16-year-old youth developed oedemata of his face, oliguria and an increased blood pressure 20
days after he recovered from scarlet fever. A urinalysis revealed an increase of relative density,
haematuria, proteinuria. On microscopic examination of a renal biopsy, a picture of intracapillary
proliferative glomerulonephritis was found out, while an electron microscopy revealed deposits on
the basal membranes. Which of the mechanisms listed below lay in the basis of this disease?
A. Anaphylactic reaction
B. Antibody-dependent cytotoxic reaction
C. Cell-dependent cytolysis
D. Granulomatosis
E. * Immunocomplex mechanism
26. Three years ago in the open skin areas of a female worker of a shop which manufactures synthetic
detergents appeared eruptions in the form of papules, vesicles and small weeping erosions, some of
them were covered with crusts. The patient notices that in case of a direct contact with detergents her
skin manifestations intensify. Skin tests with several chemical substances manufactured at the shop
are positive. A histological examination of an area of the affected skin revealed acanthosis and
hyperkeratosis, an oedema of the derma, a vascular plethora, as well as a perivascular lymphocytic
infiltration. Clinically, occupational eczema was diagnosed. Which of the immunological
mechanisms lies in the basis of this disease?
A. Anaphylactic reaction
B. Antibody-dependent cytotoxic reaction
C. Granulomatosis
D. Immunocomplex mechanism
E. * Cellular cytotoxicity
27. In a 10-year-old child, eating of strawberries was followed by appearance of some disseminated
monomorphous and severely itching urticaria (red blisters of the round and oval form), elevation of
body temperature up to 37.8(C and a gastrointestinal disturbance. A blood analysis revealed
eosinophilia. After taking of antihistamine agents the above manifestations were rapidly controlled.
Which of the immunological mechanisms lay in the basis of this disease?
A. Granulomatosis
B. Antibody-dependent cytotoxic reaction
C. Cellular cytotoxicity
D. Immunocomplex mechanism
E. * Anaphylactic reaction
28. An autopsy of a 23-year-old female, who died from puerperal sepsis, revealed an enlarged plethoric
spleen whose section gave an abundant scrape. Microscopically, hyperplasia and a plasmacytic
infiltration of both the red pulp and splenic follicles were found out; the red pulp was rich in
macrophages. Which of the immunopathological mechanisms most probably lay in the basis of the
changes in the spleen?
A. Hereditary insufficiency of the peripheral lymphoid tissue
B. Immediate hypersensitivity reaction
C. Delayed hypersensitivity reaction
D. Autoimmunization
E. * Antigenic stimulation of the organism
29. In a child, 48 hours after a tuberculin (Mantoux) test, a papule up to 10 cm in diameter formed at the
place of an injection of tuberculin. What mechanism of hypersensitivity lay in the basis of the above
changes?
A. Anaphylaxis
B. Antibody-dependent cytotoxicity
C. Immunocomplex cytotoxicity
D. Granulomatosis
E. * Cellular cytotoxicity
30. A 23-year-old male patient developed the urinary syndrome (haematuria, proteinuria, leukocyturia)
after having angina. A puncture biopsy of the kidneys revealed a picture of intracapillary proliferative
glomerulonephritis, while on electron microscopy some large subepithelial deposits were found out.
What was the pathogenesis of this disease?
A. Anaphylactic reaction
B. Antibody-dependent cytotoxic reaction
C. Cell-dependent cytolysis
D. Granulomatosis

E. * Immunocomplex mechanism
31. Nine days after receiving large doses of the antitetanic serum a female patient developed urticaria
with an elevated body temperature, skin itch, an oedema of mucosae. Indicate the immunological
mechanism of the disease.
A. Reaginic anaphylactic reaction
B. Pathogenic effect of sensitized lymphocytes
C. Granulomatosis
D. Cytotoxic effect of antibodies
E. * Immunocomplex mechanism
32. This disease develops in newborns and manifests itself in the form of persistent diarrhoeae, general
cachexia and a clinical picture of sepsis. The children die at the age of 2-3 years. On postmortem
examination, the thymus is represented with a reticular stroma, no epithelial reticulum, thymus
corpuscles and lymphocytes are present. The lymphoid organs are characterized by a severe stage of
hypoplasia. What type of immunopathological processes does the above disease belong to?
A. Autoimmune disease
B. Syndrome of insufficiency of cellular immunity
C. Antibody deficiency syndrome
D. AIDS-related syndrome
E. * Combined immunodeficiency syndrome
33. A histological examination of an increased cervical lymph node in a male patient, who worked at an
enterprise with occupational hazards, revealed a plethora and swelling of the cortical layer, the light
centre of follicles and the medullary substance contained a large amount of plasma cells, the number
of lymphocytes was reduced, there was an active proliferation of sinus cells and a significant
macrophage response. Name the character of changes in the lymph node.
A. Lymphoma
B. Acute lymphadenitis
C. Lymphogranulomatosis
D. Insufficiency of peripheral lymphoid tissue
E. * Antigenic stimulation of lymphoid tissue
34. A histological examination of a skin graft in a male patient, who underwent dermatoplasty revealed a
diffuse lymphohistiocytic infiltration with an admixture of macrophages and neutrophils, an oedema
and haemorrhages. Which of the diagnoses listed below was the most probable?
A. Delayed hypersensitivity reaction
B. Immediate hypersensitivity reaction
C. Interstitial inflammation
D. Arthus phenomenon
E. * Graft rejection reaction
35. A 45-year-old woman has had a chronic, non-productive cough for 3 months, along with intermittent
fever. She has a chest radiograph that reveals multiple small parenchymal nodules along with hilar
and cervical lymphadenopathy. A cervical lymph node biopsy is performed. Microscopic
examination of the biopsy shows noncaseating granulomatous inflammation. Cultures for bacterial,
fungal, and mycobacterial organisms are negative. Which of the following chemical mediators is
most important in the development of this inflammation?
A. Bradykinin
B. Complement C5a
C. Histamine
D. Prostaglandin E2
E. * Interferon gamma

36. At histological research of a biopsy specimen from an auricle of a heart of a patient with rheumatic
disease the foci of a mucoid swelling, fibrinoid necrosis of a connective tissue has been found out.
What immune response has developed in tissues of the auricle of the heart?
A. Hypersensitivity of a delayed type
B. Reaction of the transplantative immunity
C. Normergic reaction
D. Exudative reaction
E. * Hypersensitivity of an immediate type
37. The 30-year-old man has had for two months lacrination, pruritic palpebras, and rhinitis with mucus.
All symptoms disappeared after treatment by desensebilizators. What type of hypersensitivity
occurred in patient?
A. The type II
B. The type III
C. The type IV
D. The type V
E. * The type I
38. Histologic investigation of thyroid gland has showed destruction and atrophy of follicles, diffuse
lymphoid infiltration with formation of lymphoid follicles in the stroma. call the group of diseases
with respect to this thyroiditis.
A. autoimmune nonspecific
B. bacterial
C. infectious-allergic
D. viruses infectious
E. * autoimmune specific
39. In biopsy of stomach in patient with autoimmune gastritis it was found out: infiltration by
lymphocytes and macrophages in mucous layer. Which type of hypersensitivity is connected with
these morphologic changes?
A. The type II
B. The type V
C. The type I
D. The type III
E. * The type IV
40. In biopsy of lymph node it was found out a lot of lymphoid follicles with large centers of duplication,
increasing of mitoses. Which process is characterized by these morphologic changes?
A. Atrophy of lymphoid tissue
B. Lymphosarcoma
C. Hodgkin’s disease
D. Metastases of cancer
E. * Antigenic stimulation with follicular hyperplasia
41. In biopsy of transplantanted kidney it was found out: diffuse infiltration of stroma by lymphocytes,
plasma cells, lymphoblasts, plasmablasts and necrotic arteriitis. Which pathologic process was
appeared in organ?
A. Acute glomerulonephritis
B. Ischemic infarction
C. Tuberculosis
D. Acute pyelonephritis
E. * Immune mutilation

42. In the 30-year-old woman it was found: cough, sputum with blood, fever, increased blood pressure,
decreased urine output, edema of low extremities. All symptoms have developed for 6 weeks.
Diagnostic renal biopsy showed Goodpasture’s syndrome. Which pathologic process is characteristic
for this syndrome?
A. Autoantibodies to mitochondrias of the lungs and kidneys
B. Autoantibodies to DNA
C. Cytotoxic reaction against epithelium of renal tubules
D. Appearance of immune complexes in glomeruli of kidneys and lungs
E. * Autoantibodies to basement membrane of the lungs and kidneys
43. The 30-year-old patient with transplanted kidney has received prolonged immunosupressive therapy
and he has died because of intoxication. Microscopic examination showed giant cells with large
nuclei encircled by rings-like brightening, which looked as “owl-eye”, located in the kidneys, liver,
pancreas, lungs. Call this disease.
A. Tuberculosis
B. Syphilis
C. Leprosy
D. Bubonic plaque
E. * Cytomegalovirus infection
44. Local lymph nodules enlarged near the infected wound. Increased amount of macrophages,
lymphocytes, lymphatic follicles in the cortical layer and large amount of plasma cells were revealed
on histological examination. What process in the lymphatic nodules represents these histological
changes?
A. Tumour transformation
B. Innate insufficiency of the lymphoid tissue
C. Hypersensibility reaction
D. Acquired insufficiency of the lymphoid tissue
E. * Antigen stimulation
45. A 12-year-old boy often suffers from virus and bacterial infections and eczematous skin lesions.
Enlargement of T-lymphocytes and IgM with normal IgA and IgG was revealed on examination.
What type of immune system pathology is presented in the patient?
A. Hereditary immundeficiency of the complement system
B. Composite immunedefficiency
C. Hypoplasia of thymus
D. Turner's syndrome
E. * Bruton's hypogammaglobulinemia
46. The specimens present sections of haemopoetic and immunogenetic organs. Organ has lymph tissue
forming different structures (lymph nodes, lobules, bars). In what organ does antigen-independent
proliferation and differentiation take place?
A. Lymphatic nodes
B. Hemolymph nodes
C. Tonsil
D. Spleen
E. * Thymus
47. In the patient with drug abuse (narcomania) it was diagnosed AIDS. What type of hypersensitivity
occurred in-patient?
A. The type II
B. The type V
C. The type I
D. The type III

E. * The type IV
48. Ten years ago a male patient’s right lung was removed because of a tumour, since then the capacity
of his left lung has increased by 50 %. What process has developed in the left lung?
A. Neurohumoral hypertrophy
B. Vicarious hypertrophy
C. Work hypertrophy
D. Hypertrophic vegetations
E. * Vicarious hypertrophy
49. An autopsy of a male patient, who died from hypertensive disease, revealed an enlarged heart
weighing 600 g, with a thickened left ventricular wall up to 2 cm and a dilated cavity of the left
ventricle. Name the kind of an adaptive reconstruction in the heart.
A. Concentric hypertrophy
B. Vicarious hypertrophy
C. Eccentric atrophy
D. Vicarious hypertrophy
E. * Eccentric hypertrophy
50. As a result of falling down, a small abrasion formed of the knee of a child and some time later it
epithelialized completely without formation of any scar. What form of regeneration took place in this
case?
A. Substitution
B. Physiological
C. Pathological
D. Intracellular
E. * Restitution
51. A 74-year-old male died from chronic heart failure. On autopsy, an old postinfarction scar was found
in the heart. A histological examination revealed a focus of fibrosis and hypertrophy of
cardiomyocytes. What regeneration do the described changes manifest?
A. Physiological
B. Pathological
C. Restitution
D. Intracellular
E. * Substitution
52. Following a traumatic injury of the liver, there was a complete restoration of its structure and
functions with time. What is the name for such a kind of regeneration?
A. Substitution
B. Pathological
C. Physiological
D. Intracellular
E. * Restitution
53. A 20-year-old male patient with a posttraumatic variceal dilation and thrombosis of the subcutaneous
vein in the middle third part of the shin underwent its surgical removal. Histologically, an obstructive
thrombus was found in the lumen of the vein with growing of a connective tissue into the thrombus
from the side of the vascular wall. What process did the changes in the thrombus result from?
A. Reconstruction
B. Canalization
C. Revascularization
D. Repair
E. * Organization

54. A male patient with chronic cystitis revealed, along with the transitional epithelium, foci of the
stratified squamous one without keratinization in a biopsy of the mucosa of his bladder. What
pathological process does it indicate?
A. Leukoplakia
B. Hyperplasia
C. Dysplasia
D. Hyperkeratosis
E. * Metaplasia
55. A 30-year-old male patient, who 10 years before had undergone removal of his traumatized left
kidney, against a background of good health revealed a twice-fold enlargement of the right kidney in
comparison with the norm on prophylactic medical examination. Which of the listed processes was
the most probable one in the kidney?
A. Work hypertrophy
B. Hyperplasia
C. Neurohumoral hypertrophy
D. Pseudohypertrophy
E. * Vicarious hypertrophy
56. A microscopic examination of a myocardium revealed postinfarction transmural cardiosclerosis
surrounded by enlarged cardiomyocytes with large hyperchromatic nuclei rich in DNA. Which of the
listed morphological processes in the cardiomyocytes was the most probable?
A. Complete reparative regeneration
B. Physiological regeneration
C. Pathological regeneration
D. Work hypertrophy
E. * Regenerative hypertrophy
57. An autopsy of a 75-year-old male patient, who suffered from a prostatic adenoma and died of renal
insufficiency, revealed enlarged kidneys; on section, there was an atrophy of the parenchyma and a
lot of thin-walled cavities filled with urine. Which of the listed kinds of an atrophy in the kidney was
the most probable?
A. Dysfunctional
B. Caused by physical factors
C. Neurotic
D. Caused by insufficiency of blood circulation
E. * Caused by pressure
58. In a postoperative wound of a 10-year-old child, who was operated for acute appendicitis; 2-3 days
later appeared some granulation tissue and on the 10th day a small thin scar was formed. Name the
kind of healing.
A. Under a crust
B. Simple
C. By second intention
D. –
E. * By first intention
59. A histological examination of a scrape from the mucous membrane of the uterus was made in a 50-
year-old female patient who complained of a disorder in the ovariomenstrual cycle manifested by
irregular significant haemorrhages. A cystoglandular hyperplasia of the endometrium was diagnosed.
Name the kind of the pathological process in the endometrium.
A. Hypertrophic vegetation
B. Vicarious hypertrophy
C. Regenerative hypertrophy

D. Vicarious hypertrophy
E. * Neurohumoral hyperplasia
60. A biopsy of a bronchus of a 50-year-old male patient, who suffered from chronic bronchitis for 20
years, revealed foci of substitution of the stratified squamous epithelium for the columnar one. Which
of the pathological processes listed below took place?
A. Hyperplasia
B. Heterotopia
C. Heteroplasia
D. Dysplasia
E. * Metaplasia
61. For examination, a scrape from the mucous coat of the uterus of a 45-year-old woman suffering from
dysfunctional uterine bleedings was sent. Histologically, an increased number of endometrial glands
and their intensified branching with a cyst-like dilation of the lumens were found. Which of the
pathological processes listed below was the most probable?
A. Hypertrophy
B. Metaplasia
C. Dysplasia
D. Aplasia
E. * Hyperplasia
62. An autopsy of a 57-year-old male patient, who suffered from hypertensive disease and died of
cardiac decompensation, revealed an enlarged heart with dilated cavities. Microscopically, the
cardiomyocytes were significantly enlarged and had fatty degeneration with hyperchromatic barrel-
like nuclei. Which of the listed morphological processes in the heart was the most probable?
A. Hypertrophic vegetations
B. Brown atrophy
C. Concentric hypertrophy
D. Vicarious hypertrophy
E. * Eccentric hypertrophy
63. A 4-years-old girl was operated because of an acute appendicitis. During the operation in the
retroperitoneal space it was found that the right kidney is less by 1/3 in comparison with the left one.
The diameter of the right renal artery was 0,3 cm, the left one was 0,4 cm. What pathologic process
did occur in that case?
A. Pathologic atrophy
B. Physiologic atrophy
C. Aplasia
D. Agenesia
E. * Hypoplasia
64. A 23-years-old patient got a lesion of the liver because of trauma. In time a structure and functions of
that organ was restored completely. What kind of regeneration did occur in that case?
A. Pathologic regeneration
B. Physiologic regeneration
C. Metaplasia
D. Substitution
E. * Restitution
65. A child fell down the tree and got a simple fracture of the one of hand’s bones. In a time healing
occurred. Call the kind of regeneration and cells, which taking part in restoration.
A. Physiological regeneration, octeoclasts
B. Pathological regeneration, osteoclasts

C. Intracellular regeneration, osteocytes


D. Cellular regeneration, osteoclasts and osteoclasts
E. * Reparative regeneration, osteoclasts and osteoblasts
66. A patient has died as a result of cardiac insufficiency. In anamnesis he had got a pulmonectomy in
account of cyst of the right lung. During the autopsy enlarged left one was found out. Call the
pathologic process in the left lung.
A. Neurohumoral hypertrophy
B. Dyscirculatory atrophy
C. Dysfunctional atrophy
D. Physiologic hypertrophy
E. * Replacement hypertrophy
67. A patient has undergone to amputation of lower extremity. In a time a painful nodules appeared in a
stump. Amputatious neuromas were found out during the microscopical examination. What kind of
pathological processes do those formation relate to?
A. Dystrophy
B. Inflammation
C. Hyperemia
D. Metaplasia
E. * Regeneration
68. After deep burns of the skin a patient has got a keloid scarring. What kind of pathologic processes do
those formations relate to?
A. Complete regeneration (restitution)
B. Atrophy
C. Hypertrophy
D. Metaplasia
E. * Incomplete regeneration (substitution)
69. Under microscopic investigation the postinfarction cardiosclerosis has been found out. Around
cardiosclerotic area myocardiocytes were enlarged in size and had large hyperchromic nuclei riched
in DNA. What process taking place in myocardiocytes is more probable?
A. Physiologic regeneration
B. Complete reparative regeneration
C. Pathologic regeneration
D. Hypertrophy because of increased workload
E. * Regenerative hypertrophy
70. Dystrophic changes of the heart muscle are accompanied with cardiac cavity enlargement, decrease
of the strength of heart contraction, increased amount of blood, which remains in the heart during
systolic phase, overfilled veins. For what state of heart is it characteristic?
A. Tamponage of the heart
B. Tonogenic dilatation
C. Cardiosclerosis
D. Emergency stage of hyperfunction and hypertrophy
E. * Myogenic dilatation
71. Decreased blood supply to the organs causes hypoxia that activates fibroblasts function. Volume of
what elements is increased in this case?
A. Lymphatic vessels
B. Parenchymatous elements of the organ
C. Vessels of microcircular stream
D. Nerve elements

E. * Intercellular substance
72. At autopsy of a patient died because of a cerebral hemorrhage, strongly enlarged dense and anemic
kidneys (size: 6х3х2см, weight 60.0 g) with a uniformly small-granulated surface and with uniformly
thinned cortex on a cut-surface have been found out. Call adaptive-compensative process in this
case?
A. Hyperplasia
B. Complete regeneration (restitution)
C. Hypertrophy
D. Metaplasia
E. * Atrophy
73. During surgery in a 17-year-old patient it was revealed the tumour of 4,5х5,0х3,5 sm in size on the
lower surface of the liver with subserose localization, of dark-red color. On the section tumour has
cavities with marked amount of blood. What is preliminary diagnosis?
A. Capillar hemangioma
B. Hemangiopericytoma
C. Hemangioendothelioma
D. Lymphangioma
E. * Cavernous hemangioma
74. Examination of the anterior abdominal wall of a pregnant woman revealed a tumour-like formation
that arose on the spot of a tumour that was removed two years ago. The neoplasm was well-defined,
dense, 2х1 cm large. Histological examination revealed that the tumour was composed of
differentiated connective tissue with prevailing collagen fibres. What tumour might be suspected?
A. Lipoma
B. Fibrosarcoma
C. Hibernoma
D. Leiomyoma
E. * Desmoid
75. A 50-year-old man has felt vague abdominal discomfort within past 4 months. Physical examination
revealed no lymphadenopathy, and no abdominal masses or organomegaly at palpation. Bowel
sounds are heard. An abdominal CT scan shows a 20 cm retroperitoneal soft tissue mass obscuring
the left psoas muscle. A stool specimen tested for occult blood is negative. Which of the following
neoplasms is this man most likely to have?
A. Melanoma
B. Hamartoma
C. Adenocarcinoma
D. Lymphoma
E. * Lipoma
76. A woman suffering from dysfunctional metrorrhagia was made a diagnostic abortion. Histologically
in the scrape there were a lot of small stamped glandulars covered with multirowed epithelium. The
lumens of some glandulars were cystically extended. Choose the variant of general pathologic
process in the endometrium.
A. Atrophy of endometrium
B. Metaplasia of endometrium
C. Neoplasm of endometrium
D. Hypertrophic growth
E. * Glandular-cystic hyperplasia of endometrium

77. A 75 year old male patient consulted a surgeon about a brown nonhealing ulcer of shin. Examination
of biopsy material revealed diffuse growth of polymorphic atypic cells with brown pigment in their
cytoplasm. Pearls reaction was negative. There were also a lot of pathological mitoses and foci of
tissue necrosis. What is the most probable diagnosis?
A. Local hemosiderosis
B. Intradermal nevus
C. Trophic ulcer
D. Skin cancer
E. * Melanoma
78. A 69-year-old patient got a small plaque with subsequent ulceration on the skin of the lower eyelid.
The formation was removed. Microscopic examination of dermis revealed complexes of atypical
epitelial cells arranged perpendicularly to the basal membrane on the periphery. The cells were dark,
of polygonal prismatic shape with hyperchromic nuclei with frequent mitoses. What is the
histological form of carcinoma in this patient?
A. Keratinizing squamous cell carcinoma
B. Nonkeratinizing squamous cell carcinoma
C. Adenocarcinoma
D. Undifferentiated
E. * Basal cell carcinoma
79. Histological study of the bronchial wall and adjacent lung segments revealed sheets and strands of
squamous epithelium. The cells have moderately expressed symptoms of atypia: polymorphism,
nuclear hyperchromatism, mitoses. In the center of the complex there are concentric pink formations.
What is the most likely diagnosis?
A. Non-keratinizing squamous cell carcinoma
B. Adenocarcinoma
C. Scirrhus
D. Undifferentiated carcinoma
E. * Keratinizing squamous cell carcinoma
80. Histologically, the internal wall of a cyst localized on the upper jaw is lined with stratified squamous
epithelium with underlying granulation tissue infiltrated by lymphocytes. The external layer is
represented by loose fibrous connective tissue surrounded by cicatrical fibrous tissue. What diagnosis
can be made?
A. Simple granuloma
B. Epithelial granuloma
C. Keratocyst
D. Ameloblastoma
E. * Cystic granuloma
81. Microscopic examination of a skin tumor revealed that it invaded the underlying tissue, destroyed it
and formed nests and cords of atypical epithelium which included some pearl-like formations.
Specify the tumor:
A. Squamous cell non-keratinizing carcinoma
B. Solid carcinoma
C. Adenocarcinoma
D. Medullary carcinoma
E. * Keratinizing squamous cell carcinoma
82. Histologic analysis of uterus mucous membrane revealed twisting glands, serrated and spinned, they
were extended by stroma growth with proliferation of its cells. Formulate a diagnosis:
A. Acute endometritis
B. Leiomyoma

C. Cystic mole
D. Placental polyp
E. * Glandular hyperplasia of endometrium
83. Microscopy of colonic biopsy material revealed a tumour made up of prismatic epithelium and
forming atypical glandular structures of various shapes and sizes. The basal membrane of glands was
destroyed. Tumour cells were polymorphic, with hyperchromatic nuclei and a large number of
pathological mitoses. What is the most likely diagnosis?
A. Basal cell carcinoma
B. Solid carcinoma
C. Mucosal carcinoma
D. Undifferentiated carcinoma
E. * Adenocarcinoma
84. The man of 46 years old has a dark patch on a skin which towered and did not disturb. In course of
time a spot began to increase, pain appeared, a color became blackly-brown and a it was easy to feel
the knot. Fusiform and polymorphic cells the cytoplasm of which contained the pigment of brown
color concerned on histological research of remote fabric. What tumour is the question about?
A. Basalioma
B. Gemangioma
C. Haematoma
D. Karsinoid
E. * Melanoma
85. At the patient of 21 year the tumour of frontal part of right hemisphere is remote head brain by a
diameter 5 см, which was unclear delimited from near-by fabric. On a cut - homogeneous kind,
histological — consists of cells the numerous sprouts of which form thick interlacements. What
tumour took place in a cerebrum?
A. Oligodendroglioma
B. Ganglioneuroma
C. Epsidimoma
D. Chorionpapiloma
E. * Astrocytoma
86. At a patient on the skin of person gradual a name-plate developed with necrosis and ulcer in a center.
At pathohistologic research of bioptate excrescence of atypical ephithelial cells is exposed with
plenty of pathological mitosises. What is the most reliable diagnosis?
A. Sarcoma
B. Villoma
C. Trophic ulcer
D. Fibroma
E. * Shrines of skin
87. A dense tumour mobile is macroscopically found in a skin. At a microscopy she is presented by the
chaotically located bunches of collogens fibres with the two-bit of fusiform cells. What tumour is
remote?
A. Leiomyoma
B. Melanoma
C. Lipoma
D. angioneoplasm
E. * Hard fibroma

88. At a patient on small curvature in pyloric department found out formation of crateriform form. From
the regional area of education a biopsy is taken.Information of histological research: tumour with the
glandlike structures of different form and size, ingrowing in surrounding fabric, by expressed
атипизмом of cells. Name the histological variant of this tumour.
A. Flatcell crawfish
B. Scirrhus of stomach
C. Mucous crawfish of stomach
D. Solid crawfish of stomach
E. * Adenocarcinoma
89. Tumour in a capsule by a diameter 2 see operatively remote from amputation cults of lower
extremity, microscopically consists of fusiform cells of monomorphic kind with sticklike kernels
which form "palisades" structures together with fibres. What from the transferred types of tumours
наиболее reliable?
A. Neurofibroma
B. Malignant neurolimoma
C. Soft fibroma
D. Fibrosarcoma
E. * Of high quality neurolimoma
90. At a patient 55 years the relapsic uterine bleeding appeared. The diagnostic endometrectomy is
executed. In соскобе of endometrium among the elements of blood evidently the ferrous elements of
different size and form, formed by atypical cells with hyperchromic kernels, with numerous mitosises
(including pathological). About what process is it possible to think?
A. Ferrous hyperplasia of endometrium
B. Chorionepithelioma
C. Adenomatous polypus
D. Signs of the interrupted pregnancy
E. * Adenocarcinoma
91. At the autopsy of a deceased man who died in the result of intoxication there were found following
signs: cachexia, muscle atrophy, wrinkled skin, reduced weight of internal organs, stenotic tumor of
antral part of stomach with metastases in the liver and regional lymph nodes. What type of cachexia
is the most likely?
A. Alimentary
B. Pituitary
C. Cerebral
D. One that accompanies chronic infectious disease
E. * Cancerous
92. At microscopy study of skin neoplasm of brown colour there was found that the tumor consists of
nevus cells that are located in the dermis as conglomerations and chords. Cytoplasm of cells contains
brown pigment, which gives a negative Perl’s reaction. Which pigment is the most probably found in
the cytoplasm of cells?
A. Hematoidin
B. Hemosiderin
C. Bilirubin
D. Hemomelanin
E. * Melanin
93. On bronchoscopy, an exophytic tumour was found; it was localized in the bronchus and significantly
narrowed its lumen. Histologically, the tumour consisted of complexes of polymorphous epithelial
cells with hyperchromatic nuclei and pathological mitoses. Among the tumour cells there were
eosinophilic concentric structures. Make a diagnosis of the tumour.

A. Nonkeratinizing squamous cell carcinoma


B. Large-cell carcinoma
C. Small-cell carcinoma
D. Adenoacanthoma
E. * Keratinizing squamous cell carcinoma
94. For a histological examination, an eyeball was sent; some black tumour, 1 x 0.4 cm in size, was
revealed in its vascular membrane. Microscopically, the tumour consisted of large polymorphous
cells grouped in alveolar structures. The cytoplasm of the cells contained some brown pigment. What
is your diagnosis?
A. Neurilemmoma
B. Angiosarcoma
C. Neuroblastoma
D. Ganglioneuroblastoma
E. * Melanoma
95. A 65-year-old woman underwent removal of some tumour, 1.0 x 1.0 x 0.8 cm in size, localized under
the skin of her thigh. Macroscopically, the tumour had a connective-tissue capsule and was
represented on section with a yellowish lobate tissue. Microscopically, there were large cells, which
had the sudanophilic cytoplasm and formed lobules separated with connective-tissue layers. Name
this tumour.
A. Hibernoma
B. Liposarcoma
C. Fibroma
D. Desmoid
E. * Lipoma
96. A thick encapsulated node, 2.0 cm in diameter, was surgically removed from the mammary gland of
a female patient. On section, the tissue of the node was white-pink and fibrous. Microscopically, the
tumour consisted of glandular structures, which had no signs of cellular atypism and were
compressed with a connective tissue vegetating around. In the tumour, the stroma prevailed over the
glandular parenchyma. What is your diagnosis?
A. Adenoma
B. Nonproliferative mastopathy
C. Proliferative mastopathy
D. Adenocarcinoma
E. * Fibroadenoma
97. A male patient, who suffered from chronic bronchitis for a long period of time, revealed a pulmonary
tumour, which was closely connected with the bronchial wall and grew in the form of a polyp.
Microscopically, the tumour consisted of complexes of polymorphous epithelial cells with a large
number of mitoses. Among the tumour cells there were stratified concentric oxyphilic structures.
Name the histological type of the tumour.
A. Mucinous carcinoma
B. Solid carcinoma
C. Nonkeratinizing squamous cell carcinoma
D. Adenocarcinoma
E. * Keratinizing squamous cell carcinoma
98. A newborn baby has some red-blue flattened tumor, 5 x 4 x 0.3 cm in size, in a capsule on the skin of
its face. Microscopically, the tumour consists of large thin-walled vascular cavities which have an
endothelial lining and are filled with blood. Name the tumour.
A. Venous haemangioma
B. Capillary haemangioma

C. Hemangiopericytoma
D. Lymphangioma
E. * Cavernous haemangioma
99. On bronchoscopy in the initial part of the upper lobe bronchus of the right lung some polyp-like
formation, 1.0 cm in diameter, with a superficial ulcer was found. A histological examination
revealed a tumour consisting of lymphocyte-like cells with hyperchromatic nuclei; the cells grew in
layers and bands. Indicate the most probable tumour.
A. Undifferentiated large-cell carcinoma
B. Squamous cell carcinoma
C. Adenocarcinoma
D. Glandular squamous cell carcinoma
E. * Undifferentiated small-cell carcinoma
100. An autopsy of a female who died from cachexia, revealed some massive exophytic carcinoma on the
lesser curvature of the stomach with metastases to the ovaries. What kind of metastatic spreading
took place?
A. Haematogenous
B. Lymphogenous orthograde
C. Implantation
D. Perineural
E. * Lymphogenous retrograde
101. A histological examination of a thyroid gland revealed small cysts, which were lined with atypical
epithelium and filled with papillae, the latter originating from the walls of the cysts and growing into
their capsules. Name the tumour.
A. Papillary adenoma
B. Follicular carcinoma
C. Solid carcinoma
D. Carcinoma simplex
E. * Papillary carcinoma
102. A 47-year-old woman underwent radical mastectomy for a neoplasm. A histological examination of
the mammary gland revealed an eczematous lesion of the nipple and areola, a cancerous lesion of the
ducts of the gland and presence of large light cells in the epidermis of the nipple and areola. Make a
diagnosis.
A. Intralobular carcinoma in situ
B. Acneiform carcinoma
C. Papillary carcinoma
D. Fibrous carcinoma
E. * Paget's disease
103. A histological express examination of a tumour node of a mammary gland revealed some
encapsulated formation with proliferation of alveoli and intralobular ducts; the interstitial connective
tissue grew either around or inside the ducts. Which of the tumours took place?
A. Foliaceous tumour
B. Noninfiltrating intralobular carcinoma
C. Infiltrating intralobular carcinoma
D. Paget's disease
E. * Fibroadenoma
104. During an operation on a woman, her cyst-like changed ovary was removed; it was a thin-walled
cavity filled with some yellowish transparent fluid and having a smooth inner surface. Histologically,
the cavity wall was lined with the cubical epithelium. Name the kind of the tumour.
A. Mucinous cystadenoma

B. Serous cystadenocarcinoma
C. Pseudomucinous cystocarcinoma
D. Granulosa cell tumour
E. * Serous cystadenoma
105. A histological examination of a biopsy from a uterine cervix revealed that its tissue was covered with
a wide layer of the stratified squamous epithelium having foci of proliferation of atypical cells with
pathological mitoses, but the basal membrane of the epithelium was not affected. What is your
diagnosis?
A. Nonkeratinizing squamous cell carcinoma
B. Keratinizing squamous cell carcinoma
C. Leukoplakia
D. Epithelial dysplasia
E. * Carcinoma in situ
106. A histological examination of some spherical neoplasm located under the surface of the skin,
revealed papilliform vegetations of the epithelium with phenomena of acanthosis and
hyperkeratinization. The tumour stroma consisted of a large amount of the connective tissue and
vessels. What tumour took place?
A. Keratoacanthoma
B. Carcinoma in situ
C. Keratinizing squamous cell carcinoma
D. Nonkeratinizing squamous cell carcinoma
E. * Papilloma
107. A microscopic examination of a gastrobiopsy from a tumour of the pyloroduodenal portion of the
stomach revealed layers of atypical epithelial cells with a large number of mitoses; the tumour
architectonics is characterized by prevalence of the parenchyma over the stroma. Which of the
histological forms of carcinoma listed below was the most probable?
A. Adenocarcinoma
B. Solid carcinoma
C. Mucinous carcinoma
D. Small-cell carcinoma
E. * Medullary carcinoma
108. A 26-year-old male patient underwent surgical removal of a tumour, 4 x 5 cm in size, which was
surrounded by a capsule and located in the white matter of his brain. Microscopically, the tumour
consisted of the stellate and glia cells having various size and located among the glial fibres. Name
the tumour.
A. Oligodendroglioma
B. Astroblastoma
C. Glioblastoma
D. Ependymoma
E. * Astrocytoma
109. A 45-year-old male underwent surgical removal of a tumour, 4 x 3 cm in size, from the lateral
ventricle of his brain; the tumour surface had small papillae, and it was connected with a vascular
plexus. Microscopically, the tumour consisted of villus-like vegetations covered with epithelial cells
of the cubical and columnar shape and the monomorphous kind. Which of the tumours listed below
was the most probable?
A. Ependymoma
B. Ependymoblastoma
C. Choriocarcinoma
D. Glioblastoma

E. * Choriopapilloma
110. An encapsulated tumour, 2 cm in diameter, surgically removed from an amputation stump of a lower
extremity, microscopically consists of spindle cells of the monomorphous kind with rod-shaped
nuclei which form "fence-like" structures together with fibres. Which of the tumours listed below is
the most probable?
A. Neurofibroma
B. Malignant neurilemmoma
C. Soft fibroma
D. Fibrosarcoma
E. * Benign neurilemmoma
111. A microscopic examination of a biopsy from a deformed mucous membrane of a lobar bronchus of a
45-year-old male, who smoked for many years, revealed a carcinoma consisting of atypical epithelial
cells with hyperchromatic nuclei and numerous pathological mitoses. The growth of the tumour did
not spread to the basal membrane of the epithelium. Name the histological form of carcinoma.
A. Small-cell carcinoma
B. * Carcinoma in situ
C. Squamous cell carcinoma
D. Adenocarcinoma
E. Solid carcinoma
112. A bronchoscopy of the mucous membrane of the main bronchus revealed some tumour. A
microscopic examination of the tumour biopsy showed that it consisted of lymphocyte-like cells with
hyperchromatic nuclei growing in the form of layers or bands and involving the submucous layer.
The tumour had many pathological mitoses. Which of the histological forms of carcinoma listed
below was the most probable?
A. Squamous cell carcinoma
B. Adenocarcinoma
C. Adenoacanthoma
D. Scirrhous carcinoma
E. * Small-cell carcinoma
113. On supersonic examination of a 48-year-old male patient, a hepatic neoplasm was diagnosed and a
puncture biopsy was made. Microscopically, the tumour consisted of atypical hepatocytes which
formed trabeculae, acini or tubules. The tumour stroma was poor and had thin-walled blood vessels.
Which of the kinds of tumours listed below was the most probable?
A. Hepatocellular adenoma
B. Metastasis of adenocarcinoma
C. Cholangiocellular carcinoma
D. Solid carcinoma
E. * Hepatocellular carcinoma
114. A patient has hoarseness of voice. Duri-ng laryngoscopy a gray-white larynx tumor with papillary
surface has been detected.Microscopic investigation has shown the following: growth of connective
tissue covered with multilayer, strongly kerati-nized pavement epithelium, no cellular atypia. What is
the most likely diagnosis?
A. Fibroma
B. Polyp
C. Angioma
D. Angiofibroma
E. * Papilloma

115. A tumour removed from the white matter of the right hemisphere of the brain is some soft "motley"
node, 4 cm in diameter, without any clear borders with the substance of the brain. Microscopically,
the tumour consists of polymorphous cells with numerous pathological mitoses, and it also reveals
foci of necrosis and haemorrhages which occurred at different time. Name the tumor.
A. Oligodendroglioma
B. Oligodendroglioblastoma
C. Astrocytoma
D. Astroblastoma
E. * Glioblastoma
116. Autopsy of a man who died of intoxication revealed cachexia, muscular atrophy, wrinkled skin,
decreased mass of the inner organs, stenosing tumor of the stomach with metastases to the liver and
regional nodes. Which type of cachexia is most probable?
A. Alimentary
B. Hypophyseal
C. Cerebral
D. In chronic infectious disease
E. * Cancerous
117. In a 37-year-old female patient, an enlarged dense mammary gland was revealed, the nipple with the
areola of the mammary gland were oedematous, the skin had an appearance of an "intradermal bleb".
On microscopic examination, the gland tissues revealed layers of tumour cells with polymorphous
nuclei and a large number of pathological mitoses. The tumour stroma was poorly expressed. Make
the diagnosis.
A. Adenofibroma
B. Scirrhous carcinoma
C. Paget's disease
D. Adenoma
E. * Medullary carcinoma
118. A patient with acute myeloblast leukemia has developed liver and spleen enlargement, anemia,
myeloblasts in peripheral blood. What principal morphological sign allows differing myeloblast
leukemia from chronic one?
A. * Blast cells in peripheral blood
B. Thrombocytopenia
C. Pancytopenia
D. Anemia
E. Leukemic collapse
119. In course of an operation on account of a granuloma in the area of the right upper incisor a patient
began to bleed. The hemorrhage was stopped just only 3 hours later. The patient’s anamnesis
contains information about chronic lymphatic leukemia. What is the most probable cause of
hemorrhage?
A. * Thrombocytopenia
B. Thrombocytopathia
C. Lymphocytosis
D. Leukopenia
E. Eosinophilia
120. Fale patient, who worked for a long period of time with benzene, develops progressing anaemia and
the haemorrhagic syndrome. A biopsy of his breastbone reveals prevalence of a fatty tissue, and there
are some small islets of haemopoiesis with solitary cells of myelopoiesis. What is your diagnosis?
A. Chronic myeloleukaemia
B. Pernicious anaemia

C. Haemolytic anaemia
D. Hypoplastic anaemia
E. * Aplastic anaemia
121. The patient who long worked with benzene, progressing anemia and hemorrhagic syndrome. In the
biopsy of sternum predominant adipose tissue, revealed a few small foci with isolated blood cells
myelopoesis. Your diagnosis.
A. Chronic myeloid leukemia
B. Pernicious anemia
C. Hemolytic anemia
D. Iron deficiency anemia
E. * Aplastic anemia
122. An autopsy of a 76-year-old male, who smoked for a long period of time, lived sedentary life and had
redundant weight, revealed in the intima of the aorta some grey-yellow spots and stripes, fibrous
plaques, calcified areas with haemorrhages and calcinosis. What disease do these changes indicate?
A. Nonspecific aortoarteritis
B. Hypertensive disease
C. Systemic lupus erythematosus
D. Visceral syphilis
E. * Atherosclerosis
123. An autopsy of a 27-year-old male, who died suddenly, revealed in the intima of the abdominal aorta
some yellow foci in the form of spots and stripes, which did not rise above the surface of the intima
but after staining with sudan III became orange. What stage in the morphogenesis of atherosclerosis
was revealed?
A. Atherocalcinosis
B. Liposclerosis
C. Atheroma
D. Prelipid
E. * Lipoidosis
124. A microscopic examination of the wall of an aorta revealed a focal infiltration of the intima by lipids
and proteins. The lipids impregnated the intima and accumulated in the muscle cells and
macrophages. Determine the stage of atherosclerosis.
A. Prelipid
B. Liposclerosis
C. Atheromatosis
D. Atherocalcinosis
E. * Lipoidosis
125. A microscopic examination of the wall of an aorta revealed a focal infiltration of the intima by lipids
and proteins whith enlargement of the connective tissue around. Determine the stage of
atherosclerosis.
A. Prelipid
B. Lipoidosis
C. Atheromatosis
D. Atherocalcinosis
E. * Liposclerosis
126. Аt autopsy of a 38-year-old male, who died in a car accident, revealed in his aorta some yellow-grey
spots and stripes which did not rise above the surface of the intima. Microscopically, the intima had
an abundant deposition of proteins, plasma, fibrin, GAG, cholesterol, low-density lipoproteins; the
endothelium had foci of affection. Name the stage of morphogenesis of atherosclerosis.
A. Prelipid

B. Lipoidosis
C. Atheromatosis
D. Atherocalcinosis
E. * Liposclerosis
127. An autopsy of a 70-year-old male, who died of cardiovascular insufficiency mill during his life-time
suffered from angina pectoris, hypercholesterolaemia and obesity, revealed a chronic venous plethora
of the organs, hypertrophy of the left ventricle of the heart with microfocal cardiosclerosis,
voluminous yellow plaques in the intima of the aorta with fine-grained masses in their centre and
these masses went deep into the thickness of the wall. Which of the stages of atherosclerosis listed
below was the most probable?
A. Prelipid
B. Lipoidosis
C. Liposclerosis
D. Atherocalcinosis
E. * Atheromatosis
128. A 75-year-old male was hospitalized complaining of a sharp pain in the abdominal cavity, weakness,
filiform pulse. During an operation it was found that the paraaortic fat was imbibed with blood. The
abdominal aorta had a sac-like protrusion, its wall was thinned and had an area of rupture. What
disease caused the complication?
A. Coronary disease
B. Hypertensive disease;
C. Cardiomyopathy
D. Systemic vasculitis
E. * Atherosclerosis
129. An autopsy of a 70-year-old male patient, who died from cardiac failure, revealed deformed and
narrowed coronary arteries. On section, the inner surface of the arteries was tuberous, the wall was
whitish, fragile and stony in consistency. Which of the diagnoses listed below was the most
probable?
A. Ulceration
B. Liposclerosis;
C. Atheromatosis
D. Lipoidosis
E. * Atherocalcinosis
130. An autopsy of a 48-year-old male, who died in a car accident, revealed in his aorta some yellow-grey
spots and stripes which did not rise above the surface of the intima. Microscopically, the intima had
an abundant deposition of proteins, plasma, fibrin, GAG, cholesterol, low-density lipoproteins; the
endothelium had foci of affection. Name the stage of morphogenesis of atherosclerosis.
A. Prelipid
B. Liposclerosis
C. Atheromatosis
D. Atherocalcinosis
E. * Lipoidosis
131. A microscopic examination of the wall of an aorta revealed a focal infiltration of the intima by lipids
and proteins. The lipids impregnated the intima and accumulated in the muscle cells and
macrophages. Determine the stage of atherosclerosis.
A. Prelipid;
B. Liposclerosis;
C. Atheromatosis;
D. Atherocalcinosis;

E. * Lipoidosis
132. The patient, who has long suffered from intermittent claudication, gangrene developed dry foot. Add
a disease that led to such complications.
A. Nodular arteritis;
B. Arteriolosklerosis;
C. Arteriosclerosis
D. Coronary arteritis;
E. * Atherosclerosis
133. An autopsy of a male, who suffered from arterial hypertension during his life_time, revealed oedema
of the brain substance, arterial walls at the base were thickened, with white-yellowish plaques in the
intima, the left hemisphere had a focus, 5 x 4 x 3 cm in size, representing a cavity filled with liquid
blood and its clots. Define the pathological process in the brain.
A. Transitory ischaemia
B. Haemorrhagic infarct
C. Anaemic infarct
D. Mixed infarct
E. * Haematoma
134. Autopsy of a 75-year-old patient who had been suffering from disseminated atherosclerosis and died
under chronic cardiac failure revealed constriction and deformation of coronary arteries, tuberous
intima whose section appeared to be white and petrosal. Specify the stage of atherosclerosis
morphogenesis:
A. Atheromatosis
B. Lipoidosis
C. Bilipid
D. Liposclerosis
E. * Atherocalcinosis
135. Morphological examination of an amputated gangrenous extremity revealed that the lumen of
femoral artery was constricted due to stony, partly ulcerated plaques with obturating thrombi. What is
the most likely diagnosis?
A. Obliterating endartheriitis
B. Nodular periarthritis
C. Obliterating thromboangiitis
D. Non-specific aortoartheriitis
E. * Atherosclerosis
136. Obliterating atherosclerosis causes changes in the vessels of the lower extermities. A histological
specimen of such a vessel evidently presents both internal and external elastic membranes, middle
membrane contains a lot of myocytes. What vessel is affected in case of this disease?
A. Vein with strongly developed muscles
B. Artery of elastic type
C. Artery of mixed type
D. Lymph node
E. * Artery of muscular type
137. On autopsy of a 68-year-old male, who died from cardiac decompensation, the myocardium of the
anterior wall in left ventricle of his heart contained an irregular grey focus, 5 x 4 cm in size, with a
dense consistency, fibrous structure and clear borders. What pathological process in the myocardium
did the pathologist reveal?
A. Myocarditis
B. Microfocal cardiosclerosis
C. Infarction

D. Rheumatism
E. * Postinfarction cardiosclerosis
138. A male patient developed substernal pains at 8 a.m., and at 9 a.m. myocardial infarction was
diagnosed by ECG data at the admission department. Ten minutes later the patient died. What most
reliable sign of myocardial infarction will be found on histological examination?
A. Vacuole dystrophy of cardiomyocytes;
B. Fat infiltration of cardiomyocytes;
C. Relaxation of myofibrils in cardiomyocytes
D. Reduced activity of dehydrogenases in fibroblasts;
E. * Disappearance of glycogen in cardiomyocytes;
139. Three weeks following a myocardial infarction, a 54-year-old man presents with fever, productive
cough, and chest pain. The pain is worse with inspiration, better when he is sitting up, and not
relieved by nitroglycerin. Physical examination finds a friction rub along with increased jugular
venous pressure and pulsus paradoxus (excess blood pressure drop with inspiration). Which of the
following is the most likely explanation for these findings?
A. Caplan’s syndrome;
B. Dressler’s syndrome;
C. Ruptured papillary muscle;
D. Ventricular aneurysm.
E. * Ruptured ventricular wall;
140. On autopsy of a 66-year-old male, who died from acute cardiac failure, an acute venous plethora of
the internal organs was found. The cardiac cavities were dilated; a myocardial section revealed some
dim yellowish focus, 3.5 x 4 cm in size, in the anterior wall of the left ventricle. The coronary arteries
had stenosing atheromatous plaques. Which of the diagnoses listed below was the most probable?

A. Myocarditis
B. Microfocal cardiosclerosis
C. Macrofocal cardiosclerosis
D. Fatty degeneration of myocardium
E. * Myocardial infarction
141. A 3-month-old girl is being evaluated for feeding difficulty and failure to thrive. Physical
examination finds pallor, peripheral cyanosis, tachypnea, and fine expiratory wheezing. Chest x-ray
shows cardiac enlargement. She is admitted to the hospital, quickly develops severe cardiac failure,
and dies 3 days after admission. At the time of autopsy the endocardium is found to have a “cream
cheese” gross appearance. Histologic sections from this area reveal thickening of the endocardium
due to a proliferation of fibrous and elastic tissue. Which of the following is the most likely
diagnosis?
A. Dilated cardiomyopathy;
B. Hypertrophic cardiomyopathy;
C. Infective endocarditis;
D. Libman-Sachs endocarditis;
E. * Restrictive cardiomyopathy
142. A 49-year-old man 7 days after being admitted to the hospital for an inferior wall, transmural
myocardial infarction suddenly becomes short of breath. Physical examination reveals hypotension,
elevated jugular venous pressure, and muffled heart sounds. His systemic blood pressure drops 13
mmHg with inspiration. Which one of the following pathologic processes produced these clinical
findings?
A. Acute inflammation of the pericardium due to an autoimmune reaction
B. Acute mitral regurgitation due to rupture of a papillary muscle
C. Acute suppurative inflammation of the pericardium due to bacterial infection

D. Serous fluid accumulation in the pericardial cavity due to congestive heart failure
E. * Blood accumulation in the pericardial cavity due to rupture of the ventricular wall
143. A 78-year-old patient suffering from atherosclerosis has been delivered to a surgical ward with signs
of acute abdomen. Laparoscopy revealed blackened and flaccid small intestine loops; demarcation
line is not clear. Diagnose the changes that occurred in the patient’s small intestine:
A. Hemorrhagic infarction complicated with dry gangrene
B. Ischemic stroke complicated with humid gangrene
C. Ischemic stroke complicated with dry gangrene
D. -
E. * Hemorrhagic infarction complicated with wet gangrene
144. A 65-year-old patient had been treated for 3 days in a resuscitation unit for a cardiac pathology.
Suddenly he developed ventricular fibrillation that became the immediate cause of death of this
patient. Microscopy of the left ventricular myocard revealed a large focus of cardiomyocyte
karyolysis demarcated by the zone of hyperaemia. What cardiac pathology was the cause of death?
A. Ischemic myocardial degeneration
B. Acute myocarditis
C. Diffuse cardiosclerosis
D. Postinfarction cardiosclerosis
E. * Acute myocardial infarction
145. In the dead from complications of hypertension, during the section detected small, dense kidneys
with grainy surface, atrophied parenchyma and cortical tissue. Define such kidneys.
A. Amyloid-wrinkled
B. Atrophic
C. Secondary wrinkled
D. Pyelonephrotic wrinkled
E. * Primary wrinkled
146. At autopsy the patient died of heart failure, enlarged heart discovered a mass of 550 g, fibrinous
pericarditis and wrinkled, dense buds weighing 50 grams, with a fine surface and pronounced
Hyalinosis arterioles and glomeruli. What are the underlying disease.
A. Atherosclerosis
B. Rheumatism
C. Pericarditis
D. Cardiomyopathy
E. * Hypertension
147. At autopsy of the dead men, who suffered from hypertension, in the brain revealed a cavity whith
rusty color walls. What preceded these phenomena?
A. Diapedesis hemorrhage
B. Abscess
C. Ischemic heart attack
D. Plasmorrhage
E. * Hematoma
148. Three weeks following a myocardial infarction, a 54-year-old man presents with fever, productive
cough, and chest pain. The pain is worse with inspiration, better when he is sitting up, and not
relieved by nitroglycerin. Physical examination finds a friction rub along with increased jugular
venous pressure and pulsus paradoxus (excess blood pressure drop with inspiration). Which of the
following is the most likely explanation for these findings?
A. Caplan’s syndrome
B. Dressler’s syndrome

C. Ruptured papillary muscle


D. Ventricular aneurysm
E. * Ruptured ventricular wall
149. In a patient with hypertension disease, in the context of a hypertensive crisis, acute renal failure, from
which he died, developed. What are the most likely morphological changes in kidney arterioles?

A. Constrictive atherosclerosis
B. Hyperelastosis
C. Sclerosis
D. Hyalinosis
E. * Fibrinoid necrosis
150. At the autopsy of the deceased who suffered from hypertension, a cavity of round shape of 4 ? 5 cm
with a rusty wall, filled with a yellowish transparent liquid, was found in the left hemisphere of the
brain. What is the pathology that has developed in the brain of the patient?
A. Hematoma
B. Hemorrhagic leakage
C. Ischemic heart attack
D. Abscess
E. * Cyst
151. Autopsy has revealed shrunken kidneys weighing 50 mg, with finegrained surface and uniformly
thinned substance. Microscopic investigation has shown the thickening of arteriole walls due to
accumulation of homogeneous anhistic pink-colored masses in them. Glomerules were undersized,
sclerotic, with atrophied tubules. What disease are these changes characteristic of?
A. Membranous nephropathy
B. Pyelonephritis with kidney shrinkage
C. Renal amyloidosis
D. Acute glomerulonephritis
E. * Essential hypertension
152. At autopsy the occipital lobe of brain was found to have a cavity 2,5x1,5 cm large filled with a
transparent liquid. The cavity had smooth brownish walls. What process had developed in the brain?
A. A cyst on the site of the softening of the cerebrocortical grey matter
B. Softening of the cerebrocortical grey matter
C. Brain abscess
D. Paracephalia
E. * Cyst on the site of a hemorrhage
153. An autopsy of a male patient, who died from heart failure, revealed an enlarged heart weighing 550
g, fibrinous pericarditis, as well as contracted dense kidneys weighing 50 g each and having a fine-
grained surface. Microscopically, the kidneys were characterized by an expressed hyalinosis of
arteioles and glomeruli. Name the basic disease.
A. Pericarditis
B. Rheumatism
C. Atherosclerosis
D. Cardiomyopathy
E. * Hypertensive disease
154. Against a background of hypertensive crisis, a male patient with hypertensive disease developed
acute renal insufficiency which caused his death. What morphological changes in the renal arteioles
were the most probable?
A. Sclerosis
B. Hyperelastosis
C. Stenosing atherosclerosis
D. Hyalinosis
E. * Fibrinoid necrosis
155. A 52-year-old male patient with elevated blood pressure (250/120 mm Hg) died from an impairment
of his cerebral circulation. An autopsy of the brain revealed a red focus in the thalamus, 2.5 cm in
diameter, which sank on section. Microscopically, there was fibrinoid necrosis of the vascular walls
and impregnation of the necrotized brain tissue with blood. Which of the diagnoses listed below was
the most probable?
A. Mixed infarct
B. Cerebral haematoma
C. Anaemic infarct of brain
D. -
E. * Haemorrhagic infarct of brain
156. An autopsy of a male, who suffered from arterial hypertension during his life-time, revealed oedema
of the brain substance, arterial walls at the base were thickened, with white-yellowish plaques in the
intima, the left hemisphere had a focus, 5 x 4 x 3 cm in size, representing a cavity filled with liquid
blood and its clots. Define the pathological process in the brain.
A. Mixed infarct
B. Haemorrhagic infarct
C. Anaemic infarct
D. Transitory ischaemia
E. * Haematoma
157. At autopsy the diminished kidneys with weight of 50.0 have been found out, the surface has been
closed-grained, cortex has been uniformly thinned. At microscopic examination a wall of arterioles
has been considerably thickened because of deposition of homogeneous unstructured pink colored
masses, the lumen has sharply narrowed down, the glomeruli have been reduced, with sclerosis and
atrophy of tubules. What disease the described changes are typical for?
A. Acute necrotic nephrosis
B. Chronic glomerulonephritis
C. Amyloidosis of kidney
D. Pyelonephritis with shrinkage of kidneys
E. * Hypertensive disease
158. The patient, aged 74, with the history of hypertensive syndrome, lost consciousness suddenly and
died of increasing disturbance of respiration and heart activity. The autopsy has demonstrated a dark-
red focus in the trunk of the brain measuring 2x1x5 cm. The weight of the heart is 550 g. the
thickness of left ventricle wall is 2,5 cm. The vessels of the brain base are considerably thickened,
whitish-yellow, the lumen is narrowed. What is the diagnosis?
A. Chronic bronchitis
B. Ischemic heart disease
C. Glomerulonephritis
D. Atherosclerosis
E. * Hypertensive disease
159. An autopsy of a 9-year-old child, who suffered from rheumatism and died of heart failure, revealed
dilatation of cavities in the ventricles of his heart. Microscopically, the myocardial stroma was
characterized by a plethora, oedema, diffuse infiltrations of histiocytes, lymphocytes, neutrophils and
eosinophils. What diagnosis was the most probable one?
A. Focal interstitial exudative myocarditis
B. Granulomatous productive myocarditis
C. Interstitial productive myocarditis
D. Alterative myocarditis
E. * Diffuse interstitial exudative myocarditis
160. An autopsy of a female revealed morphological manifestations of chronic heart failure in the right
ventricle, stenosis of the left atrioventricular aperture, insufficiency of the mitral valve.
Histologically, a connective-tissue disorganization in the form of some mucoid and fibrinoid swelling
was found with presence of blooming Aschoff s bodies against a background of focal cardiosclerosis
in the myocardium. Which of the diagnoses listed below was the most probable?
A. Scleroderma
B. Dermatomyositis
C. Polyarteritis nodosa
D. Systemic lupus erythematosus
E. * Rheumatism
161. A room for dissections received the body of a 56-year-old male who was ill with rheumatism during
8 years and died from cardiovascular insufficiency. An autopsy revealed the rheumatic defect of the
heart -mitral incompetence. A histological examination revealed oedema of the interstice, its diffuse
infiltration by lymphocytes, histiocytes, neutrophilic and eosinophilic leukocytes, as well as
parenchymatous protein and fatty degeneration of the cardiomyocytes. The left atrial auricle had foci
of fibrinoid necrosis surrounded by large macrophages which were located in the form of a veil.
Which of the diagnoses listed below was the most probable?
A. Diffuse isolated myocarditis
B. Focal isolated myocarditis
C. Acute serous myocarditis
D. Acute purulent myocarditis
E. * Productive granulomatous myocarditis
162. Two weeks after angina, a 14-year-old child developed pains in the heart, cyanosis, dyspnoea,
oedemata of his lower extremities. The death was caused by cardiac arrest. On autopsy, the heart was
enlarged, its cavities were dilated, the myocardium was flaccid. A microscopic examination revealed
an acute oedema and plethora of the interstice, an expressed infiltration by lymphocytes, histiocytes,
neutrophils and eosinophils, foci of dystrophy of cardiomyocytes. Which of the diagnoses listed
below was the most probable?
A. Ischaemic dystrophy of myocardium
B. Nodular productive myocarditis
C. Idiopathic myocarditis
D. Focal interstitial exudative myocarditis
E. * Diffuse interstitial exudative myocarditis
163. A room for dissections received the body of a 56-year-old male who was ill with rheumatism during
8 years and died from cardiovascular insufficiency. An autopsy revealed the rheumatic defect of the
heart -mitral incompetence. A histological examination revealed oedema of the interstice, its diffuse
infiltration by lymphocytes, histiocytes, neutrophilic and eosinophilic leukocytes, as well as
parenchymatous protein and fatty degeneration of the cardiomyocytes. The left atrial auricle had foci
of fibrinoid necrosis surrounded by large macrophages which were located in the form of a veil
(Aschoff’s granulomA.. Which of the diagnoses listed below was the most probable?
A. Diffuse isolated myocarditis
B. Focal isolated myocarditis
C. Acute serous myocarditis
D. Acute purulent myocarditis
E. * Productive granulomatous myocarditis
164. What form of rheumatism myocardial lesions observed more frequently in adults?
A. Eosinophilic myocarditis

B. Diffuse acute serous myocarditis


C. Exudative focal myocarditis
D. Alterative myocarditis
E. * Poductive (granulomatous) myocarditis
165. During autopsy of the body of a patient, who had died due to heart failure, the following has been
detected: myogenic dilatation of the heart left ventricle, microfocal cardi- osclerosis, vasculitis,
Aschoff bodies with disorganization of connective tissue, myocardosis. Make the diagnosis:
A. Rheumatic exudative myocarditis
B. Cardiac infarction
C. Systemic lupus erythematosus
D. Myocardial ischemic dystrophy
E. * Rheumatic productive myocarditis
166. Postmortem examination of a patient with a long history of rheumatism revealed thickening and
shortening of the mitral valve leaflets with abundant thrombotic deposits. Histological examnation of
the valve leaflets confirmed sclerosis and revealed multiple foci of connective tissue disorganization
in form of mucoid and fibrinoid swelling, as well as deendothelization foci. Endothelium defects
were covered with thrombotic deposits of 1-2 mm. What type of valvular endocarditis is the case?
A. Acute verrucous endocarditis
B. Fibroplastic endocarditis
C. Diffuse valvulitis
D. Polypous-ulcerative endocarditis
E. * Recurrent verrucous endocarditis
167. Examination of a patient who had been suffering from rheumatism for a long time revealed stenosis
of mitral orifice, death was caused by cardiac and pulmonary insufficiency. Autopsy has shown
brown induration of lungs. What type of circulation disturbance provokes such changes in lungs?
A. Chronic right ventricular insufficiency
B. Acute left ventricular insufficiency
C. Acute right ventricular insufficiency
D. Portal hypertension
E. * Chronic left ventricular insufficiency
168. A 36-year-old female suffers from an expressed deformity of joints of her lingers und Iocs.
Histologically, the periarticular connective tissue reveals some mucoid swelling, foci of fibrinoid
necrosis, clusters of macrophages and areas of sclerosis, the synovial membrane has an oedema of
villi, as well as their mucoid and fibrinoid swelling, the synovial cavity contains "rice bodies". Make
a diagnosis of the disease.
A. -
B. Rheumatism
C. Bekhterev's disease
D. Infectious polyarthritis
E. * Rheumatoid arthritis
169. An autopsy of a 45-year-old female, who died from cardiac failure, revealed on the lateral surfaces of
her both cheeks some reddish-brownish spots which fused on the bridge of the nose. The heart was
enlarged, the myocardium was flaccid in consistency, the cusps of the aortic valve were thickened
and had thrombotic superpositions. The kidneys were motley and had focal haemorrhages. The
pyramids of the medullary layer were dark red, the cortical layer was greyish and had red specks. A
microscopic examination of the kidneys revealed haematoxylin bodies in the tubular epithelial nuclei,
the basal membranes of capillaries of the glomeruli were thickened and gave an appearance of wire
loops, somewhere the walls of the capillaries contained hyaline thrombi and foci of fibrinoid
necrosis. Which diagnosis was the most probable?
A. Rheumatic defect of heart

B. Atherosclerotic defect of heart


C. Septic endocarditis
D. Subacute glomerulonephritis
E. * Systemic lupus erythematosus
170. Histological investigation of skeletal muscle detected lymphocytic infiltration of the walls arteries,
sometimes circular and segmental fibrinoid necrosis, cell proliferation segments of the outer shell of
a transition sclerosis and the formation of small plots thickening of the walls of arteries. Define
pathological process.
A. Wegener's granulomatosis
B. Takayasu's disease
C. Horton disease
D. Syphilitic vasculitis
E. * Polyarteritis nodosa
171. A clinical examination of a 41-year-old male patient revealed some deformity in small joints of his
extremities. A microscopic examination of a biopsy of the synovial coat revealed foci of mucoid and
fibrinoid swelling and fibrinoid necrosis in the stroma of villi and vascular walls, proliferation of
synoviocytes, a perivascular infiltration by lymphocytes, plasmacytes and neutrophils; there was IgG
in the plasmacytes. Which of the diagnoses listed below was the most probable?
A. Polyarthritic form of rheumatism
B. Arthritis in systemic lupus erythematosus
C. Rheumatoid arthritis (stage II)
D. Rheumatoid arthritis (stage III)
E. * Rheumatoid arthritis (stage I)
172. Male 47 years died of a heart attack. Histologically in the wall of the aortic arch and the vessels
departing from it revealed granulomatous arteritis with a mononuclear infiltrate predominance in a
small number of giant multi-cell destruction of elastic fibers, smooth myocyte necrosis and parietal
thrombosis. What disease in this case is most likely?
A. Nonspecific aortitis
B. Rheumatism
C. C Polyarteritis nodosa
D. Wegener's granulomatosis
E. * Takayasu's disease
173. Histological examination of a kidney patient 26 years old who died from renal failure revealed
capillary membranes of glomeruli in the form of wire loops, hyaline thrombi in capillaries, centers of
fibrinoidal necrosis. What is the most likely diagnosis?
A. Nodular periarthritis
B. Rheumatism
C. Scleroderma
D. Rheumatoid arthritis
E. * Systemic lupus erythematosus
174. In a patient with systemic connective tissue disease, the dryness of the conjunctiva and oral cavity is
noted. Puncture biopsy revealed the immune destruction of the salivary glands. Which disease is
most probable?
A. Rheumatism
B. Scleroderma
C. Calculous sialoadenitis
D. Systemic lupus erythematosus
E. * Shegren's syndrome

175. The histological examination of the deformed mitral valve revealed a pronounced basophilic reaction
in the connective tissue of the valve when colored with hematoxylin and eosin, and when the
toluidine blue colored, the reaction was metachromasia. What changes in connective tissue reveal
these reactions?
A. Amyloidosis
B. Fibrinoid swelling
C. Gyalinosis
D. Fibrinoid necrosis
E. * Mucoid swelling
176. In a patient after supercooling, deformity of the joints, pain and limitation of movements in the
fingers of the extremities developed; small dense nodules appeared around the joints. At the biopsy,
nodules found tricus of fibrinous necrosis surrounded by histiocytes. Your diagnosis?
A. Deforming arthrosis
B. Dermatomyositis
C. Rheumatism
D. Gout
E. * Rheumatoid arthritis
177. A patient with high-titer antinuclear antibodies died from progressing renal impairment. Autopsy
revealed mesangioproliferative glomerulonephritis and abacterialpolypous endocarditis. Periarterial
bulbar sclerosis was detected in spleen and productive proliferative vasculitis in skin. What is the
most likely diagnosis?
A. Periarteritisnodosa
B. Nephrotic syndrome
C. Rheumatism
D. Dermatomyositis
E. * Systemic lupus erythematosus
178. A 70-year-old male patient died from acute coronary insufficiency. He had knee joint swelling,
gonycampsis and gonalgia during his lifetime. Pathomorphologic examination of the deformed joints
and synovial membranes revealed membrane hyperaemia with multiple perivascular inflammatory
infiltrations made by lymphocytes, plasmocytes and macrophagocytes. There was an accumulation of
organized fibrin covering some areas of synovium membrane and looking like rice grains in the
articular liquid. What is the most likely diagnosis?
A. Deforming arthrosis
B. Periarteritis nodosa
C. Ankylosing spondylitis
D. Tuberculous arthritis
E. * Atrophic arthritis
179. In autopsy of the 40-year-old woman suffering from rheumatic arthritis, the enlarged solid spleen was
revealed. On section its tissue is of the mahogany color with enlarged follicles, which look like
semitransparent grayish-whitish grains. What pathological process is the most likely?
A. * Sago spleen
B. Glaze spleen
C. Hyaline spleen
D. Porphyric spleen
E. Waxy spleen
180. A 45-year-old patient has died because of uremia in a systemic lupus erythematosus. Formation of
what antigens is most probable in the mechanism of development of this disease?
A. * To native DNA
B. To myocytes

C. To a myosin
D. To endothelial cells
E. Of the rheumatoid factor
181. A patient ill with tuberculosis died from progressing cardiopulmonary decompensation. Autopsy in
the area of the right lung apex revealed a cavity 5 cm in diameter communicating with lumen of a
segmental bronchus. On the inside cavity walls are covered with caseous masses with epithelioid and
Langhans cells beneath them. What morphological form of tuberculosis is it?
A. Tuberculoma
B. Caseous pneumonia
C. Infiltrative tuberculosis
D. Acute focal tuberculosis
E. * Acute cavernous tuberculosis
182. A male patient is 28 years old. Histological study of a cervical lymph node revealed a change of its
pattern due to the proliferation of epithelioid, lymphoid cells and macrophages having nuclei in form
of a horseshoe. In the center of some cell clusters there were non-structured light-pink areas with
fragments of nuclei. What disease are these changes typical for?
A. Hodgkin’s disease
B. Actinomycosis
C. Tumor metastasis
D. Syphilis
E. * Tuberculosis
183. A 63-year-old man, who has been suffering from chronic fibrous-cavernous pulmonary tuberculosis
for 24 years, has been delivered to a nephrology department with uremia. Intravital diagnostic test for
amyloid in the kidneys was positive. What amyloidosis is it in this case?
A. Primary systemic
B. Localized (focal)
C. Hereditary (genetic)
D. Senile
E. * Secondary systemic
184. A 3-year-old child with meni-ngeal symptoms died. Postmortem macroscopy of the pia matter
revealed miliary nodules which were microscopically represented by a focus of caseous necrosis with
masses of epithelioid and lymphoid cells with some crescent-shaped large cells inbetween having
peripheral nuclei. Specify the type of meningitis in the child:
A. Syphilitic
B. Brucellar
C. Grippal
D. Meningococcal
E. * Tuberculosis
185. A 40-year-old prisoner died of tuberculosis in the corrective labor camp. Autopsy of the body
revealed deformation and diminishing of both lung apices; in the both upper lobes there are multiple
cavities with dense walls 2-3 mm thick; in the lower lung lobes there are disseminated foci of caseous
necrosis varying from 5 mm to 2 cm in diameter. Diagnose the type of tuberculosis:
A. Secondary fibrous-focal tuberculosis
B. Hematogenous macrofocal pulmonary tuberculosis
C. Primary tuberculosis, primary affect development
D. Secondary cirrhotic tuberculosis
E. * Secondary fibro-cavitary tuberculosis

186. Autopsy of a man who had tuberculosis revealed a 3-2 cm large cavity in the superior lobe of the
right lung. The cavity was interconnected with a bronchus, its wall was dense and consisted of three
layers: the internal layer was pyogenic, the middle layer was made by tuberculous granulation tissue
and the external one was made by connective tissue. What is the most likely diagnosis?
A. Fibrous focal tuberculosis
B. Tuberculoma
C. Acute focal tuberculosis
D. Acute cavernous tuberculosis
E. * Fibrous cavernous tuberculosis
187. Autopsy of a young man revealed some lung cavities with inner walls made up of granulation tissue
with varying degrees of maturity; pronounced pneumosclerosis and bronchiectasis. Some cavities had
caseation areas. What is your presumptive diagnosis?
A. Infiltrative tuberculosis
B. Caseous pneumonia
C. Acute cavernous tuberculosis
D. Bronchiectasis
E. * Fibrous cavernous tuberculosis
188. A patient was suffering from primary tuberculosis 5 years ago. Radiography has revealed a sharply
marginated nodular shadow with diameter of 4 cm in the 2nd segment of the right lung. Focus was
surgically removed. Histological study has revealed the following: the focus of caseous necrosis
surrounded by the thick capsule of connective tissue. What kind of secondary tuberculosis has
occurred in patient?
A. Cirrhotic tuberculosis
B. Acute cavernous tuberculosis
C. Fibro-cavernous tuberculosis
D. Caseous pneumonia
E. * Tuberculoma
189. A 4-year-old child with meningeal symptoms died. Postmortem macroscopy of the pia matter
revealed miliary nodules which were microscopically represented by a focus of caseous necrosis with
masses of epithelioid and lymphoid cells with large cells containing crescent-shaped peripheral
nuclei situated between them. Specify the type of meningitis in the child:
A. Syphilitic
B. Brucellar
C. Grippal
D. Meningococcal
E. * Tuberculosis
190. A male patient is 20 years old. Histological study of a cervical lymph node revealed a change of its
pattern due to the proliferation of epithelioid, lymphoid cells and macrophages having nuclei in form
of a horseshoe. In the center of some cell clusters there were non-structured light-pink areas with
fragments of nuclei. What disease are these changes typical for?
A. Hodgkin’s disease
B. Actinomycosis
C. Tumor metastasis
D. Syphilis
E. * Tuberculosis
191. Autopsy of a 18 year old girl who died from pulmonary failure revealed a small area of caseous
necrosis in the inferior lobe of the right lung, and occurrences of caseous necrosis in the
bronchopulmonary, bronchial and bifurcational lymph nodes. What is the most probable postmortem
diagnosis?

A. Hematogenous progression of primary tuberculosis


B. Hematogenous tuberculosis with predominant lung affection
C. Tuberculoma
D. Caseous pneumonia under secondary tuberculosis
E. * Primary tuberculosis
192. A patient with tuberculosis died from progressing cardiopulmonary decompensation. Autopsy in the
region of the right lung apex revealed a cavity 5 cm in diameter communicating with lumen of a
segmental bronchus. On the inside cavity walls are covered with caseous masses with epithelioid and
Langhan's cells beneath them. What morphological form of tuberculosis is it?
A. Tuberculoma
B. Caseous pneumonia
C. Infiltrative tuberculosis
D. Acute focal tuberculosis
E. * Acute cavernous tuberculosis
193. A man is 28 years old. Histological investigation of the cervical lymph node revealed a change of its
pattern due to proliferation of epithelioid, lymphoid cells and macrophages with horseshoe-shaped
nuclei. In the center of some cell clusters there were non-structured light-pink areas with fragments
of nuclei. What disease are these changes typical of?
A. Hodgkin’s disease
B. Actinomycosis
C. Tumor metastasis
D. Syphilis
E. * Tuberculosis
194. 48 hours after performing tuberculin test (Mantoux test) to a child a 10 mm papule appeared on the
spot of tuberculin introduction. What hypersensitivity mechanism underlies these changes?
A. Anaphylaxis
B. Antibody-dependent cytotoxicity
C. Immune complex cytotoxicity
D. Granulomatosis
E. * Cellular cytotoxicity
195. Autopsy of a 17 year old girl who died from pulmonary failure revealed a small area of caseous
necrosis in the inferior lobe of the right lung, and occurences of caseous necrosis in the
bronchopulmonary, bronchial and bifurcational lymph nodes. What is the most probable postmortem
diagnosis?
A. Hematogenous progression of primary tuberculosis
B. Hematogenous tuberculosis with predominant lung affection
C. Tuberculoma
D. Caseous pneumonia under secondary tuberculosis
E. * Primary tuberculosis
196. A 20 year old patient died from intoxication 8 days after artificial illegal abortion performed in her
14-15th week of pregnancy. Autopsy of the corpse revealed yellowish colour of eye sclera and of
skin, necrotic suppurative endometritis, multiple pulmonary abscesses, spleen hyperplasia with a big
number of neutrophils in its sinuses. What complication after abortion was developed?
A. Chroniosepsis
B. Hemorrhagic shock
C. Septicemia
D. Viral hepatitis type A
E. * Septicopyemia

197. A man with a wound of his limb that had been suppurating for a long time di-ed from intoxication.
Autopsy revealed extreme emaciation, dehydration, brown atrophy of liver, myocardium, spleen and
cross-striated muscles as well as renal amyloidosis. What diagnosis corresponds with the described
presentations?
A. Septicopyemia
B. Septicemia
C. Chernogubov’s syndrome
D. Brucellosis
E. * Chroniosepsis
198. A 71-year-old man had been presenting with diarrhea for 10 days. The feces had admixtures of blood
and mucus. He was delivered to a hospital in grave condition and died 2 days later. Autopsy of the
body revealed the following: diphtheritic colitis with multiple irregularly-shaped ulcers of different
depth in both sigmoid colon and rectus. Bacteriological analysis revealed Shigella.What was the main
disease?
A. Typhoid fever
B. Salmonellosis
C. Nonspecific ulcerous colitis
D. Yersiniosis
E. * Dysentery
199. Autopsy of a 42-year-old man revealed a distinctly dilated lumen of small intestine filled with rice-
water-like liquid. The intestine wall was edematic with lots of petechial haemorrhages on the
mucosa.What infectious disease is the described enteritis typical for?
A. Dysentery
B. Salmonellosis
C. Amebiasis
D. Typhoid fever
E. * Cholera
200. A worker of a cattle farm consulted a surgeon about fever up to 40oC, headache, weakness. Objective
examination of his back revealed hyperaemia and a dark red infiltration up to 5 cm in diameter with
black bottom in the centre and some pustules. What disease are these presentations typical for?

A. Plaque
B. Tularemia
C. Furuncle
D. Abscess
E. * Anthrax
201. A 42-year-old man died with symptoms of severe intoxication and respiratory failure. A slide of lung
tissue was heterogenous, with multiple microfocal hemorrhages and foci of emphysema. Histological
examination of lungs revealed hemorrhagic abscessing bronchopneumonia; eosinophilic and
basophilic granules in the cytoplasm of epithelial cells of bronchi. What is the most likely diagnosis?
A. Parainfluenza
B. Adenovirus infection
C. Respiratory syncytial virus infection
D. Staphylococcal bronchopneumonia
E. * Influenza
202. Autopsy of a man who died from intraintestinal hemorrhage revealed necrosis of grouped and
solitary follicles, dead tissues imbibed with bile and blood in the ileum; sequestration and rejection of
necrotic masses with defect formation in the lower segment of the intestine. Which of the following
diagnoses is most likely?

A. Typhoid fever, "clean ulcer"stage


B. Typhoid fever, necrosis stage
C. Abdominal typhoid salmonellosis
D. Crohn’s disease
E. * Typhoid fever, ulcerative stage
203. On the base of the clinical data a child was diagnosed with atypical pneumonia resistant to the effects
of beta-lactam antibiotics. The patient’s sputum was cultured and incubated in a special medium,
which resulted in growth of microorganisms formingmicroscopic colonies with a dense center
(looking like fried eggs). What microorganism caused the disease?
A. Klebsiella pneumoniae
B. Streptococcus pneumoniae
C. Legionella pneumophila
D. Chlamidia pneumoniae
E. * Mycoplasma pneumoniae
204. A patient with marked manifestations of exsicosis died in the infectious disease hospital. Postmortem
examination results: the corpse with contracted muscles, dry skin and mucous membranes, thick and
dark blood in veins, edematous plethoric mucosa, distended bowel loops, the lumen contains about 4
liters of ricewater fluid. What is the most likely diagnosis?
A. Enteric fever
B. Dysentery
C. Anthrax, intestinal form
D. Yersiniosis
E. * Cholera
205. A 47-year-old patient with symptoms of severe intoxication and respiratory failure died. A section of
lung tissue had a mottled pattern with multiple small focal hemorrhages and foci of emphysema.
Histological examination revealed hemorrhagic bronchopneumonia accompanied by abscess; the
cytoplasm of bronchial epithelial cells had eosinophil and basophil inclusions. According to the
section analysis, make your diagnosis:
A. Adenovirus infection
B. Parainfluenza
C. Respiratory syncytial
D. Staphylococcal bronchopneumonia
E. * Influenza
206. A 47-year-old male patient consulted a dentist about difficult mouth opening (lockjaw). The patient
has a history of a stab wound of the lower extremity. What infection can be manifested by these
symptoms?
A. Brucellosis
B. Whooping cough
C. Anaerobic wound infection
D. Tularemia
E. * Tetanus
207. A patient, who works as a milkmaid, has made an appoinment with a dentist with complaints of
aphtha-shaped rash on the mucosa of oral cavity. The doctor detected rash on her hands in the area of
nail plates. What agent causes this disease?
A. Herpesvirus
B. Vesicular stomatitis virus
C. Cytomegalovirus
D. Coxsackie A virus
E. * Foot-and-mouth disease virus

208. Autopsy of the body of a 46-year-old man, who had been suffering from typhoid fever and died of
intestinal hemorrhage, has revealed sequestration areas, tissue rejection in the areas of lymphoid
follicle clusters. What stage of typhoid fever is it?
A. Healing
B. Arain-like swelling of the follicles
C. Necrosis
D. Clean ulcers
E. * Dirty ulcers
209. During autopsy approximately 2,0 liters of pus have been found in the abdominal cavity of the body.
Peritoneum is dull and of grayish shade, serous tunic of intestines has grayish-colored coating that is
easily removable. Specify the most likely type of peritonitis in the patient:
A. Hemorrhagic peritonitis
B. Serous peritonitis
C. Tuberculous peritonitis
D. Necrosis
E. * Fibrinopurulent peritonitis
210. A patient, having suffered a thermal burn, developed painful boils filled with turbid liquid in the skin.
What morphological type of inflammation has developed in the patient?
A. Proliferative
B. Croupous
C. Granulomatous
D. Diphtheritic
E. * Serous
211. A 7-year-old child has acute onset of disease: temperature rise up to 38oC, rhinitis, cough,
lacrimation, and large-spot rash on the skin. Pharyngeal mucosa is edematous, hyperemic, with
whitish spots in the buccal area. What kind of inflammation caused the changes in the buccal
mucosa?
A. Suppurative inflammation
B. Fibrinous inflammation
C. Hemorrhagic inflammation
D. Serous inflammation
E. * Catarrhal inflammation
212. During autopsy of a man, who died of acute transmural cardiac infarction, the following has been
detected on the pericardium surface: fibrous whitish-brown deposit connecting parietal and visceral
pericardial layers. What kind of inflammation occurred in the pericardium?
A. Diphtheritic
B. Serous
C. Suppurative
D. Granulomatous
E. * Croupous
213. A 39-year-old man who had been operated for the stomach ulcer died 7 days after the surgery.
Autopsy revealed that peritoneal leaves were dull, plethoric, covered with massive yellow-greenish
films, the peritoneal cavity contained about 300 ml of thick yellow-greenish liquid. What pathologic
process was revealed in the peritoneal cavity?
A. Serous peritonitis
B. Fibrinous serous peritonitis
C. Peritoneal commissures
D. Fibrinoushaemorrhagic peritonitis

E. * Fibrinoussuppurative peritonitis
214. A patient has undergone surgical removal of a cavitary liver lesion 2 cm in diameter. It was revealed
that the cavity wall was formed by dense fibrous connective tissue; the cavity contained murky thick
yellowish-green fluid with an unpleasant odor. Microscopically the fluid consisted mainly of
polymorphonuclear leukocytes. What pathological process are these morphological changes typical
for?
A. Acuteabscess
B. Phlegmon
C. Empyema
D. Furuncle
E. * Chronicabscess
215. Granulomas containing lymphocytes and macrophages were detected during analysis of skin biopsy
material. Among macrophages there are large cells with fat inclusions, which contain
microorganisms in spherical packages (Virchow’s cells). The following disease is based on the
described type of hypersensitivity:
A. Syphilis
B. Tuberculosis
C. Rhinoscleroma
D. Epidemic typhus
E. * Leprosy
216. As a result of careless handling of an iron, a 34-year-old female patient has got acute pain, redness,
swelling of her right index finger. A few minutes later, there appeared a blister filled with a
transparent liquid of straw-yellow color. The described changes are a manifestation of the following
pathological process:
A. Traumatic edema
B. Alternative inflammation
C. Proliferative inflammation
D. Vacuolar degeneration
E. * Exudative inflammation
217. A 7-year-old boy got ill with diphtheria. On the third day he died of asphyxiation. At autopsy the
mucosa of the larynx, trachea and bronchi had thickened, edematous, lusterless appearance and was
covered with gray films which could be easily removed. Specify the type of laryngeal inflammation:
A. Diphtheritic
B. Purulent
C. Catarrhal
D. Intermediate
E. * Croupous
218. There is a 7-year-old child with complains of cough, lacrimation, rhinitis, skin rash, photophobia and
three-day-long fever as high as 38°N. Physical examination has revealed the following:
conjunctivitis; bright red maculopapular rash covering the skin of face, neck and torso; hyperemic
pharynx; serous purulent secretions from the nose; dry rales in the lungs. What is the most probable
diagnosis?
A. Adenovirus infection
B. Scarlet fever
C. Chicken pox
D. Rubella
E. * Measles

219. Autopsy of a dead 6-year-old child revealed a marked edema of the soft tissues of neck and enlarged
tonsils. Pharyngeal mucosa was covered with numerous dense whitish-yellow pellicles exposing deep
ulcers after their removal. Histological examination of the pharyngeal mucosa revealed necrosis of
the upper epithelial layers, impregnation of the mucous memrane with the fibrinous exudate and
moderate leukocyte infiltration. What infectious disease caused the death of the child?
A. Parainfluenza
B. Scarlet fever
C. Whooping cough
D. Measles
E. * Diphtheria
220. A 4-year-old girl died suddenly with symptoms of asphyxia. Autopsy revealed white spots on the
buccal mucosa; large blotches of rash on the skin of face, trunk and extremities; conjunctivitis, edema
with foci of necrosis on the laryngeal mucosa; giant-cell pneumonia on microscopy. What is the most
likely diagnosis?
A. Scarlet fever
B. Influenza
C. Meningococcal infection
D. Typhus
E. * Measles
221. Such presentations as catarrhal conjunctivitis, pharyngitis, laryngotracheobronchitis, white spots on
the buccal mucosa in the region of lower premolar teeth, maculopapular rash on face, body and
extremities are typical for the following disease:
A. Spotted fever
B. Scarlet fever
C. Meningococcal infection
D. Influenza
E. * Measles
222. A child is 10 years old. The followi-ng presentations have developed: sharp pain during swallowing,
swollen neck, body temperature rise up to 39, 0oC , bright-red fi-nely papular rash all over the body.
Pharynx and tonsils are sharply hyperemic ("flaming pharynx"), "crimson tongue". On the tonsi-ls
surface there are isolated greyish necrosis focuses. What disease it might be?
A. Meningococcal nasopharyngitis
B. Diphtheria
C. Influenza
D. Measles
E. * Scarlet fever
223. Autopsy of a Middle-Eastern woman, who had been suffering from wasting fever for a long time,
revealed enlarged blackened liver and spleen. Bone marrow was hyperplastic and black-colored as
well. Cerebral cortex was smoky grey. What disease is it characteristic of?
A. AIDS
B. Epidemic typhus
C. Sepsis
D. Hepatitis
E. * Malaria
224. A child is 7 years old. The following presentations have developed: sharp pain during swallowing,
swollen neck, body temperature rise up to 39,0°C, bright and red finelypapular rash all over the body.
Pharynx and tonsils are sharply hyperemic ("flaming pharynx"), "crimson tongue". On the tonsils
surface there are isolated grayish necrosis focuses. What disease it might be?
A. Meningococcal nasopharyngitis

B. Diphtheria
C. Influenza
D. Measles
E. * Scarlet fever
225. A 9-year-old boy has acute onset of disease: sore throat, body temperature rise up to 39, 5oC; on the
second day diffuse skin rash was detected all over his skin except for nasolabial triangle. On
examination of oral cavity: crimson tongue, "flaming pharynx", necrotic tonsillitis. What diagnosis is
the most likely?
A. Measles
B. Diphtheria
C. Influenza
D. Meningococcemia
E. * Scarlet fever
226. During autopsy the following has been revealed: the meninges of the upper cerebral hemispheres are
extremely plethoric, of yellow-green color and are soaked with purulent effluent. What kind of
meningitis is characterized by such clinical presentations?
A. Tuberculous meningitis
B. Grippal meningitis
C. Anthrax-induced
D. Epidemic typhus-induced
E. * Meningococcal meningitis
227. A 10-year-old child has painful swallowing, neck edema, temperature rise up to 39,0oC, the whole
body is covered with bright-red petechial rash. Back of the throat and tonsils are hyperemic, the
tongue is crimson colored. Tonsillar surface is covered with isolated grayish-colored necrosis nidi.
What disease is it?
A. Meningococcal nasopharyngitis
B. Diphtheria
C. Influenza
D. Measles
E. * Scarlet fever
228. On the 24th day since the onset of disease, a male patient diagnosed with typhoid fever and
undergoing treatment in an infectious diseases hospital has suddenly developed clinical presentations
of acute abdomen leading to the death of the patient. During autopsy peritonitis has been revealed,
with numerous ulcers covering the colon mucosa and reaching as deep as muscular and, in places,
serous tunic. The ulcers have smooth edges and even floor. The intestinal wall is perforated. What
stage of typhoid fever has the lethal complication arisen at?
A. Medullary swelling
B. Necrosis
C. Dirty ulcer
D. Regeneration
E. * Clean ulcer
229. A diseased child has a high fever, sore throat, swelling of submandibular lymph nodes. Objectively:
pharyngeal mucosa is edematous, moderately hyperemic, the tonsils are enlarged, covered with
grayish membrane tightly adhering to the tissues above. Attempts to remove the membrane produce
the bleeding defects. What disease are these presentations typical for?
A. Catarrhal tonsillitis
B. Scarlet fever
C. Meningococcal disease
D. Measles

E. * Diphtheria
230. A 60-year-old male patient was suffering from chronic tibial osteomyelitis during 10 years. Three
years ago he developed the nephrotic syndrome. His death was caused by uraemia. On autopsy, the
kidneys were moderately reduced in size, white, dense and had some cicatrices in their cortical layer.
Indicate the renal pathology which developed.
A. Primary amyloidosis
B. Senile amyloidosis
C. Periodic disease
D. Acute glomerulonephritis
E. * Secondary amyloidosis
231. A 23-year-old male patient acutely fell ill after supercooling. An increased blood pressure,
haematuria and oedemata on his face were observed. The treatment was not effective. Six months
later he died from uraemia. On autopsy, the kidneys were enlarged, flaccid, their cortical layer was
yellow-grey, dim, with some red specks, broad, oedematous and well delimited from the dark red
medullary substance. Microscopically, there was proliferation of the epithelium of the glomeruli,
podocytes and macrophages with formation of (crescents(. Make a diagnosis.
A. Chronic glomerulonephritis
B. Acute glomerulonephritis
C. Acute pyelonephritis
D. Amyloid shrunk kidneys
E. * Subacute glomerulonephritis
232. In a 52-year-old female patient, whose case history had arterial hypertension, phenomena of uraemia
and oedemata of her face began to develop during the past year. A renal biopsy revealed a diffuse
affection of all the studied glomeruli, whose main part had signs of focal and total sclerosis and
hyalinosis, and only in solitary intact glomeruli there was dilation of the mesangium with
proliferation of the mesangial cells. Diagnose the kind of glomerulonephritis.
A. Chronic mesangial proliferative
B. Subacute extracapillary
C. Acute intracapillary
D. Chronic mesangial capillary
E. * Chronic fibroplastic
233. An autopsy of a 62-year-old male patient revealed that his skin was grey-sallow with microfocal
haemorrhages, his face was as if covered with some whitish powder, the patient had fibrinous-
haemorrhagic laryngitis, tracheitis, fibrinous pericarditis, gastritis, enterocolitis. What syndrome is
characterized by this complex of morphological changes?
A. Cushing’s
B. Acute renal failure
C. Chronic cardiac insufficiency
D. Chronic hepatic insufficiency
E. * Chronic renal insufficiency
234. An autopsy of a male patient, who suffered from hypertensive disease and died of an intracerebral
haemorrhage, revealed small dense kidneys with a fine-grained surface, the parenchyma and cortical
substance were atrophic. Indicate the kind of changes in the kidneys.
A. Secondary shrunk
B. Pyelonephritic
C. Amyloid shrunk
D. Nephritic
E. * Primary shrunk

235. A 30-year-old male patient was hospitalized complaining of a headache and faints. During the
previous year, high indices of blood pressure were observed. A renal biopsy revealed an
extracapillary proliferation of the epithelium of the glomerular capsule with formation of (crescents(.
What disease is characterized by such signs?
A. Acute glomerulonephritis
B. Lupus erythematosus glomerulonephritis
C. Membranous nephropathy
D. Chronic glomerulonephritis
E. * Subacute glomerulonephritis
236. A 63-year-old male patient, who was suffering from infectious polyarthritis for a long period of time,
died under the phenomena of renal insufficiency. Macroscopically, his kidneys were significantly
enlarged, dense and waxy. A histological examination of the kidneys revealed sclerosis of the
intermedial region and malpighian pyramids, a large number of lipids and much congophilic
substance in the stroma of the tubules. What pathological process complicated the course of
infectious polyarthritis?
A. Chronic glomerulonephritis
B. Nephrosclerosis
C. Nephrotic syndrome
D. Chronic pyelonephritis
E. * Amyloid shrunk kidneys
237. Soon after she had acute pharyngitis, a 3-year-old girl developed diffuse oedemata (anasarca),
massive proteinuria, hypoalbuminaemia and hyperlipidaemia which were controlled by corticosteroid
preparations. An electron microscopy of a renal biopsy revealed absence of small processes of the
podocytes. What was the most probable diagnosis?
A. Subacute glomerulonephritis
B. Acute glomerulonephritis
C. Focal segmental glomerulosclerosis
D. Membranous nephropathy
E. * Lipoid nephrosis
238. Fourteen days following angina, a 15-year-old child developed oedemata on the face and an
increased blood pressure, the urine resembled (broth slops(. An immunohistological examination of a
renal biopsy revealed deposition of immune complexes on the basal membranes of the tubules and in
the glomerular mesangium. What disease developed in the patient?
A. Lipoid nephrosis
B. Acute pyelonephritis
C. Necrotic nephrosis
D. Subacute glomerulonephritis
E. * Acute glomerulonephritis
239. A microscopic examination of a renal biopsy revealed proliferation of the nephrothelium of the
Bowman’s capsule, podocytes and macrophages with formation of (crescent( structures which
squeezed the glomeruli. The glomerular capillaries were characterized by necrosis, some of them by
sclerosis or hyalinization. An oedema and a lymphoplasmacytic infiltration of the renal stroma were
observed. Name the renal pathology.
A. Chronic glomerulonephritis
B. Chronic pyelonephritis
C. Tubulointerstitial nephritis
D. Acute glomerulonephritis
E. * Subacute glomerulonephritis

240. On macroscopic examination, the kidneys are enlarged and motley because of alternation of light
areas with foci of petechial haemorrhages, their surface is smooth. Microscopically, some vascular
loops of the glomeruli were in the state of fibrinoid necrosis, the basal membranes of the capillaries
were thickened and appeared like (wire loops(, the lumens of some capillaries contained hyaline
thrombi. Make a diagnosis.
A. Chronic glomerulonephritis
B. Necronephrosis
C. Acute glomerulonephritis
D. Lipoid nephrosis
E. * Lupus erythematosus nephritis
241. A 12-year-old boy, who had had streptococcal tonsillitis before, developed oliguria, an increased
blood pressure and oedemata in the facial region. A renal biopsy revealed diffuse affection of all the
glomeruli studied: hyperaemia, solitary fibrin microthrombi in the lumens of the capillaries, dilation
of the mesangium and its infiltration by segmented leukocytes. Indicate the kind of
glomerulonephritis.
A. Chronic mesangial proliferative
B. Chronic mesangial capillary
C. Chronic fibroplastic
D. Subacute extracapillary
E. * Acute exudative
242. A 29-year-old male was admitted to a nephrological department complaining of oedemata, headache,
vomiting and nausea. His death was caused by uraemia. On autopsy, his kidneys were significantly
reduced in size and dense, their capsule was removed with difficulty exposing a fine-grained surface,
the cortical layer was thinned on section. A microscopic examination of a renal biopsy revealed that
most of the glomeruli were scleroid, some of them were characterized by dilation of the mesangium
and proliferation of the mesangial cells. What disease should be suspected?
A. Amyloid shrunk kidneys
B. Chronic pyelonephritis
C. Necrotic nephrosis
D. Fulminating glomerulonephritis
E. * Chronic glomerulonephritis
243. On autopsy of a male, who suffered from hypertensive disease during his lifetime, the kidneys were
reduced in size and dense, their capsule was removed with difficulty, and the surface was fine-
grained. On section, there was an atrophy of the renal parenchyma, particularly that of the cortical
layer, the blood vessels were in the form of thick tubules which did not collapse. A histological
examination revealed hyalinosis and sclerosis of small arteries, hyalinosis and sclerosis of the larger
part of the glomeruli, a reduced number of tubules, a vegetation of the connective tissue in the
stroma. Some glomeruli were characterized by compensatory hypertrophy and their tubules were
dilated. What is your diagnosis?
A. Amyloid shrunk kidneys
B. Secondary shrunk kidneys
C. Fulminating glomerulonephritis
D. Chronic glomerulonephritis
E. * Primary shrunk kidneys
244. On autopsy of a 32-year-old female, who died from renal insufficiency, it was found that the kidneys
were enlarged and flaccid, their capsule was easily removed exposing a motley surface. On section,
the cortical layer was oedematous and definitely delimited from the medullary one. A histological
examination revealed an expressed plethora of vessels, enlarged and oedematous glomeruli, with
clusters of erythrocytes and accumulation of some serous fluid in the lumen of the Bowman’s
capsule. Some vascular loops of the glomeruli were necrotized. An expressed peritubular and
periglomerular lymphoidocytic infiltration was observed. What is your diagnosis?

A. Acute productive glomerulonephritis


B. Acute tubulopathy
C. Acute tubulointerstitial nephritis
D. Acute fibrinous glomerulonephritis
E. * Acute haemorrhagic glomerulonephritis
245. An autopsy of a 70-year-old female, who died from renal insufficiency, revealed dense kidneys
which were reduced in size (each weighing 90 g). Their fibrous capsule was removed with difficulty,
exposing a rough-grained surface; on section, the cortical substance was thinned, the tissue was dim.
Histologically, some glomeruli developed a proliferation of the epithelium in the Bowman’s capsule
with crescent formations, while other glomeruli were characterized by hyalinization. The stroma
contained some lymphoid infiltrates, the vascular walls were thickened, their lumens were narrowed.
What is your diagnosis?
A. Atherosclerotic nephrosclerosis
B. Chronic glomerulonephritis
C. Acute glomerulonephritis
D. Arterial nephrosclerosis
E. * Subacute glomerulonephritis
246. On autopsy of a female, who was suffering from bronchiectatic disease during 15 years and died of
renal insufficiency, it was found that the kidneys were reduced in size, dense and (sebaceous( by
appearance, their surface was fine-grained. Microscopically, the glomeruli were atrophied and
substituted for a connective tissue. There was accumulation of congophilic masses under the intima
and adventitia of the vessels, as well as in the basal membranes of the tubules and in the renal stroma.
Which of the diagnoses listed below was the most probable?
A. Chronic glomerulonephritis
B. Subacute glomerulonephritis
C. Chronic tubulointerstitial nephritis
D. Malignant nephrosclerosis
E. * Amyloid shrunk kidneys
247. Urinalyses of a 44-year-old male patient, who suffers from a chronic renal disease, demonstrate
persistent proteinuria. A microscopic examination of a renal biopsy reveals some diffuse and even
thickening of the basal membranes in the glomerular capillaries with formation of their processes in
the direction of the podocytes ((small thorns(); the proliferation of the mesangiocytes was absent or
poorly expressed; there were small foci of sclerosis in the cortical layer and a fatty degeneration of
the epithelium of the proximal tubules. Make a diagnosis.
A. Mesangial capillary glomerulonephritis
B. Mesangial proliferative glomerulonephritis
C. Lipoid nephrosis
D. Focal segmental glomerular sclerosis
E. * Membranous glomerulonephritis
248. A microscopic examination of a renal biopsy revealed an expressed proliferation of the mesangial
cells with expulsion of their processes to the periphery of the capillary loops (mesangial
interposition), a diffuse thickening and doubling of the basal membranes in the capillaries, and an
accumulation of the mesangial matrix. Indicate the diagnosis which was the most probable.
A. Mesangial proliferative glomerulonephritis
B. Membranous glomerulonephritis
C. Mesangial glomerulonephritis
D. Acute intracapillary glomerulonephritis
E. * Mesangial capillary glomerulonephritis

249. A 38-year-old male patient complained of dyspnoea, a cough with production of a large amount of
sputum, an elevated body temperature up to 38(C, and a reduced urination. An X-ray examination
revealed some focal shadow in the lungs, an increased amount of urea and uric acid in the blood. The
urine was characterized by its low specific weight, presence of proteinuria and casts. The patient died
two weeks later. On autopsy, fibrinous-necrotic laryngotracheobronchitis, fibrinous-haemorrhagic
pneumonia and contracted kidneys were found. Which of the diagnoses listed below was the most
probable?
A. Fibrinous-haemorrhagic pneumonia
B. Influenza with pneumonic complications
C. Chronic glomerulonephritis
D. Chronic destructive bronchitis
E. * Chronic renal insufficiency
250. An electron microscopy of a renal biopsy of a 10-year-old boy with expressed oedemata and
proteinuria revealed absence of small processes of the podocytes in the glomeruli, the basal
membranes of the capillaries were thickened, the mesangium was insignificantly dilated. The lumens
of dilated proximal tubules contained hyaline and granular casts, the epithelium of the tubules had
phenomena of hyaline-drop, hydropic and fatty degeneration. The interstice contained a lot of lipids
and lipophages. Which of the diagnoses listed below was the most probable?
A. Subacute glomerulonephritis
B. Acute glomerulonephritis
C. Membranous glomerulonephritis
D. Mesangial capillary glomerulonephritis
E. * Lipoid nephrosis
251. An autopsy of a 60-year-old female, who had suffered from bronchiectatic disease for 15 years and
died of renal insufficiency, revealed that the kidneys were reduced in size, dense and (sebaceous( by
appearance, their surface was fine-grained. Microscopically, the glomeruli were atrophied and
substituted for a connective tissue. There was accumulation of congophilic masses under the intima
and adventitia of the vessels, as well in the basal membranes of the tubules and in the renal stroma.
Name the most probable diagnosis.
A. Lipoid nephrosis
B. Chronic pyelonephritis
C. Chronic tubulointerstitial nephritis
D. Postnecrotic nephrosclerosis
E. * Amyloid shrunk kidneys
252. Three weeks following a severe supercooling, a young male developed an elevated body temperature,
a pain in his lumbar region, oedemata, proteinuria and haematuria. A renal biopsy demonstrated
enlargement of the glomeruli, a plethora of their capillaries, some eosinophilic fluid, fibrin and
erythrocytes in the cavity of the Bowman’s capsule, the mesangium was infiltrated by
polymorphonuclear leukocytes. What diagnosis was the most probable?
A. Subacute glomerulonephritis
B. Acute renal failure
C. Acute productive glomerulonephritis
D. Acute pyelonephritis
E. * Acute exudative glomerulonephritis
253. In a 25 year-old female patient with severe edemas, hyperproteinuria, hyperlipidemia the biopsy of
kidney established: well-developed thickening of the glomerular capillary wall with the presence of
electron-dense immunoglobulin-containing deposits along the epithelial surface of the basement
membrane. Epithelial cells lost their foot processes. Described changes are characteristic for…
A. Acute Poststreptococcal glomerulonephritis
B. Crescentic glomerulonephritis

C. Chronic glomerulonephritis
D. Acute pyelonephritis
E. * Membranous glomerulonephritis
254. A 12-year-old child has developed nephritic syndrome (proteinuria, hematuria, cylindruria) after
angina of 2 weeks ago. In a biopsy of kidney it was established: diffuse proliferation of endothelial
and mesangial, epithelial cells in glomeruli, infiltration with leukocytes, both neutrophils and
monocytes, interstitial edema. Described changes are characteristic for…
A. Membranous glomerulonephritis
B. Chronic glomerulonephritis
C. Acute necrotic nephrosis
D. Crescentic glomerulonephritis
E. * Acute poststreptococcal glomerulonephritis
255. A 55 year-old man has died after chronic glomerulonephritis, chronic renal insufficiency. In the
autopsy the pathologist has found out characteristic changes in kidneys for this disease, also fibrinous
pericarditis, pleuritis, and bronchitis. Call the cause of the fibrinous inflammation in serosal and
mucosal layers.
A. Hypolipidemia
B. Hyperlipidemia
C. Arterial hypertension
D. Arterial plethora
E. * Uremia
256. For a long time a 49-year-old woman was suffering from glomerulonephritis which caused death.On
autopsy it was revealed that kidneys size was 7x3x2.5 sm, weight is 65,0 g, they are dence and small-
grained. Microscopically: fibrinogenous inflammation of serous and mucous capsules, dystrophic
changes of parenchymatous organs, brain edema. What complication can cause such changes of
serous capsules and inner organs?
A. Thrombopenia
B. Anemia
C. Sepsis
D. DIC-syndrome
E. * Uremia
257. On macroscopic examination of the kidneys in a male, who died from renal insufficiency, a
pathologist found some asymmetrical corticomedullary cicatrices and dilation of the pelvicocaliceal
system. A microscopic examination revealed atrophic tubules which had cyst-like dilations in some
parts, were filled with eosinophilic masses and resembled the thyroid gland, as well as there was
periglomerular sclerosis. Make a diagnosis.
A. Chronic tubulointerstitial nephritis
B. Amyloid shrunk kidneys
C. Acute tubulointerstitial nephritis
D. Necronephrosis
E. * Chronic pyelonephritis
258. On autopsy of a male patient, who died from ethylene glycol poisoning, the kidneys were somewhat
enlarged and oedematous, their capsule was easily removed, the cortical substance was pale grey and
wide, the medullary one was dark red. Microscopically, there were diffuse necroses of the tubules
with ruptures of their basal membranes. What renal pathology developed in the patient?
A. Acute pyelonephritis
B. Acute glomerulonephritis
C. Acute tubulointerstitial nephritis
D. Acute glomerulosclerosis

E. * Necrotic nephrosis
259. In a male patient, who died from chronic renal insufficiency against a background of a chronic renal
disease, the kidneys are symmetrically reduced in size and dense in consistency, their capsule is
removed with difficulty exposing a grained surface. On section, the cortical and medullary layers are
thin, the renal tissue is grey, dry and ischaemic. Name the pathological process in the kidneys.
A. Arteriolosclerotic nephrosclerosis
B. Atherosclerotic nephrosclerosis
C. Amyloid shrunk kidneys
D. Pyelonephritic kidneys
E. * Secondary shrunk kidneys
260. A histological examination of a 56-year-old male, who died from chronic renal insufficiency,
revealed the following changes: focal sclerosis and a lymphoplasmacytic infiltration of the interstice,
mucous membranes of the pelves and calyces, a metaplasia of the transitional epithelium into the
stratified squamous one, numerous encapsulated abscesses. The epithelium of the tubules was in the
state of dystrophy and necrosis. The lumens of solitary tubules were dilated and filled with some
colloid contents, the epithelium was flattened (the tissue resembled the thyroid gland). The glomeruli
were focally scleroid. Name the pathological process.
A. Chronic tubulointerstitial nephritis
B. Necronephrosis
C. Acute pyelonephritis
D. Chronic glomerulonephritis
E. * Chronic pyelonephritis
261. An autopsy of a male revealed a prostatic adenoma and large kidneys with sharply enlarged pelves
and calyces filled with some transparent fluid. Name the process in the kidneys.
A. Polycystic kidneys
B. Glomerulonephritis
C. Tuberculosis
D. Pyelonephritis
E. * Hydronephrosis
262. An autopsy of a male, who died from chronic renal insufficiency, enlarged kidneys with large tubera
on their surface were found. On section, the kidneys revealed numerous smooth-walled cysts filled
with some transparent fluid. Name the pathological process in the kidneys.
A. Hydroureteronephrosis
B. Hydronephrosis
C. Pyonephrosis
D. Hydrocalycosis
E. * Polycystic kidneys
263. On autopsy of a male, who died from burn disease, enlarged swollen kidneys were revealed; their
fibrous capsule was easily removed and tense. On section, the cortical layer was pale grey, wide and
sharply delimited from the dark red medullary one. Microscopically, there was necrosis of the renal
tubules, in some places with a destruction of the basal membranes, an oedema of the interstice, a
leukocytic infiltration and haemorrhages. Which of the diagnoses listed below was the most
probable?
A. Acute pyelonephritis
B. Acute tubulointerstitial nephritis
C. Chronic tubulopathy
D. Chronic pyelonephritis
E. * Acute tubulopathy
264. An autopsy of a male, who died from renal insufficiency, revealed enlarged kidneys with a tense
fibrinous capsule which was easily removed. The cortical layer was pale grey, the malpighian
pyramids were dark red; there were microfocal haemorrhages in the intermediary area of the kidneys
and in the renal pelves. A histological examination revealed dystrophy and necrosis in the epithelium
of the convoluted tubules, in some places with a destruction of the basal membranes mostly in the
distal tubules (tubulorrhexis), there were casts in the lumens of the tubules at different levels of the
nephrones, an oedema of the interstice with its leukocytic infiltration and haemorrhages. The lumens
of the glomerular capsules had an accumulation of the ultrafiltrate. Which of the diagnoses listed
below was the most probable?
A. Acute glomerulonephritis
B. Necronephrosis, the shock stage
C. Acute pyelonephritis
D. Necronephrosis, the stage of restoration of diuresis
E. * Necronephrosis, the oligoanuric stage
265. An autopsy of a 55-year-old female, who died from uraemia, revealed that her kidneys were unevenly
reduced in size, had a dense consistency, large tubera on their surface and whitish cicatrices in the
cortical layer. On section, the renal pelves were dilated, their mucous membrane was whitish and
thickened. Microscopically, the renal interstice was characterized by a focal inflammation, an atrophy
and sclerosis of the renal tissue; the tubules were dilated and had hyaline casts in their lumens. The
arteries and veins were scleroid. Which of the pathological processes in the kidneys was the most
probable?
A. Diabetic nephrosclerosis
B. Acute glomerulonephritis
C. Primary shrunk kidneys
D. Amyloid shrunk kidneys
E. * Pyelonephritic kidneys
266. An autopsy of a male with a syndrome of prolonged compression revealed enlarged swollen kidneys
whose fibrous capsule was easily removed exposing a pale grey cortical layer sharply delimited from
dark red malpighian pyramids. Histologically, there was necrosis in the epithelium of the tubules
with phenomena of tubulorrhexis, an obstruction of the tubules with detritus and crystals of
myoglobin, an oedema of the interstice and its infiltration by heterophilic leukocytes. Which of the
diagnoses was the most probable?
A. Acute pyelonephritis
B. Acute glomerulonephritis
C. Tubulointerstitial nephritis
D. Haemolytic kidneys
E. * Acute renal failure
267. On autopsy of a female, who died from uraemia, it was found that her kidneys differed in size and
their surface had large tubera; there were dense adhesions between the renal surface and capsule.
Microscopically, the renal tissue had foci of the following changes: numerous encapsulated
abscesses, an expressed lymphohistiocytic infiltration of the interstice and a vegetation of the
connective tissue. The mucous membrane of the pelves had foci of metaplasia of the transitional
epithelium into the stratified one. The tubules were partly atrophied, partly stretched and filled with
some colloid-like contents. Which of the diagnoses listed below was the most probable?
A. Chronic glomerulonephritis
B. Acute pyelonephritis
C. Acute glomerulonephritis
D. Tubulointerstitial nephritis
E. * Chronic pyelonephritis
268. A patient suffered with urolithiasis has died as a result uremia. Autopsy has showed an enlargement
of right kidney, its parenchyma has been thinned; the pelvis and calices have been extended, filled
with fluid. In the orifice of ureter there has been a calculus. Call the pathologic process in the right
kidney.
A. Replacement hypertrophy
B. Hyperplasia
C. Neoplasm of kidney
D. Pyelonephritis
E. * Hydronephrosis
269. A 4 year-old child has died because of an uremia. In autopsy the pathologist has found out: the
kidneys have been increased and consisted of large multiple cavities with thin walls contained fluid.
Call this pathological process in kidneys.
A. Necrotic nephrosis
B. Pyelonephritis
C. Glomerulonephritis
D. Infarction
E. * Policystosis
270. On autopsy it is revealed that kidneys are enlarged, surface is large-granular because of multiple
cavities with smooth wall, which are filled with clear fluid. What kidney disease did the patient have?

A. Infarction
B. Necrotic nephrosis
C. Glomerulonephritis
D. Pyelonephritis
E. * Polycystic kidney
271. An autopsy has revealed that kidneys are enlarged, surface is large-granulary because of multiple
cavities with smooth wall, which are filled with clear fluid. What kidney disease did the patient have?

A. Infarction
B. Glomerulonephritis
C. Pyelonephritis
D. Necrotic nephrosis
E. * Polycystic kidney
272. Autopsy of a man who died from ethylenglycol poisoning revealed that his kidneys are a little bit
enlarged, edematic; their capsule can be easily remove. Cortical substance is broad and light-grey.
Medullary substance is dark-red. What pathology had this man?
A. Acute tubular-interstitial nephritis
B. Lipoid nephrosis
C. Acute glomerulonephritis
D. Acute pyelonephritis
E. * Necrotic nephrosis
273. Autopsy of a man who died from burn disease revealed brain edema, liver enlargement as well as
enlargement of kidneys with wide light-grey cortical layer and plethoric medullary area.Microscopic
examination revealed necrosis of tubules of main segments along with destruction of basal
membranes, intersticium edema with leukocytic infiltration and haemorrhages. What is the most
probable postmortem diagnosis?
A. Myeloma kidney
B. Tubulointerstitial nephritis
C. Pyelonephritis
D. Gouty kidney
E. * Necrotic nephrosis
274. A 39-year-old female with a clinical picture of acute abdomen underwent surgical removal of an
enlarged uterine tube. On examination, the serous coat of the uterine tube was dark purple, the lumen
contained some blood clots. A histological examination of the wall of the tube revealed that the
mucous membrane had layers of the decidual cells, and there were villi of the chorion among the
blood clots. What is the most probable diagnosis?
A. Placental polyp
B. Choriocarcinoma
C. Haematosalpinx
D. Salpingitis
E. * Tubal pregnancy
275. A histological examination of a scrape from the uterine cavity of a 45-year-old woman with clinical
manifestations of a disordered ovariomenstrual cycle revealed that the number of the endometrial
glands was increased, the lumens in some of them were significantly dilated and rounded, the
epithelium of the glands had morphological signs of the phase of proliferation. What is your
diagnosis?
A. Atypical hyperplasia of endometrium
B. Glandular polyp of endometrium
C. Glandular hyperplasia of endometrium
D. Endometrial adenocarcinoma
E. * Glandulocystic hyperplasia of endometrium
276. A histological examination of the vaginal portion of the uterine cervix revealed substitution of the
stratified squamous epithelium for the simple columnar one with numerous underlying glands
without any signs of their new formation. Which of the diagnoses listed below was the most
probable?
A. Papillary endocervicosis
B. Progressing endocervicosis
C. Healing endocervicosis
D. Adenosis
E. * Simple endocervicosis
277. A 34-year-old female took medical advice of a gynaecologist complaining of a uterine haemorrhage
for 10 days after her menses. A scrape from the uterine cavity revealed a large number of the
endometrial glands which were spirally convoluted, dichotomically divided and ramified in a tree-
like manner. The stroma of the endometrium was hyperplastic and had a great number of predecidual
cells. Which of the diagnoses was the most probable?
A. Mixed hyperplasia of endometrium
B. Atypical hyperplasia of endometrium
C. Glandulocystic hyperplasia of endometrium
D. Remnants of abortion
E. * Glandular hyperplasia of endometrium
278. A mammary gland of a 25-year-old female revealed a slightly tender node, 3 cm in diameter; a
sectorial resection of the mammary gland was made. Macroscopically, the node was whitish and had
round hollows 0.1-0.3 cm in diameter. Microscopically, there was a vegetation of the connective
tissue with foci of hyalinosis, an atrophy of the glandular lobules and dilation of the lumens in some
places. What disease are the above morphological changes typical for?
A. Fibrous mastopathy
B. Sclerosing adenosis
C. Fibroadenoma of mammary gland

D. Acute mastitis
E. * Fibrocystic mastopathy
279. A woman suffered with dysfunctional metrorrhagia was made a diagnostic abortion. Histologically in
the scrape there were a lot of small stamped glandulas covered by multirowed epithelium. The
lumens of some glandulas were cystically extended. Call the variant of general pathologic process in
the endometrium.
A. Atrophy of endometrium
B. Metaplasia of endometrium
C. Neoplasm of endometrium
D. Hypertrophic growth
E. * Glandular-cystic hyperplasia of endometrium
280. During histological investigation of prostate gland, that has been resected in 72-year-old patient with
symptoms of difficult urination the following signs were found: enlargement of glandular and
muscle’s elements with disturbance of glandular structure. What process is more probable?
A. Muscle-fibrotic hyperplasia
B. Adenocarcinoma
C. Mixed variant of prostatopathy
D. Prostatitis
E. * Glandular hyperplasia
281. At examination of 35-year-old woman it was found out red area about 1 cm in diameter with uneven
edges in cervix. Microscopic examination showed accumulation of numerous glands, look like
uterina ones, under epithelium layer. Call this pathologic process.
A. Endocervicosis
B. Vaginitis
C. Adenomatosis
D. Candidosis
E. * Endometriosis
282. A 25-year-old woman palpates a left breast "lump" on self-examination. Her physician palpates an
ill-defined mass. There is no pain or tenderness. No axillary lymphadenopathy is noted. Fine needle
aspiration is performed and cytologic examination shows cells that appear benign. The lesion
persists, and 6 months later another biopsy is taken and shows ductal epithelial proliferation with
ductal apocrine metaplasia, stromal fibrosis, and sclerosing adenosis. Which of the following is the
most likely diagnosis?
A. Fat necrosis
B. Ductal carcinoma in situ
C. Lobular carcinoma in situ
D. Intraductal papilloma
E. * Fibrocystic changes
283. A clinical study is peformed involving subjects who women are ranging in age from 15 to 45 years
who palpated breast "lumps" on self-examination. The presence of breast mass lesions in these
subjects was subsequently confirmed by physical examination and by mammography. All subjects
had a biopsy or excision of their lesion performed, with a definitive pathologic diagnosis made.
Which of the following diagnoses is likely to be the most frequent in these subjects?
A. Abscess
B. Fibroadenoma
C. Lobular carcinoma in situ
D. Infiltrating ductal carcinoma
E. * Fibrocystic changes

284. A 19-year-old woman gave birth to a healthy male infant at term following an uncomplicated
pregnancy. She has now been breast feeding the baby for a month, but notes that her left breast has
gradually become swollen and painful to touch over the past week. On physical examination her
temperature is 38.2 C. Which of the following is the most likely diagnosis?
A. Fibrocystic disease
B. Fat necrosis
C. Intraductal papilloma
D. Galactocele
E. * Acute mastitis
285. A 35-year-old woman has noted a palpably firm, irregular mass in her right breast for the past 3
months. On physical examination there is no tenderness or swelling. By mammography there is an
irregular 2 cm density that demonstrates scattered microcalcifications. Biopsy of this mass reveals
extensive fat necrosis. Which of the following is the most likely cause for this breast lesion?
A. Pregnancy
B. Prolactinoma
C. Fibrocystic changes
D. Lobular carcinoma in situ
E. * Trauma
286. A 29-year-old woman has been taking oral contraceptives for the past 12 years. She now has a
palpable "lump" in the left breast that has persisted for the past 3 months. Which of the following
breast lesions is most likely to be associated with her oral contraceptive use?
A. Acute mastitis
B. Fat necrosis
C. Hypertrophy
D. Cyst formation
E. * Galactocele
287. A 20-year-old woman gives birth to a term girl infant following an uncomplicated pregnancy. She
breast feeds the infant. Six weeks later, her left breast becomes painful and slightly swollen. On
physical examination there is a tender 3-cm mass in the left breast beneath a nipple that shows
several painful fissures. Which of the following pathologic findings is most likely to be present in
this breast?
A. Infiltrating ductal carcinoma
B. Numerous plasma cells
C. Sclerosing adenosis
D. Fat necrosis
E. * Staphylococcus aureus infection
288. A 30-year-old woman is claiming in a civil lawsuit that her husband has abused her for the past year.
A workup by her physician reveals a 2 cm left breast mass. There is no lymphadenopathy. No skin
lesions are seen, other than a bruise to her upper arm. An excisional biopsy of the breast mass is
performed. On microscopic examination, the biopsy shows fat necrosis. This biopsy result is most
consistent with which of the following etiologies?
A. Physiologic atrophy
B. Lactation
C. Radiation injury
D. Hypoxic injury
E. * Breast trauma

289. A scrape from the uterine cavity revealed in the blood some particles of a neoplasm which had an
organoid structure and consisted of the columnar epithelial cells which formed glandular structures;
the stroma of the neoplasm was significantly developed, the area of its (pedicle( had glomi of thick-
walled vessels. Which of the tumours was the most probable?
A. Glandular hyperplasia of endometrium
B. Uterine fibroadenoma
C. Endometriosis
D. Uterine adenocarcinoma
E. * Endometrial polyp
290. On gynaecological examination of a 36-year-old female, a red focus, which was 0.8 x 0.5 cm in size
and had uneven contours, was found in the mucous coat of the vaginal portion of the uterine cervix
on its border with the mouth of the womb. A microscopical examination revealed substitution of the
stratified squamous epithelium for the columnar one, as well as accumulation and new formation of
glands from the cambial elements of the columnar epithelium in the cervical channel were observed
under the integmentary epithelium. Which of the diagnoses listed below was the most probable?
A. Adenomatosis of uterine cervix
B. Healing endocervicosis
C. Simple endocervicosis
D. Dysplasia of epithelium of uterine cervix
E. * Proliferative endocervicosis
291. In a biopsy of сervix of a 26-year-old woman the diagnosis following was established: pseudo-
erosion. What microscopical changes has the pathologist revealed?
A. Cell-atypia of an epithelium of an mucosal epithelium
B. Keratinization of an epithelium
C. “Carcinomatous pearls”
D. Local inflammation and necrosis in mucosa
E. * Local changes of a stratified squamous epithelium on single-layer prismatic one
292. Diagnostic scraping was performed to the woman with dysfunctional uterine bleeding. Multiple
convoluted glands, ganglially dilated cavities of some glands were revealed histologically in the
scrape. Name the type of general pathological process.
A. Hypertrophic excrescence
B. Metaplasia
C. Displasia
D. Atrophy
E. * Glandular- cystic hyperplasia
293. A patient with gastric juice hypersecretion has been recommended to exclude from the diet rich
broths and vegetable infused water. A doctor recommended it, because these food products stimulate
production of the following hormone:
A. * Gastrin
B. Secretin
C. Cholecystokinin
D. Somatostatin
E. Neurotensin
294. A 49-year-old patient was found to have a disproportionate enlargement of hands, feet, nose, ears,
superciliary arches and cheek bones. Blood test revealed hyperglycemia, impaired glucose tolerance.
What is the most likely cause of this pathology development?
A. * Hypersecretion of growth hormone
B. Posterior pituitary hormone hypersecretion
C. Insulin hyposecretion

D. Vasopressin hyposecretion
E. Glucocorticoid hypersecretion
295. An autopsy of the body of an elderly man, who was suffering from acute intestinal disorder during
his last 2 weeks of life, has revealed the following change in the rectum and sigmoid colon: brown
and green film covering the mucosa is detected. The itestinal wall is thickened; the cavi- ty sharply
narrows down. Microscopy reveals mucosa necrosis of varying depth, necrotic tissue is pierced
through with fibrin threads, leucocytic infiltrati- on is observed. What diagnosis is the most probable?

A. * Fibrinous colitis
B. Catharrhal colon
C. Ulcerative colitis
D. Follicular colitis
E. -
296. A 43-year-old woman complains of weight loss, hyperhidrosis, low-grade fever, increased irritability.
She has been found to have hyperfunction of the sympathetic-adrenal system and basal metabolism.
These disorders can be caused by hypersecretion of the followi- ng hormone:
A. * Thyroxine
B. Somatotropin
C. Corticotropin
D. Insulin
E. Aldosterone
297. Specify the type of jaundice, during which there is no direct bilirubin in blood, and urine
urobilinogen level is high:
A. * Suprahepatic
B. Hepatic
C. Subhepatic
D. Mechanical
E. -
298. A 35-year-old woman complai- ns of swollen neck. Subtotal thyreoidectomy is performed. On
histological examination of the removed part of the thyroid gland the following was detected: atrophy
of parenchyma, moderate sclerosis development, duffuse infiltration by lymphocytes and plasma
cells leading to formation of lymphatic follicles. What pathology has developed in the thyroid gland?

A. * Hashimoto’s thyroiditis
B. Follicular adenoma
C. Riedel’s thyroiditis
D. Papillary carcinoma of the thyroid gland
E. Diffuse toxic goiter
299. A 55-year-old man had been suffering from chronic glomerulonephritis. He died from chronic renal
failure. Macroscopical examination revealed on the surface of epicardium and pericardium some
greyish-white villous depositions. After their removal dilated and plethoric vessels were uncovered.
What process took place in the pericardium?
A. Organization
B. Haemorrhagic inflammation
C. Proliferative inflammation
D. Arterial hyperemia
E. * Fibrinous inflammation

300. Preventive examination of a 55-year-old patient revealed type II diabetes mellitus. An


endocrinologist revealed an increase in body weight and liver enlargement. The man is non-smoker
and doesn't abuse alcohol but likes to have a good meal. Histological examination by means of
diagnostic liver puncture revealed that the hepatocytes were enlarged mostly on the lobule periphery,
their cytoplasm had transparent vacuoles showing positive reaction with sudan III. What liver
pathology was revealed?
A. Acute viral hepatitis
B. Chronic viral hepatitis
C. Alcohol hepatitis
D. Portal liver cirrhosis
E. * Fatty hepatosis
301. A female patient with a tumour of pancreas has developed mechanic jaundice resulting from
compression of a bile-excreting duct. Which duct is compressed?
A. Ductus hepaticus dexter
B. * Ductus choledochus
C. Ductus hepaticus sinister
D. Ductus cysticus
E. Ductus hepaticus communis
302. In a young male, an abundant quantity of the somatotropic hormone and enlargement of the nose,
lips, ears, loWER jaw, hands and feet were revealed. What is your diagnosis?
A. Pituitary dwarfismB.
B. shing's disease
C. Addison's disease
D. Adiposogenital dystrophy
E. * Acromegaly
303. In a male patient with an increased level of the parathormone, a histological examination in the area
of a pathological fracture of his femur revealed foci of a lacunar resolution of the osteoid beams and
new formation of a fibrous tissue. What is your diagnosis?
A. Multiple myeloma
B. Osteoblastoclastoma
C. Paget's disease
D. Osteopetrosis
E. * Parathyroid osteodystrophy
304. A male patient with phenomena of hypothyroidism died from heart failure. On histological
examination, his thyroid gland revealed a diffuse infiltration of the gland by lymphocytes and
plasmacytes, an atrophy of the parenchyma and a vegetation of the connective tissue. What disease
was it?
A. Acute nonsuppurative thyroiditis
B. Ligneous thyroiditis
C. Nodular goiter
D. Toxic goiter
E. * Hashimoto's disease
305. An autopsy of a 45-year-old female, who was suffering from arterial hypertension, diabetes mellitus
and ovarian dysfunction during past 15 years, revealed obesity by the upper type, a pituitary basophil
adenoma in the anterior lobe of the hypophysis, hyperplasia of the adrenal cortex. Which of the
diagnoses listed below was the most probable?
A. Cushing's syndrome
B. Hypertensive disease
C. Addison's disease

D. Adiposogenital dystrophy
E. * Cushing's disease
306. A 50-year-old female took medical advice complaining of excretion of a large amount of urine and
excessive thirst. On examination, her nourishment was reduced, the skin was dry, density of the urine
ranged from 1001 to 1010, data of an ultrasound examination and computed tomography of the brain
revealed a tumour in the posterior lobe of the hypophysis. Indicate the most probable disease.
A. Acromegaly
B. Babinsky-Frelich disease
C. Simmonds disease
D. Cushing's disease
E. * Diabetes insipidus
307. A 46-year-old male patient, who suffered from bulimia, polydipsia, polyuria with glucosuria and
albuminuria, died of renal insufficiency. On autopsy, the kidneys were reduced in size, dense and had
a fine-grained surface. The pancreas was reduced and partially substituted for a fatty tissue.
Microscopically, islets of Langerhans were fine, in some places they were substituted for a
connective tissue, solitary ones were hypertrophic. The kidneys reveal intracapillary
glomerulosclerosis. Which of the diagnoses was the most probable?
A. Subacute glomerulonephritis
B. Chronic indurative pancreatitis
C. Diabetes insipidus
D. Chronic glomerulonephritis
E. * Diabetes mellitus
308. A 36-year-old female patient underwent resection of the both lobes of her thyroid gland; each of
them was 5 x 6 cm in size, pink-yellow, moderately dense and had a tuberous surface. A microscopic
examination revealed follicles of various size, some of them were dilated like cysts and filled with
some colloid; the follicular walls were lined with the smoothed cuboidal epithelium; the stroma of the
gland was redundantly developed owing to the connective tissue, there were foci of calcinosis. Which
of the diseases listed below corresponded most to the changes found?
A. Parenchymatous goiter
B. Toxic goiter
C. Hashimoto's disease
D. Ligneous thyroiditis
E. * Colloid goiter
309. An autopsy of a 24-year-old female (from her case history it is known that a year before the woman
had given birth to a child) revealed a sharp decrease of the body weight down to 38 kg, the skin was
dry and thin, the weight of the internal organs was loWER ed. Also, there was a sharp decrease in the
weight of the adenohypophysis, the latter had cicatrices; there were foci of dystrophy, necrobiosis
and hyalinosis in the diencephalon. The ovaries, thyroid and adrenal glands had phenomena of
hypotrophy, the mucous membrane of the intestines was atrophied. Which of the diagnoses was the
most probable?
A. Nutritional dystrophy
B. Suprarenal cachexia
C. Cachexia associated with chronic amoebiasis
D. Cachexia associated with pellagra
E. * Cerebrohypophysial cachexia

310. An autopsy of a 45-year-old female patient, who suffered from obesity by the upper type, steroid
diabetes mellitus, arterial hypertension and secondary ovarian dysfunction, revealed hypertrichosis,
hirsutism, striae on the skin of the thighs and abdomen. The anterior lobe of the hypophysis
contained a white-pink encapsulated tumour, 2.5 cm in diameter (microscopically, it was a pituitary
basophil adenoma); the adrenal glands were characterized by bilateral hyperplasia of the fascicular
layer. Which of the diagnoses was the most probable?
A. Cushing's syndrome
B. Adiposogenital dystrophy
C. Simmonds disease
D. Pituitary dwarfism
E. * Cushing's disease
311. An autopsy of a male, who died from chronic renal insufficiency, revealed atherosclerosis of the
aorta and large arteries, small and dense kidneys with a finegrained surface, an enlarged yellow-
brown and flaccid liver, the pancreas was reduced in size. Microscopically, there was
atherocalcinosis of the aorta and arteries, an atrophy of the parenchyma, sclerosis and lipomatosis of
the pancreas; the kidneys were characterized by hyalinosis of the mesangium and glomeruli, a
glycogenic infiltration of the epithelium of the tubules, with large-drop adiposis in the hepatocytes.
What pathological process took place in the kidneys?
A. Arterial nephrosclerosis
B. Chronic pancreatitis
C. Chronic glomerulonephritis
D. Steatosis
E. * Diabetic nephrosclerosis
312. On autopsy of a male, who died from uraemia, it was found that the pancreas was reduced in size, his
contracted kidneys had a fine-grained surface, the liver was enlarged, yellow and flaccid.
Microscopically, the pancreatic tissue revealed an atrophy of the parenchyma, including islets of
Langerhans, the atrophied parenchyma was substituted for hyperplastic connective and fatty tissues.
The kidneys were characterized by sclerosis and hyalinosis of the glomeruli, as well as by a
glycogenic infiltration of the tubules; there was a fatty degeneration in the liver and a fibrinous
inflammation in the mucous coats of the trachea, bronchi and stomach. What disease did the died
person suffer from?
A. Chronic indurative pancreatitis
B. Chronic glomerulonephritis
C. Hypertensive disease
D. Steatosis
E. * Diabetes mellitus
313. An autopsy of a male revealed a tumour in the anterior lobe of the hypophysis, enlarged adrenal
glands, a reduction of the gonads in size, a hypertrophy of the left cardiac ventricle, the pancreas was
reduced in size and thickened. Histologically, there was a pituitary basophil adenoma and a
hyperplasia of the cortical layer in the adrenal glands. The pancreas was characterized by a
moderately expressed atrophy of the parenchyma, including islets of Langerhans. What disease did
the patient suffer from?
A. Diabetes mellitus
B. Adiposogenital dystrophy
C. Cushing's syndrome
D. Simmonds disease
E. * Cushing's disease

314. For a histological examination, a lobe and a part of the isthmus of the thyroid gland were received.
The tissue of the gland was dense and tuberous, on section it was pale brown and had grey-whitish
foci. Microscopically, against a background of an atrophy of the follicles of the gland, there was
some diffuse lymphoplasmacytic infiltration of the stroma with formation of lymphoid follicles.
What pathological process were these changes typical for?
A. Toxic goiter
B. Thyroid adenoma
C. Colloid goiter
D. Sporadic goiter
E. * Allergic thyroiditis
315. A histological examination of a thyroid gland revealed follicles of various size and shape which were
lined with the columnar epithelium; the latter proliferated and formed papillae of various size. The
follicular lumens contained some liquid and vacuolized colloid. The stroma of the gland was
characterized by a lymphoplasmacytic infiltration, in some places with formation of lymphatic
follicles having light centres. Which of the diagnoses was the most probable?
A. Colloid goiter
B. Nodular goiter
C. Hashimoto's disease
D. Ligneous thyroiditis
E. * Toxic goiter
316. An autopsy of a 48-year-old male, who died from vascular collapse, revealed an increased
pigmentation of the skin, the adrenal glands were reduced in size, the brown-yellow liver was
enlarged. On histological examination, foci of necrosis with a tuberculous granulation tissue were
found in the adrenal glands. The liver was characterized by phenomena of fatty degeneration. Which
of the diagnoses was the most probable?
A. Steatosis
B. Primary aldosteronism
C. Cushing's syndrome
D. Lipofuscinosis
E. * Addison's disease
317. An autopsy of a male, who suffered from right-sided pneumonia in the lower lobe during his life-
time and for a long period of time expectorated sputum of a purulent character, revealed some cavity
with dense edges that was located in the 9th-10th segments of the lung and was filled with yellowish
cream-like masses. There was some whitish path from the cavity to the root of the lung.
Microscopically, the cavity was separated from the intact pulmonary tissue with a membrane which
consisted of a fibrous connective tissue from the outside and a granulation one from inside. Which of
the diagnoses was the most probable?
A. Bronchiectatic disease
B. Pulmonary gangrene
C. Acute pulmonary abscess
D. Chronic pneumonia
E. * Chronic abscess
318. A male was treated for purulent otitis. On the 9th day of his staying at an inpatient department he
died from a brain oedema. On autopsy, the temporal region of the left hemisphere revealed a cavity
with uneven rough inner edges which was filled with some yellowish-greenish thick dull fluid. The
outer wall of the cavity was represented with the cerebral tissue. What pathological process was it?
A. Colliquative necrosis
B. Phlegmon
C. Empyema
D. Chronic abscess

E. * Acute abscess
319. An autopsy revealed 0.5 I of some yellowish transparent fluid with small white crumble clots in the
right pleural cavity. The parietal and visceral pleurae were covered with a white crumble coat. What
kind of exudative inflammation was it?
A. Suppurative
B. Serous
C. Putrid
D. Catarrhal
E. * Croupous
320. A 6-year-old girl fell ill with diphtheria and three days later died of asphyxia resulting from
membranous croup. On autopsy, the mucous membranes of the larynx, trachea and bronchi were
thickened, oedematous and covered with greyish films which were easily separated. What kind of
inflammation did the morphological changes in the larynx indicate?
A. Serous
B. Haemorrhagic
C. Diphtheritic
D. Croupous
E. * Catarrhal
321. An autopsy of a 77-year-old male, who died from dysentery, revealed some grey-yellow films which
were closely connected with the underlying tissues in the colon and separated with formation of
ulcers. Name the kind of inflammation.
A. Serous
B. Catarrhal
C. Croupous
D. Suppurative
E. * Diphtheritic
322. A microscopic examination of the tissue dissected from some postoperative infiltrate revealed
granulomata with giant multinucleate cells around the suture material. What kind of granulomata did
they belong to?
A. Tuberculous
B. Rheumatic
C. Lepromatous
D. Mycotic
E. * Foreign-body
323. An examination of a renal biopsy revealed some mostly perivascular and periglomerular
lymphocytic, plasmacytic and macrophagal infiltration of the interstice against a background of its
sclerosis. Name the most probable kind of inflammation.
A. Productive diffuse
B. Granulomatous
C. Exudative diffuse
D. Exudative focal
E. * Productive focal
324. A microscopic examination of the myocardium in a male, who died from cardiac decompensation,
revealed sclerosis of the perivascular connective tissue and its diffuse infiltration by lymphocytes,
macrophages, plasmacytes and solitary neutrophils. Which of the listed kinds of inflammation was
the most probable?
A. Granulomatous productive
B. Alterative
C. Exudative diffuse

D. Exudative focal
E. * Interstitial productive
325. A little girl died of asphyxia resulting from membranous croup. A girl illed with diphtheria. On
autopsy, the mucous membranes of the larynx, trachea and bronchi were thickened, oedematous and
covered with greyish films which were easily separated. What type of inflammation
pathomorpholodist investigated?
A. Serous
B. Haemorrhagic
C. Diphtheritic
D. Croupous
E. * Catarrhal
326. A surgeon removed a tumour in the liver of a 47-year-old male patient. It was macroscopically
revealed that the wall of the cavity was formed by a dense fibrous connective tissue; the cavity
contained some yellow-greenish dull thick fluid which had an unpleasant odour and microscopically
consisted mainly of polymorphonuclear leukocytes. What pathological process did such
morphological changes correspond to?
A. Phlegmon
B. Acute abscess
C. Empyema
D. Colliquative necrosis
E. * Chronic abscess
327. A children died from asphyxia at the diphtheria. On autopsy were identified membranous plates in
space of respiratory ways, the mucous membranes of the larynx, trachea and bronchi were thickened,
oedematous and covered with greyish films which were easily separated. What kind of inflammation
did the morphological changes in the larynx indicate?
A. Serous
B. Haemorrhagic
C. Diphtheritic
D. * Croupous
E. Catarrhal
328. An autopsy of a 58-year-old male, who suffered from croupous pneumonia during his life-time and
died of cardiopulmonary insufficiency, revealed 900 ml of some yellow-greenish dull fluid in his
right pleural cavity. The pleural leaves were dull and plethoric. Name the clinical-morphological
form of the inflammation in the pleural cavity.
A. Dry pleurisy
B. Phlegmon
C. Chronic abscess
D. Acute abscess
E. * Empyema
329. An autopsy of a woman, who suffered from the right-sided purulent otitis during her life-time,
revealed a cavity 4 x 3 cm in size in the region of the right temporal lobe that contained some yellow-
green dull viscous fluid. The inner layer of the wall was represented with a yellowish crumble tissue,
the outer layer consisted of a whitish dense tissue. What process were the described changes in the
brain characteristic of?
A. Acute abscess
B. Empyema
C. Grey softening of the brain
D. Haemorrhage
E. * Chronic abscess

330. A pathology-histology laboratory received a vermiform appendix up to 2,0 cm thick. Its serous
membrane was pale, thick and covered with yellowish-green films. The wall was flaccid, of grayish-
red colour. The appendix lumen was dilated and filled with yellowish-green substance. Histological
examination revealed that the appendix wall was infiltrated with neutrophils. Specify the appendix
disease:
A. * Acute phlegmonous appendicitis
B. Acute gangrenous appendicitis
C. Acute superficial appendicitis
D. Acute simple appendicitis
E. Chronic appendicitis
331. Autopsy of a 50-year-old man revealed the following changes: his right lung was moderately
compact in all parts, the dissected tissue was found to be airless, fine-grained, dryish. Visceral pleura
had greyish-brown layers of fibrin. What is the most likely diagnosis?
A. * Croupous pneumonia
B. Tuberculosis
C. Bronchopneumonia
D. Interstitial pneumonia
E. Pneumofibrosis
332. A patient with android-type obesity had been suffering from arterial hypertension, hyperglycemia,
glycosuria for a long time and died from the cerebral haemorrhage. Pathologic examination revealed
pituitary basophil adenoma, adrenal cortex hyperplasia. What is the most likely diagnosis?
A. * Itsenko-Cushing's syndrome
B. Diabetes mellitus
C. Acromegalia
D. Pituitary nanism
E. Adiposogenital dystrophy
333. Mucous membrane of the right palatine tonsil has a painless ulcer with smooth lacquer fundus and
accurate edges of cartlaginous consistency. Microscopically: inflammatory infiltrate that consists of
lymphocytes, plasmocytes, a small number of neutrophils and epithelioid cells; endovasculistis and
perivasculitis. What disease is in question?
A. * Syphilis
B. Actinomycosis
C. Tuberculosis
D. Pharyngeal diphtheria
E. Necrotic (Vincent's) tonsillitis
334. At patomorphological microscopic examination of the myocardium in a male, who died from cardiac
decompensation, revealed sclerosis of the perivascular connective tissue and its diffuse infiltration by
lymphocytes, macrophages, plasmacytes and solitary neutrophils. Which of the listed kinds of
inflammation was the most probable?
A. * Interstitial productive
B. Granulomatous productive
C. Alterative
D. Exudative diffuse
E. Exudative focal
335. A microscopic examination of some inflammatory infiltrate of skin revealed an accumulation of
purolekocytic exudate in several hair follicles. What type of inflammation is this?
A. Phlegmon
B. Furuncle
C. * Carbuncle

D. Wandering abscess
E. Abscess
336. A 77-year-old male died from dysentery. An autopsy revealed some grey-yellow films which were
closely connected with the underlying tissues in the colon and separated with formation of ulcers.
Name the kind of inflammation.
A. * Diphtheritic
B. Serous
C. Catarrhal
D. Croupous
E. Suppurative
337. A tumour in the liver was investigated. Macroscopically was revealed that the wall of the cavity was
formed by a dense fibrous connective tissue; the cavity contained some yellow-greenish dull thick
fluid which had an unpleasant odour and microscopically consisted mainly of polymorphonuclear
leukocytes. What pathological process did such morphological changes correspond to?
A. Phlegmon
B. Acute abscess
C. * Chronic abscess
D. Empyema
E. Colliquative necrosis
338. During patomorphological examination examination of a renal biopsy revealed some mostly
perivascular and periglomerular lymphocytic, plasmacytic and macrophagal infiltration of the
interstice against a background of its sclerosis. Name the kind of inflammation.
A. Productive diffuse
B. * Productive focal
C. Granulomatous
D. Exudative diffuse
E. Exudative focal
339. The myocardium in a male, who died from cardiac decompensation, revealed sclerosis of the
perivascular connective tissue and its diffuse infiltration by lymphocytes, macrophages, plasmacytes
and solitary neutrophils. What kind of inflammation is this most probable?
A. * Interstitial productive
B. Granulomatous productive
C. Alterative
D. Exudative diffuse
E. Exudative focal
340. A microscopic examination of some inflammatory infiltrate of skin revealed an accumulation of
purolekocytic exudate in several hair follicles. What is the name of such an inflammation?
A. Phlegmon
B. Furuncle
C. Abscess
D. Wandering abscess
E. * Carbuncle
341. During pathomorphological investigation of a male, who died from vascular collapse, revealed an
increased pigmentation of the skin, the adrenal glands were reduced in size, the brown-yellow liver
was enlarged. On histological examination, foci of necrosis with a tuberculous granulation tissue
were found in the adrenal glands. The liver was characterized by phenomena of fatty degeneration.
Which of the diagnoses was the most probable?
A. * Addison's disease
B. Steatosis

C. Primary aldosteronism
D. Cushing's syndrome
E. Lipofuscinosis
342. During pathomorphological investigation of a 50-year-old male, who suffered from numerous
pathological fractures during his lifetime, revealed changes in his long tubular bones: the bones of the
thigh and shin were bent, in some places they resembled spirals, their surface was tuberous, a section
revealed an obliterated medullary channel and a change in the compact structure of the cortical layer
by the spongy type. Microscopically, there was a mosaic type of the bone structures: against a
background of a disordered thin-fibrous or lamellar structure of the bone fragments there were
numerous cavities of sinusal resorption combined with signs of new formation of the osseous tissue.
The arteries, which supplied the bone tissue, were dilated and convoluted. Name a diagnosis.
A. * Deforming osteodystrophy
B. Osteopetrosis
C. Parathyroid osteodystrophy
D. Fibrous dysplasia
E. Chronic osteomyelitis
343. A death of a 7-year-old boy resulted from acute posthaemorrhagic anaemia caused by a profuse
bleeding from the gastrointestinal tract. A postmortem examination revealed: macroscopically - an
anaemia of the internal organs, an enlargement of lymph nodes in different groups, thymomegaly, a
moderately manifested hepatosplenomegaly, a bright red bone marrow; microscopically - a
hypercellular bone marrow with some monomorphous infiltrate of blast cells, diffuse-focal tumour
infiltrates in the liver, spleen, lymph nodes, meninges and substance of the brain. Make a diagnosis
for this form of leukaemia.
A. * Acute lymphoblastic
B. Acute myeloblastic
C. Acute stem cell
D. Acute monoblastic
E. Acute plasmablastic
344. An autopsy of 78-year-old male patient, who died from cardiopulmonary insufficiency, revealed
an enlarged right lung with massive fibrinous superpositions on the pleura. Histologically, the
alveolar lumen had accumulations of fibrin and neutrophils. The lymph nodes in the lung roots were
pale pink and somewhat enlarged. Name the stage of croupous pneumonia.
A. Red hepatization
B. Influx
C. * Grey hepatization
D. Resolution
E. Edema
345. An autopsy of a 45-year-old male patient, who had had double bronchopneumonia and died under the
phenomena of intoxication, revealed in the lower lobe of the right lung some thick-walled cavity, 4
cm in diameter, filled with liquid yellowish masses. What pathological process complicated the
course of pneumonia?
A. Tuberculoma
B. * Gangrene
C. Abscess
D. Sequester
E. Empyema
346. On autopsy, 380 ml of some yellow fluid with an unpleasant odour were found in the right pleural
cavity. Microscopically, the liquid contained a lot of neutrophilic granulocytes. What is your
diagnosis?
A. Pulmonary abscess

B. Phlegmon
C. * Pleural empyema
D. Pulmonary gangrene
E. Purulent catarrh of pleura
347. An autopsy of a 34-year-old male, who died from cardiopulmonary insufficiency, revealed an
enlarged grey dense air-free lower lobe of the left lung, the vincernl pleura of this lobe had
superpositions of fibrin. Microscopically, the lumens of alveoli revealed some fibrinous-leukocytic
exudate. Name the stage of croupous pneumonia.
A. Influx
B. Red hepatization
C. Resolution
D. * Grey hepatization
E. Edema
348. An autopsy of a male patient, who died from cardiopulmonary insufficiency, revealed some dim
liquid with grey flakes in the pleural cavities, the lungs were enlarged, air-free and had some grey-
whitish films on the pleura. On section, the lungs were white-greyish and dense, their surface was
granular. The peribronchial lymph nodes were enlarged and whitish-grey on section.
Microscopically, the alveoli contained some fibrinous-leukocytic exudate. Name the stage of
croupous pneumonia.
A. * Grey hepatization
B. Red hepatization
C. Influx
D. Resolution
E. Caseous pneumonia
349. During pathomorphological investigation of a male, who suffered from right-sided pneumonia,
revealed in the right lung some cavity 3.5 cm in diameter, that had uneven edges, communicated with
the bronchus and was filled with some purulent exudate. Microscopically, the wall of the cavity was
formed by the granulation tissue diffusely infiltrated by leukocytes. Which of the diagnoses, listed
below, was the most probable?
A. Acute pneumogenic abscess
B. * Acute bronchiogenic abscess
C. Chronic bronchiogenic abscess
D. Chronic pneumogenic abscess
E. Pulmonary echinococcosis
350. A 7 year old child was taken to the infectious disease hospital with complaints of acute pain during
swallowing, temperature rise up to 390С, neck edem. Objective signs: tonsills are enlarged, their
mucous membrane is plethoric and covered with a big number of whitish-yellowish films that are
closely adjacent to the mucous membran. After removal of these films the deep bleeding defect
remains. What type of inflammation is it?
A. Purulent
B. Crupous
C. Serous
D. Hemorrhagic
E. * Diphteritic
351. Histologic examination revealed in all layers of appendix a big number of polymorthonuclear
leukocytes; hyperemia, stases. What disease are these symptoms typical for?
A. Superficial appendicitis
B. * Phlegmonous appendicitis
C. Gangrenous appendicitis

D. Chronic appendicitis
E. Simple appendicitis
352. Post-mortem examination of a 5 year old boy who died from acute pulmonary and cardiac
insufficiency revealed the following: serohemorrhagic tracheobronchitis with some necrotic areas of
mucous membrane, multiple foci of hemorrhagic pneumonia in lungs. What disease is in question?
A. Diphtheria
B. Croupous pneumonia
C. Scarlet fever
D. * Influenza
E. Measles
353. At the operation the doctor opened a patient's abdominal cavity and revealed for about 2,0 L of
purulent fluid. Peritoneum is dull, greyish, serous tunic of intestines has grayish layers that can be
easily removed. It is most likely to be:
A. Serous peritonitis
B. Hemorrhagic peritonitis
C. Tuberculous peritonitis
D. * Fibrinopurulent peritonitis
E. -
354. During the histologic examination of thyroid gland of a man who died of cardiac insufficiency
together with hypothyroidism there was found the diffusive infiltration of gland by lymphocytes and
plasmocytes, parenchyma atrophy and growth of connective tissue. Formulate a diagnosis:
A. * Hashimoto's thyroiditis
B. Thyroid gland adenoma
C. Purulent thyroiditis
D. Thyrotoxic goiter
E. -
355. During pathomorphological investigation was revealed a big number of polymorphonuclear
leukocytes in all layers of appendix; hyperemia, stases. What disease are these symptoms typical for?
A. * Phlegmonous appendicitis
B. Gangrenous appendicitis
C. Superficial appendicitis
D. Simple appendicitis
E. Chronic appendicitis
356. During pathomorphological investigation of a 45 old-age man microscopic analysis of tissue
sampling from patient's skin reveals granulomas that consist of epithelioid cells surrounded mostly by
T-lymphocytes. Among epithelioid cells there are solitary giant multinuclear cells of Pirogov-
Langhans type. In the centre of some granulomas there are areas of caseous necrosis. Blood vessels
are absent. What disease are the described granulomas typical for?
A. * Tuberculosis
B. Syphilis
C. Leprosy
D. Rhinoscleroma
E. Glanders
357. Autopsy of a man who died of typhoid fever revealed ulcers along the ileum. These ulcers have even
sides, clean fundus formed by muscle layer or even by serous tunic of an intestine. What stage of
disease does the described presentation correspond with?
A. * Stage of "clean" ulcers
B. Stage of medullary swelling

C. Stage of necrosis
D. Stage of "dirty" ulcers
E. Stage of ulcer healing
358. Mucous membrane of the right palatine tonsil has a painless ulcer with smooth lacquer fundus and
accurate edges of cartlaginous consistency. Microscopically: inflammatory infiltrate that consists of
lymphocytes, plasmocytes, a small number of neutrophils and epithelioid cells; endovasculistis and
perivasculitis. What disease is in question?
A. * Syphilis
B. Actinomycosis
C. Tuberculosis
D. Pharyngeal diphtheria
E. Necrotic (Vincent's) tonsillitis
359. A 48 old-age years woman patient with android-type obesity had been suffering from arterial
hypertension, hyperglycemia, glycosuria for a long time and died from the cerebral haemorrhage.
Pathologic examination revealed pituitary basophil adenoma, adrenal cortex hyperplasia. What is the
most likely diagnosis?
A. Pituitary nanism
B. Diabetes mellitus
C. Acromegalia
D. * Itsenko-Cushing's syndrome
E. Adiposogenital dystrophy
360. Enzymatic jaundices are characterized by disbalanced activity of UDP-glucuronyl transferase. What
compound is accumulated in the blood serum in case of these pathologies?
A. * Indirect bilirubin
B. Direct bilirubin
C. Biliverdin
D. Mesobilirubin
E. Verdoglobin
361. A patient with apparent icteritiousness of skin, sclera and mucous membranes was admitted to the
hospital. The patient's urine was of brown ale colour, analysis revealed presence of direct bilirubin.
Feces had low concentration of bile pigments. What type of jaundice is it?
A. * Obturative
B. Parenchymatous
C. Haemolytic
D. Conjugated
E. Absorbtion
362. During pathomorphological investigation a doctor laboratory received a vermiform appendix up to
2,0 cm thick. Its serous membrane was pale, thick and covered with yellowish-green films. The wall
was flaccid, of grayish-red colour. The appendix lumen was dilated and filled with yellowish-green
substance. Histological examination revealed that the appendix wall was infiltrated with neutrophils.
Specify the appendix disease:
A. Acute gangrenous appendicitis
B. * Acute phlegmonous appendicitis
C. Acute superficial appendicitis
D. Acute simple appendicitis
E. Chronic appendicitis
363. A pathohistological examination of the thyroid gland detected a significant infiltration of its tissue by
lymphocytes, formation of lymphoid follicles, an atrophy of parenchymatous elements and a
significant vegetation of the connective tissue. What disease is characterized by this picture?

A. Colloid goiter
B. Endemic goiter
C. Diffuse toxic goiter
D. Parenchymatous goiter
E. * Hashimoto's disease
364. A young female died from adrenal insufficiency. An autopsy was performed in pathoanatomical
bureau detected diffuse hypermelanosis of the skin, hyperplasia of the cells in islets of Langerhans in
the pancreas, the adrenal glands were sharply reduced in size and their thinned cortical substance had
foci of necrosis, haemorrhages and sclerosis. What is your diagnosis?
A. Waterhouse-Friderichsen syndrome
B. Primary aldosteronism
C. Cushing's disease
D. Pheochromocytoma
E. * Addison's disease
365. A 50-year-old male died from vascular collapse. An autopsy was performed in the pathoanatomical
bureau and detected an increased pigmentation of the skin, the adrenal glands were reduced in size,
the brown-yellow liver was enlarged. On histological examination, foci of necrosis with a
tuberculous granulation tissue were found in the adrenal glands. The liver was characterized by
phenomena of fatty degeneration. Which of the diagnoses was the most probable?
A. Steatosis
B. Primary aldosteronism
C. Cushing's syndrome
D. Lipofuscinosis
E. * Addison's disease
366. A 54-year-old male suffered from numerous pathological fractures during his lifetime. After his
death an autopsy detected the changes in his long tubular bones: the bones of the thigh and shin were
bent, in some places they resembled spirals, their surface was tuberous, a section revealed an
obliterated medullary channel and a change in the compact structure of the cortical layer by the
spongy type. Microscopically, there was a mosaic type of the bone structures: against a background
of a disordered thin-fibrous or lamellar structure of the bone fragments there were numerous cavities
of sinusal resorption combined with signs of new formation of the osseous tissue. The arteries, which
supplied the bone tissue, were dilated and convoluted. Name a diagnosis.
A. Osteopetrosis
B. Parathyroid osteodystrophy
C. Fibrous dysplasia
D. Chronic osteomyelitis
E. * Deforming osteodystrophy
367. A 43-year-old male suffered from right-sided pneumonia in the lower lobe during his life-time and
for a long period of time expectorated sputum of a purulent character. After his death an autopsy
detected some cavity with dense edges that was located in the 9th-10th segments of the lung and was
filled with yellowish cream-like masses. There was some whitish path from the cavity to the root of
the lung. Microscopically, the cavity was separated from the intact pulmonary tissue with a
membrane which consisted of a fibrous connective tissue from the outside and a granulation one from
inside. Which of the diagnoses was the most probable?
A. Bronchiectatic disease
B. Pulmonary gangrene
C. Acute pulmonary abscess
D. Chronic pneumonia
E. * Chronic abscess

368. An autopsy of a male, who died from sepsis, revealed a purulent melting of the tissue of the neck and
the fat of the anterior mediastinum. Histologically, there were a lot of small abscesses with a
honeycomb structure, whose centres had basophilic homogenous formations to which some short
rod-like structures were connected with one end. What is your diagnosis?
A. * Actinomycosis
B. Tuberculosis
C. Diphtheria
D. Leishmaniasis
E. Scarlet fever
369. A 45-year-old female furrier died from brain oedema. On autopsy, the pia mater in the vault and base
of the skull were dark red and impregnated with blood, as if they were covered with a (red cap(; the
brain substance was oedematous, the stem was strangulated in the great foramen. Microscopically,
the pia mater had a haemorrhagic inflammation, the cerebral tissue was characterized by a serous-
haemorrhagic inflammation, necrosis of the walls of small vessels, numerous haemorrhages. What
disease was the patient most probably ill with?
A. * Anthrax
B. Meningococcal leptomeningitis
C. Meningococcal meningoencephalitis
D. Subarachnoid haemorrhage
E. Tuberculous leptomeningitis
370. An autopsy of a cachectic 36-year-old male, who during 10 years was suffering from femoral
osteomyelitis with periodic elevations of the body temperature up to 38(C, general weakness, a
headache, and manifestations of intoxication and died from a progressing renal failure, revealed
amyloid shrunk kidneys, a brown atrophy of the myocardium and liver, a sago spleen. What kind of
sepsis was the complication of osteomyelitis in this case?
A. * Chronic septicaemia
B. Fulminant sepsis
C. Septicopyaemia
D. Septicaemia
E. Septic endocarditis
371. An autopsy of a female, who died 4 days after a criminal abortion, revealed jaundice, an expressed
thrombohaemorrhagic syndrome, fibrinous-purulent endometritis, necronephrosis in the kidneys, a
septic hyperplasia of the spleen. Name the clinical-morphological form of sepsis.
A. * Septicaemia
B. Bacterial shock
C. Septicopyaemia
D. Chronic septicaemia
E. Septic endometritis
372. A 40-year-old male butcher died from sepsis. His right cheek had a dark red, conical and dense
infiltrate, 6 cm in diameter, with a black crust in its centre.. The right halves of the face and neck
were oedematous and dense. A microscopic examination revealed an acute serous-haemorrhagic
inflammation in the infiltrate, the centre of the latter had necrosis of the epidermis and underlying
tissues. What diagnosis did a pathologist make?
A. * Anthrax
B. Plague
C. Tularaemia
D. Phlegmon of neck
E. Furuncle

373. During an operation for elimination of phimosis, a round ulcer with a smooth bottom and dense edges
was found on the glans penis. On a microscopic examination of the tissue taken from an edge of the
ulcer a pathologist found a polymorphous infiltrate consisting of disorderly located plasma and
lymphoid elements with some admixture of heterophilic leukocytes and epithelioid cells; the infiltrate
was located mostly around small vessels. The vessels were characterized by phenomena of
endangiitis. What disease are the described changes typical for?
A. * Syphilis
B. Actinomycosis
C. Tuberculosis
D. Polyarteritis nodosa
E. Leprosy
374. On visual examination of a male, who was admitted in a severe state and with a high body
temperature, the physician noticed that the inguinal lymph nodes were enlarged up to 8 cm in
diameter, tightly united with the surrounding tissue and immobile, the skin above them was
hyperaemic and sharply tender. A microscopic examination revealed the most acute serous-
haemorrhagic inflammation in a lymph node. From his case history it was known that the patient had
spent some time before on a mountain pasture. What disease is characterized by the changes
described?
A. * Plague
B. Tularaemia
C. Anthrax
D. Brucellosis
E. –
375. A 40-year-old male hunter at first noticed appearance of some pustule on the index finger of his right
hand, later it turned into an ulcer with uneven edges. His right axillary lymph nodes were
significantly enlarged and tightly united among themselves. A biopsy of the finger skin was made. Its
microscopic examination revealed necrosis of the epithelium and underlying tissues, the derma had
granulomata consisting of epithelioid cells, lymphocytes, a large number of segmented leukocytes
and giant cells. There were areas of necrosis and suppuration in the centres of the granulomata. What
are these changes typical for?
A. * Tularaemia
B. Syphilis
C. Tuberculosis
D. Brucellosis
E. Actinomycosis
376. A 35-year-old milkmaid died from cardiovascular insufficiency. On autopsy, a pathologist diagnosed
polypous-ulcerous endocarditis of the aortic valve. Microscopically, the aortic valve revealed an
inflammatory infiltration by polymorphonuclear leukocytes, areas of ulceration and thrombi with
organization. The myocardial stroma was characterized by sclerosis and granulomata consisting of
disorderly located epithelioid, giant, plasma cells and eosinophils, the vessels had vasculitis. What
disease is characterized by the described changes in the heart?
A. * Brucellosis
B. Chronic sepsis
C. Rheumatism
D. Systemic lupus erythematosus
E. Systemic scleroderma

377. A 40-year-old male butcher died from sepsis. His right cheek had a dark red, conical and dense
infiltrate, 5 cm in diameter, with a black coal-like centre. The right halves of the face and neck were
oedematous and dense. The cervical lymph nodes were enlarged, on section they were dark red, the
surrounding fat was red and oedematous. A microscopic examination revealed the most acute serous-
haemorrhagic inflammation in the infiltrate, the centre of the latter had necrosis of the epidermis and
underlying layers, the lymph nodes and fat were characterized by a serous-haemorrhagic
inflammation. What disease should be suspected?
A. * Anthrax
B. Tularaemia
C. Plague
D. Carbuncle
E. Furuncle
378. The mucous membrane of the right tonsil revealed a painless ulcer with a smooth vanished bottom
and even edges of the chondroid density. Microscopically, a biopsy from the ulcer edge had a
perivascular inflammatory infiltrate consisting of lymphocytes, plasmacytes, some neutrophils and
epithelioid cells. The vessels were characterized by a proliferation of endotheliocytes up to the
absolute closure of the lumens. Name the disease.
A. * Syphilis
B. Actinomycosis
C. Ulceronecrotic angina
D. Candidiasis
E. Faucial diphtheria
379. In a male cattle-breeder, the disease began with appearance of some red spot on the skin of his right
arm; at first, a vesicle with some serous-haemorrhagic fluid formed in the centre of the spot, but later
the central part of the vesicle necrotized and became black. The axillary lymph nodes were sharply
enlarged and dark red on section. Microscopically, a biopsy of the skin and lymph nodes had a
picture of acute serous-haemorrhagic inflammation. What disease was the most probable?
A. * Anthrax
B. Streptococcal pyoderma
C. Plague
D. Actinomycosis of skin
E. Tularaemia
380. For a long time a 45-year-old countrywoman complained of weakness, insomnia, irritability, pains in
her muscles and joints, an elevation of the body temperature up to 37.1-37.3(C. Her death came
under the phenomena of multiorgan insufficiency. On autopsy, the organs revealed granulomata
formed by epithelioid cells, disorderly located Pirogov-Langhans giant cells and a small number of
plasma cells and eosinophils. The granulomata had a lot of vessels, there were no foci of necrosis,
and systemic productive-destructive vasculitis was revealed. What is your diagnosis?
A. * Brucellosis
B. Tularaemia
C. Plague
D. Tuberculosis
E. Anthrax
381. Three days after a criminal abortion a female developed an elevation of her body temperature up to
40(C, cloudiness of consciousness, petechial haemorrhages on her skin. Two days later she died
under increasing phenomena of intoxication. An autopsy revealed jaundice, petechial haemorrhages
in the serous and mucous membranes, a sharp enlargement of the spleen and lymph nodes.
Microscopically, the spleen and lymph nodes were characterized by a proliferation of the lymphoid
and reticular cells, as well as a large number of immature forms of haemopoiesis. Besides, there were
dystrophic changes and an interstitial inflammation in the liver, kidneys, heart, as well as
disseminated necrotic vasculititides. What was the most probable form of sepsis?

A. * Septicaemia
B. Chernogubov’s disease
C. Septicopyaemia
D. Chronic sepsis
E. Fulminant sepsis
382. An autopsy of 2-month-old infant revealed papules and pustules on the skin, white dense lungs
(white pneumonia), a brown dense liver (a brimstone liver), signs of osteochondritis. Indicate the
most probable diagnosis.
A. * Congenital syphilis
B. Congenital toxoplasmosis
C. Congenital cytomegalovirus infection
D. Congenital herpetic infection
E. Congenital mononucleosis
383. An autopsy of an 8-year-old child, who suffered from deafness during his life-time and died of
pneumonia, revealed signs of parenchymatous keratitis and twisted teeth with formation of crescent
grooves on the central incisors (Hutchinson’s triad). Microscopically, there was a chronic diffuse
interstitial inflammation in the liver, lungs and ovarian tissue, as well as Dubois’s abscesses in the
thymus. Which of the diagnoses listed below was the most probable?
A. * Congenital syphilis
B. Congenital toxoplasmosis
C. Congenital cytomegalovirus infection
D. Congenital herpetic infection
E. Congenital mononucleosis
384. A male furrier, who dressed coypu skins (nutrias), died from respiratory insufficiency. On autopsy,
his lung had dense foci whose microscopic examination revealed serous-haemorrhagic pneumonia,
coagulation necroses, granulomata consisting of epithelioid, lymphoid and giant cells, as well as foci
of suppuration. The enlarged regional lymph nodes had the same granulomata, necroses and
suppuration. Which of the diagnoses listed below was the most probable?
A. * Tularaemia
B. Plague
C. Tuberculosis
D. Sarcoidosis
E. Actinomycosis
385. A 32-year-old female died during a puerperal period. An autopsy revealed purulent endometritis,
purulent thrombophlebitis of the uterine veins, numerous pulmonary abscesses, abscesses of the
kidneys and spleen, apostematous myocarditis and purulent meningitis. Make a diagnosis of the
clinical-morphological form of sepsis.
A. * Septicopyaemia
B. Septicaemia
C. Chronic sepsis
D. Chernogubov’s disease
E. Fulminant sepsis
386. A female, who had a phlegmon of her thigh, died from intoxication. An autopsy revealed jaundice,
petechiae on the skin, haemorrhages in the serosae, mucosae and internals. Microscopically, the
spleen and lymph nodes had a proliferation of the lymphoid and reticular cells with accumulation of
immature cells of haemopoiesis in the marrow. The parenchymatous organs were characterized by a
diffuse interstitial inflammation, manifestations of cloudy swelling and fatty degeneration. Which of
the diagnoses listed below was the most probable?
A. * Septicaemia

B. Septicopyaemia
C. Chronic sepsis
D. Plague
E. AIDS
387. During his life-time, a 40-year-old male had a high body temperature, jaundice, clouded
consciousness, bacteraemia. On autopsy, his sclerae and skin were yellow, there were numerous
haemorrhages in the serous and mucous membranes, as well as enlarged lymph nodes and spleen.
Microscopically, the spleen and lymph nodes had a proliferation of the reticular cells. The heart, liver
and kidneys were characterized by an interstitial inflammation, a parenchymatous cloudy swelling;
the vascular walls had vasculititides and a fibrinoid swelling. Which of the diagnoses listed below
was the most probable?
A. * Septicaemia
B. Septicopyaemia
C. Acute lymphocytic leukaemia
D. Acute stem cell leukaemia
E. Chronic sepsis
388. An autopsy of a 40-year-old male, who worked as a sheep-breeder and died from a severe
intoxication, revealed on the skin some papulopustular haemorrhagic rash with necroses and
ulcerations, as well as enlarged immovable dark red lymph nodes in the inguinal region which were
united with one another and had a doughy consistency. The spleen was enlarged, flaccid and had an
abundant scrape. Microscopically, the lungs revealed a focal serous-haemorrhagic inflammation with
necroses; the heart, liver and kidneys were characterized by dystrophic and necrotic changes; the
lymph nodes had haemorrhagic-necrotic lymphadenitis with some purulent melting. Make a
diagnosis.
A. * Bubonic plague
B. Bubocutaneous form of plague
C. Primary pneumonic plague
D. Septicaemic plague
E. Septicaemia
389. A professional male hunter developed an acute elevation of body temperature and enlarged united
lymph nodes in his left axilla. A microscopic examination of the lymph nodes revealed granulomata
consisting of epithelioid, lymphoid, giant cells and polymorphonuclear leukocytes, as well as foci of
necrosis and purulent inflammation. Which of the diagnoses listed below was the most probable?
A. * Tularaemia
B. Plague
C. Tuberculosis
D. Anthrax
E. Brucellosis
390. On autopsy of a 65-year-old male, who for many years was ill with a rheumatic heart defect and died
from cardiovascular insufficiency, it was found that the weight of the heart was 900 g, the patient had
hypertrophy of the left ventricle of the heart, polypous-ulcerous endocarditis of the aortic and mitral
valves with their petrification and deformation, hyperplasia of the spleen, “old” renal and pulmonary
infarcts. A microscopic examination revealed diffuse vasculititides, proliferative isolated
myocarditis, nephritis, hepatitis, glomerulonephritis, the thrombohaemorrhagic syndrome. Which of
the diagnoses listed below was the most probable?
A. * Secondary bacterial endocarditis
B. Chernogubov’s disease
C. Fibroplastic endocarditis
D. Acute verrucous endocarditis
E. Aortomitral heart defect

391. An autopsy of a male revealed that the right upper extremity was oedematous, the skin in the middle
third part of the arm was dark red, dense and diffusely impregnated with blood, in the centre there
was a vesicle with some serous-haemorrhagic exudate (a “phlyctena”). The regional lymph nodes
were enlarged, united with one another and immobile. Microscopically, the lymph nodes were
characterized by a serous-haemorrhagic inflammation, a large number of microbes, a proliferation of
the reticular cells, foci of necrosis. There were dystrophic processes in the parenchymatous organs,
the spleen was enlarged and flaccid. Which of the diagnoses listed below was the most probable?
A. * Bubocutaneous plague
B. Bubonic plague
C. Primary pneumonic plague
D. Septicaemic plague
E. Secondary bubonic pneumonia
392. A male cattle-farm worker developed a dark red spot on the skin of his thigh; in the centre of the spot
there was formation of a vesicle filled with some serous-haemorrhagic fluid, a bit later the centre
necrotized and became dark. The inguinal lymph nodes were enlarged, dense and dark red on section.
Under the phenomena of severe intoxication the patient died. A microscopic examination of the skin
vesicles and lymph nodes revealed a diffuse haemorrhagic infiltration and a sharp oedema of the
surrounding tissues. Which of the forms of anthrax was the most probable?
A. * Cutaneous
B. Septicaemic
C. Enteric
D. Primary pneumonic
E. Mixed
393. An autopsy of a male, who had a prolonged suppuration of the wound following an injury of his
extremity and died under the phenomena of intoxication, revealed cachexia, dehydration, a brown
atrophy of the liver, myocardium, spleen and striated muscles, as well as renal amyloidosis. Which of
the diagnoses was the most probable?
A. * Chronic sepsis
B. Chernogubov’s disease
C. Septicaemia
D. Septicopyaemia
E. Brucellosis
394. A cattle worker has died with signs of severe intoxication in a 2nd day after beginning of disease.
Autopsy was showed the enlarged flabby spleen, on the surface of the cut of dark-cherry colored; the
scrape of the pulp is abundant. The leptomeninges are swelling, infiltrating by blood of dark-red
color (“a red cap”). It was defined a microorganism – bacillus anthracis – and diagnosed anthrax.
What kind of inflammation does occur in that disease?
A. * Hemorrhagic
B. Phlegmonous
C. Fibrinous
D. Purulent
E. Putrefactive
395. A 36-year-old farmer had been suffering from unknown disease for 3,5 months. Microscopical
investigation: there were granulomas in internal organs consisting of epithelioid, Langhans giant-
cells, plasma cells, eosinophils. In granulomas there are a lot of vessels with features of productive-
destructive vasculitis. Diagnose this disease
A. * Brucellosis
B. Malignant anthrax
C. Tuberculosis
D. Classical typhus

E. Leprosy
396. An experimental animal received a subcutaneous dose of an antigen preceded by sensitization. At the
place of the injection, some fibrinous inflammation developed with an alteration of the vascular
walls, the main substance and fibrous structures of the connective tissue in the form of a mucoid and
fibrinoid swelling, a fibrinoid necrosis. Which of the diagnoses listed below was the most probable?
A. Delayed hypersensitivity
B. Transplantation immunoreaction
C. Normergy
D. Granulomatosis
E. * Immediate hypersensitivity
397. The disease in a male hunter began with an elevation of his body temperature up to 37-38(C,
increased reflex excitability, a disturbance of sleep and hydrophobia. Later these signs were
accompanied by spasms of the muscles of the larynx and pharynx, as well as those of respiration. The
patient’s death was caused by arrest of respiration. On autopsy, an oedema and plethora of the brain,
as well as small haemorrhages in the region of the myelencephalon were found. On histological
examination of the brain, its stem part, walls of the 3rd ventricle and hippocampus revealed necrosis
of the nerve cells which were surrounded (as well as small vessels) by nodules consisting of clusters
of microglial and lymphoid cells. The cytoplasm of the nerve cells of the hippocampus contained
some rounded eosinophilic inclusions (Babes-Negri bodies). What disease is characterized by the
picture described?
A. Typhoid fever
B. Epidemic typhus
C. Poliomyelitis
D. Tick-borne encephalitis
E. * Rabies
398. The disease in a 67-year-old woman acutely began with an expressed oedema and tenderness of the
skin and soft tissues of the neck. A phlegmon of neck and mediastinitis were diagnosed. The patient
died under the increasing phenomena of intoxication. On autopsy, the left tonsil was slightly enlarged
and dense; on section, it was yellowish-greenish and had a lot of small cavities which imparted a
honeycomb structure to it. The soft tissues of the neck and the fat of the anterior mediastinum had
signs of purulent melting. Microscopically, the tissue had a lot of small abscesses, their centres
having intensive basophilic formations, which consisted of short rod-like elements connected with
their one end to the common centre. What is your diagnosis?
A. Giardiasis
B. Leishmaniasis
C. Brucellosis
D. Amoebiasis
E. * Actinomycosis
399. A male patient, who came from the Central Asia, had persistent diarrhoeae, a loss of body weight and
signs of intoxication against whose background he died. An autopsy revealed numerous hepatic
abscesses, the caecum was characterized by dingy green areas of necrosis of its mucous membrane,
these areas slightly rose above its surface and penetrated into the muscular layer. The ulcers resulting
from the necrosis were characterized by undermined edges which hung over their bottom. The
inflammatory reaction in the intestinal wall was poorly expressed. What was the most probable
disease in that case?
A. Salmonellosis
B. Cholera
C. Typhoid fever
D. Bacterial dysentery
E. * Amoebiasis

400. An autopsy of a 45-year-old male, who had had a fever with signs of intoxication during his life-time,
revealed an enlarged dense spleen (500 g); on section, its pulp had numerous grey-white and white-
yellow miliary necroses of follicles, and there were infarct-like foci of necrosis under the capsule. A
histological examination revealed hyperplasia of the follicles with breakdown of leukocytes and
accumulation of neutrophils, and numerous thrombi in the vessels. Which of the diagnoses listed
below was the most probable?
A. Plague
B. Haematogenous general miliary tuberculosis
C. Tularaemia
D. Typhoid fever
E. * Relapsing fever
401. The examination of the child with measles showed the non-clear border edematous fluctuated areas
of red-black color in the soft tissues of the cheeks and perineum. What complication did develop in
the child?
A. Dry gangrene
B. Gas gangrene
C. Bedsore
D. Trophic ulcer
E. * Wet gangrene (noma)
402. A 8 year-old child was ill acutely with clinical signs of vomiting, headache and severe intoxication.
After two days of the disease he has died. In autopsy the pathologist has found out: meninges
thickened, yellowish color on basal surface, edema and hyperemia. Meningococcus was detected
from liquor fluid. Diagnose this disease.
A. Scarlet fever
B. Pertussis
C. Diphtheria
D. Measles
E. * Meningococcal infection
403. A 5 year-old girl has died because of asphyxia owing to true croup. In the autopsy it was established;
mucosa of larynx, trachea and bronchi dwarfed, edematous, dull, coated by grayish fibrinous plaques,
which were easily removed. Described morphological changes are characteristic for…:
A. Flu
B. Measles
C. Pertussis
D. Scarlet fever
E. * Diphtheria
404. A 6 year-old child, was ill acutely with signs of intoxication. In 2 day the patient has died. In autopsy
the pathologist has found out: meninges of brain with edema, hyperemia, yellow-grey exudate.
Tissue of brain was edematous. Microscopic investigation: there were neutrophils, hyperemia,
hemorrhages and edema in meninges. Described changes are most typical for:
A. Flu
B. Pertussis
C. Diphtheria
D. Measles
E. * Meningococcal meningitis
405. Patient has suffered from cholera. Clinical dates are dehydratating, cyanosis and convulsions. In the
result of massive infusion therapy the exicosis has been diminished, but anuria has been remained.
Patient has dead because of uremia. What morphological features in kidney have been found out?
A. Choleric typhoid is developed

B. Development of uremia is connected with acute glomerulonephritis


C. Fibrinous colitis is found in autopsy
D. Exicosis is due to action of virus exotoxin
E. * Necrotic nephrosis with cortical necrosis takes place in the kidneys
406. An autopsy of a 45-year-old male, who suffered from numerous pathological fractures during his
lifetime, revealed changes in his long tubular bones: the bones of the thigh and shin were bent, in
some places they resembled spirals, their surface was tuberous, a section revealed an obliterated
medullary channel and a change in the compact structure of the cortical layer by the spongy type.
Microscopically, there was a mosaic type of the bone structures: against a background of a disordered
thin-fibrous or lamellar structure of the bone fragments there were numerous cavities of sinusal
resorption combined with signs of new formation of the osseous tissue. The arteries, which supplied
the bone tissue, were dilated and convoluted. Name a diagnosis.
A. Osteopetrosis
B. Parathyroid osteodystrophy
C. Fibrous dysplasia
D. Chronic osteomyelitis
E. * Deforming osteodystrophy
407. A 20-year-old girl developed complaints about an expressed fatiguability of her ocular, masticatory,
speech and deglutitive groups of muscles, when the normal contraction of the muscles after great
activity absolutely discontinued, but after some rest the functioning of the muscles was restored
again. Some time later the pathological process involved the muscles of the extremities and
intercostal ones. An inadequate ventilation of the lungs resulted in development of the secondary
lobular pneumonia which caused the patient’s death. An autopsy revealed an atrophy of the striated
muscles, their dystrophy with focal clusters of the lymphocytes in the interstice. An enlarged thymus
was characterized by follicular hyperplasia. What was the most probable diagnosis?
A. Pseudohypertrophic muscular dystrophy
B. Amyotrophic lateral sclerosis
C. Werdnig-Hoffman spinal amyotrophy
D. Erb’s muscular dystrophy
E. * Myasthenia
408. An autopsy of a male, who died from uraemia, revealed deformity of the spinal column with a sharp
limitation of mobility. The articular cartilages of small joints of the spinal column were destroyed,
there were some expressed signs of a prolonged chronic inflammation in the articular tissues, the
cavities of the joints were filled with the connective tissue, but in some places with the osseous one
together with formation of ankyloses. The aorta, heart and lungs revealed a chronic inflammation and
focal sclerosis. The kidneys were characterized by amyloidosis. What diagnosis was the most
probable in this case?
A. Rheumatoid arthritis
B. Paget’s disease (deforming osteosis)
C. Parathyroid osteodystrophy
D. Osteopetrosis (marble bone disease)
E. * Bekhterev’s disease
409. On autopsy of a male, who died from uraemia, it was found that the pancreas was reduced in size, his
contracted kidneys had a fine-grained surface, the liver was enlarged, yellow and flaccid.
Microscopically, the pancreatic tissue revealed an atrophy of the parenchyma, including islets of
Langerhans, the atrophied parenchyma was substituted for hyperplastic connective and fatty tissues.
The kidneys were characterized by sclerosis and hyalinosis of the glomeruli, as well as by a
glycogenic infiltration of the tubules; there was a fatty degeneration in the liver and a fibrinous
inflammation in the mucous coats of the trachea, bronchi and stomach. What disease did the died
person suffer from?
A. Chronic glomerulonephritis

B. Hypertensive disease
C. Chronic indurative pancreatitis
D. Steatosis
E. * Diabetes mellitus
410. An autopsy of a male, who died from chronic renal insufficiency, revealed atherosclerosis of the
aorta and large arteries, small and dense kidneys with a fine-grained surface, an enlarged yellow-
brown and flaccid liver, the pancreas was reduced in size. Microscopically, there was
atherocalcinosis of the aorta and arteries, an atrophy of the parenchyma, sclerosis and lipomatosis of
the pancreas; the kidneys were characterized by hyalinosis of the mesangium and glomeruli, a
glycogenic infiltration of the epithelium of the tubules, with large-drop adiposis in the hepatocytes.
What pathological process took place in the kidneys?
A. Arterial nephrosclerosis
B. Chronic pancreatitis
C. Chronic glomerulonephritis
D. Steatosis
E. * Diabetic nephrosclerosis
411. A 52-year-old male died from renal insufficiency. On microscopic examination of his organs, the
pancreas revealed lipomatosis and sclerosis with an atrophy of islets of Langerhans, the kidneys had
hyalinosis of the mesangium and glomeruli (Kimmelstiel-Wilson syndrome) and a glycogenic
infiltration of the epithelium of the tubules, the liver was characterized by fatty degeneration. Which
of the diagnoses listed below was the most probable?
A. Arterial nephrosclerosis
B. Amyloid shrunk kidneys
C. Chronic glomerulonephritis
D. Goodpasture’s syndrome
E. * Diabetic glomerulosclerosis
412. A 53-year-old male patient, who suffered from peptic ulcer of the stomach for more than 25 years,
was admitted to a surgical department with complaints about frequent vomiting after taking food,
progressing loss of weight, severe thirst. At the hospital, the signs of oliguria and later anuria
developed. The patient died. An autopsy revealed a cicatricial stenosis of the pylorus and a sharp
enlargement of the stomach which practically reached the pelvic region. Which of the complication
of peptic ulcer listed below caused the patient’s death?
A. Peritonitis
B. Penetration of ulcer
C. Erosive haemorrhage
D. Malignancy
E. * Chlorhydropenic uraemia
413. A woman with a clinical picture of acute abdomen underwent surgical removal of an enlarged uterine
tube. On examination, the serous coat of the uterine tube was dark purple, the lumen contained some
blood clots. A histological examination of the wall of the tube revealed that the mucous membrane
had layers of the decidual cells, and there were villi of the chorion among the blood clots. Which of
the following contributes to this disease?
A. antigenic incompatibility of tissues of woman and fetus
B. the age is over 30 years old
C. chromosomal aberrations in gametes
D. all answers are correct
E. * the use of intrauterine contraceptives
414. Microscopically, a scrape from the uterine cavity, taken in a 36-year-old female against a background
of uterine bleeding, revealed a neoplasm which consisted of a large number of light epithelial cells of
Langhans and multinuclear symplasts, the number of figures of mitosis was increased. The stroma
was absent, the vascular cavities were lined with the above cells. Make a diagnosis of the uterine
tumour.
A. Endometrial polyp
B. Endometrial adenocarcinoma
C. Simple hydatidiform mole
D. Invasive hydatidiform mole
E. * Choriocarcinoma
415. Within the first stage of labour, the blood pressure in a female elevated (220/110 mm Hg), she
developed convulsions and a loss of consciousness. Her death resulted from a sharp impairment of
cerebral circulation. An autopsy revealed jaundice, an intracerebral haemorrhage, a pulmonary
oedema with microfocal haemorrhages into the pulmonary parenchyma, an enlarged yellow liver
with numerous haemorrhages, enlarged flaccid kidneys with a swollen cortical layer, where on
microscopic examination symmetrical necroses were found. Which of the diagnoses listed below was
the most probable?
A. Viral hepatitis
B. Haemorrhagic insult
C. Necrotic nephrosis
D. Haemorrhagic pneumonia
E. * Eclampsia
416. A 30-year-old woman had ectopic tubal pregnancy which finished with a location of a fetus in the
tubal cavity with bleeding. Call this pathology of pregnancy
A. Complete tubal abortion
B. Spontaneous abortion
C. Induced abortion
D. Criminal abortion
E. * Incomplete tubal abortion
417. The body of a young woman after delivery was taken for autopsy procedure. In autopsy, the
following features were found: enlarged parti-colored dimmed liver with areas of necrosis.
Microscopically the following signs were found in the liver: hemorrhages, thrombosis of the vessels,
proteinous and fatty degeneration of hepatocytes; necrosis of epithelium of kidney’s canals with
fibrinoid necrosis of vessels’ walls and hemorrhages in interstitial tissue; the hemorrhages also can be
seen in brain, heart, lungs and serous membranes. The main cause of death – hepatocellular
insufficiency. What determines the expressed hemodynamic disorders in the liver?
A. specific angioarchitectonics
B. high responsiveness of the liver parenchyma to hypoxia
C. high regenerative capacity of hepatocytes
D. mechanical pressure of the uterus of pregnant woman on the liver
E. * antigenic familiarity of basal membranes of the liver and placenta
418. A woman, 40 weeks of pregnancy, had a cesarian section. From the uterine cavity there were
delivered symmetrically formed twins with split heads while their bodies were divided. What is the
variant of twin deformity?
A. heteropagus
B. homopagus
C. blastopagus
D. all of the terms listed are synonyms
E. * diplopagus
419. A woman, 42 weeks of pregnancy, had a cesarian section. From the uterine cavity there were
delivered symmetrically formed twins with split heads while their bodies were divided. In what
period of development such an anomaly occured?
A. fetogenesis
B. early neonatal period
C. late neonatal period
D. embryogenesis
E. * blastogenesis
420. At the examination of the child in pediatric hospital clinic-morphologic triad was found: congenital
heart defect, deafness and cataracts. Choose the teratogenic factor that was most likely to cause such
changes:
A. X-ray examination in the third trimester of pregnancy
B. alcohol abuse
C. smoking
D. taking thalidomide
E. * rubella in the first trimester of pregnancy
421. A patient W., born in 1960, has defects of the development of the upper limbs - shoulders and
forearms are absent, arms attach directly to the body, are widespread and resemble a walrus flippers.
Choose the teratogenic factor that was most likely to cause such changes:
A. X-ray examination in the second trimester of pregnancy
B. abuse of surrogate alcohol
C. smoking cannabis
D. influenza in the second trimester of pregnancy
E. * taking thalidomide
422. At the ultrasound examination were found multiple congenital malformations of the fetus. Abortion
has been done on medical indications. At the pathoanatomical investigation: the lungs of the fetus are
reduced, the volume of each is approximately ? of the volume of the heart. Histologically structural
elements of the pulmonary tissue are visualized. How to characterize such defect of the respiratory
system?
A. ectopia of the lungs
B. agenesis of the lungs
C. neonatal respiratory distress syndrome
D. aplasia of the lungs
E. * hypoplasia of the lungs
423. At the pathoanatomical examination of the stillborn it was revealed that the esophagus is represented
by two isolated fragments, which are connected by thin cord of connective tissue. The proximal
fragment of the esophagus is combined with the trachea in the region of its bifurcation. What
complication is typical for such congenital defect?
A. hemorragia per diabrosin
B. squamous cell carcinoma of the esophagus
C. such a defect does not affect the health status
D. neonatal respiratory distress syndrome
E. * aspiration pneumonia
424. During the section of the deceased newborn there was found an aperture in the left dome of the
diaphragm, through which the left lobe of the liver and the loop of the small intestine shifted to the
chest cavity, squeezing the left lung. The right lung is compressed by mediastinal organs, the size of
both lungs is reduced. Set diagnosis:
A. situs inversus
B. congenital myotonia

C. birth trauma
D. gastroschisis
E. * false congenital diaphragmatic hernia
425. A patient was hospitalized with blunt abdominal trauma and signs of intra-abdominal bleeding. The
surgeon discovered a spleen rupture, and at its lower pole – extra spleen. In what period of
development such an anomaly occured?
A. blastogenesis
B. fetogenesis
C. early neonatal period
D. late neonatal period
E. * embryogenesis
426. A lung of a premature infant is presented on electronic photomicrography of biopsy material.
Collapse of the alveolar wall caused by the deficiency of surfactant was revealed. Dysfunction of
what cells of the alveolar wall caused it?
A. Alveolar macrophages
B. Secretory cells
C. Alveocytes type I
D. Fibroblasts
E. * Alveocytes type II
427. Autopsy of a fetus who died of intranatal asphyxia due to acute disturbance of utero-placental
circulation revealed small perivascular punctuates hemorrhages in the pia mater, under the
epicardium and pleura. What is the most probable mechanism of the vascular wall damage?
A. Spasm
B. Erosion
C. Edema
D. Rupture
E. * Increased permeability
428. The Respiratory Distress Syndrome often takes place in immature newborns. What is the most
probable cause of this syndrome?
A. Aspiration of amniotic fluid
B. Intrauterine asphyxia
C. Imperfection of nervous regulation of respiration
D. Intrauterine hypercapnia
E. * Immaturity of alveolar-parenchyma connected with deficiency of surfactant
429. Autopsy of the newborn showed jaundice of the skin, signs of the bilirubin encephalopathy in the
brain substance, bilirubin infarctions in the kidneys, enlarged liver and spleen. His mother is Rh-
negative. The child died on the third day after birth. What is your diagnosis?
A. Birth injury
B. Pneumonia in newborn
C. Respiratory distress syndrome of newborn
D. Edematous hemorrhagic syndrome
E. * Hemolytic disease of newborn
430. Microscopic examination of the lungs of a dead 2-day-old newborn showed: pinkish masses that line
the respiratory bronchioles, alveoli. These masses are largely made up of fibrinogen and fibrin,
admixed with cell debris chiefly from necrotic alveolar lining pneumocytes. Areas of dys- and
atelectasis are found out too. The deficiency of which substance plays a leading role in the
development of disease
A. vitamin K

B. fetal hemoglobin
C. immunoglobulins A and G
D. tyrosine
E. * surfactant
431. In autopsy of a 6-month-old infant it was found out: small cysts in pancreatic glands, signs of chronic
bronchitis with atelectasis, bronchiectasis, fatty changes and cholestasis in liver, coprostasis in
intestine. Described changes are typical for…:
A. Hemolytic disease of newborn
B. Pneumonia in newborn
C. Respiratory distress syndrome of newborn
D. Edimatous hemorrhagic syndrome
E. * Mucoviscidosis
432. In microscopic examination of a 2-day-old pre-term-newborn it was found out: diffuse edema and
numerous hemorrhages in the lungs, pulmonary capillaries overfull by blood. What is the probable
diagnosis?
A. Hemolytic disease of newborn
B. Birth injury
C. Pneumonia in newborn
D. Respiratory distress syndrome of newborn
E. * Edematous-hemorrhagic syndrome
433. In autopsy of still-born it was found out: teratoma in ovary, nevus on skin, rupture of tentorium
cerebelli, hypoplasia of thymus, polydactilia. What is the cause of death?
A. Hypoxia
B. Anoxia
C. Birth injury
D. Pneumonia
E. * Rupture of cerebellar palatinum
434. In autopsy of still-born it was found out: teratoma in ovary, nevus on skin, rupture of tentorium
cerebelli, hypoplasia of thymus, polydactilia. What is the variant of perinatal pathology?
A. Hemolytic disease of newborn
B. Pneumonia in newborn
C. Respiratory distress syndrome of newborn
D. Edimatous hemorrhagic syndrome
E. * Birth injury
435. On autopsy of a 71-year-old male, who had worked as a stone grinder for 14 years and died from
cardiopulmonary insufficiency, his lungs were enlarged and dense, they revealed numerous miliary
and larger grey or grey-black nodules rounded, oval or irregular in shape and dense in consistency. A
microscopic examination revealed some peribronchial and perivascular vegetation of the connective
tissue, scleroid interalveolar septa, catarrhal-desquamative bronchitis, bronchoectases, a diffuse
emphysema. The nodules were represented by clusters of coniophages with collagen fibres among
them, some nodules had a fibrous structure. What disease is characterized by the above changes in
the lungs?
A. Anthracosis
B. Bronchiectatic disease
C. Chronic bronchitis
D. Bronchial asthma
E. * Silicosis

436. An autopsy of a 59-year-old male, who had worked in mine for 19 years, revealed the following
changes in the lungs: chronic deforming bronchitis, chronic bronchopneumonia, an expressed
emphysema, sclerotic changes in the vascular walls, irregular caverns with crumbling black walls and
some black contents. The lymph nodes were enlarged and black. Histologically, the lungs revealed
perivascular and peribronchial sclerosis, sclerosis of interalveolar septa and a large number of
macrophages whose cytoplasm contained some grey-black or slate pigment. Name the most probable
pathology.
A. Siderosis
B. Silicosis
C. Aluminosis
D. Asbestosis
E. * Anthracosis
437. An autopsy of a young male, who served aboard a nuclear submarine and during his life-time
revealed severe anaemia, leukopenia, thrombocytopenia and an expressed haemorrhagic syndrome,
revealed the following changes: panmyelophthisis, decomposition of lymphocytes in the lymph
nodes, spleen and lymphatic apparatus of the gastrointestinal tract, as well as haemorrhages in the
mucosal membranes of the stomach, intestines and adrenal glands. What disease developed in this
case?
A. Acute hypoplastic anaemia
B. Vibration disease
C. Acute leukaemia
D. Decompression sickness
E. * Acute radiation sickness
438. An autopsy of a 56-year-old male, who had worked as a driller of boreholes during his life-time,
revealed dry gangrene of the toes and feet, an atrophy of the forearm, deltoid and rhomboid muscles.
Microscopically, the muscles were characterized by an atrophy, the distal epiphyses of the radial and
ulnar bones had foci of osteoporosis and sclerosis, the carpal bones had numerous cysts, marks of
pathological fractures and deformities; in the arteries there was a sharp narrowing of their lumens up
to the absolute obliteration. What disease developed in this case?
A. Decompression sickness
B. Deforming arthrosis
C. Myopathy
D. Obliterating endarteritis
E. * Vibration disease
439. An autopsy of a 43-year-old male, who had worked as a diver during his life-time, revealed in the
long bones of his lower extremities some foci of osteoporosis surrounded by an area of sclerosis, as
well as foci of aseptic necrosis of the bone tissue and osteomyelitis. The joints were characterized by
deformity, an atrophy of the cartilage, phenomena of arthritis. What disease developed in this man?
A. Obliterating endarteritis
B. Deforming arthrosis
C. Myopathy
D. Vibration disease
E. * Decompression sickness
440. An autopsy of a female, who had worked as a radiologist for 20 years and died from double
pneumonia, revealed numerous haemorrhages of various remoteness on the skin, mucous and serous
membranes, general haemosiderosis. The marrow of the sternum was yellow, the liver and kidneys
were yellow-clay and flaccid, the myocardium was flaccid, from the side of the endocardium it had
yellow-white lines. Microscopically, the marrow was characterized by panmyelophthisis, there was
fatty degeneration of the parenchymatous organs and purulent pneumonia in the lungs. What disease
developed in this case?

A. Hypoplastic anaemia
B. Aplastic anaemia
C. Acute radiation sickness
D. AIDS
E. * Chronic radiation sickness

You might also like